You are on page 1of 435

SEALES Y SISTEMAS

Jos Morn

SEALES Y SISTEMAS

Universidad Rafael Urdaneta


Autoridades Rectorales
Dr. Jess Esparza Bracho, Rector
Ing. Maulio Rodrguez, Vicerrector Acadmico
Ing. Salvador Conde, Secretario
Lic. Nancy Villarroel M.L.S. Directora de Biblioteca

2011 Fondo Editorial Biblioteca Universidad Rafael Urdaneta

Portada: Luz Elena Hernndez


Universidad Rafael Urdaneta, Fondo Editorial Biblioteca
Vereda del Lago, Maracaibo, Venezuela.

ISBN: 978-980-7131-06-3
Deposito Legal:lfi2382011620521

CAPTULO UNO

SEALES Y SISTEMAS
1.1

Introduccin 1

1.2

Seales y Clasificacin de Seales 2

1.3

Seales Peridicas y No Peridicas 6

1.4

Seales de Potencia y de Energa 8

1.5

Transformaciones de la Variable Independiente 12

1.6

Escalamiento en el Tiempo 16

1.7

Seales Pares e Impares 18

1.8

Seales de Tiempo Continuo Bsicas 21


1.8.1
1.8.2
1.8.3
1.8.4

1.9

Seales de Tiempo Discreto Bsicas 33


1.9.1
1.9.2
1.9.3
1.9.4
1.9.5
1.9.6
1.9.7
1.9.8

1.10

Seales Exponenciales Complejas


21
Seales Exponenciales Complejas Generales 26
La Funcin Escaln Unitario 27
La Funcin Impulso Unitario 27

Secuencias Exponenciales Complejas Generales 33


Secuencias Exponenciales Reales 33
Seales Sinusoidales 34
Seales Exponenciales Complejas Generales 34
Periodicidad de las Exponenciales Complejas 35
Periodicidad de la Exponencial Compleja 36
La Secuencia Escaln Unitario 38
La Secuencia Impulso Unitario 38

Sistemas y Clasificacin de Sistemas 39


1.10.1 Sistemas en Tiempo Continuo y en Tiempo Discreto 40
1.10.2 Sistemas Con y Sin Memoria 42
1.10.3 Invertibilidad y Sistemas Inversos 43

ii

1.10.4
1.10.5
1.10.6
1.10.7

1.11

Sistemas Causales 44
Sistemas Estables 46
Invariabilidad en el Tiempo 47
Sistemas Lineales 49

Interconexin de Sistemas 51
Problemas 53

CAPTULO DOS

SISTEMAS LINEALES E INVARIANTES EN EL


TIEMPO
2.1
2.2

Introduccin 61
Sistemas LIT en Tiempo Discreto 62
2.2.1

2.3

Sistemas LIT Discretos: la Suma de Convolucin 63


2.3.1
2.3.2

2.4

Propiedades de la Integral de Convolucin 78


Evaluacin de la Integral de Convolucin 79
Respuesta al Escaln 83

Propiedades de los Sistemas LIT 84


2.5.1
2.5.2
2.5.3
2.5.4

2.6

Propiedades de la Suma de Convolucin 73


Respuesta al Escaln 77

Sistemas de Tiempo Continuo: la Integral de Convolucin 77


2.4.1
2.4.2
2.4.3

2.5

La Representacin de Seales de Tiempo Discreto Mediante Impulsos Unitarios 62

Sistemas LIT Con y Sin Memoria 84


Causalidad 85
Estabilidad 87
Invertibilidad 89

Funciones Propias de Sistemas LIT de Tiempo Continuo 90

iii

2.7

Funciones Propias de Sistemas LIT de Tiempo Discreto 91

2.8

Sistemas Descritos por Ecuaciones Diferenciales 92


2.8.1
2.8.2
2.8.3
2.8.4
2.8.5

2.9

Ecuaciones Diferenciales Lineales con Coeficientes Constantes 93


Linealidad 94
Causalidad 95
Invariabilidad en el Tiempo 95
Respuesta al Impulso 96

Sistemas Descritos por Ecuaciones en Diferencias 101


2.9.1
2.9.2
2.9.3

Solucin Homognea de la Ecuacin en Diferencias 104


La Solucin Particular 106
Determinacin de la Respuesta al Impulso 109

2.10 Simulacin de Sistemas 112


2.10.1
2.10.2
2.10.3

Componentes Bsicas: Sistemas de Tiempo Continuo 112


Diagramas de Simulacin: Sistemas de Tiempo Continuo 114
Componentes Bsicas: Sistemas de Tiempo Discreto 116

2.11 Representacin Mediante Variables de Estado: Tiempo Continuo 120


2.11.1
2.11.2
2.11.3
2.11.4
2.11.5
2.11.6

Definiciones 121
Solucin General de la Ecuacin de Estado 122
Solucin de la Ecuacin de Estado Mediante Integracin 125
Mtodo de los Valores y Vectores Caractersticos 127
Solucin Mediante Diagonalizacin de Matrices 135
Solucin por Reduccin a la Forma Cannica de Jordan 138

Problemas 147

CAPTULO TRES

ANLISIS DE FOURIER (TIEMPO CONTINUO)


Introduccin 161
3.1

Respuesta de Sistemas LIT a Exponenciales Complejas 163

iv

3.2

Representacin de Seales Usando Series de Fourier 164


3.2.1 Seales Peridicas y Combinaciones Lineales de Exponenciales Complejas 164
3.2.2 Series de Fourier 167
3.2.3. Condiciones para la Convergencia de las Series de Fourier 175

3.3

Propiedades de las Series de Fourier 181


3.3.1
3.3.2
3.3.3
3.3.4
3.3.5
3.3.6

3.4

Efectos de la Simetra 181


Linealidad 182
Diferenciacin 183
Teorema de la Potencia de Parseval 184
Integracin en el Tiempo 185
Manipulacin de Seales 186

Transformadas de Fourier y Espectros Continuos 187


3.4.1 La Transformada de Fourier 187
3.4.2 Convergencia de las Transformadas de Fourier 191
3.4.3 Ejemplos de Transformadas de Fourier en Tiempo Continuo 193

3.5

La Transformada de Seales Peridicas 196


3.5.1 Los Coeficientes de la Serie de Fourier como Muestras de la Transformada 197
3.5.2 La Transformada de Fourier de Seales Peridicas 199

3.6

Propiedades Adicionales de la Transformada de Fourier 201


3.6.1
3.6.2
3.6.3
3.6.4
3.6.5

3.7

Retardo en el Tiempo y Cambio de Escala 202


Diferenciacin en el Dominio del Tiempo 104
Integracin en el Dominio del Tiempo 205
Dualidad 205
La Relacin de Parseval 207

La Propiedad de Convolucin 208


3.7.1 Las Funciones Escaln y Signo 211

3.8

Modulacin 213

3.9

Generacin de Otros Pares de Transformadas 215

3.10 Densidad Espectral de Potencia 217


Problemas 222

CAPTULO CUATRO

ANLISIS DE FOURIER (TIEMPO DISCRETO)


4.1

Introduccin 237

4.2

Seales Peridicas 237

4.3

Serie de Fourier Discreta 238


4.3.1 Secuencias Peridicas 238
4.3.2 Representacin en Serie de Fourier Discreta 239
4.3.3 Convergencia de la Serie de Fourier Discreta 243

4.4

PROPIEDADES DE LA SERIE DE FOURIER DISCRETA 243


4.4.1
4.4.2
4.4.3
4.4.4

Periodicidad de los Coeficientes de Fourier 243


Dualidad 243
Otras Propiedades 244
Secuencias Pares e Impares 244

4.5

Teorema de Parseval 247

4.6

La Transformada de Fourier Discreta 248


4.6.1
4.6.2
4.6.3
4.6.4

4.7

Transformacin de la Serie de Fourier Discreta en la Transformada de Fourier 248


Par de Transformadas de Fourier 250
Espectros de Fourier 252
Convergencia de X() 252

Propiedades de la Transformada de Fourier 253


4.7.1 Periodicidad 253
4.7.2 Linealidad 253

vi

4.7.3
4.7.4
4.7.5
4.7.6
4.7.7
4.7.8
4.7.9
4.7.10
4.7.11
4.7.12
4.7.13
4.7.14
4.7.15

4.8

Desplazamiento o Corrimiento en el Tiempo 253


Desplazamiento en Frecuencia 255
Conjugacin 256
Inversin en el Tiempo 256
Escalamiento en el Tiempo 256
Dualidad 257
Diferenciacin en Frecuencia 258
Diferencias 258
Acumulacin 260
Convolucin 260
Multiplicacin o Modulacin 262
Propiedades Adicionales 263
Relacin de Parseval 263

La Respuesta de Frecuencia de Sistemas LIT Discretos 264


4.8.1 Sistemas LIT Caracterizados por Ecuaciones de Diferencias 265
4.8.2 Naturaleza Peridica de la Respuesta de Frecuencia 266

4.9

Respuesta del Sistema a Muestras de Sinusoides de Tiempo Continuo 266


4.9.1 Respuestas del Sistema 266

4.10 La Transformada de Fourier Discreta 267


4.10.1
4.10.2
4.10.3
4.10.4

Definicin 268
Relacin entre la TFD y la Serie de Fourier de Tiempo Discreto 270
Relacin entre la TFD y la Transformada de Fourier 270
Propiedades de la TFD 271

Problemas 276

CAPTULO CINCO

LA TRANSFORMACIN DE LAPLACE
5.1

Introduccin 281

5.2

Definicin de la Transformada de Laplace 282

vii

5.3

Condiciones para la Existencia de la Transformada de Laplace 285


5.3.1 Funciones Seccionalmente Continuas 285
5.3.2 Regin de Convergencia de la Transformada 289

5.4

Teoremas de la Derivada y de la Integral 289


5.4.1
5.4.2
5.4.3
5.4.4
5.4.5

5.5

La Transformada de Laplace Bilateral 291


La Funcin Impulso 291
El Teorema de la Derivada 291
El Teorema de la Integral 294
Traslacin Compleja 295

El Problema de Inversin 297


5.5.1 Inversin de Transformadas Racionales (Fracciones Parciales) 298
5.5.2 Inversin de Funciones Impropias 303

5.6

Los Valores Inicial y Final de f(t) a partir de F(s) 304


5.6.1 El Teorema del Valor Inicial 304
5.6.2 El Teorema del Valor Final 306

5.7

Teoremas Adicionales 307


5.7.1
5.7.2
5.7.3
5.7.4

El Teorema de Traslacin Real o de Desplazamiento 307


El Teorema de Escala 309
Derivadas de Transformadas 310
La Transformada de una Funcin Peridica 311

5.8

Aplicacin de la Transformada de Laplace a Ecuaciones Diferenciales Ordinarias 313

5.9

La Convolucin 317

5.10 Propiedades de la Integral de Convolucin 321


5.11 Ecuaciones Diferenciales e Integrales 322
5.12 Polos y Ceros de la Transformada 327
Problemas 329

viii

CAPTULO SEIS

LA TRANSFORMADA Z
6.1

Introduccin 333

6.2

La Transformada Z 333
6.2.1.
6.2.2.
6.2.3.

6.3

Transformadas Z de Secuencias Importantes 340


6.3.1.
6.3.2.
6.3.3.
6.3.4.

6.4

6.5

Secuencia Impulso unitario [n] 340


Secuencia Escaln Unitario u[n] 340
Funciones Sinusoidales 340
Tabla de Transformadas Z 341

Propiedades de la Transformada Z 341


6.4.1
6.4.2
6.4.3

Linealidad 341
Desplazamiento (Corrimiento) en el Tiempo o Traslacin Real 344
Inversin en el Tiempo 344

6.4.4

Multiplicacin por z 0n o Corrimiento en Frecuencia 345

6.4.5
6.4.6
6.4.7

Multiplicacin por n (o Diferenciacin en el Dominio de z) 346


Acumulacin 347
Convolucin 348

La Transformada Z Inversa 349


6.5.1.
6.5.2.
6.5.3.
6.5.4.

6.6

Definicin 334
La Regin de Convergencia de la Transformada Z 335
Propiedades de la Regin de Convergencia 338

Frmula de Inversin 349


Uso de Tablas de Pares de Trasformadas Z 349
Expansin en Series de Potencias 349
Expansin en Fracciones Parciales 351

La Funcin del Sistema: Sistemas LIT de Tiempo Discreto 356


6.6.1.
6.6.2.

La Funcin del Sistema 356


Caracterizacin de Sistemas LIT de Tiempo Discreto 359
Causalidad 359

ix

6.6.3.
6.6.4.

6.7

La Transformada Z Unilateral 366


6.7.1.
6.7.2.
6.7.3.
6.7.4.

6.8

Estabilidad 360
Sistemas Causales y Estables 361
Funcin del Sistema para Sistemas LIT Descritos por Ecuaciones de Diferencias Lineales
con Coeficientes Constantes. 361
Interconexin de Sistemas 364

Definicin 366
Propiedades Bsicas 367
La Funcin del Sistema 367
Valores Inicial y Final 367
Teorema del Valor Inicial 367
Teorema del Valor Final 367

La Transformada de Laplace y la Transformada Z 370


Pares Ordinarios de Transformadas Z 372
Problemas 373

CAPTULO SIETE

MODULACIN DE AMPLITUD
7.1

Introduccin 379
7.1.1 Necesidad de la Modulacin 180

7.2

Tipos de Modulacin Analgica 381

7.3

Transmisin de Seales de Banda Base Analgicas 381


7.3.1
7.3.2
7.3.3
7.3.4

7.4

Distorsin de la Seal en la Transmisin en la Banda Base 382


Distorsin Lineal 383
Compensacin 384
Distorsin No Lineal y Compansin 385

Esquemas de Modulacin Lineales OC 386


7.4.1 Modulacin de Banda Lateral Doble (DSB) 387

7.4.2
7.4.3
7.4.4
7.4.5
7.4.6
7.4.7

Modulacin de Amplitud Ordinaria 392


ndice de Modulacin 393
Potencia y Ancho de Banda de la Seal Transmitida 393
Demodulacin de Seales AM 396
Modulacin de Banda Lateral nica (SSB) 400
Modulacin de Banda Lateral Residal (VSB) 409

7.5

Conversin de Frecuencias (Mezclado) 411

7.6

Multicanalizacin por Divisin de Frecuencias 413


Problemas
Referencias 425

CAPTULO UNO

SEALES Y SISTEMAS
1.1 Introduccin

Los conceptos de seales y sistemas surgen en una gran variedad de campos y las ideas y tcnicas
asociadas con estos conceptos juegan un papel importante en reas tan diversas de la ciencia y la
tecnologa como las comunicaciones, la aeronutica, sistemas de generacin y distribucin de energa,
diseo de circuitos, acstica, etc. En este captulo introducimos la idea bsica sobre la descripcin y
representacin matemtica de seales y sistemas y sus clasificaciones. Tambin se definen varias
seales bsicas importantes, especialmente sobre sistemas lineales, las cuales son esenciales para
nuestros estudios posteriores.
El anlisis de un sistema lineal se facilita frecuentemente utilizando un tipo especfico de seales de
excitacin o una determinada representacin de seales. Por esta razn, es conveniente incluir el
anlisis de seales y sus propiedades en un estudio de sistemas lineales. Adems del anlisis nos
interesa tambin la sntesis de sistemas. De hecho, la sntesis o diseo de sistemas constituye la parte
creativa de la ingeniera. De aqu que para abordar el diseo de sistemas primero se debe aprender a
analizarlos. Este texto est orientado principalmente al anlisis de ciertos tipos de sistemas lineales; sin
embargo, debido a que los tpicos de diseo y anlisis estn ntimamente relacionados, este estudio
proporciona las bases para un diseo elemental.
El anlisis de sistemas puede dividirse en tres aspectos:
1.

El desarrollo de un modelo matemtico apropiado para el problema fsico bajo consideracin. Esta
parte del anlisis se dedica a la obtencin de ecuaciones dinmicas, condiciones iniciales o de
frontera, valores de parmetros, etc. En este proceso es donde el juicio, la experiencia y laa
experimentacin se combinan para lograr el desarrollo de un modelo apropiado. En esta forma,
este primer aspecto es el ms difcil de desarrollar formalmente.

2.

Despus de obtener un modelo apropiado, se resuelven las ecuaciones resultantes para encontrar
soluciones de diversas formas.

3.

Luego, la solucin del modelo matemtico se relaciona o interpreta en funcin del problema fsico.
Es conveniente que el desarrollo del modelo sea lo ms exacto posible de manera que se puedan
hacer interpretaciones y predicciones significativas concernientes al sistema fsico. No obstante, se
debe sealar que mientras ms exacto sea un modelo, mayor es la dificultad para obtener una
solucin matemtica y una realizacin fsica.

1.2 Seales y Clasificacin de Seales

Los trminos seales y sistemas, en la forma en que se usan generalmente, tienen diferentes
significados. En consecuencia, cualquier intento para dar una definicin general precisa, o una
definicin en el contexto de la ingeniera no sera muy productivo. Normalmente el significado de estos
trminos se extrae del contenido del texto. Una seal es una funcin de una variedad de parmetros,
uno de los cuales es usualmente el tiempo, que representa una cantidad o variable fsica, y tpicamente
contiene informacin o datos sobre la conducta o naturaleza de un fenmeno. Las seales pueden
describir una variedad muy amplia de fenmenos fsicos. Aunque las seales pueden representarse en
muchas formas, en todos los casos, la informacin en una seal est contenida en un patrn que vara
en alguna manera. Por ejemplo, el mecanismo vocal humano produce sonidos creando fluctuaciones en
la presin acstica. Diferentes sonidos, usando un micrfono para convertir la presin acstica en una
seal elctrica, corresponden a diferentes patrones en las variaciones de la presin acstica; el sistema
vocal humano produce sonidos inteligibles, generando secuencias particulares de estos patrones. Otros
ejemplos son una imagen monocromtica; en este caso es importante el patrn de variaciones en el
brillo y los diferentes matices existentes entre los colores blanco y negro.
Matemticamente, una seal se puede representar como una funcin de una o ms variables
independientes. Por ejemplo, una seal de audio puede representarse mediante la presin acstica en
funcin del tiempo, y una imagen como una funcin del brillo de dos variables espaciales. En estas
notas slo consideraremos seales que involucran una sola variable independiente. Una seal se
denotar por x(t). Por conveniencia, generalmente nos referiremos a la variable independiente como el
tiempo, aun cuando ella no represente al tiempo en operaciones especficas. Por ejemplo, las seales
que representan variaciones de cantidades fsicas con la profundidad, tales como la densidad, porosidad
y resistividad elctrica, se usan en geofsica para estudiar la estructura de la tierra. Tambin, el
conocimiento de las variaciones en la presin del aire, la temperatura y la velocidad del viento con la
altitud son de extrema importancia en investigaciones meteorolgicas.
Has dos tipos bsicos de seales: seales en tiempo continuo o seales analgicas y seales en
tiempo discreto o digitales. Una seal x(t) es una seal en tiempo continuo si la variable independiente t
es una variable continua y, por ende, estas seales estn definidas para un continuo de valores de esa
variable; es decir, el valor de x(t) es especificado en todo instante t de un intervalo de tiempo dado, ya
sea mediante una expresin matemtica o grficamente por medio de una curva; en otras palabras, la
variable independiente puede tomar cualquier valor real. Si la variable independiente t es una variable
discreta, es decir, x(t) est definida en puntos del tiempo discretos, entonces x(t) es una seal en tiempo
discreto, a menudo generada por muestreo de una seal de tiempo continuo. Como una seal de tiempo
discreto est definida solamente en tiempos discretos, con frecuencia se identifica como una secuencia
de nmeros, denotada por {xn} o x[n], donde, para nuestros propsitos, n es un entero. En la Fig. 1.1 se
ilustran una seal de tiempo continuo y una de tiempo discreto. La msica proveniente de un disco
compacto es una seal analgica, pero la informacin almacenada en el disco compacto est en forma
digital. sta debe procesarse y convertirse en forma analgica antes de que pueda escucharse.
Una seal de tiempo discreto x[n] puede representar un fenmeno para el cual la variable
independiente es inherentemente discreta. Por ejemplo, el promedio diario de los valores de cierre de la
bolsa de valores es, por su naturaleza, una seal que evoluciona en puntos discretos en el tiempo (es
decir, el cierre del da). Una seal de tiempo discreto, x[n], tambin puede obtenerse mediante el
muestreo de una seal de tiempo continuo x(t) para obtener los valores

x(t
)

x[n]
3

3 2 1 0

(a)

1 2 3

(b)

Figura 1.1. Seales de tiempo continuo y de tiempo discreto.

x ( t0 ), x ( t1 ),, x ( tn ),

o en una forma abreviada como


x [0], x [1],, x [ n ],

o
x0 , x1 ,, xn ,

y a los valores xn se les denomina muestras; el intervalo de tiempo entre muestras se llama el intervalo
de muestreo. Cuando estos intervalos son iguales (muestreo uniforme), entonces
xn x [ n ] x [ nTs ]

donde la constante Ts es el intervalo de muestreo. Un dispositivo que convierta informacin analgica a


forma digital mediante cuantizacin (redondeo) se denomina un convertidor analgico-digital.
Una seal de tiempo discreto con muestreo uniforme puede ser especificada de dos maneras:
1. Podemos especificar una regla para calcular el n-simo valor de la secuencia. Por ejemplo,

1
x [ n ] xn 2

n0
n0

{ xn } , 0, 0,1, 12 , 14 ,,

1
2

2. Podemos dar una lista explcita de los valores de la secuencia. Por ejemplo, la secuencia mostrada
en la Fig. 1.1b puede escribirse como
{ xn } {, 0, 0, 2,3,3, 2,1, 0, 0,}

{ xn } {2,3,3, 2,1}

Se usa la flecha para indicar el trmino correspondiente a n = 0. Se usar la convencin de que si


no aparece la flecha, entonces el primer trmino corresponde a n = 0 y todos los valores son
iguales a cero para n < 0.

Ejemplo 1. Dada la seal en tiempo continuo especificada por

1 t
x (t )

1 t 1
t 1

Determine la secuencia de tiempo discreto resultante obtenida mediante muestreo uniforme de x(t) con
un intervalo de muestreo de (a) 0.25 s; (b) 0.5 s.

Solucin: Es ms fcil resolver este problema grficamente. La seal x(t) se grafica en la Fig. 1.2a. Las
Figs. 1.2b y c muestran grficos de las secuencias de las muestras resultantes obtenidas para los
intervalos de muestreo especificados.

x(t)

x[n) = x(n/4)

3 2 1 0 1 2 3 4

(b)

(a)
x[n) = x(n/2)

(c)

Figura 1.2. Las seales para el Ejemplo 1.

(a) Ts = 0.25 s. De la Fig. 1.2b obtenemos


x [n] {, 0, 0.25, 0.5, 0.75,1, 0.75, 0.5, 0.25, 0,}

(b) Ts = 0.5 s. De la Fig. 1.2c, obtenemos


x [n] {, 0, 0.5,1, 0.5, 0,}

Con frecuencia, se procesan seales para producir nuevas seales para diferentes propsitos. A
continuacin se da un ejemplo de cmo se generan nuevas seales a partir de seales conocidas.

Ejemplo 2. Usando las seales de tiempo discreto x1[n] y x2[n] mostradas en la Fig. 1.3, represente
cada una de las siguientes seales mediante una grfica y mediante una secuencia de nmeros.
(a) y1 [ n ] x1 [ n ] x2 [ n ] ;

(b) y2 [ n ] 2 x1 [ n ] ;

(c) y3 [ n ] x1 [ n ] x2 [ n ] .

x1[n]

x2[n]

3
2

2
2 1

1 0

3
0

Figura 1.3. Seales para el Ejemplo 2

Solucin:
(a) y1[n] se dibuja en la Fig. 1.4a. A partir ella obtenemos
y1 [ n ] {, 0, 2, 2,3, 4,3, 2, 0, 2, 2, 0, }

(b) y2[n] se dibuja en la Fig. 1.4b. De ella obtenemos

y2 [ n ] {, 0, 2, 4, 6, 0, 0, 4, 4, 0,}

(c) y3[n] se dibuja en la Fig. 1.4c. De ella obtenemos


y3 [ n ] {, 0, 2, 4, 0,}

y3[n]

y2[n]
y1[n]

3 2 1

3
0

(a)

1 0

2 3

(b)

Figura 1.4

7
7

1 0

(c)

2 3

1.3 Seales Peridicas y No-Peridicas

Una seal peridica de tiempo continua x(t) tiene la propiedad de que existe un nmero positivo T para
el cual
x t x t T para todo t

(1.1)

x(t)

...

...
T

Figura 1.5

En este caso decimos que la seal x(t) es peridica con perodo T. En la Fig. 1.5 se ilustra un ejemplo
de esta clase de seales. Observe que una seal peridica repite un mismo patrn durante un tiempo
mltiplo de T y contina hacindolo por tiempo infinito.
De la figura se deduce que si x(t) es peridica con perodo T, entonces
x ( t ) x ( t mT )

(1.2)

para todo T y cualquier entero m. Por ello, x(t) tambin es peridica con perodo 2T, 3T, . El perodo
fundamental T0 es el mnimo valor de T para el cual se cumple la Ec. (1.1). Observe que esta definicin
de T0 funciona excepto cuando x(t) es una constante. En este caso, el perodo fundamental no est
definido puesto que x(t) es peridica para cualquier seleccin de T (es decir, no hay un valor positivo
mnimo). La Ec. (1.2) dice simplemente que si la seal se desplaza un nmero entero de perodos hacia
la derecha o hacia la izquierda no cambia la forma de la onda. La frecuencia fundamental (cclica) f0 es
el recproco del perodo fundamental, f0 = 1/T0, y se mide en hertz (ciclos por segundo). La frecuencia
fundamental en radianes por segundo es 0 = 2f0 = 2/T0. Finalmente, a una seal que no exhiba
periodicidad se le referir como una seal no peridica o aperidica.
Ejemplos conocidos de seales peridicas son las seales sinusoidales; como ejemplo est la seal
x ( t ) A sen ( 0 t )

donde
A = amplitud.
0 = frecuencia angular (rad/s).
= ngulo de fase inicial con respecto al origen del tiempo (rad).
Observe que
sen[ 0 ( t T ) ] sen ( 0 t 0 T ) sen ( 0 t )

si
0 T m 2 o T m

2
0

, m un entero positivo

As que el perodo fundamental T0 de x(t) est dado por

T0

2
0

Ejemplo 3. Sean x1(t) y x2(t) dos seales peridicas con perodos fundamentales T1 y T2,
respectivamente. Cules son las condiciones para que la suma z(t) = x1(t) + x2(t) sea peridica y cul
es el perodo fundamental de z(t)?

Solucin: Puesto que x1(t) y x2(t) son peridicas con perodos fundamentales T1 y T2, respectivamente,
se tiene que
x1 ( t ) x1 ( t T1 ) x1 ( t mT1 ) , m un entero positivo
x2 ( t ) x2 ( t T2 ) x2 ( t nT2 ) , n un entero positivo

Entonces,
z ( t ) x1 ( t mT1 ) x2 ( t nT2 )

Para que z(t) sea peridica con perodo T, se necesita que

z ( t ) z t T x1 ( t T ) x2 ( t T ) x1 ( t mT1 ) x2 ( t nT2 )
y entonces se debe cumplir que
mT1 nT2 T

(1.3)

T1
T2

n
m

nmero racional

(1.4)

En otras palabras, la suma de dos seales peridicas es peridica solamente si la relacin entre sus
periodos respectivos es un nmero racional. El perodo fundamental es entonces el mnimo comn
mltiplo de T1 y T2, y est dado por la Ec. (1.3) si los enteros m y n son primos relativos. Si la relacin
T1/T2 es un nmero irracional, entonces las seales x1(t) y x2(t) no tienen un perodo comn y z(t) no
puede ser peridica.

Las seales peridicas de tiempo discreto se definen en forma similar. Especficamente, una seal de
tiempo discreto x[n] es peridica con perodo N, si existe un entero positivo N para el cual

x n x n N para toda n

(1.5)

En la Fig. 1.6 se ilustra un ejemplo de este tipo de seal.

x[n]

...

...
n

Figura 1.6. Una seal de tiempo discreto peridica.

El perodo fundamental N0 de x[n] es el menor entero positivo N para el cual se cumple la Ec. (1.5).
cualquier secuencia (seal de tiempo discreto) que no sea peridica se conoce como una secuencia noperidica (o aperidica).
1.4 Seales de Potencia y de Energa

En muchas aplicaciones, no en todas, las seales que consideraremos estn directamente relacionadas
con cantidades fsicas que representan potencia y energa. Por ejemplo, si v(t) e i(t) son,
respectivamente, el voltaje y la corriente en un resistor de resistencia R, entonces la potencia
instantnea p(t) viene dada por

p (t ) v (t ) i (t )

1
R

v2 ( t ) R i 2 ( t )

(1.6)

La energa total disipada en el intervalo de tiempo t1 t t2 est dada por


t2

t2

p ( t ) dt
t1

t2

v ( t ) dt R i 2 ( t ) dt
2

(1.7)

t1

t1

y la potencia promedio en ese intervalo es


1
t2 t1

t2

p ( t ) dt
t1

t1

t2 t1

t2

1
R

v ( t ) dt
2

1
t2 t1

t1

Ri

( t ) dt

(1.8)

t2

En una forma similar, la potencia disipada por friccin es p ( t ) b v 2 ( t ) , donde v(t) es la velocidad, y
podemos definir la energa y la potencia promedio en un intervalo de tiempo dado en la misma forma
que en las Ecs. (1.7) y (1.8).
Se acostumbra usar una terminologa parecida para cualquier seal, ya sea de tiempo continuo x(t) o
de tiempo discreto x[n], normalizando la energa y la potencia promedio de una seal arbitraria (en el
caso de seales elctricas, esto se hace tomando un valor de R = 1 ). Adicionalmente, con frecuencia

ser conveniente considerar seales de valores complejos. En este caso, la energa total normalizada en
el intervalo t1 t t2 se define como
t2

x t

(1.9)

dt

t1

La potencia promedio normalizada se obtiene dividiendo la Ec. (1.9) por la longitud o duracin t2 t1
del intervalo. En la misma forma, la energa total normalizada para una seal de tiempo discreto x[n]
en el intervalo n1 n n2, se define como
n2

x[n]

(1.10)

n n1

y al dividir la Ec. (1.10) por el nmero de puntos en el intervalo, ( n 2 n1 1) , se obtiene la potencia


promedio en ese intervalo.
Adicionalmente, en muchos sistemas nos interesa examinar la potencia y la energa de seales en un
intervalo de tiempo infinito. En estos casos, definimos la energa total normalizada E como los
lmites de las Ecs. (1.9) y (1.10) conforme el intervalo de tiempo aumenta indefinidamente. Para
tiempo continuo, tenemos

E lm

x ( t ) dt
2

x ( t ) dt

(1.11)

y en tiempo discreto,

E lm

x[n]

n N

x[n]

(1.12)

De la misma forma se puede definir la potencia promedio normalizada en un intervalo infinito como
P lm

1
2T

(1.13)

x ( t ) dt

para tiempo continuo y

P lm

2 N 1
1

x[n]

(1.14)

n N

para tiempo discreto.


Con base en las definiciones dadas por las Ecs. (1.11) a (1.14), se pueden definir tres clases
importantes de seales:
1.

Se dice que x[t] o x[n] es una seal de energa si y slo si 0 < E < (energa finita). Una seal de
este tipo debe tener una potencia promedio igual a cero, ya que, en el caso de tiempo continuo, por
ejemplo, de la Ec. (1.13) vemos que

P lm

E
2T

10

2.

Se dice que una seal x(t) o x[n] es una seal de potencia si y slo si 0 < P < (potencia promedio
finita). Entonces, si P > 0, por necesidad E . Esto tiene sentido, ya que si se tiene una
energa promedio por unidad de tiempo diferente de cero (es decir, potencia promedio diferente de
cero), entonces integrando o sumando en un intervalo de tiempo infinito produce una cantidad de
energa infinita.

3.

Las seales que no satisfacen ninguna de las dos propiedades anteriores se conocen, por supuesto,
como seales que no son ni de energa ni de potencia.

Se deben sealar las propiedades que contemplan una energa nula. Es claro que si x(t) = 0, la energa
E es cero, pero lo contrario no es estrictamente cierto. Slo es posible decir que si E = 0, entonces
x(t) es igual a cero casi en todas partes. Desde un punto de vista puramente matemtico, la propiedad
E = 0 no define una sola seal sino una clase de seales equivalentes. En estas notas no consideramos
este punto de vista, y todos los elementos de esta clase de seales equivalentes son considerados como
una sola seal. Por lo tanto, una seal de energa nula es tambin considerada como una seal igual a
cero.
Ejemplo 4. Si x(t) es una seal peridica con perodo fundamental T0, entonces la integral en la Ec.
(1.13) tiene el mismo valor para cualquier intervalo de longitud T0. Tomando el lmite en una forma tal
que 2T sea un mltiplo entero del perodo, es decir, 2T = mT0, entonces la energa total en un intervalo
de longitud 2T es m veces la energa en un perodo. Como consecuencia, la potencia promedio es
T0
1
P lm
m x (t )
m mT
0 0

1
dt
T0

T0

x (t )

dt

Observe que una seal peridica es de potencia si su contenido de energa por perodo es finito.

EJEMPLO 5. Considere las seales en la Fig. 1.7. Se quiere clasificar cada seal calculando la energa
y la potencia en cada caso.

x1(t)
A

x2(t)

A exp( t )
A

T0
t1

Figura 1.7. Seales de energa y de potencia.

Solucin: La seal en la Fig. 1.7a es aperidica y su energa total es

11

A2

E A exp( 2 t ) dt
2

la cual es finita. La potencia promedio es

1
P lm
T 2 T

2T

A2
A2 exp( 2 t ) dt lm
0
T 2 T

En consecuencia, la seal en la Fig. 1.7a es una seal de energa con una energa igual a A2/2 y
potencia promedio cero.
La seal en la Fig. 1.7b es peridica con perodo T0. Su potencia promedio es
P

1
T0

T0

x2 ( t )

t1

2 A2
1 2
dt A dt A2 dt

T0 0
T0
t1

As que x2(t) es una seal de potencia con energa infinita y potencia promedio igual a 2 A2 T0 .

Ejemplo 6. Considere las dos seales aperidicas mostradas en la Fig. 1.8. Estas dos seales son
ejemplos de seales de energa.

x1(t)

x2(t)

/
2

/2

A exp a t

(a)

(b)

Figura 1.8. Ejemplos de seales de energa.

La funcin pulso rectangular rect(t/) mostrada en la Fig. 1.8a est estrictamente limitada en el
tiempo, ya que x1(t) es igual a cero para t fuera de la duracin del pulso. La otra seal est
asintticamente limitada en el sentido de que x(t) 0 conforme t . En cualquiera de los casos, la
potencia promedio es igual a cero. La energa para la seal x1(t) es
2

E1 lm

y para x2(t) es

2
1

( t ) dt

A2 dt A2

12

E1 lm

exp( 2 a t ) dt lm

A2
a

[1 exp( 2 aT ) ]

A2
a

Puesto que E1 y E2 son finitas, las seales x1(t) y x2(t) son seales de energa.

Aqu se debe sealar que la energa como la define la Ec. (1.11) o la Ec. (1.12) no indica la energa
real de la seal a que la energa de la seal depende no slo de la seal sino tambin de la carga. La
interpretamos como la energa normalizada disipada en un resistor de 1 ohmio si a ste se le aplicase
un voltaje x(t) o si por el pasase una corriente x(t). Observaciones similares aplican a la potencia de la
seal definida en la Ec. (1.13) o en la Ec. (1.14). Por lo planteado, las ecuaciones para la energa o la
potencia no tienen las dimensiones correctas. Las unidades dependen de la naturaleza de la seal. Por
ejemplo, si x(t) es una seal de voltaje, entonces su energa E tiene unidades de V2s (voltios al
cuadrado-segundos) y su potencia P tiene unidades de V2 (voltios al cuadrado)

1.5

Transformaciones de la Variable Independiente

En muchas ocasiones es importante considerar analtica y grficamente seales relacionadas por una
modificacin de la variable independiente, mediante operaciones tales como desplazamiento o
corrimiento e inversin. Por ejemplo, como se ilustra en la Fig. 1.9, la seal x[n] se obtiene a partir de
la seal x[n] por una reflexin o inversin en n = 0 (es decir, una inversin de la seal).
x[n]

x[n]

n
(a)

n
(b)

Figura 1.9. Inversin en tiempo discreto.

De igual forma, como se muestra en la Fig. 1.10, x(t) se obtiene a partir de la seal x(t) por reflexin
en t = 0. Entonces, si x(t) representa una seal de audio en un grabador de cinta, la seal x(t) es la
misma grabacin reproducida en reversa.
Esta operacin se conoce como reflexin y es equivalente a doblar la seal (rotacin de 180) en
torno a la lnea t 0 o simplemente a intercambiar el pasado y el futuro de la seal de tiempo.
Observe que cualquier cosa que suceda en la Fig. 1.10(a) en el instante t tambin ocurre en la Fig.
1.10(b) en el instante t. Como esta operacin significa intercambiar el pasado y el futuro, es obvio
que ningn sistema fsico puede ejecutarla.

13

x(t)

x(t)

t0

t0

(a)

(b)

Figura 1.10. Inversin en tiempo continuo.

Otra operacin es la de desplazamiento. La seal x(t t0 ) representa una versin desplazada de x(t),
Fig. 1.11. El desplazamiento en el tiempo es t0, donde t0 es un nmero real. Si t0 > 0, entonces la seal
es retrasada en t 0 unidades de tiempo. Fsicamente, t0 no puede tomar valores negativos, pero desde un
punto de vista analtico, x(t t0), t0 < 0, representa una rplica adelantada de la seal x(t). Las seales
que estn relacionadas en esta forma (t0 > 0) surgen en aplicaciones tales como el radar, sonar, sistemas
de comunicacin y procesamiento de seales ssmicas. Un sistema cuya seal de salida es idntica a la
de su entrada pero retrasada por una constante se denomina una unidad de retardo. Por otra parte, si la
seal de salida es idntica a la de entrada pero avanzada por una constante, el sistema se denomina un
predictor. Sin embargo, un sistema que prediga (adivine) es fsicamente imposible de construir.
x(t t0)

x(t)

t1

t1

t0 t1

(a)

t0

t0 + t1

(b)

Figura 1.11. Desplazamiento de una seal de tiempo continuo.

Ejemplo 7. Considere la seal x(t) mostrada en la Fig. 1.12. Se desea graficar x(t 2) y x(t + 3).
x(t)
1

1 0 1

2 3

Figura 1.12

Solucin: Es fcil verificar que

14

1 t 0
t 1
1
0t 2

x (t )
t 3 2 t 3
0
otros valores de t
Para realizar la operacin de desplazamiento, se reemplaza t por t 2 en la expresin para x(t):

1 t 2 0
( t 2) 1

1
0t2 2

x ( t 2)
( t 2) 3 2 t 2 3

0
otros valores de t
o, equivalentemente,

t 1

x ( t 2)
t 3

1 t 2
2t 4
4t 5
otros valores de t

x(t+3)

x(t2)
1

0 1 2 3 4 5

4 3 2 1 0

(b)

(a)
Figura 1.13

La seal x(t) se grafica en la Fig. 1.13a y puede describirse como la funcin x(t) desplazada dos
unidades hacia la derecha. En la misma forma se puede demostrar que

4 t 3
t4
1
3 t 1

x ( t 3)
1 t 0
t
0 otros valores de t
Esta ltima seal se grafica en la Fig. 1.13b y representa una versin de x(t) desplazada tres unidades
hacia la izquierda.

15

Ejemplo 8. Se desea dibujar x(t) y x(3 t) si x(t) es como se muestra en la Fig. 1.14.
x(t)
1

Figura 1.14

Solucin: La seal x(t) se puede escribir como

t 1

x (t ) 1
0

1 t 0
0t 2
otros valores de t

Reemplazando ahora t por t, se obtiene

t 1

x ( t ) 1
0

1 t 0

t 1 0 t 1

0 t 2
1
2 t 0
otros valores de t
0 otros valores de t

La seal x(t) se muestra en la Fig. 1.15a.


x(t)

x(3t)

(a)

3
(b)

Figura 1.15

En la misma forma se puede demostrar que

4t

x (3 t ) 1
0

3t 4
1 t 3
otros valores de t

y x(3 t) es como se muestra en la Fig. 1.15b.


La figura es primero reflejada y luego trasladada. Este resultado se obtiene escribiendo la operacin
completa como

16

x (3 t ) x t 3

Observe que si primero desplazamos la seal y luego reflejamos la seal desplazada, se obtiene como
resultado la seal x(t 3) (Fig. 1.16).
De lo anterior se deduce que las operaciones de inversin y desplazamiento no son conmutativas. No
obstante, una seal puede ser invertida y retardada simultneamente. Las operaciones son equivalentes
a reemplazar t o n por t + t0 o n n0 . Para ver esto, consideramos una seal de tiempo continuo x(t)
que se desea invertir y trasladar por t0 unidades de tiempo. Para producir la seal invertida
reemplazamos t por t en x(t), lo que resulta en x(t). La seal invertida x(t) es entonces retrasada por
t0 unidades para obtener x[ ( t t0 )] x ( t t0 ) , como se afirm.
x(t3)
1

5 4 3 2 1

Figura 1.16

1.6 Escalamiento en el Tiempo


La operacin de compresin o expansin en el tiempo se conoce como escalamiento en el tiempo.
Considere, por ejemplo, las seales x(t), x(3t) y x(t/2), mostradas en la Fig. 1.17. Como se puede ver,
x(3t) puede describirse como x(t) comprimida por un factor de 3. En forma similar, x(t/2) puede
describirse como expandida por un factor de 2. Se dice que ambas funciones, x(3t) y x(t/2), son
versiones de x(t) escaladas en el tiempo.
En general, si la variable independiente es escalada por un parmetro , entonces x(t) es una versin
comprimida de x(t) si 1 y es una versin expandida de x(t) si 1 . Si consideramos a x(t) como
si fuese la seal de salida de un grabador de video, por ejemplo, entonces x(3t) se obtiene cuando la
grabacin se reproduce a tres veces la velocidad con la cual fue grabada, y x(t/2) se obtiene cuando la
grabacin se reproduce a la mitad de esa velocidad. Tambin se puede decir, por ejemplo, que lo que le
pase a x(t) en el instante t, tambin le suceder a x(t/2) en el instante t/2.

x(3t)
A

x(t)

1 t

1/3

1/3

x(t/2)

Figura 1.17. Ejemplos de escalamiento en el tiempo.

17

Ejemplo 9. Se desea graficar la seal x(3t 6), donde x(t) es la seal del Ejemplo 7. Usando la
definicin de x(t) dada en el Ejemplo 7, obtenemos

t 2
3t 5
3

2t 8 3
1
x (3t 6)
3t 9 8 t 3

otros valores de t
0
La seal x(3t 6) se grafica en la Fig. 1.18 y puede considerarse como x(t) comprimida por un factor
de 3 (o escalada en el tiempo por un factor de 1/3) y luego desplazada dos unidades hacia la derecha;
observe que si x(t) es desplazada primero y luego escalada por una factor de 1/3, hubisemos obtenido
una seal diferente; en consecuencia, las operaciones de desplazamiento y de escalamiento en el tiempo
no son conmutativas. El resultado obtenido se puede justificar escribiendo la operacin en la forma
siguiente:
x (3t 6) x (3( t 2))
la cual indica que se ejecuta primero la operacin de escalamiento y despus la de desplazamiento.
x(t)
1

5/3 2

8/3 3

Figura 1.18

Ejemplo 10. El tiempo que le toma a una seal para alcanzar 90% de su valor final, T90, es una
caracterstica muy importante. Determine T90 para las seales siguientes: (a) x(t); (b) x(2t); x(t/2),
donde x ( t ) 1 e t .
Solucin
(a) El valor final de x(t) es igual a 1. Para hallar el tiempo requerido por la funcin para alcanzar el
valor de 0.90, tenemos que resolver la ecuacin

0.90 1 e T90
la cual produce T90 = 2.3.
(b) Para la seal x(2t) tenemos que resolver
0.90 1 e2 T90

18

la cual produce T90 = 1.15.


(c) La seal x(t/2) tiene un T90 dado por
0.90 1 e T90

la cual resulta en T90 = 4.6.


Estos resultados eran de esperarse. En la parte (b) comprimimos la seal por un factor de 2, y en la
parte (c) la expandimos por el mismo factor.

En conclusin, para cualquier seal general x(t), la transformacin (mltiple) de la variable


independiente en la forma t puede realizarse de la manera siguiente:

x ( t ) x ( ( t )
donde se supone que y son nmeros reales. Las operaciones deben ejecutarse en el orden siguiente:
1. Escale por . Si es negativo, refleje tambin con respecto al eje real.
2. Desplace hacia la derecha por si y son de signos diferentes y hacia la derecha si tienen el
mismo signo.
El orden de las operaciones es importante. Observe que las operaciones de reflexin y escalamiento en
el tiempo son conmutativas, mientras que las de desplazamiento y reflexin o las de desplazamiento y
escalamiento, como ya se mencion, no lo son. Observe tambin que no definimos la operacin de
escalamiento en el tiempo para una seal de tiempo discreto (por qu?).
1.7 Seales Pares e Impares
Adicionalmente a su uso en la representacin de fenmenos fsicos (como en el ejemplo del
grabador), la reflexin es extremadamente til para examinar las propiedades de simetra que pueda
poseer una seal. Una seal x(t) o x[n] se conoce como una seal par si es idntica a su reflexin
respecto del origen, es decir, si
x ( t ) x (t )
x[ n] x[ n]

(1.15)

lo que equivale a decir que una seal par, x(t) o x[n], es invariante bajo la operacin de reflexin (o
inversin) en el tiempo..
Una seal se denomina impar si
x ( t ) x (t )
x[ n] x[ n]

(1.16)

Observe que una seal impar debe ser necesariamente igual a cero en el origen. En la Fig. 1.19 se
muestran ejemplos de una seal par y una impar.

19

x(t)

x[n]

Figura 1.19. Ejemplos de una funcin par y una impar.

Un hecho importante es que cualquier seal, que no sea par ni impar, puede ser expresada como una
suma de dos seales, una de las cuales es la parte par y la otra la parte impar. Para ver esto, considere la
seal
1
x p ( t ) [ x ( t ) x ( t )]
2
la cual se conoce como la parte par de x(t). En forma similar, la parte impar de x(t) est dada por
1
xi ( t ) [ x ( t ) x ( t )]
2

Es muy sencillo comprobar que, efectivamente, la parte par es par y que la parte impar es impar, y que
x(t) es la suma de las dos. Para el caso de tiempo discreto se cumplen definiciones completamente
anlogas. En resumen, tenemos las siguientes identidades:

x ( t ) x p ( t ) xi ( t )
x [ n ] x p [ n ] xi [ n ]

(1.17)

1
x p ( t ) [ x ( t ) x ( t )]
2
1
xp [ n ] [ x[ n ] x[ n ]
2

(1.18)

1
xi ( t ) [ x ( t ) x ( t )]
2
1
xi [ n ] { x [ n ] x [ n ]}
2

(1.19)

Observe que la suma de dos seales pares es par y de dos seales impares es impar, y tambin que el
producto de dos seales pares o dos impares es una seal par y que el producto de una seal par y una
seal impar es una seal impar; tambin se puede demostrar que la derivada de cualquier funcin par es
impar, y la derivada de una funcin par es impar (la demostracin de todo lo anterior se deja como un
ejercicio).

20

Ejemplo 11. Considere la seal x(t) definida por


1, t 0
x (t )
0, t 0

Las partes par e impar de esta seal, conocida como la funcin escaln, estn dadas por

xp (t )

para todo t, excepto en t = 0


2
1
2 , t 0
xi ( t )
1, t 0
2
El nico problema aqu radica en el valor de las funciones en x = 0. Si definimos x (0) 1 2 , entonces
1
x p (0)
y xi (0) 0
2
Las seales xp(t) y xi(t) se grafican en la Fig. 1.20.
xp(t)

xi(t)

1/2

1/2

0
1/2

Figura 1.20. Descomposicin de la funcin escaln en sus partes par e impar.

Ejemplo 12. Considere la seal


A exp ( t ), t 0
x (t )
0,
t0

La parte par de x(t) est dada por


1

2 A exp ( t )
xp (t ) 1

2 A exp ( t )

t 0
t0

1
2

A exp t

y la parte impar por


1

exp ( t ), t 0
xi ( t ) 2 1

2 exp ( t ). t 0

Las seales xp(t) y xi(t) se muestran en la Fig. 1.21.

21

xp(t
)

xi(t)
A

Figura 1.21

Ejemplo 13. Determine las componentes par e impar de x ( t ) e jt .


Solucin: La parte par es
y la parte impar es

x p ( t ) 12 e jt e jt cos t
xi ( t ) 12 e jt e jt j sen t

1.8 Seales en Tiempo Continuo Bsicas

En esta seccin se introducen varias seales de tiempo continuo de particular importancia. Estas
seales no slo ocurren frecuentemente en la naturaleza, sino que ellas tambin sirven como bloques
bsicos para la construccin de otras seales. En ste y en los captulos subsiguientes encontraremos
que al construir seales de esta forma se podrn examinar y comprender ms profundamente las
propiedades de seales y sistemas.

1.8.1

Seales Exponenciales Complejas

La seal exponencial compleja de tiempo continuo es de la forma

x ( t ) Aest

(1.20)

donde A y s son, en general, nmeros complejos. Dependiendo del valor de estos parmetros, la
exponencial compleja puede tomar varias caractersticas diferentes. En el anlisis a continuacin, para
simplificar, se tomar A = 1.
Si s se restringe a ser puramente imaginaria, s = j0 por ejemplo, se obtiene la seal
x ( t ) e j0 t cos 0 t j sen 0 t

(1.21)

Usando ahora la identidad de Euler, esta seal puede ser definida como
x ( t ) e j0 t cos 0 t j sen 0 t

(1.22)

22

O sea que x(t) es una seal compleja cuyas partes real e imaginaria son cos 0 t y sen 0 t ,
respectivamente. Una propiedad importante de la seal exponencial compleja es su periodicidad. Para
comprobar esto, recuerde de la Sec. 1.3 que una funcin x(t) ser peridica con perodo T si

x (t ) x (t T )
o, para la funcin exponencial
e j0 t e j0 ( t T )

(1.23)

Puesto que
e j0 ( t T ) e j0 t e j0T

se concluye que para tener periodicidad, se debe cumplir que

e j0T 1
Si 0 = 0, entonces x(t) = 1, la cual es peridica para cualquier valor de T. Si 0 0 , entonces el
perodo fundamental T0 de x(t) es
T0

(1.24)

As que las seales e j0 t y e j0 t tienen el mismo perodo fundamental. Observe tambin que x(t) es
peridica para cualquier valor de 0.
Una seal ntimamente relacionada con la seal exponencial compleja peridica es la sinusoidal
x ( t ) A cos ( 0 t )

(1.25)

ilustrada en la Fig. 1.22 y ya estudiada en la Seccin 1.3.

x(t
)

T0

A cos

Figura 1.22

Las seales sinusoidales y las exponenciales complejas tambin se usan para describir las
caractersticas de muchos procesos fsicos en particular, sistemas fsicos en los cuales se conserva la
energa. Por ejemplo, la respuesta natural de una red constituida solamente por inductores y capacitores
o el movimiento armnico simple de un sistema mecnico consistente de una masa conectada por un
resorte a un soporte estacionario. Las variaciones de la presin acstica correspondientes a un solo tono
musical tambin son sinusoidales.

23

Como ya se vio, si se usa la relacin de Euler, la exponencial compleja en la Ec. (1.21) puede
escribirse en trminos de seales sinusoidales con el mismo perodo fundamental, es decir,

e j0 t cos 0 t j sen 0 t

(1.26)

En forma similar, la seal sinusoidal en la Ec. (1.26) puede escribirse en funcin de exponenciales
complejas peridicas con el mismo perodo fundamental:
A

A cos( 0 t ) e j e j0 t e j e j0 t
2

(1.27)

Observe que las dos exponenciales en la Ec. (1.28) tienen amplitudes complejas. Alternativamente, una
sinusoide puede expresarse en funcin de una seal exponencial compleja como

A cos( 0 t ) A Re e j ( 0 t )

(1.28)

donde A es real y Re se lee la parte real de. Tambin se usar la notacin Im para denotar la
parte imaginaria de. Entonces

A sen ( 0 t ) A Im e j ( 0 t )

(1.29)

De la Ec. (1.25) vemos que el perodo fundamental T0 de una seal sinusoidal o de una seal
exponencial peridica (ambas funciones de tiempo continuo) es inversamente proporcional a 0 , a la
cual llamaremos la frecuencia fundamental (rad/s). De la Fig. 1.23 vemos grficamente lo que esto
significa. Si disminuimos la magnitud de 0, el ritmo de oscilacin se hace ms lento y, por tanto, el
perodo aumenta. Ocurren efectos exactamente opuestos si se aumenta la magnitud de 0. Considere
ahora el caso cuando 0 = 0. Como ya se mencion, aqu x(t) representa una constante y, por ello, es
peridica con perodo T para cualquier valor positivo de T, lo que significa que el perodo de una seal
constante no est definido. Por otra parte, no hay ambigedad al definir la frecuencia fundamental de
una constante como igual a cero; es decir, la tasa de oscilacin de una constante es igual a cero (perodo
infinito).

x1 (t ) cos 1t

x2 (t ) cos 2 t

(a)

(b)

x3 (t ) cos 3t

t
(c)

Figura 1.23

1 > 2 > 3
T1 < T2 < T3

24

Las seales peridicas y en particular, la seal exponencial compleja en la Ec. (1.21) y la seal
sinusoidal en la Ec. (1.26) proporcionan ejemplos importantes de seales con energa total infinita
pero potencia promedio finita. Por ejemplo, considere la exponencial peridica de la Ec. (1.21) y
suponga que calculamos la energa total y la potencia promedio en un perodo:
T0

Eperodo

T0
j0 t 2

(1.30)

Eperodo 1

(1.31)

dt (1) dt T0

Pperodo

1
T0

Puesto que hay un nmero infinito de perodos conforme t vara de a +, la energa total
integrada para todo el tiempo es infinita. Sin embargo, cada perodo de la seal es idntico a los dems.
Como la potencia promedio de la seal por perodo es igual a 1, promediando en periodos mltiples
producir un promedio igual a 1; es decir,
P lm

1
2T

e j0 t

dt 1

(1.32)

Ejemplo 14. Algunas veces es deseable expresar la suma de dos exponenciales complejas como el
producto de una sola exponencial compleja. Por ejemplo, suponga que se quiere graficar la magnitud
de la seal
x (t ) e j 2 t e j 3t

Para hacer esto, primero extraemos un factor comn del lado derecho de la ecuacin, tomando como
frecuencia de ese factor el promedio de las dos frecuencias de las exponenciales en la suma, y se
obtiene
x ( t ) e j 2.5 t ( e j 0.5 t e j 0.5 t )

la cual, por la relacin de Euler, se puede escribir como


x ( t ) 2 e j 2.5 t cos 0.5t

y de aqu se obtiene directamente la expresin para la magnitud de x(t):

x ( t ) 2 cos 0.5t
As que x ( t ) es lo que se conoce comnmente como una sinusoide rectificada de onda completa,
como se muestra en la Fig. 1.24.

25

x(t)
2

...

...
0 2

Figura 1.24

Para la seal compleja definida en la Ec. (1.20), si A es real y s = (tambin real), entonces la
expresin para la seal se reduce a
x ( t ) Aet

(1.33)

vale decir, x(t) es una funcin exponencial real. Si > 0, entonces x(t) es una exponencial creciente,
una forma usada en la descripcin de muchos procesos fsicos diferentes, incluyendo las reacciones en
cadena en explosiones atmicas y en reacciones qumicas complejas. Si < 0, entonces x(t) es una
exponencial decreciente, la cual tambin se usa para describir fenmenos tales como el proceso de
decaimiento radiactivo y las respuestas de redes elctricas formadas por resistores-capacitores (RC) o
resistores-inductores (RL). Observe tambin que para = 0, x(t) es una constante. En la Fig. 1.25 se
ilustran curvas tpicas para > 0 y < 0.
x(t)

x(t)

>0

< 0

Figura 1.25

Las exponenciales complejas jugarn un papel importante en mucho de nuestro tratamiento sobre
seales y sistemas, principalmente porque sirven como bloques sumamente tiles en la construccin de
otras seales. Con frecuencia hallaremos de utilidad el considerar conjuntos de exponenciales
complejas relacionadas armnicamente es decir, conjuntos de exponenciales peridicas con un
perodo comn T0. Especficamente, ya vimos que una condicin necesaria para que la exponencial
compleja e jt sea peridica con perodo T0 es que
e jT0 1

lo que implica que T0 debe ser un mltiplo de 2, es decir,


T0 2 k ,

Entonces, si definimos la frecuencia fundamental

k 0, 1, 2,

(1.34)

26

2
T0

(1.35)

vemos que, para satisfacer la Ec. (1.34), debe ser un mltiplo entero de 0. Es decir, un conjunto de
exponenciales complejas relacionadas armnicamente es un conjunto de exponenciales peridicas con
frecuencias fundamentales que son mltiplos de una sola frecuencia positiva 0:
k ( t ) e jk0 t ,

k 0, 1, 2,

(1.36)

Para k = 0, k ( t ) es una constante, mientras que para cualquier otro valor de k, k ( t ) es peridica con
frecuencia fundamental k 0 y perodo fundamental
T0
k

2
k 0

(1.37)

el k-simo armnico k ( t ) todava es peridico con perodo T0, a medida que recorre k de sus
perodos fundamentales durante cualquier intervalo de duracin T0.
1.8.2

Seales Exponenciales Complejas Generales

El caso ms general de una seal exponencial compleja puede expresarse e interpretarse en funcin de
los casos examinados hasta ahora: la exponencial real y la exponencial compleja peridica.
Especficamente, considere una seal exponencial compleja Aest , donde A se expresa en forma polar y
s en forma rectangular; es decir,

A A e j
y

s j

Entonces,

Aest A e j e( j ) t A et e j ( t )

(1.38)

Usando la identidad de Euler, se puede expandir esta relacin para obtener

Aest A et cos ( t ) j A et sen ( t )

(1.39)

As que para = 0, las partes real e imaginaria de una exponencial compleja son seales sinusoidales.
Para > 0, ellas corresponden a seales sinusoidales multiplicadas por una exponencial real creciente
y, para < 0, corresponden a seales sinusoidales multiplicadas por una exponencial real decreciente.
Ambos casos se ilustran en la Fig. 1.26. Las lneas punteadas representan las funciones A et . De la
Ec. (1.39) vemos que A et es la magnitud de la exponencial compleja. As que las lneas de puntos
se comportan como una envolvente para las curvas oscilatorias en la figura, donde los picos de las
oscilaciones justo tocan estas curvas y, de esta manera, la envolvente proporciona una forma
conveniente de visualizar la tendencia general en la amplitud de la oscilacin.

27

<0

>0

Figura 1.26. Seales sinusoidales multiplicadas por seales exponenciales.

Las seales sinusoidales multiplicadas por exponenciales decrecientes comnmente se conocen como
sinusoides amortiguadas. Ejemplos de ellas se encuentran en la respuesta de redes elctricas
compuestas de resistores-inductores-capacitores (RLC) y en sistemas mecnicos que contienen fuerzas
de amortiguamiento y de restauracin (el amortiguamiento de los automviles, por ejemplo). Estos
sistemas poseen mecanismos que disipan energa (resistores, friccin, etc.).
1.8.3

La Funcin Escaln Unitario

La funcin escaln unitario u(t) se define como


t 0

1
u (t )
0

(1.40)

t 0

y se muestra en la Fig. 1.27a. Observe que es discontinua en t = 0 y que el valor en t = 0 no est


definido. En la misma forma se define la funcin escaln unitario desplazado u(t t0):

1 t t0
u ( t t0 )
0 t t0

(1.41)

y la cual se muestra en la Fig. 1.27b.


u(t t0)

u(t)
1

0
(a)

t0

t
(b)

Figura 2.27. La funcin escaln unitario.

1.8.4

La Funcin Impulso Unitario

En aplicaciones de modelado prcticas, con frecuencia encontramos discontinuidades en una seal x(t)
de tiempo continuo. Una seal as no posee derivadas finitas en sus discontinuidades. No obstante, por

28

razones conceptuales y operacionales, es deseable incluir la derivada de la seal x(t) en nuestras


consideraciones; por lo tanto, introducimos el concepto de la funcin impulso unitario. Esta funcin,
tambin conocida como la funcin delta de Dirac, se denota por ( t ) y se representa grficamente
mediante una flecha vertical, como en la Fig. 1.28.
(t)

Figura 1.28

Tradicionalmente, (t) se define como el lmite de una funcin convencional seleccionada


adecuadamente y la cual tiene un rea unitaria en un intervalo de tiempo infinitesimal, como la funcin
ilustrada en la Fig. 1.29.
En un sentido matemtico estricto, la funcin impulso es un concepto bastante sofisticado. Sin
embargo, para las aplicaciones de inters es suficiente comprender sus propiedades formales y
aplicarlas correctamente. En lo que se expone a continuacin se presentan estas propiedades,
recalcando no el rigor sino la facilidad operacional. En las aplicaciones prcticas de algunos modelos,
con frecuencia encontramos discontinuidades abruptas en una seal f(t) (como la de la Fig. 1.29). Esta
seal no posee derivadas finitas en esas discontinuidades. No obstante, muchas veces es deseable
incluir las derivadas de la seal en nuestras consideraciones. Es aqu donde tiene su aplicacin el
concepto de la funcin impulso unitario. Antes de enunciar algunas de las propiedades de la funcin
impulso considere la funcin dada por

0,

xn ( t ) n ,

0,

t0
0t
t

1
n

1
n

Para n =1, 2 y 3, los pulsos x1(t), x2(t) y x3(t) se muestran en la Fig. 1.29(b). Conforme n aumenta, la
anchura del pulso disminuye y la altura aumenta. Como consecuencia, el rea del pulso para toda n es
igual a la unidad:

xn ( t ) dt 1,

1
n

En el lmite, conforme n , para un nmero positivo, tenemos que

lm xn ( t ) dt 1
n

29

f(t)

xn(t)
4

2
1
t

(a)

(b)

Figura 1.29. Funciones modelos para obtener una funcin impulso.

lo que nos da una forma de definir la funcin impulso unitario como


( t ) lm xn ( t )
n

La funcin impulso (t), tambin conocida como funcin delta de Dirac, tiene las siguientes
propiedades:
1. Es una seal de rea unitaria con valor cero en todas partes excepto en el origen:
0, t 0
(t )
no est definida en t 0

(1.42)

( t ) dt 1

(1.43)

Pero una funcin ordinaria que es igual a cero en todas partes excepto en el origen debe tener
una integral de valor cero (en el sentido de la integral de Riemann). As que (t) no puede ser
una funcin ordinaria y matemticamente se define por

( t ) ( t ) dt (0)

(1.44)

donde (t) es una funcin continua en el origen. Esta propiedad se conoce como la propiedad
de seleccin o de filtrado de la funcin impulso unitario.
Una definicin alterna de (t) est dada por

(0), ab 0

( t ) ( t ) dt 0,
ab 0

a
no definida, a 0 o b 0
b

(1.45)

30

Observe que la Ec. (1.45) o la Ec. (1.46) es una expresin simblica y no debe ser considerada
una integral de Riemann ordinaria. En este sentido, a (t) se le refiere con frecuencia como una
funcin generalizada y a (t) como una funcin de prueba. Tome nota que la funcin impulso
es una funcin ficticia con propiedades ideales que ninguna funcin real posee.
2. La funcin delta es la derivada de la funcin escaln unitario, es decir,

(t )

d u (t )

(1.46)
dt
La demostracin de esta propiedad se deja como un ejercicio para el lector. Esta ltima ecuacin
tambin puede usarse para definir la funcin (t) como
t

() d u(t )

(1.47)

Al igual que (t), la funcin delta retrasada, (t t0), se define por

( t ) ( t t ) dt ( t )
0

(1.48)

A continuacin se presentan algunas consecuencias de las propiedades anteriores:


De la propiedad de la definicin en (1.44), se tiene que la funcin (t) es una funcin par; es decir,
(t ) ( t )

(1.49)

Tambin,
( at )

1
a

(t )

(1.50)

La funcin (t t0) es la derivada de la funcin escaln unitario retrasado:


( t t0 )

d u ( t t0 )
dt

(1.51)

Si (t) es continua en t = 0,
( t ) ( t ) (0) ( t )

(1.52)

( t ) ( t t0 ) ( t0 ) ( t t0 )

(1.53)

y si es continua en t = t0,

Estas dos ltimas ecuaciones representa la propiedad de muestro de la funcin delta, es decir, la
multiplicacin de cualquier funcin (t) por la funcin delta resulta en una muestra de la funcin en los
instantes donde la funcin delta no es cero. El estudio de los sistemas en tiempo discreto se base en esta
propiedad.
Una funcin impulso de nsimo orden se define como la nsima derivada de u(t), es decir

dn
(t ) u (t )
dt
(n)

(1.54)

31

La funcin '(t) se denomina doblete, ''(t) triplete, y as sucesivamente.


Usando las Ecs. (1.47) y (1.48), se obtiene que cualquier funcin continua x(t) puede expresarse
como

x ( ) (t ) d

x(t )

(1.55)

Esta identidad es bsica. Diferencindola con respecto a t, se obtiene

x ( t )

x ( ) ( t ) d

(1.56)

y para t = 0,

x (0)

x ( ) ( ) d

(1.57)

Puesto que (t) es una funcin par, su derivada ( t ) , el doblete, es impar, es decir,
( t ) ( t )

(1.58)

por lo que al usar esta propiedad, la Ec. (1.55) se convierte en

x ( t ) ( t ) dt x (0)

(1.59)

Tambin se puede demostrar que (hgalo Ud.!)

x ( ) ( t ) d x ( t )

(1.60)

Si g(t) es una funcin generalizada, su n-sima derivada generalizada g ( n ) ( t ) d n g ( t ) dt n se define


mediante la siguiente relacin:

(t ) g

(n)

( t ) dt ( 1)

(n)

( t ) g ( t ) dt

(1.61)

donde (t) es una funcin de prueba que puede ser diferenciada un nmero arbitrario de veces y se
anula fuera de algn intervalo fijo. Como una aplicacin de la Ec. (1.59) y la Ec. (1.58), si g ( t ) ( t ) ,
entonces

(t )

(n)

( t x ) dt ( 1) n ( n ) ( x )

De la Ec. (1.46) se tiene que la funcin escaln unitario u(t) puede expresarse como

(1.62)

32

u(t )

( ) d

(1.63)

Ejemplo 15. Halle y dibuje la primera derivada de las seales siguientes:


(a) x ( t ) u ( t ) u ( t a ) a 0
(b) x ( t ) t [ u ( t ) u ( t a )] a 0
Solucin:
(a) Usando la Ec. (1.46), tenemos que
u (t ) (t )

Entonces,

u (t a ) (t a )

x(t ) u (t ) u (t a ) (t ) (t a )

Las seales x(t) y x(t ) se dibujan en la Fig. 1.30.


(b) Usando la regla para la derivada del producto de dos funciones y el resultado de la parte (a), se
obtiene

x ( t ) u ( t ) u ( t a ) t [ ( t ) ( t a )]
Pero, por las Ecs. (1.51) y (1.52),

t ( t ) (0) ( t ) 0 y t ( t a ) a ( t a )
Y, por ello,
x(t ) u (t ) u (t a ) a (t a )

Las seales x(t) y x(t ) se grafican en la Fig. 1.30b.

x(t)

x(t)
a

1
0

x'(t)
(t)

x'(t)
1

a
t

(t a)
(a)

(t a)
(b)

Figura 1.30

33

1.9

Seales de Tiempo Discreto Bsicas

1.9.1

Secuencias Exponenciales Complejas Generales

Igual que en tiempo continuo, una seal importante en tiempo discreto es la secuencia exponencial
compleja

x [ n ] A n

(1.64)

donde A y son, en general, cantidades complejas. Esto podra expresarse alternativamente en la forma

x [ n ] Aen

(1.65)

donde e .
Aunque la forma de la secuencia exponencial compleja dada en la Ec. (1.63) es ms parecida a la
forma de la funcin exponencial en tiempo continuo, a menudo es ms conveniente expresarla en la
forma de la Ec. (1.62).
1.9.2

Secuencias Exponenciales Reales

Si en la Ec. (1.62) A y son reales, podemos tener diferentes tipos de conducta para las secuencias,
como se ilustra en la Fig. 1.31. Si 1 , la magnitud de la seal crece exponencialmente con n,
mientras que si 1 , tenemos una exponencial decreciente. Adicionalmente, si es positiva, todos
los valores de A n tienen el mismo signo, pero si es negativa, entonces los signos de x[n] se
alternan. Observe tambin que si = 1, entonces x[n] es una constante, mientras que si = 1, el valor
de x[n] se alterna entra +A y A. Las exponenciales en tiempo discreto de valores reales con frecuencia
se usan para describir el crecimiento de una poblacin en funcin de su tasa de generacin, el retorno
de una inversin en funcin del da, mes, etc.
<1

>1

(a)

(b)

1 < < 0

< 1

(c)

(d)

Figura 1.31

34

1.9.3

Seales Sinusoidales

Otra exponencial compleja importante se obtiene usando la forma dada en la Ec. (1.63) y restringiendo
a ser puramente imaginaria (de modo que 1 ). Especficamente, considere la expresin

x [ n ] e j 0 n

(1.66)

Igual que en el caso de tiempo continuo, esta seal est ntimamente relacionada con la seal sinusoidal
x [ n ] A cos ( 0 n )

(1.67)

Si tomamos al parmetro n como adimensional, entonces 0 y tienen las dimensiones de radianes. En


la Fig. 1.32 se ilustra un ejemplo de una secuencia sinusoidal.
x[n] cos

6
9

6
0

6 n

9
12

Figura 1.32

La relacin de Euler nos permite escribir


e j0 n cos 0 n j sen 0 n

(1.68)

A cos ( 0 n ) 12 e j e j0 n 12 e j e j0 n

(1.69)

Las seales en las Ecs. (1.64) y (1.65) son ejemplos de seales de tiempo discreto con energa total
infinita pero potencia promedio finita. Por ejemplo, puesto que e j 0 n 1 , toda muestra de la seal en
la Ec. (1.64) contribuye con 1 a la energa de la seal, por lo que la energa total para n es
infinita, mientras que la potencia promedio para algn perodo de tiempo es obviamente igual a 1.
1.9.4

Seales Exponenciales Complejas Generales

La exponencial compleja de tiempo discreto general puede escribirse e interpretarse en funcin de


seales exponenciales reales y de seales sinusoidales. Especficamente, si escribimos A y en forma
polar,

35

A A e j
y

e j 0
entonces
A n A cos ( 0 n ) j A sen ( 0 n )
n

As que para

(1.70)

1 , las partes real e imaginaria de una secuencia exponencial compleja son

sinusoides. Para

1 , ellas corresponden a secuencias sinusoidales multiplicadas por una

exponencial decreciente (Fig. 1.33a), mientras que para 1 , ellas corresponden a secuencias
sinusoidales multiplicadas por exponenciales crecientes (Fig. 1.33b).

(b)

(a)

Figura 1.33

1.9.5

Periodicidad de las Exponenciales Complejas

Aunque hay muchas semejanzas entre las seales exponenciales de tiempo continuo y las de tiempo
discreto, tambin hay diferencias importantes. Una de ellas se relaciona con la seal e j0 n . En la
Seccin 1.8.1 se sealaron las dos propiedades siguientes de su contraparte de tiempo continuo e j 0 t :
(1) Mientras mayor sea la magnitud de 0, ms grande ser la tasa de oscilacin de la seal; y (2)
e j 0 t es peridica para cualquier valor de 0. Ahora se describirn las versiones en tiempo discreto de
estas propiedades y, como se ver, hay diferencias bien definidas entre ellas y sus equivalentes en
tiempo continuo.
El hecho de que la primera de estas propiedades sea distinta en tiempo discreto, es una consecuencia
directa de otra diferencia extremadamente importante entre las exponenciales complejas de tiempo
discreto y las de tiempo continuo. Especficamente, considere la exponencial compleja con frecuencia
igual a 0 2 k , donde k es un entero:
e j ( 0 2 k ) n e j0 n e j 2 k n e j0 n

(1.71)

36

puesto que e j 2 k n 1 . De la Ec. (1.69) vemos que la secuencia exponencial compleja con frecuencia 0
es la misma que las secuencias con frecuencias iguales a 0 2 , 0 4 , etc. As que tenemos una
situacin muy diferente de la del caso en tiempo continuo, donde las seales e j 0 t son todas distintas
para distintos valores de 0. En tiempo discreto, las seales e j0 n no son todas distintas. Como lo
indica la Ec. (1.69); las seales que estn separadas por 2 radianes son idnticas y, por ello, al tratar
con secuencias exponenciales en tiempo discreto, solamente necesitamos considerar un intervalo de
longitud 2 en el cual seleccionar 0. En la mayora de los casos se usar el intervalo 0 0 2 o el
intervalo 0 .
Debido a la periodicidad implicada por la Ec. (1.69), la seal e j0 n no tiene una tasa de oscilacin
que aumenta continuamente conforme 0 aumenta en magnitud. Ms bien, a medida que aumentamos
0 desde 0, obtenemos seales con tasas de oscilacin crecientes hasta alcanzar el valor 0 = . De all
en adelante, al continuar aumentando 0, disminuye la tasa de oscilacin hasta llegar al valor 0 = 2
que es la misma que en 0 = 0. el proceso comienza de nuevo!
1.9.6

Periodicidad de la Exponencial Compleja

Para que la seal e j0 n sea peridica con perodo N > 0, se debe cumplir que

e j0 ( n N ) e j0 n
o, equivalentemente, que
e j0 N 1

(1.72)

Esta ecuacin se satisface si 0N es un mltiplo entero de 2, es decir,


0 N 2 m m un entero positivo

u
0
2

m
N

un nmero racional

(1.73)

Por ello, la secuencia e j0 n no es peridica para cualquier valor de 0; es decir, si 0 satisface la


condicin de periodicidad en la Ec. (1.71), 0 0 y si N y m no tienen factores en comn, el perodo
fundamental N0 de la secuencia e j0 n est dado por

2
N0 m

(1.74)

De acuerdo con la Ec. (1.71), la seal e j0 n es peridica si 0/2 es un nmero racional, y no lo es


para cualquier otro valor. Estas mismas observaciones tambin son vlidas para sinusoides de tiempo
discreto. Por ejemplo, la secuencia en la Fig. 1.34, x[ n ] cos n 6 , es peridica con perodo
fundamental igual a 12, pero la secuencia dada por x[ n ] cos n 2 no lo es.

37

x[n] cos n 6
6

Figura 1.34

Ejemplo 16. Sea

x[ n] e j ( 7 9 )n

Entonces
0
2

7 9
2

7
18

m
N

As pues, x[n] es peridica y su perodo fundamental, obtenido al hacer m = 7, es igual a 18.


Si x[n] es la suma de dos secuencias peridicas x1[n] y x2[n], las cuales tienen periodos fundamentales
N1 y N2, respectivamente, entonces si

mN1 kN2 N

(1.75)

donde m y k son enteros, x[n] es peridica con perodo N (demustrelo!). Puesto que siempre podemos
encontrar enteros m y k que satisfagan la Ec. (1.73), se deduce que la suma de dos secuencias
peridicas es tambin peridica y su perodo fundamental es el mnimo comn mltiplo de N1 y N2.
Igual que en el caso de tiempo continuo, en el anlisis de sistemas y seales en tiempo discreto
tambin es muy importante considerar conjuntos de exponenciales relacionadas armnicamente es
decir, exponenciales peridicas con un perodo comn N0. De la Ec. (1.71) sabemos que stas son
precisamente las seales con frecuencias que son mltiplos de 2/N0. Es decir,

k [ n ] e jk 0 n ,

2
N0

k 0, 1, 2,

(1.76)

En el caso de tiempo continuo, todas las exponenciales complejas relacionadas armnicamente,


e j k ( 2 T0 ) , k 0, 1, 2,, son distintas. Sin embargo, debido a la Ec. (1.69), ste no es el caso en
tiempo discreto. Especficamente,

k N0 [ n ] e j ( k N0 )( 2 N0 ) n e j k ( 2 N0 ) n k [ n ]

(1.77)

la cual implica que slo hay N0 exponenciales peridicas distintas en el conjunto dado por la Ec. (1.74)
y, por ello, se tiene que

38

0 [ n ] N0 [ n ], 1 [ n ] N0 1 [ n ], , k [ n ] N0 k [ n ],
1.9.7

(1.78)

La Secuencia Escaln Unitario

La secuencia escaln unitario u[n] se define como


1, n 0
u[n]
0, n 0

(1.79)

la cual se muestra en la Fig. 1.35a. Observe que el valor de u[n] est definido en n = 0 (a diferencia de
la funcin escaln unitario de tiempo continuo, que no lo est en t = 0). En forma similar, la secuencia
escaln unitario desplazado u[n k] se define como
1, n k
u[n k ]
0, n k

(1.80)

y se muestra en la Fig. 1.35b.


u[n k]

u[n ]

...

2 1 0 1 2 3

...

...

(a)

...

...

(b)

Figura 1.35

1.9.8

La Secuencia Impulso Unitario

Una de las seales ms sencillas de tiempo discreto es la secuencia impulso unitario (o muestra
unitaria), la cual se define como
1 n 0
(1.81)
[ n ]
0 n0
y se ilustra en la Fig. 1.36a. En forma similar, la secuencia impulso unitario desplazado (o muestra
unitaria que ocurre en n = k, [n k] se define como
1 n k
[ n k ]
0 nk

la cual se muestra en la Fig. 1.36b.

(1.82)

39

[n ]

...

[n k]

...

2 1 0

1 2 3

...

1 0 1

(a)

...

...
k

(b)

Figura 1.36

A diferencia de la funcin impulso unitario de tiempo continuo (t), [n] se define para todos los
valores de n sin complicaciones o dificultades analticas; observe que la magnitud del impulso discreto
es siempre finita. A partir de las definiciones (1.79) y (1.80) se ve rpidamente que

x [ n ] [ n ] x [0] [ n ]
x [ n ] [ n k ] x [ k ] [ n k ]

las cuales representan la propiedad de seleccin de la secuencia impulso unitario, es decir, la secuencia
impulso unitario puede usarse para tomar muestras de la seal x[n].
La relacin en tiempo discreto entre el impulso y el escaln unitarios viene dada por la llamada
primera diferencia; ella es
[ n ] u [ n ] u [ n 1]

(1.83)

Inversamente, el escaln unitario es la suma acumulada de la muestra unitaria; es decir,


u[n]

[ m ]

(1.84)

Observe en la Ec. (1.82) que la suma acumulada es igual a 0 para n < 0 y 1 para n 0 ,
Adicionalmente, si se cambia la variable de la sumatoria de m a k = n m, la Ec. (1.82) se convierte en

u [ n ] [ n k ]

(1.85)

k 0

En la Ec. (1.83) el valor diferente de cero de [n k ] ocurre cuando k = n, as que de nuevo vemos
que la sumatoria es 0 para n < 0 y 1 para n 0. Una interpretacin de la Ec. (1.83) es verla como una
superposicin de impulsos retardados, es decir, podemos considerar la ecuacin como la suma de un
impulso unitario [ n ] en n = 0, un impulso unitario [ n 1] en n = 1, otro, [ n 2] en n = 2, etc.

1.10

Sistemas y Clasificacin de Sistemas

Los sistemas fsicos en el sentido ms amplio son un conjunto de componentes o bloques funcionales
interconectados para alcanzar un objetivo deseado. Para nuestros propsitos, un sistema es un modelo
matemtico que relaciona las seales de entrada (excitaciones) al sistema con sus seales de salida
(respuestas). Por ejemplo, un sistema de alta fidelidad toma una seal de audio grabada y reproduce esa
seal. Si el sistema tiene controles de tono, se puede cambiar la calidad tonal de la seal reproducida;

40

en otras palabras, el sistema procesa la seal de entrada. De igual modo, la red sencilla de la Fig. 1.37
se puede considerar como un sistema que procesa un voltaje de entrada ve(t) y produce un voltaje de
salida vs(t). Un sistema de realzamiento de imgenes transforma una imagen de entrada en una imagen
de salida con algunas propiedades deseadas como, por ejemplo, un mayor contraste entre los colores.
R

ve

vs

Figura 1.37

Si x y y son las seales de entrada y de salida, respectivamente, de un sistema, entonces el sistema se


considera como una transformacin de x en y. Esta representacin se denota por
y T[ x ]

(1.86)

donde T es el operador que representa alguna regla bien definida mediante la cual la excitacin x es
transformada en la respuesta y. La relacin (1.84) se ilustra en la Fig. 1.38a para el caso de un sistema
de una sola entrada y una sola salida. La Fig. 1.38b ilustra un sistema con entradas y salidas mltiples.
En estas notas solamente nos ocuparemos de sistemas con una sola entrada y una sola salida.

Sistema
T

x1
.
.
.

Sistema
T

xn

y1
.
.
.
yn

(a)

(b)

Figura 1.38

1.10.1 Sistemas en Tiempo Continuo y en Tiempo Discreto


Un sistema en tiempo continuo es un sistema en el cual las seales de entrada y de salida son de tiempo
continuo (Fig. 1.39a). Si las seales de entrada y de salida son de tiempo discreto, entonces el sistema
se llama un sistema en tiempo discreto (Fig. 1.40b). Ambos sistemas tambin se denotarn
simblicamente por
x (t ) y (t )

(a)

x n y n

(b)

(1.87)

41

x(t)

y(t)

Sistema de
tiempo continuo

x[n]

Sistema de
tiempo discreto

(a)

y[n]

(b)

Figura 1.39

Ejemplo 17. Considere la red RC de la Fig. 1.37. Si tomamos al voltaje ve(t) como la seal de entrada y
al voltaje vs(t) como la seal de salida, entonces, aplicando la ley de Ohm, la corriente que pasa por el
resistor R es
v ( t ) vs ( t )
i (t ) e
R
Esta corriente est relacionada con el voltaje en el capacitor, vs(t), por
d v (t )
i (t ) C s
dt
y de estas dos ltimas relaciones, obtenemos la ecuacin diferencial que conecta la entrada con la
salida:
d vs ( t ) 1
1
(1.88)

vs ( t )
ve ( t )
dt
RC
RC
En general, los sistemas en tiempo continuo en una sola variable estn descritos por ecuaciones
diferenciales ordinarias. En el Ejemplo 17, la ecuacin diferencial ordinaria es una con coeficientes
constantes, lineal y de primer orden, de la forma
d y (t )
dt

a y (t ) b x (t )

(1.89)

en la cual x(t) es la entrada y y(t) es la salida y a y b son constantes.

Ejemplo 18. Un ejemplo sencillo de un sistema de tiempo discreto, lo da un modelo simplificado para
el balance mensual de una cuenta bancaria de ahorros. Especficamente, sea y[n] el balance al final del
n-simo mes y suponga que y[n] evoluciona mensualmente de acuerdo con la ecuacin
y [ n ] 1.01 y [ n 1] x [ n ]

o
y [ n ] 1.01 y [ n 1] x [ n ]

(1.90)

donde x[n] representa el depsito neto (es decir, depsitos menos retiros) durante el n-simo mes y el
trmino 1.01y[n] modela el hecho del aporte del 1% de inters mensual

42

La Ec. (1.88) es un ejemplo de una ecuacin en diferencias lineal de primer orden y de coeficientes
constantes, vale decir, una ecuacin en diferencias de la forma

y [ n ] a y [ n 1] b x [ n ]
Como lo sugieren los Ejemplos 17 y 18, las descripciones matemticas de sistemas provenientes de
una gran variedad de aplicaciones, con frecuencia tienen mucho en comn, y este hecho es una de las
mayores motivaciones para el desarrollo de herramientas que faciliten el anlisis de seales y sistemas.
Aqu la clave del xito est en identificar clases de sistemas que posean dos caractersticas importantes:
1. Los sistemas deben tener propiedades y estructuras que se puedan explotar para obtener una mejor
comprensin de su comportamiento y para desarrollar herramientas efectivas para el anlisis.
2. Los sistemas de importancia prctica deben poder modelarse con la mayor precisin posible
usando modelos tericos bsicos.
La mayor parte de este texto est enfocada en la primera de estas caractersticas y su aplicacin a
sistemas lineales e invariantes en el tiempo (sistemas LIT). En la prxima seccin se introducirn las
propiedades que caracterizan este tipo de sistemas como tambin otras propiedades bsicas de mucha
importancia.
La segunda caracterstica mencionada es de una importancia obvia para que cualquier tcnica de
anlisis tenga valor prctico. Los sistemas que estudiaremos pueden modelar bastante bien una gran
variedad de sistemas fsicos. Sin embargo, un punto crtico es que cualquiera sea el modelo utilizado
para analizar un sistema fsico, ese modelo es una idealizacin y, por consiguiente, cualquier anlisis
basado en el modelo ser tan bueno como lo sea el modelo. En el caso de resistores y capacitores
reales, por ejemplo, los modelos idealizados son bastante precisos para muchas aplicaciones y
proporcionan resultados y conclusiones tiles, siempre y cuando las variables fsicas voltajes y
corrientes permanezcan dentro de las bandas de operacin establecidas por los modelos. Por ello, es
importante en la prctica de ingeniera tener siempre presente los intervalos de validez de las
suposiciones hechas para elaborar el modelo y tambin asegurarnos que cualquier anlisis o diseo no
viola esas suposiciones.
1.10.2 Sistemas Con y Sin Memoria
Se dice que un sistema es instantneo o sin memoria si su salida en cualquier instante depende
solamente de su excitacin en ese instante, no de ningn valor pasado o futuro de la excitacin. Si esto
no es as, se dice que el sistema tiene memoria. Un ejemplo de un sistema sin memoria es un resistor R;
con la entrada x(t) tomada como la corriente y el voltaje tomado como la salida y(t), la relacin de
entrada-salida (ley de Ohm) para el resistor es
y (t ) R x (t )

(1.91)

Un sistema que no es instantneo se dice dinmico y que tiene memoria. As pues, la respuesta de un
sistema dinmico depende no slo de la excitacin presente sino tambin de los valores de la entrada
pasada. Un ejemplo de un sistema con memoria es un capacitor C con la corriente como la entrada x(t)
y el voltaje como la salida y(t); entonces,

43

y (t )

x ( ) d

(1.92)

En tiempo discreto, un ejemplo de un sistema con memoria es un acumulador, en el cual las secuencias
de entrada y salida estn relacionadas por
n

x[k ]

(1.93)

y [ n ] x [ n 1]

(1.94)

y [n]

y otro ejemplo es un retardo

El concepto de memoria en un sistema, expuesto someramente, corresponde a la presencia de algn


mecanismo que permite el almacenamiento de informacin sobre los valores de la excitacin en
tiempos diferentes del presente. Por ejemplo, el retardo en la Ec. (1.92) retiene el valor pasado
inmediato. Del mismo modo, el acumulador de la Ec. (1.91), recuerda la informacin sobre todas las
excitaciones hasta el momento presente; la relacin (1.91) puede escribirse en la forma equivalente
n 1

y [n]

x {k ] x [ n ]

o
y [ n ] y [ n 1] x [ n ]

(1.95)

En estas dos ltimas ecuaciones se observa que para obtener la salida en el tiempo presente, el
acumulador debe recordar la suma acumulada de los valores previos, y esa suma es exactamente el
valor precedente de la salida del acumulador.
1.10.3 Invertibilidad y Sistemas Inversos
Se dice que un sistema es invertible si excitaciones distintas producen respuestas distintas. Como se
ilustra en la Fig. 1.40a, si un sistema es invertible, entonces existe un sistema inverso, el cual, al ser
excitado con la salida del sistema invertible, reproduce la seal original; es decir, en un sistema
invertible siempre es posible recuperar la entrada si se conoce la salida; si las excitaciones diferentes
(nicas) producen respuestas diferentes (nicas), entonces es posible, en principio, si se da la respuesta,
asociarla con la excitacin que la produjo.
Un ejemplo de un sistema de tiempo continuo invertible es
y (t ) 2 x (t )

(1.96)

y su inverso es
w t

Los dos sistemas se ilustran en la Fig. 1.40b.

1
2

y t

(1.97)

44

Otro ejemplo de un sistema invertible es el acumulador de la Ec. (1.91). En este sistema, la diferencia
entre dos valores sucesivos es precisamente el ltimo valor de la entrada. En consecuencia, para este
caso el sistema inverso es
w[ n ] y [ n ] y [ n 1]

(1.98)

como se muestra en la Fig. 1.40c.

x(t)

Sistema
invertible

y(t)

y(t)

x(t)

y(t ) 2 x(t )

y(t)

y(t)

x[n]

y[n]

n
k

x[k ]

y[n]

y[n]

Sistema
invertible

w(t )

1
y(t )
2

x(t)

w(t) = x(t)

w[n] y[n] y[n 1]

w[n] = x[n]

Figura 1.40

Ejemplos de sistemas no invertibles son

y n 0

(1.99)

y ( t ) x2 ( t )

(1.100)

En el primer caso, Ec. (1.97), el sistema produce la secuencia cero para cualquier entrada y, en el
segundo caso, Ec. (1.98), no se puede determinar el signo de la funcin de entrada a partir del
conocimiento de la seal de salida. Observe que en el primer caso, si y n c , donde c es una
constante, el sistema no es invertible.
El concepto de invertibilidad es muy importante. Un ejemplo bastante claro proviene de los sistemas
para codificacin utilizados en una gran variedad de aplicaciones en los sistemas de comunicacin. En
esos sistemas, se codifica primero la seal que se va a transmitir; para que el sistema no cometa errores
(sistema ideal), debe ser posible recuperar completamente la seal original a partir de la seal
codificada. En otras palabras, el codificador debe ser invertible.
1.10.4 Sistemas Causales
El trmino causalidad connota la existencia de una relacin causa-efecto. Se dice que un sistema es
causal si su salida en cualquier instante arbitrario depende solamente de los valores de la entrada en ese
instante y en el pasado. Es decir, la salida de un sistema causal en el tiempo presente depende slo de

45

los valores presente y pasados de la entrada. A estos sistemas tambin se les refiere como noanticipativos, ya que el sistema no anticipa, ni depende de valores futuros de la entrada. Como
consecuencia, si dos entradas a un sistema causal son idnticas hasta algn punto en el tiempo t0 o n0,
entonces las salidas correspondientes a esas entradas tambin deben ser idnticas. Todos los sistemas
fsicos son causales ya que no pueden ver el futuro y anticipar una excitacin. El circuito RC de la Fig.
1.37 es un sistema causal puesto que, por ejemplo, el voltaje en el capacitor responde solamente a los
valores presentes y pasados del voltaje de la fuente.
Un sistema se denomina no-causal, si no es causal. Ejemplos de sistemas no-causales son
y ( t ) x ( t 1)

(1.101)

y [ n] x[ n]

(1.102)

Observe que todos los sistemas sin memoria son causales; sin embargo, lo inverso no es cierto.
Aunque los sistemas causales son de gran importancia, ellos no constituyen los nicos sistemas de
significacin prctica. Por ejemplo, la causalidad no es con frecuencia una restriccin esencial en
aplicaciones en las cuales la variable independiente no es el tiempo, como, por ejemplo, en el
procesamiento de imgenes. Tambin, en el procesamiento de datos grabados con anterioridad, como
sucede a menudo con seales de voz, geofsicas o meteorolgicas, para nombrar algunas, no estamos en
modo alguno restringidos a un procesamiento causal. Como otro ejemplo, en muchas aplicaciones,
incluyendo el anlisis histrico de la bolsa de valores o de estudios demogrficos, podramos estar
interesados en determinar alguna tendencia de variacin lenta, la cual podra contener fluctuaciones de
alta frecuencia con respecto a la tendencia. En este caso, un enfoque comn es promediar los datos para
suavizarlos y mantener solamente la tendencia. Un ejemplo de un sistema causal que promedia est
dado por

y [n]

x[n k ]

2 M 1 k M

(1.103)

Para comprobar la causalidad de un sistema es importante observar cuidadosamente la relacin de


entrada-salida. Como ilustracin, se comprobar la causalidad de dos sistemas especficos. Ejemplo 19.
El primer sistema lo define la Ec. (1.100), y [ n ] x [ n ] . Observe que la salida y[n0], para un tiempo
positivo n0, depende slo del valor de la seal de entrada x [ n0 ] en el tiempo ( n0 ) , el cual es
negativo y, por tanto, est en el pasado de n0. Aqu podra surgir la tentacin de concluir que el sistema
es causal. No obstante, debemos ser cuidadosos y proceder a comprobar la relacin de entrada-salida
para todo el tiempo. En particular, para n0 < 0, vemos que n0 0 y y n0 x n0 , y la salida
en n0 < 0 depende del valor futuro de la entrada. Por lo tanto, el sistema es no-causal.
Tambin es importante distinguir cuidadosamente los efectos sobre la entrada de cualesquiera otras
funciones usadas para definir un sistema. Por ejemplo, considere el sistema definido por la relacin
y ( t ) x ( t ) cos ( t 1)

(1.104)

En este caso, la salida en cualquier instante t, y(t), depende de la entrada x(t) multiplicada por un
nmero que vara con el tiempo. Es decir, solamente el valor presente de x(t) influye en el valor
presente de la salida y(t) y concluimos entonces que este sistema es causal (y sin memoria!).

46

1.10.5 Sistemas Estables


Bsicamente, un sistema estable es aqul en el cual pequeas excitaciones producen respuestas que no
divergen (no aumentan sin lmite). Considere, por ejemplo, el pndulo en la Fig. 1.41a, en el cual la
excitacin es la fuerza aplicada x(t) y la respuesta es la desviacin angular y(t) con respecto a la
normal. En este caso, la gravedad aplica una fuerza que tiende a regresar al pndulo a la posicin
vertical (posicin de equilibrio) y las prdidas por friccin tienden a frenar el movimiento oscilatorio.
Es obvio que si la fuerza aplicada es pequea, la desviacin resultante tambin lo ser. Adems, al
dejar de aplicar esa fuerza, el pndulo regresar a su posicin de equilibrio. En contraste, para el
pndulo invertido en la Fig. 1.41b, la gravedad ejerce una fuerza que tiende a aumentar cualquier
desviacin de la posicin de equilibrio. Por ello, la aplicacin de cualquier fuerza, por muy pequea
que sea, conduce a un aumento de la desviacin y hace que el pndulo caiga y no regrese a la posicin
original.
El sistema de la Fig. 1.41a es un ejemplo de un sistema estable, mientras que el de la Fig. 1.41b, es
inestable. La estabilidad de los sistemas fsicos resulta de la presencia de mecanismos que disipan
energa. El circuito RC de la Fig. 1.37 es un sistema estable porque el resistor disipa energa, la
respuesta est acotada para un voltaje de la fuente acotado y al desaparecer la excitacin proporcionada
por la fuente, desaparece la respuesta.

y(t)
x(t)
y(t)

x(t)

(a)

(b)

Figura 1.41

Los ejemplos mencionados dan una idea intuitiva del concepto de estabilidad. Expresado ms
formalmente, un sistema es estable si para una entrada acotada, la salida correspondiente tambin
est acotada. Es decir, si la entrada est definida por

x k1

(1.105)

y k2

(1.106)

entonces la salida est definida por

donde k1 y k 2 son constantes reales finitas.


Para demostrar que un sistema es estable, una estrategia vlida es buscar una excitacin acotada
especfica y verificar si la salida resultante est acotada o no. Considere, por ejemplo, el sistema
descrito por la relacin

47

y[ n] n x[ n]

(1.107)

Si seleccionamos una entrada constante, x[n] = k, la respuesta del sistema ser y[n] = kn, que no est
acotada, puesto que sea cual sea el valor (finito) de k, y [n] exceder ese valor para algn valor de n.
Esta propiedad y las anteriores, sern analizadas con mayor detalle en captulos posteriores.
1.10.6 Invariabilidad en el Tiempo
Conceptualmente, un sistema es invariable en el tiempo si su conducta y sus caractersticas son fijas en
el tiempo. El circuito RC de la Fig. 1.37 es un sistema con esta propiedad si los valores de los
parmetros R y C son constantes; es de esperar que experimentos idnticos produzcan los mismos
resultados sin importar el momento en que se realicen. Por supuesto, esos resultados no seran los
mismos si R o C o ambos cambian con el tiempo.
Esta propiedad puede expresarse en una forma muy sencilla para las seales y sistemas que hemos
estudiado hasta ahora. Especficamente, se dice que un sistema es invariable en el tiempo si un
desplazamiento en el tiempo (retraso o avance) en la seal de entrada resulta en un desplazamiento
igual en la seal de salida. Entonces, para un sistema de tiempo continuo, el sistema no vara con el
tiempo si
x ( t t0 ) y ( t t0 )

(1.108)

para cualquier valor real t0, y, para un sistema de tiempo discreto,


x [ n n0 ] y [ n n0 ]

(1.109)

para cualquier entero n 0 . Un sistema que no cumpla con la Ec. (1.108) (tiempo continuo) o la Ec.
(1.109), se conoce como un sistema variable en el tiempo. Para comprobar si un sistema es invariable
en el tiempo, sencillamente comparamos la salida producida por la entrada desplazada

Ejemplo 20. El sistema de la Fig. 1.42 se conoce como el elemento retardo unitario. Determine si el
sistema es invariable en el tiempo.
Respuesta: Sea y1[n] la respuesta a x1 [ n ] x [ n n0 ] . Entonces
y1 [ n ] x [ n n0 1]

y tambin
y [ n n0 ] x [ n n0 1] y1 [ n ]

Por consiguiente, el sistema es invariable en el tiempo.

x[n]

Retardo
unitario
Figura 1.42

y[n] = x[n 1]

48

Ejemplo 21. Considere el sistema definido por

y ( t ) sen x ( t )
Sea y1(t) la respuesta a la entrada x1(t); esto es
y1 ( t ) sen x1 t

(1.110)

y sea y2(t) la respuesta a la entrada desplazada x1 ( t t0 ) ; es decir,


y2 ( t ) sen x1 t t0

De la Ec. (1.110) se obtiene


y1 ( t t0 ) sen x1 t t0 y2 ( t )

y, por lo tanto, el sistema es invariable en el tiempo.

Ejemplo 22. Un sistema tiene una relacin de entrada-salida dada por


y[ n] n x[ n]

Determine si el sistema es invariable en el tiempo.


Respuesta. Sea y1[n] la respuesta a x1 [ n ] x [ n n0 ] . Entonces
y1 [ n ] n x [ n n0 ]

Pero
y [ n n0 ] ( n n0 ) x [ n n0 ] y1 [ n ]

y el sistema vara con el tiempo


El sistema de este ejemplo representa uno cuya ganancia vara con el tiempo. Si, digamos, el valor de
la entrada es igual a 1, no podemos determinar el valor de la salida sin conocer el instante en que se
aplic la excitacin. Continuando con esta idea, considere la seal de entrada x1 [ n ] [ n ] ; sta
produce una salida igual a cero

n [ n ] 0 . Sin embargo, la entrada

x2 [ n ] [ n 1] produce la

salida y2 [ n ] n [ n 1] [ n 1] . As que, mientras que x 2 [n] es una versin desplazada de x 1 [n], la


respuesta y 2 [n] no es una versin desplazada de y1[n]. Lo que se quiere sealar con esto es que para
demostrar si un sistema cumple con la condicin (1.106) o la (1.107), slo basta con encontrar un
ejemplo, y slo uno, que indique lo contrario

49

1.10.7 Sistemas Lineales

Un sistema lineal, en tiempo continuo o discreto, es aqul que posee la importante propiedad de la
superposicin. Para esta clase de sistemas, si una entrada consiste de la suma ponderada de varias
seales, entonces la salida es la superposicin es decir, la suma ponderada de las respuestas del
sistema a cada una de esas seales. En forma ms precisa, en tiempo continuo, si y1(t) es la respuesta a
la entrada x1(t), y y2(t) es la respuesta a x2(t), entonces el sistema es lineal si
La respuesta a x1 ( t ) x2 ( t ) es y1 ( t ) y2 ( t ) . Propiedad de aditividad.
La respuesta a x1 ( t ) es y1 ( t ) , donde es cualquier constante. Propiedad de homogeneidad o
de escalamiento.

1.
2.

Las dos propiedades que definen un sistema lineal pueden combinarse en una sola expresin:
Tiempo continuo:

x1 ( t ) x2 ( t ) y1 ( t ) y2 ( t )

(1.111)

Tiempo discreto:

x1 [ n ] x2 [ n ] y1 [ n ] y2 [ n ]

(1.112)

donde y son constantes. Adicionalmente, es muy sencillo demostrar a partir de la definicin de


linealidad que si xk [ n ], k 1, 2, , forman un conjunto de entradas a un sistema lineal de tiempo
discreto con salidas correspondientes yk [ n ], k 1, 2, , entonces la respuesta a una combinacin
lineal de estas entradas,
x[n]

x [ n ]

(1.113)

y [ n ]

(1.114)

es

y[n]

Este hecho, de mucha importancia en el anlisis de sistemas lineales, se conoce como la propiedad de
superposicin, y se cumple para sistemas tanto de tiempo continuo como discreto.
Una consecuencia directa de la propiedad de superposicin es que, para sistemas lineales, una entrada
que es cero para todo el tiempo resulta en una salida que tambin es igual a cero para todo el tiempo
(propiedad de homogeneidad).

Ejemplo 23. Un sistema tiene la relacin de entrada-salida


y x2

Demuestre que el sistema es no-lineal.


Solucin: Tenemos que

50

x1 y1 x12
x2 y2 x22
y, por lo tanto,
x1 x2 ( x1 x2 )2 x12 2 x1 x2 x22 x12 x22

As que el sistema es no-lineal.

Ejemplo 24. Considere un sistema cuya relacin de entrada-salida est dada por
y (t ) t x (t )

Entonces,
x1 ( t ) y1 ( t ) t x1 ( t )
x2 ( t ) y2 ( t ) t x2 ( t )

y si
x3 ( t ) a x1 ( t ) b x2 ( t )

donde a y b son constantes arbitrarias, entonces, para la entrada x3 ( t ) se obtiene


t x3 ( t ) a t x1 ( t ) bt x2 ( t )
a y1 ( t ) b y2 ( t )

y el sistema es lineal
Ejemplo 25. Considere el sistema cuya relacin de entrada-salida est dada por la ecuacin lineal
y = ax + b
donde a y b son constantes. Si b 0, el sistema es no-lineal porque x = 0 implica que y = b 0
(propiedad de homogeneidad). Si b = 0, el sistema es lineal.
Puede parecer sorprendente que el sistema en el ejemplo anterior no sea lineal puesto que su ecuacin
de definicin, y = ax + b, es una lnea recta. Por otra parte, como se muestra en la Fig. 1.43, la salida de
este sistema puede representarse como la suma de la salida de un sistema lineal y otra seal igual a la
respuesta de entrada cero del sistema. Para el sistema del ejemplo, el sistema lineal es
y = ax
y la respuesta de entrada cero es
y0(t) = b
Hay grandes clases de sistemas que pueden representarse como en la Fig. 1.43 y para los cuales la
salida completa del sistema consiste de la superposicin de la respuesta de un sistema lineal y una
respuesta de entrada cero. Estos sistemas corresponden a la clase de sistemas lineales incrementales, es

51

decir, sistemas que responden linealmente a cambios en la entrada. Dicho de otra forma, la diferencia
entre las respuestas a dos entradas cualesquiera a un sistema lineal incremental es una funcin lineal de
la diferencia entre las dos entradas.

y0(t)
Sistema
lineal

x(t)

y(t)

Figura 1.43. Sistema lineal incremental.

1.11

Interconexin de Sistemas

Una idea que se usar a travs del texto es el concepto de la interconexin de sistemas. Muchos
sistemas reales son conformados como interconexiones de varios subsistemas; un ejemplo es un
sistema de audio, el cual involucra la interconexin de un radio receptor, un reproductor de discos
compactos o un grabador con un amplificador y una o ms bocinas. Considerando tales sistemas como
una interconexin de sus componentes, podemos usar nuestros conocimientos de los componentes y de
su interconexin para analizar la operacin y conducta del sistema completo. Adicionalmente, al
describir un sistema en funcin de una interconexin de subsistemas ms sencillos, podemos de hecho
definir formas tiles para sintetizar sistemas complejos a partir de bloques de construccin bsicos ms
simples.
Aun cuando se puede construir una gran variedad de interconexiones de sistemas, hay varias formas
bsicas que se encuentran con mucha frecuencia. Una interconexin en serie o en cascada de dos
sistemas se ilustra en la Fig. 1.44a. Esta clase de diagramas se conoce como diagramas de bloques.
Aqu, la salida del sistema 1 es la entrada al sistema 2. Un ejemplo de una interconexin en serie es un
receptor de radio conectado a un amplificador. En la misma forma se puede definir una interconexin
en serie de tres o ms sistemas.
En la Fig. 1.44b se ilustra una interconexin en paralelo de dos sistemas. Aqu, la seal de entrada se
aplica simultneamente a los sistemas 1 y 2. El smbolo en la figura denota la operacin de adicin,
as que la salida de la interconexin en paralelo es la suma de la salida de los sistemas 1 y 2. Un
ejemplo de esta interconexin es un sistema de audio sencillo, en el cual varios micrfonos alimentan
un solo amplificador. Adicionalmente a la interconexin en paralelo sencilla de la Fig. 1.44b, podemos
definir la interconexin en paralelo de ms de dos sistemas y podemos combinar interconexiones en
cascada y en paralelo para obtener interconexiones ms complicadas. Un ejemplo es este tipo de
interconexin se ilustra en la Fig. 1.44c.
Otro tipo importante de interconexin de sistemas es la llamada interconexin de realimentacin; un
ejemplo de ella se muestra en la Fig. 1.45. Aqu, la salida del sistema 1 es la entrada al sistema 2,
mientras que la salida del sistema 2 es realimentada y sumada a la entrada externa para producir la
entrada efectiva al sistema 1.

52

Sistema 1
Entrada

Sistema 1

Salida

Sistema 2

Salida
Entrada
Sistema 2

(a
)
Sistema 1

(b)

Salida

Sistema 2

Entrada

Sistema 4

Salida

Sistema 3
(c)

Figura 1.44. Interconexiones.


Entrada

Salida

Sistema 1

Sistema 2

Figura 1.45. Sistema realimentado.

Los sistemas realimentados surgen en una gran variedad de aplicaciones. Por ejemplo, el sistema de
control de la velocidad de crucero de un automvil mide la velocidad del vehculo y ajusta el flujo de
combustible para mantener la velocidad en el nivel deseado. Tambin, los circuitos elctricos a menudo
son considerados como si tuviesen interconexiones realimentadas. Como un ejemplo, considere el
circuito mostrado en la Fig. 1.46a. Como se indica en la Fig. 1.46b, este sistema puede analizarse
considerndolo como la interconexin realimentada de los elementos del circuito.

i(t)

i (t )

i2 (t )

i1 ( t )

v (t )

i1(t)

Capacitor

1
v(t )
C

i2(t
)

v(t)

i (t ) dt
1

Resistor

i 2 (t )

(a)

v(t )
R

(b)
Figura 1.46

53

Problemas
1.1. Las siguientes desigualdades se usan con frecuencia en estas notas. Demuestre su validez.

1 N
, 1

n 1
n 0
N,
1

N 1

(a)

nf

(c)

n n0

n0

(b)

n 0

1
1

, 1

n f 1

, 1

1.2. Halle el menor perodo positivo T de las siguientes seales:


cos nt ,

sen nt ,

cos

2 t
k

sen

2 t
k

cos

2 nt
k

sen

2 nt
k

1.3. Bosqueje las seales siguientes:


(a) sen t ,
(b) sen 2 t 2sen 6 t ,

t 0
4 ,
(c) x ( t )
y x (t 2 ) x (t )
,
0t
4
(d) x ( t ) exp t , t , y x ( t 2 ) x ( t )
1.4. Demuestre que si x(t) es peridica con perodo T, entonces tambin es peridica con
perodo nT, n = 2, 3, .
1.5. Si x1(t) y x 2 (t) tienen perodo T, demuestre que x3(t) = ax 1(t) +bx 2(t) (a, b constantes) tiene
el mismo perodo T.
1.6. Son peridicas las seales siguientes? Si lo son, determine sus perodos.
(a) x ( t ) sen 23 t 2sen 163 t

(b) x ( t ) 4exp j 25 t 3exp j 3t


1.7. Sea x(t) una seal peridica con perodo fundamental T. Determine cules de las siguientes
seales son peridicas y halle sus perodos fundamentales:
(a) y1 ( t ) x (2 t )
(c) y2 ( t ) x ( t 2)
n2
(e) x [ n ] cos
8

(b) x ( t ) cos2 t
(d) x [ n ] e j [( n 4 ) ]

54

n
n
n
(f) x [ n ] cos
sen
2cos

4
8
2

1.8. Si x(t) es una seal peridica en t con perodo T, demuestre que x(at), a > 0, es una seal
peridica en t con perodo T/a, y que x(t/b), b > 0, es una seal peridica en t con perodo
bT. Verifique estos resultados para x ( t ) sen t , a b 2 .
1.9. Demuestre que si x[n] es peridica con perodo N, entonces se cumple que
n

(a)

n N

x[k ]

k n0

(b)

x[k ]

k n0 N

n N

k 0

k N

x[ k ] x[ k ]

1.10. Determine si las siguientes seales son seales de potencia o de energa o de ninguno de los
dos tipos. Justifique sus respuestas.
(a) x ( t ) A u ( t a ) u ( t a )

(b) r ( t ) r ( t 1)

(c) x ( t ) exp( at ) u ( t ),
(e) x ( t ) u ( t )

(d) x ( t ) t u ( t )

a0

(f) x ( t ) A exp bt ,

b0

1.11. Para una seal de energa x(t) con energa Ex, demuestre que la energa de cualquiera de las
seales x(t), x(t) y x(t T) es Ex. Demuestre tambin que la energa de x(at) y de
x(at b) es Ex/a. Cul es el efecto sobre la energa de la seal si se multiplica por una
constante a?
1.12. Para la seal x(t) dada como

t,

x ( t ) 1,
0,

1 t 0
1 t 2
otros valores de t

grafique y halle expresiones analticas para las siguientes funciones:


(a) x ( t 2)
(c) x (2 t 4)

(b) x (3 t )

(d) x 3t 12

1.13. Repita el Problema 1.11 para la seal

1,

x ( t ) exp( t )
0

1 t 0
t0
otros valores de t

1.14. Para la seal de tiempo discreto mostrada en la Fig. P1.14, dibuje cada una de las seales
siguientes:
(a) x 2 n

(b) x 3n 4

55

(c) x 23 n 1

n 8
(d) x

(e) xp[n]

(f) xi[n]

(g) x 2 n x 3n 4
3
2
1
2

1
1 0

3
2

n
1

Figura P1.14

1.15. Grafique las siguientes seales:


(a) x2 ( t ) r ( t ) r ( t 1) u ( t 2)
(c) x4 (t ) 2u (t ) (t 1)

(b) x3 ( t ) exp ( t ) u ( t )
(d) x6 ( t ) u ( t ) u ( a t ),

a0

(f) x1 ( t ) x2 t 12

(e) x7 ( t ) u (cos t )
t 1
(g) x1 x3 (2 t )
3 2

1.16. (a) Demuestre que


xp (t )

1
2

x ( t ) x ( t )

es una seal par.


(b) Demuestre que

xi ( t )

1
2

x ( t ) x ( t )

es una seal impar.


1.17. Determine las partes par e impar de las seales siguientes:
(a) x(t) = u(t)
(b) x [ n ] e j ( 0 n 2 )
(c) x [ n ] [ n ]
1.18. Determine y grafique las partes par e impar de las seales mostradas en la Fig. P1.18.
Identifique sus grficas cuidadosamente.

56

x(t)

x(t) = 2t, t < 0

Figura P1.18

1.19. Sea x[n] una secuencia arbitraria con partes par e impar denotadas por xp[n] y xi[n],
respectivamente. Demuestre que

x2 [ n ]

x 2p [ n ]

x [n]
2
i

1.20. Considere el transmisor FM estreo sencillo mostrado en la Fig. P1.20.


(a) Grafique la seal (I + D) e (I D).
(b) Si las salidas de los dos sumadores se aaden, dibuje la forma de onda resultante.
(c) Si la seal I D se invierte y se suma a la seal I + D, dibuje la forma de onda
resultante.
I
Seal de audio
izquierda

Amplificador

Sumador

I+D

Sumador

ID

D
Seal de audio
derecha

Amplificador
D

I(t)

D(t)

0
1

Inversor

0
1

2 3

Figura P1.20

1.21. Para cada una de las seales mostradas en la Fig. P1.21, escriba una expresin en trminos
de funciones escaln y rampa unitarios.

57

x2(t)

x1(t)
1

x3(t)
a

a b a

x5(t)

x4(t)
a

x6(t)

ac

a 0
t

a a+b t

b/2

a
t

a b a

a a+b

Figura P1.21

1.22. Sea x[n] una secuencia arbitraria con partes par e impar denotadas por x p [n] y xi [n] ,
respectivamente. Demuestre que

x2 [ n ]

x 2p [ n ]

x [n]
2
i

1.23. Si la duracin de x(t) se define como el tiempo en el cual x(t) cae a 1/e del valor en el
origen, determine la duracin de las siguientes seales:
(a) x1 ( t ) A exp( t T ) u ( t )
(b) x2 ( t ) x1 (3t )
(c) x3 ( t ) x1 ( t 2)
1.24. Demuestre las siguientes identidades: (a) t ( t ) ( t ) ; (b) ( t ) ( t ) .
1.25. Verifique si alguna de las expresiones siguientes puede usarse como un modelo matemtico
de una funcin delta.
t2
exp
2
2 2
2
1

(a) p1 ( t ) lm
0

(b) p2 ( t ) lm
0

(c) p3 ( t ) lm
0

2
4 t 2
2 2

t 2 2

(d) p4 ( t ) lmexp t
0

(e) p5 ( t ) lm
0

1 sen t

1.26. Simplifique las expresiones siguiente:

sen t
(a) 2
(t )
t 2

j 5
(b) 2
()
4

(c) et cos 3t 60 (t )

sen 2 t 2
(d)
(t 2)
t2 4

4
(e)
( 1)
j 5

sen k
(f)
()

1.27. Evale las integrales siguientes:

(a)

(t 1) ( t 1) dt

(b)

(c)


t sen t t dt
2
4

cos t u ( t 1) ( t ) dt

(d)

exp (5t ) ( t ) dt

(e)

t
t sen 2 ( t ) dt


0
2

(f)

t sen ( t 2) ( t ) dt
0

1.28. La probabilidad de que una variable aleatoria x sea menor que se determina integrando la
funcin de densidad de probabilidades f (x) para obtener

P ( x )

f ( x ) dx

Dado que
f ( x ) 0.2 ( x 2) 0.3 ( x ) 0.2 ( x 1) 0.1 u ( x 3) u ( x 6)
determine
(a) P x 3

59

(b) P x 1.5
(c) P x 4
1.29. Grafique la primera y segunda derivadas de las seales siguientes:
(a) x(t) = t, 0 < t < 1, y x(t) es peridica con perodo 2.
(b) x(t) = u(t) u(t 2) y x(t) es peridica con perodo 4.
0 t 1
t,

(c) x ( t ) 1,
1 t 2
0,
otros valores de t

1.30. D un ejemplo de un sistema lineal variable en el tiempo tal que con una entrada peridica
la salida correspondiente no es peridica.
1.31. Considere el sistema de tiempo continuo cuya relacin de entrada-salida es

y (t )

a x ( t kT )
k

a 1

k 0

Calcule la salida y(t) correspondiente a la entrada x ( t ) exp( jt ) . Es este sistema lineal?


1.32. Si x(t) y y(t) denotan la entrada y la salida de un sistema, respectivamente, diga si los
siguientes sistemas son lineales o no, causales o no, variables en el tiempo o no, tienen
memoria o no. Justifique su respuesta.
(a) y ( t ) tx ( t )

(b) y [ n ] x 2 [ n ]

dx ( t )

(c) y ( t )

dt

(d) y [ n ] n x [ n ]

(e) y ( t )

x ( ) d

(f) y ( t ) x ( t a )

(g) y ( t ) cos x ( t )
(j)

dy ( t )
dt

(h) y ( t ) x ( t ) cos t

ay ( t ) bx ( t ) (k) y ( t )
2

1
T

(i)

dy ( t )
dt

ay ( t ) bx ( t )

t T 2

x ( ) d

(l) y ( t )

t T 2

x ( t ) ( t kT )
s

1.33. Demuestre que un sistema que tiene como respuesta la magnitud de su excitacin es no
lineal, estable, causal y no invertible.
1.34. Para los sistemas descritos por las ecuaciones que se dan a continuacin, donde la entrada
es x(t) y la salida es y(t), determine cules de ellos son invertibles y cules no lo son. Para
los sistemas invertibles, halle la relacin de entrada salida del sistema inverso.
t

(a) y (t )

x() d

(b) y(t ) xn (t ), n un entero

(c) y(t ) x 3t 6

(d) y(t ) cos x(t )

CAPTULO DOS

SISTEMAS LINEALES E INVARIANTES


EN EL TIEMPO
2.1 Introduccin
En este captulo se introducen y discuten varias propiedades bsicas de los sistemas. Dos de ellas, la
linealidad y la invariabilidad en el tiempo, son atributos muy importantes y juegan un papel
fundamental en el anlisis de seales y sistemas porque muchos procesos fsicos poseen estas
propiedades y por ello pueden ser modelados como sistemas lineales e invariantes en el tiempo
(sistemas LIT) y porque esos sistemas LIT pueden ser analizados con bastante detalle. Los objetivos
primordiales de este texto son desarrollar la comprensin de las propiedades y herramientas para
analizar seales y sistemas LIT y proporcionar una introduccin a varias de las aplicaciones
importantes en las que se usan estas herramientas. En este captulo comenzamos este desarrollo
derivando y examinando una representacin fundamental y extremadamente til de los sistemas LIT e
introduciendo una clase importante de tales sistemas.
Una de las principales razones para lo amigable que resulta el anlisis de los sistemas LIT es el hecho
de cumplir con la propiedad de superposicin. Por ello, si la entrada x(t) a un sistema LIT de tiempo
continuo consiste de una combinacin lineal de seales,
x ( t ) a1 x1 ( t ) a2 x2 ( t ) a3 x3 ( t )

(2.1)

entonces, por la propiedad de superposicin, la salida est dada por


y ( t ) a1 y1 ( t ) a2 y2 ( t ) a3 y3 ( t )

(2.2)

donde yk(t) es la respuesta del sistema a la excitacin xk(t), k = 1, 2, . En consecuencia, si podemos


representar la entrada a un sistema LIT en funcin de un conjunto de seales bsicas, podemos
entonces usar la superposicin para calcular la salida del sistema en funcin de sus respuestas a estas
seales bsicas.
Como veremos en la prxima seccin, una de las caractersticas importantes del impulso unitario,
tanto en tiempo continuo como discreto, es que puede usarse para representar seales muy generales.
Este hecho, unido a las propiedades de superposicin e invariabilidad en el tiempo, nos permitir
desarrollar una caracterizacin completa de cualquier sistema LIT en trminos de su respuesta a un
impulso unitario. Esta representacin, a la cual se le refiere como la suma de convolucin en tiempo
discreto y como la integral de convolucin en tiempo continuo, proporciona gran facilidad analtica al
tratar sistemas LIT. Posteriormente se discutir la especificacin de las relaciones de entrada-salida de
sistemas LIT mediante ecuaciones diferenciales y ecuaciones de diferencias.

62

2.2

Sistemas LIT en Tiempo Discreto

2.2.1

La Representacin de Seales en Tiempo Discreto Mediante Impulsos Unitarios

La idea clave para visualizar cmo se puede usar la funcin impulso unitario para construir cualquier
seal de tiempo discreto es considerar a sta como una sucesin de impulsos individuales. Para ver
cmo esta imagen puede convertirse en una representacin matemtica, considere la seal en tiempo
discreto x[n] mostrada en la Fig. 2.1a. En las otras partes de la figura se muestran cinco secuencias de
impulsos unitarios escalados y desplazados en el tiempo, donde el escalamiento de cada impulso es
igual al valor de x[n] en el instante especfico en que ocurre la muestra. Por ejemplo,
x [ 1], n 1
x [ 1] [ n 1}
0, n 1
x [0], n 0
x [0] [ n}
0, n 0
x [1], n 1
x [1] [ n 1}
0, n 1

Por lo tanto, la suma de las tres secuencias en la figura, es decir,


x [ 2] [ n 2] x [ 1] [ n 1] x [ n ] [ n ]

(2.3)

es igual a x[n] para 2 n 0 . En forma ms general, incluyendo impulsos escalados adicionales,


podemos escribir que
x [ n ] x [ 3] [ n 3} x [ 2] [ n 2} x [ 1] [ n 1] x [0] [ n ]

(2.4)

x [1] [ n 1]

x[n]
...

x[2][n + 2]

...
2 1
4

1
0

...

...

2
3

1 0

(b)

(a)

x[1][n + 1]

x[0][n]

...

...
4 3 2 1

1 2

12

...

...

(c)

4 3 2 1 0
(d)

Figura 2.1

1 2

63

Para cualquier valor de n, solamente uno de los trminos en el lado derecho de la Ec. (2.4) es diferente
de cero y la ponderacin en ese trmino es precisamente x[n]. Escribiendo esta suma en una forma ms
compacta, se obtiene

x[n]

x [ k ][ n k ]

(2.5)

sta corresponde a la representacin de una secuencia arbitraria como una combinacin lineal de
impulsos unitarios desplazados, [n k], donde los pesos en esta combinacin son los valores x[k].
Como un ejemplo, considere la secuencia x[n] = u[n], la secuencia escaln unitario. En este caso, u[k]
= 0 para k < 0 y u[k] = 1 para k 0 y la Ec. (2.5) se convierte en

[ n k ]

u[n]

k 0

la cual es idntica a la expresin derivada en la Sec. 1.9.8 [ver la Ec. (1.81)].


La Ec. (2.5) se conoce como la propiedad de seleccin del impulso unitario de tiempo discreto.
Como la secuencia [n k] es diferente de cero solamente cuando n = k, la sumatoria en el lado
derecho de la Ec. (2.5) selecciona a travs de la secuencia de valores x[n] y preserva slo el valor
correspondiente a k n .
2.3 Sistemas LIT Discretos: La Suma de Convolucin
Considere un sistema lineal en tiempo discreto y una entrada arbitraria x[n] a ese sistema. Como vimos
en la Sec. 2.2, cualquier seal arbitraria x[n] puede expresarse como una combinacin lineal de
muestras desplazadas en la forma de la Ec. (2.5), la cual repetimos aqu por conveniencia;

x[n]

x [ k ][ n k ]

Usando la propiedad de superposicin de los sistemas lineales [Ecs. (1.109) y (1.110)], se deduce que
la salida y[n] puede expresarse como una combinacin lineal de las respuestas del sistema cuando la
excitacin est constituida por muestras unitarias desplazadas en el tiempo. Especficamente, si
denotamos por hk[n] la respuesta de un sistema lineal a la muestra unitaria desplazada [n k],
entonces la respuesta del sistema a una entrada arbitraria x[n] puede expresarse como

y [n]

x [k ]h [n ]
k

(2.6)

De acuerdo con la Ec. (2.6), si conocemos la respuesta de un sistema lineal al conjunto de muestras
unitarias desplazadas, entonces podemos construir la respuesta a una entrada arbitraria. Una
interpretacin de la Ec. (2.6) se ilustra en la Fig. 2.2. En la Fig. 2.2a se dibuja una seal particular
x[n], la cual es diferente de cero solamente para n = 1, 0 y 1. Esta seal se aplica a la entrada de un
sistema lineal cuyas respuestas a las seales [n + 1], [n ] y [n 1] se muestran en la Fig. 2.2b.
Como x[n] puede escribirse como una combinacin lineal de [n + 1], [n ] y [n 1], el principio de

64

superposicin nos permite escribir la respuesta a x[n] como una combinacin lineal de las respuestas a
los impulsos individuales desplazados. Los impulsos individuales desplazados y escalonados que
conforman a x[n] se ilustran en el lado izquierdo de la Fig. 2.2c, mientras que las respuestas a estas
seales componentes se dibujan en el lado derecho.
x[n]

...

...
-1
-2

(a)
h 0 [n ]

h 1 [n ]

...

...

...
n

012

...

...

...

x [ 1] [ n 1]

...

h1 [n]

x [ 1] h 1 [ n ]

(b)

...

...

...
n

0
x [0] [n]

x [ 0] h 0 [n ]

...

...

...

x [1] h1 [ n ]

...

...
n

(c)

...
0

...
n

x[n]

...

x [1] [ n 1]

...

y[n]

...
n

...
0

(d)

Figura 2.2

65

Finalmente, en la Fig. 2.2d se muestra la entrada real x[n], la cual es la suma de sus componentes en
la Fig. 2.2c y la salida real y[n], la cual, por superposicin, es la suma de sus componentes en la Fig.
2.2c. Por consiguiente, la respuesta en el tiempo de un sistema lineal es simplemente la superposicin
de las respuestas debidas a cada valor sucesivo de la entrada.
En general, por supuesto, las respuestas hk [ n ] no tienen que estar relacionadas entre ellas para
diferentes valores de k. No obstante, si el sistema tambin es invariable en el tiempo, entonces

hk [ n ] h0 [ n k ]

(2.7)

Especficamente, como [n k] es una versin desplazada de [n], la respuesta hk[n] es una rplica
desplazada en el tiempo de h0[n]. Por conveniencia en la notacin, no se usar el subndice en h0[n] y
se definir la respuesta al impulso (muestra) unitario h[n] como
h [ n ] h0 [ n ]

(2.8)

(es decir, [n] h[n]). Entonces, para un sistema LIT, la Ec. (2.6) se convierte en

y [ n ] x[ n ] h[ n ]

x [k ]h[n k ]

(2.9)

Este ltimo resultado se conoce como la suma de convolucin o suma de superposicin y la


operacin en el lado derecho de la Ec. (2.9) se conoce como la convolucin de las secuencias x[n] y
h[n] y que se representar simblicamente por y [ n ] x [ n ] h [ n ] . Observe que la Ec. (2.9) expresa la
respuesta de un sistema LIT a una entrada arbitraria en funcin de su respuesta al impulso unitario. En
ste y en los prximos captulos se desarrollarn algunas de las implicaciones de esta observacin.

h[n - k]

h[k]

(a)

0 n

k
x[k]

(b)

(c)

Figura 2.3

La interpretacin de la Ec. (2.9) es que la respuesta debida a la entrada x[k] en el instante k es

x k h n k , y sta es sencillamente una versin desplazada y escalada de h[n]. La respuesta real es la

66

superposicin de todas estas respuestas. Para cualquier instante fijo n, la salida y[n] consiste de la suma
para todos los valores de k de los nmeros x k h n k . Como se ilustra en la Fig. 2.3, esta
interpretacin es una forna til de visualizar el clculo de la respuesta usando la suma de convolucin.
Especficamente, considere el clculo de la respuesta para algun valor especfico de n. Observe que h[n
k] se obtuvo mediante una reflexin en torno al origen seguida por un desplazamiento en el tiempo.
En la Fig. 2.3a se muestra h[k] y en la Fig. 2.3b se muestra h[n k] como una funcin de k con n fija.
En la Fig. 2.3c se ilustra x[k]. La salida para este valor especfico de n se calcula entonces ponderando
cada valor de x[k] pora el valor correspondiente de h[n k] y luego sumando estos productos. El
proceso se ilustrar mediante ejemplos.

Ejemplo 1. Consideremos una entrada x[n] y la respuesta al impulso unitario h[n] dadas por
x [ n ] n u [ n ]
h[ n ] u [ n ]

donde 0 < < 1.


h[k] = u[k]

...

...
k

(a)

h[-k]

...

...

h[-1 - k]

...

...

(b)

-1

(c)

h[1 - k]

h[n - k]

...

...
0

(d)

...

...

n>0

k
(e)x [ k ] u [ k ]

h[n - k]
n<0

...

...

...
k

(f)

Figura 2.4

...
0

(g)

67

En la Fig. 2.4 se muestran h[k], h[k] y h[1 k], es decir, h[n k] para n = 0, 1 y h n k para
cualquier valor positivo arbitrario de n. Finalmente, x[k] se ilustra en la Fig. 2.4g. En la figura se
observa que para n < 0 no hay solapamiento entre los puntos que no son iguales a cero en x[k] y h[n
k]. Por ello, para n < 0, x[k]h[n k] = 0 para todos los valores de k y, en consecuencia, y[n] = 0 para
n < 0. Para n 0, x[k]h[n k] est dada por

k ,
x[ k ] h[ n k ]
0,

0k n
otros valores de n

Entonces, para n 0,
n

y [ n]

k 0

El resultado se grafica en la Fig. 2.5.


1
1
n

...
y[ n ]

...

k 0

n
Figura 2.5

Ejemplo 2. Considere ahora las dos secuencias x[n] y h[n] dadas por
1, 0 n 4
x[ n]
0, otros valores de n

n ,
h[ n ]
0,

0n6
otros valores de n

Estas seales se muestran en la Fig. 2.6. Para calcular la convolucin de las dos seales, conviene
considerar cinco intervalos separados para n. Esto se ilustra en la Fig. 2.7.

x[n]

...

h[n]

...
2 1 0 1 2 3 4

...

Figura 2.6

...
2 1 0 1 2 3 4 5 6

68

Intervalo 1. Para n < 0 no hay solapamiento entre las porciones diferentes de cero de x[k] y h [ n k ]
y, por lo tanto, y[n] = 0.
Intervalo 2. Para 0 n 4, el producto x k h n k est dado por

nk ,
x[ k ] h[ n k ]
0,

0k n
otros valores de k

Por lo que en este intervalo, se tiene


n

y[n]

n k

k 0

Intervalo 3. Para n > 4 pero n 6 0 (es decir, 4 < n 6), x[k]h[n k] est dada por

nk ,
x[ k ] h[ n k ]
0,

0k 4
otros valores de k

As que en este intervalo,


4

y[n]

n k

k 0

Intervalo 4. Para n > 6 pero n 6 4 (es decir, para 6 < n 10),

nk ,
x[ k ] h[ n k ]
0,

( n 6) k 4
otros valores de k

de modo que
y [ n]

nk

k n 6

Intervalo 5. Para (n 6) < 4 o, equivalentemente, n > 10, no hay solapamiento entre las porciones
diferentes de cero de x[k] y h[n k] y, por tanto,
y [n] 0

El resultado grfico de la convolucin se muestra en la Fig. 2.7.


y[n]

Figura 2.7

10

69

Estos dos ejemplos ilustran la utilidad de interpretar grficamente el clculo de la suma de


convolucin. En el resto de esta seccin examinaremos varias propiedades importantes de la
convolucin que sern de mucha utilidad en diferentes ocasiones.

Ejemplo 3. Sean
x [ n ] n u [ n ] y h [ n ] n u [ n ]

Entonces

u [ k ]

y[n]

n k

u[n k ]

Como u[k] = 0 para k < 0 y u[n k] = 0 para k > n, podemos escribir la sumatoria como
n

y[n]

n k

k 0

1 k

k 0

Claramente, y[n] = 0 si n < 0.


Para n 0, si = , tenemos
n

y [ n ] n

(1) ( n 1)

k 0

Si , la sumatoria puede escribirse en forma compacta usando la frmula


n2

k n1

a n1 a n2 1
1 a

a 1

(2.10)

Suponiendo que 1 1 , entonces podemos escribir


y [ n ] n

1 ( 1 ) n1
1 1

n1 n1

Como un caso especial de este ejemplo, sea = 1, de modo que x[n] representa a la funcin escaln
unitario. La respuesta al escaln para este sistema se obtiene haciendo = 1 en la ltima expresin para
y[n] y es
y[n]

1 n 1
1

Observe que el Ejemplo 1 es un caso especial de esta relacin.

Resumiendo, se tiene entonces que la suma de convolucin est compuesta de cuatro operaciones
bsicas:

70

1.

Tomar la imagen especular de h[k] sobre el eje vertical a travs del origen para obtener h[k].

2.

Desplazar h[n] en una cantidad igual al valor de n, en donde la secuencia de salida se evala para
calcular h[n k].

3.

Multiplicar la secuencia desplazada h[n k] por la secuencia de entrada x[k].

4.

Sumar la secuencia de valores resultantes para obtener el valor de la convolucin en n.

5.

Los pasos 1 a 4 se repiten conforme n vara de a + para producir toda la salida h[n].

Existe otro algoritmo que se puede usar para evaluar convoluciones discretas (este mtodo es
especialmente til para secuencias finitas). Suponga que se desea determinar la convolucin de x[n] y
h[n], en donde
n

12 , n 0
h[ n ]
n0

0,

x [ n ] 3, 2, 1
Se puede construir una matriz donde h[n] se localice en la parte superior de la matriz y x[n] ocupe la
parte izquierda de la misma, como se indica en la Fig. 2.8. En este caso, la matriz es infinita porque
h[n] es infinita. Los valores dentro de la matriz se obtienen multiplicando los encabezados
correspondientes a la fila y a la columna. Para calcular la convolucin de las dos secuencias, basta con
girar y sumar siguiendo las lneas diagonales punteadas. As, por ejemplo, el primer trmino y[0] es
igual a 3. El segundo trmino, y[1], es igual a 2 + 3/2 = 7/2, que es la suma de los trminos contenidos
entre la primera y la segunda diagonal. Procediendo en esta forma, se obtiene la secuencia de salida

y[n] 3

7 11 11 11
2 4 8 16

211k

En el caso de secuencias bilaterales, el trmino de orden cero correspondiente a la salida se localiza


entre las diagonales en las cuales se encuentra el trmino correspondiente a la interseccin de los
ndices de orden cero para las secuencias de las filas y columnas.
h[n]
1

2
2

1
3

4
4

1
3

x[n]
2

Figura 2.8

71

Ejemplo 4. Se desea determinar la convolucin de la muestra unitaria [n] con una secuencia arbitraria
x[n]. De la Ec. (2.9), el n-simo trmino de la secuencia resultante ser

y[n]

x [ k ][ n k ]

Sin embargo, cada trmino de [n k] es cero excepto cuando n = k. En este caso se tiene que
[0] 1 , por lo que el nico trmino que es diferente de cero en la sumatoria aparece cuando n k y,
en consecuencia,
y [ n] x[ n]

En otras palabras, la convolucin de x[n] y [n] reproduce la secuencia x[n].

Ejemplo 5. Determinar la convolucin de las secuencias x[n] y h[n], donde

an , n 0
x[n]
0, n 0
y

bn ,
h[n]
0,

n0
n0

Solucin: La secuencia resultante, y[n], est dada por

y [n]

x [k ]h[n k ]

x [k ]h[n k ]
k 0

Los lmites en la ltima sumatoria se deben a que x[n] = 0 para n < 0 y h[n] = 0 para k > n. En
consecuencia,

n0
0,
n
y[n]
a k bnk , n 0

k 0

Ejemplo 6. Determinar, empleando la suma de convolucin, la salida del circuito digital de la Fig. 2.9,
correspondiente a la secuencia de entrada x [n] 3 1 3 . Suponga que la ganancia G es igual a 1/2.
Solucin: La ecuacin que describe al sistema se puede obtener igualando la salida del sumador y[n]
con las dos entradas, es decir,
y [ n ] 12 y [ n 1] x [ n ]

(2.11)

72

x[n]

y[n]

+
+

Gy[n 1]

Ganancia
G

Unidad de
retardo

Figura 2.9

La Ec. (2.11) es un ejemplo de una ecuacin en diferencias. Se supone que el sistema est inicialmente
en reposo, de modo que y[1] = 0. Para emplear la suma de convolucin, primero se debe calcular la
funcin de respuesta al impulso h[n]. Un mtodo para obtener dicha respuesta es emplear la ecuacin
en diferencias y determinar la salida en forma iterativa. De la Ec. (2.11) se tiene que
h{0} [0] 12 h[ 1] 1 0 1
h [1] [1] 12 h [0]] 0 12 1

1
2

h [2] [2] 12 h [1] 0 12 12

1
4

h [ n ] [ n ] 12 h [ n 1] 12

La funcin de respuesta al impulso es entonces


n

12 , n 0
h[ n ]
n0

0,

y la salida estar dada por


y [ n ] 3 1 3

,
1 n
2

n0

Una forma sencilla de calcular esta convolucin es emplear la matriz con el mtodo de gira y suma,
como se ilustra en la Fig. 2.10. De esta figura se obtiene la secuencia de salida como

y[n] 3

1
2

13
4

13
8

13
16

13
2n

Este mtodo iterativo tiene la desventaja de que no siempre es posible reconocer la forma del trmino
general. En esos casos, la solucin para h[n] no se obtiene en una forma cerrada, como en este ejemplo,
y puede no ser una solucin aceptable.

73

h[n]

x[n]

1 1
3

1
16

16

12

14

18

116

16

Figura 2.10

2.3.1 Propiedades de la Suma de Convolucin


La Ec. (2.9) define la convolucin de las dos secuencias x[n] y h[n]:

y [n] x[n]h[n ]

x [k ]h [n k ]

(2.12)

La primera propiedad bsica de la suma de convolucin es que es una operacin conmutativa, es


decir,
x[ n] h [ n] h [ n] x[ n]

(2.13)

Esto se demuestra en una forma directa mediante una sustitucin de variables en la Ec. (2.12).
Haciendo m n k , la Ec. (2.12) se convierte en

x[n]h[n]

x [k ]h [n k ] x [n m ]h [m ] h [n ] x [n ]

De acuerdo con esta ltima ecuacin, la salida de un sistema LIT con entrada x[n] y respuesta al
impulso h[n] es idntica a la salida de un sistema LIT con entrada h[n] y respuesta al impulso x[n].
Una segunda propiedad til de la convolucin es que es asociativa, es decir,

x [ n ] h1 [ n ] h2 [ n ] x [ n ] h1 [ n ] h2 [ n ]
Para demostrar esta propiedad, sean x [ n ] h1 [ n ] f1 [ n ] y h1 [ n ] h2 [ n ] f 2 [ n ] . Entonces

f1 [ n ]

x [k ]h [n k ]
1

(2.14)

74

x [ n ] h1 [ n ] h2 [ n ] f1 [ m ] h2 [ n m ]
m

x [ k ] h1 [ m k ] h2 [ n m ]

m k

Sustituyendo r = m k e intercambiando el orden de las sumatorias, tenemos

x [ n ] h1 [ n ] h2 [ n ] x [ k ] h1 [ r ] h2 [ n k r ]
r

y ahora, puesto que

f2 [ n ]

h [ r ]h [n r ]
1

tenemos

f2 [ n k ]

h [ r ]h [n k r ]
1

y, por lo tanto,

x [ n ] h1 [ n ] h2 [ n ] x [ k ] f 2 [ n k ]
k

x [ n ] f 2 [ n ] x [ n ] h1 [ n ] h2 [ n ]

La interpretacin de la propiedad asociativa se indica en las Figs. 2.11a y b. Los sistemas mostrados
en estos diagramas de bloques son sistemas LIT cuyas respuestas al impulso son las indicadas.
En la Fig. 2.11a,

y [ n ] w[ n ] h2 [ n ]

x [ n ] h1 [ n ] h2 [ n ]

x[n]

h1[n]

w[n]

h2[n]

x[n]

y[n]

h[ n ] h1 [ n ] h2 [ n ]

(a)
x[n]

h[ n ] h2 [ n ] h1 [ n ]

y[n]

(b)
y[n]

x[n]

(c)

y[n]
h2[n]

h1[n]
(d)

Figura 2.11

75

En la Fig. 2.11b,
y [ n ] x[ n ] h[ n ]
x [ n ] h1 [ n ] h2 [ n ]

Segn la propiedad asociativa, la interconexin en cascada de los dos sistemas en la Fig. 2.11a es
equivalente al sistema nico en la Fig. 2.11b. Tambin, como una consecuencia de la propiedad
asociativa en conjunto con la propiedad conmutativa, la respuesta completa al escaln de sistemas LIT
en cascada es independiente del orden en el cual los sistemas estn conectados (Figs. 2.11c y d).
Una tercera propiedad de la convolucin es la distributiva con respecto a la suma, es decir,

x [ n ] h1 [ n ] h2 [ n ] x [ n ] h1 [ n ] x [ n ] h2 [ n ]

(2.15)

la cual se verifica fcilmente usando la propiedad de linealidad de la suma.


De nuevo, esta propiedad tiene una interpretacin til. Considere los dos sistemas LIT en paralelo
mostrados en la Fig. 2.12a. Los dos sistemas h 1 [n] y h 2 [n] tienen entradas idnticas y sus salidas se
suman.
Como
y1 [ n ] x [ n ] h1 [ n ]

y
y2 [ n ] x [ n ] h2 [ n ]

la salida del sistema de la Fig. 2.12a es

y [ n ] x [ n ] h1 [ n ] x n ] h2 [ n ]
que corresponde al lado derecho de la Ec. (2.15). La salida del sistema de la Fig. 2.12b es

y [ n ] x [ n ] h1 [ n ] h2 [ n ]
lo que corresponde al lado izquierdo de la Ec. (2.15). En consecuencia, por la propiedad distributiva de
la convolucin, una combinacin en paralelo de sistemas LIT puede ser reemplazada por un solo
sistema LIT cuya respuesta al impulso es la suma de las respuestas al impulso individuales de la
combinacin en paralelo.

h1[n]

y1[n]

x[n]

y[n]

x[n]

h1[n] + h2[n]

h2[n]
y2[n]
(b)

(a)

Figura 2.12

y[n]

76

Ejemplo 7. Considere el sistema mostrado en la Fig. 2.13 con


h1 [ n ] [ n ] a [ n 1]

h2 [ n ] 12 u [ n ]
n

h3 [ n ] a n u [ n ]

h4 [ n ] ( n 1) u [ n ]
h5 [ n ] [ n ] nu [ n 1] [ n 2]
h4[n]

h1[n]

h2[n]

h3[n]

h5[n]

Figura 2.13

De la figura est claro que


h [ n ] h1 [ n ] h2 [ n ] h3 [ n ] { h5 [ n ] h4 [ n ]}

Para evaluar h[n], calculamos primero la convolucin h1 [ n ] h3 [ n ]

h1 [ n ] h3 [ n ] [ n ] a [ n 1] a n u [ n ]
a n u [ n ] a n u [ n 1] [ n ]

Tambin,
h5 [ n ] h4 [ n ] [ n ] nu [ n 1] [ n 2] ( n 1) u [ n ]
[ n ] [ n 2] u [ n ]

de modo que

h [ n ] [ n ] h2 [ n ] [ n ] [ n 2] u [ n ]
h2 [ n ] h2 [ n 2] s2 [ n ]

donde s2 representa la respuesta al escaln correspondiente a h 2 [n]; En consecuencia, tenemos que

h[n]

1 n
2

u[n]

1 n 2
2

u [ n 2]

k 0

Usando la Ec. (2.10), este resultado puede escribirse como

1 k
2

77

h [ n ] 12 n2 u [ n 2] 2 u [ n ]
2.3.2 Respuesta al Escaln
La respuesta al escaln s[n] de un sistema LIT de tiempo discreto cuya respuesta al impulso es h[n]
se obtiene rpidamente a partir de la Ec. (2.9) como

h [ k ]u [ n k ] h [ k ]

s[n] h[n]u[n]

(2.16)

puesto que u[k n] = 0 para k > n. De la Ec. (2.16) tenemos que


h [ n ] s [ n ] s [ n 1]

(2.17)

2.4 Sistemas de Tiempo Continuo: La Integral de Convolucin


En el dominio del tiempo, un sistema lineal se describe en trminos de su respuesta al impulso, la cual
se define como la respuesta del sistema (con cero condiciones iniciales) a una funcin impulso
unitario o funcin delta (t) aplicada a la entrada del sistema. Si el sistema es invariable en el tiempo,
entonces la forma de la respuesta al impulso es la misma sin importar cuando se aplica el impulso
unitario al sistema. As pues, suponiendo que la funcin impulso unitario se aplica en el instante t = 0,
podemos denotar la respuesta al impulso de un sistema LIT por h(t). Suponga que el sistema est
sometido a una excitacin arbitraria x(t). Entonces. igual a como se hizo en la seccin precedente, el
objetivo de sta es obtener una caracterizacin completa de sistemas LIT de tiempo continuo en
funcin de la respuesta al impulso. Por la Ec. (1.51) sabemos que

x (t )

x ( ) (t ) d

(2.18)

La respuesta al impulso h(t) de un sistema LIT de tiempo continuo (representado por ) se define
como la respuesta del sistema cuando la entrada es (t), es decir,

h ( t ) { ( t )}

(2.19)

Puesto que el sistema es lineal, la respuesta y(t) del sistema a una excitacin arbitraria x(t) puede ser
expresada como

y ( t ) x ( t ) x ( ) ( t ) d

x ( ) { ( t )}d

Como el sistema no vara con el tiempo, entonces

(2.20)

78

h ( t ) ( t )

(2.21)

y sustituyendo la Ec. (2.21) en la Ec. (2.20), se obtiene

y (t )

x ( ) h (t ) d

(2.22)

La Ec. (2.22) indica que un sistema LIT de tiempo continuo est completamente caracterizado por su
respuesta al impulso h(t) y se conoce como la integral de convolucin o la integral de superposicin y
es la contraparte de la Ec. (2.9) para la convolucin en tiempo discreto. Tenemos entonces el resultado
fundamental que la salida de cualquier sistema LIT de tiempo continuo es la convolucin de la entrada
x(t) con la respuesta al impulso h(t) del sistema. La respuesta a cualquier entrada x(t) puede calcularse
usando la integral de la Ec. (2.22). La Fig. 2.14 ilustra esta definicin.
La convolucin de dos seales x(t) y h(t) se representar simblicamente por
y (t ) x (t ) h (t )

(t)
x(t)

Sistema
LIT

(2.23)

h(t)
y(t) = x(t) h(t)

Figura 2.14

2.4.1 Propiedades de la Integral de Convolucin


La convolucin en tiempo continuo satisface las mismas propiedades ya discutidas para la
convolucin de tiempo discreto. En particular, es conmutativa, asociativa y distributiva:
Conmutativa:
x (t ) h (t ) h (t ) x (t )

(2.24)

x ( t ) h1 ( t ) h2 ( t ) x ( t ) h1 ( t ) h2 ( t )

(2.25)

x ( t ) h1 ( t ) h2 ( t ) x ( t ) h1 ( t ) x ( t ) h2 ( t )

(2.26)

Asociativa:

Distributiva:

Estas propiedades tienen las mismas implicaciones que las discutidas para la convolucin en tiempo
discreto. Como una consecuencia de la propiedad conmutativa, los papeles de la seal de entrada y de
la respuesta al impulso son intercambiables. Por la propiedad asociativa, una combinacin en cascada
de sistemas LIT puede agruparse en un solo sistema cuya respuesta al impulso es la convolucin de las
respuestas al impulso individuales. Tambin, la respuesta al impulso total no es afectada por el orden
que tienen los sistemas en la conexin en cascada. Finalmente, como un resultado de la propiedad

79

distributiva, una combinacin en paralelo de sistemas LIT es equivalente a un solo sistema cuya
respuesta al impulso es la suma de las respuestas al impulso individuales en la combinacin en
paralelo.
2.4.2 Evaluacin de la Integral de Convolucin
La convolucin es una operacin integral que puede evaluarse analtica, grfica o numricamente.
Aplicando la propiedad de conmutatividad de la convolucin, Ec. (2.24), a la Ec., se obtiene

y (t ) h (t ) x (t )

h ( ) x (t ) d

(2.27)

la cual en algunos casos puede ser ms fcil de evaluar que la Ec. (2.22). De esta ltima ecuacin
observamos que el clculo de la integral de convolucin involucra los cuatro pasos siguientes:
1. La respuesta al impulso h() es invertida en el tiempo (es decir, reflejada con respecto al
origen) para obtener h() y luego desplazada por t para formar h(t ), la cual es una
funcin de con parmetro t.
2. Las seal x() y la respuesta al impulso h(t ) se multiplican para todos los valores de con t
fijo en algn valor.
3. El producto x() h(t ) es integrado en para producir un solo valor de salida y(t).
4. Los pasos 1 a 3 se repiten conforme t vara desde hasta para producir toda la salida y(t).
Tenga siempre presente que al evaluar la integral, x() y h(t ) son funciones de y no de t; t es una
constante con respecto a .

Ejemplo 8. La entrada x(t) y la respuesta al impulso h(t) de un sistema LIT de tiempo continuo estn
dadas por
h ( t ) e t u ( t ), 0

x (t ) u (t )

Calcule la salida y(t).


Solucin: Por la Ec. (2.22)

y (t )

x ( ) h (t ) d

Las funciones x() y h ( t ) se muestran en la Fig. 2.15 para t < 0 y t > 0.


De la figura vemos que para t < 0, x() y h ( t ) no se solapan, mientras que para t > 0, se solapan
desde 0 hasta t . En consecuencia, para t < 0, y(t) = 0. Para t > 0, tenemos
t

y (t ) e
0

( t )

d e

e d

1 e

80

y podemos escribir la salida y(t) como

y (t )

1 e u (t )

(2.28)

x()

h()

t >0

t <0
t

0
h(t )

h(t )

Figura 2.15

Ejemplo 9. Calcule la respuesta y(t) para un sistema LIT de tiempo continuo cuya respuesta al impulso
h(t) y la entrada x(t) estn dadas por
h ( t ) e t u ( t )

x ( t ) et u ( t ), 0

Solucin: Por la Ec. (2.22)

y (t )

x ( ) h (t ) d

As que,

y (t )

u ( ) e ( t ) u ( t ) d

Las funciones x() y h(t ) se muestran en la Fig. 2.16a para t < 0 y t > 0.
De la Fig. 2.16a vemos que para t < 0, x() y h(t ) se solapan desde = hasta = t, mientras que
para t > 0, se solapan desde = hasta = 0. En consecuencia, para t < 0, tenemos
t

y (t )

e e

( t )

d e

e 2 d

1
2

et

y para t > 0,
0

y (t )

( t )

d e

2 t

dt

1
2

e t

81

x()

h(t )

y(t)
t<0

h(t )

(b)

t>0

(a)

Figura 2.16

Combinando las dos ltimas relaciones, y(t) se puede escribir como


y (t )

1
2

Este resultado se muestra en la Fig. 2.16b.


Ejemplo 10. Evale la convolucin y ( t ) x ( t ) h ( t ) , donde x(t) y h(t) se muestran en la Fig. 2.17,
mediante una tcnica analtica.
x(t)

h(t)

Figura 2.17

Solucin: Primero expresamos x(t) y h(t) como funciones del escaln unitario:
x ( t ) u ( t ) u ( t 3)

Entonces, por la Ec. (2.22), tenemos que

y (t )

x ( ) h (t ) d

h ( t ) u ( t ) u ( t 2)

82

[ u ( ) u ( 3) ][ u ( t ) u ( t 2) ] d

u ( ) u (t ) d

u ( ) u (t 2 ) d

u ( 3) u ( t ) d

u ( 3) u ( t 2 ) d

Puesto que
1, 0 t , t 0
u ( ) u (t )
0, otros valores de t
1, 0 t , t 2
u ( ) u (t 2 )
0, otros valores de t
1, 3 t , t 3
u ( 3) u ( t )
0, otros valores de t
1, 3 t 2, t 5
u ( 3) u ( t 2 )
0, otros valores de t

podemos expresar a y(t) como

t
t 2
t
t 2
y ( t ) d u ( t ) d u ( t 2) d u ( t 3) d u ( t 5)
0
0
3
3

t u ( t ) ( t 2) u ( t 2) ( t 3) u ( t 3) ( t 5) u ( t 5)
la cual se grafica en la Fig. 2.18.

y(t)
2u(t)

(t 5)u(t 5)

1
0
1

(t 2)u(t 2)

(t 3)u(t 3)

Figura 2.18

Intente resolver este ejemplo mediante la tcnica grfica usada en el Ejemplo 9.

83

Ejemplo 11, Si x1(t) y x2(t) son ambas seales peridicas con un perodo comn T0, la convolucin de
x1(t) y x2(t) no converge. En este caso, definimos la convolucin peridica de x1(t) y x2(t) como
T0

f ( t ) x1 ( t ) x2 ( t ) x1 ( ) x2 ( t ) d

(2.29)

(a) Demuestre que f (t) es peridica con perodo T0.


(b) Demuestre que
a T0

f (t )

x1 ( ) x2 ( t ) d

(2.30)

para cualquier a.
Solucin:
(a) Como x2(t) es peridica con perodo T0, tenemos que
x2 ( t T0 ) x2 ( t )

Entonces, de la Ec. (2.29) tenemos


T0

f ( t T0 )

x ( ) x (t T
1

) d

T0

x () x (t ) d f (t )
1

As pues, f (t) es peridica con perodo T0.


(b) Puesto que ambas x1(t) y x2(t) son peridicas con el mismo perodo T0, x1()x2(t ) es peridica
con perodo T0 y entonces, igual que toda funcin peridica x(t) con perodo T tiene la propiedad de
que
T

a T

x ( t ) dt x ( t ) dt
y para cualquier a real, se tiene que
a T0

T0

f (t )

x ( ) x (t ) d
1

x1 ( ) x2 ( ) d

2.4.3 Respuesta al Escaln


Otra seal que se usa con frecuencia para describir el comportamiento de sistemas LIT de tiempo
continuo es la funcin escaln unitario. La respuesta al escaln s(t) de un sistema LIT de tiempo

84

continuo (representado por ) se define como la respuesta del sistema cuando la entrada es u(t); es
decir,

s ( t ) u ( t )

(2.31)

En muchas aplicaciones, la respuesta al escaln s(t) tambin es una caracterizacin til del sistema y
por ello es importante relacionarla con la respuesta al impulso. La respuesta al escaln se puede
determinar fcilmente a partir de la integral de convolucin, Ec. (2.22):

s (t ) h (t ) u (t )

h ( ) u (t ) d h ( ) d

(2.32)

As que la respuesta al escaln s(t) puede obtenerse por integracin de la respuesta al impulso h(t).
Diferenciando la Ec. (2.32) con respecto a t, se obtiene
h (t ) s (t )

d s (t )
dt

(2.33)

Esta ecuacin es la contraparte de la Ec. (2.17) en tiempo discreto.


2.5

Propiedades de los Sistemas LIT

En las secciones anteriores se desarrollaron representaciones muy importantes para los sistemas LIT de
tiempo discreto y de tiempo continuo. Esta representacin en tiempo discreto toma la forma de la suma
de convolucin, mientras que su contraparte en tiempo continuo es la integral de convolucin. En esta
seccin usamos la caracterizacin de sistemas LIT en funcin de sus respuestas al impulso para
examinar otras propiedades de los sistemas.
2.5.1 Sistemas LIT Con y Sin Memoria
Recuerde que la salida y(t) de un sistema sin memoria en un instante dado depende solamente de la
entrada y(t) en ese mismo instante. Esta relacin slo puede ser de la forma
y (t ) K x (t )

(2.34)

donde K es una constante (ganancia del sistema). Por ello, la respuesta al impulso correspondiente h(t)
es simplemente
h (t ) K (t )

(2.35)

En consecuencia, si h ( t0 ) 0 para t 0 0, el sistema LIT de tiempo continuo tiene memoria.


Para sistemas LIT de tiempo discreto sin memoria, la relacin equivalente a la Ec. (2.34) es
y[ n] K x[ n]

(2.36)

donde K es una constante (ganancia del sistema) y la respuesta al impulso correspondiente h[n] es
h [ n ] K [ n ]

(2.37)

85

Por lo tanto, si h[n0] 0 para n0 0, el sistema LIT de tiempo discreto tiene memoria.
2.5.2 Causalidad
Como ya se estudi en el Cap. 1, un sistema causal no responde a un evento en su entrada hasta que
este evento efectivamente ocurra; en otras palabras, la respuesta de un sistema causal depende
solamente de los valores presente y pasados de la excitacin. Usando la suma y la integral de
convolucin, podemos relacionar esta propiedad con la propiedad correspondiente de la respuesta al
impulso de un sistema LIT de tiempo discreto o de tiempo continuo. Especficamente, para que un
sistema LIT de tiempo discreto sea causal, su salida y[n] no debe depender de la entrada x[k] para
k n . De la ecuacin para la suma de convolucin

y[n]

x [k ]h[n k ]

se deduce que ste ser el caso si


h [ n ] 0 para n 0

(2.38)

y, aplicando esta condicin, la suma de convolucin se convierte en


n

k 0

x [ k ]h [ n k ] h [ k ] x [ n k ]

y [n]

(2.39)

La segunda sumatoria en el lado derecho de la Ec. (2.39) muestra que los nicos valores de x[n] usados
para evaluar la salida y[n] son aquellos para k n.
Se dice entonces que cualquier secuencia x[n] es causal si
x [ n ] 0,

n0

(2.40)

x [ n ] 0,

n0

(2.41)

y se llama anticausal si

Entonces, cuando la entrada x[n] es causal, la salida y[n] de un sistema LIT de tiempo discreto est
dada por

y [n]

k 0

k 0

h [ k ] x [ n k ] x [ k ]h [ n k ]

(2.42)

Para que un sistema LIT de tiempo continuo sea causal se debe cumplir que la respuesta al impulso
cumpla con la condicin
h ( t ) 0,

t 0

(2.43)

y, en este caso, la integral de convolucin se convierte en

y (t ) h ( ) x (t ) d
0

x ( ) h (t ) d

(2.44)

86

Por la condicin de causalidad, Ec. (2.43), cualquier seal x(t) es causal si


x ( t ) 0,

t 0

(2.45)

x ( t ) 0,

t 0

(2.46)

y se llama anticausal si

Entonces, cuando la entrada x(t) es causal, la salida y(t) de un sistema LIT causal de tiempo continuo
est dada por
t

y (t ) h ( ) x(t ) d x ( ) h (t ) d
0

(2.47)

Ejemplo 11. Considere un sistema LIT de tiempo continuo descrito por

y (t )

t T 2

x ( ) d

(2.48)

t T 2

(a) Determine y dibuje la respuesta al impulso h(t) del sistema.


(b) Es causal este sistema?
Solucin:
(a) La Ec. (2.44) puede escribirse como

y (t )

1
T

t T 2

x ( ) d

1
T

t T 2

x ( ) d

(2.49)

Ahora bien,
t t0

x ( t ) u ( t t0 )

x ( ) u (t t ) d
0

x ( ) d

por lo que la Ec. (2.49) puede expresarse como


y (t )

T 1
T
x (t ) u t x (t ) u t
T
2 T
2
1

x (t )

1 T T
u t u t x (t ) h (t )
T 2 2

y obtenemos
T
T
1
1 T T , t
h (t ) u t u t T
2
2
T 2 2
0, otros valores de t

(2.50)

87

h(t)
1

T/2

T/2

Figura 2.19

(c) De la Ec. (2.50) o de la Fig. 2.19 vemos que h (t ) 0 para t 0 . En consecuencia, el sistema no
es causal.

Ejemplo 12. Considere un sistema LIT de tiempo discreto cuya entrada x[n] y salida y[n] estn
relacionadas por la ecuacin
n

y[n]

k n

x [ k 1]

Determine si el sistema es causal.


Solucin: Por definicin, la respuesta al impulso h[n] del sistema est dada por
n

h[n]

2k n [ k 1]

2 ( n 1 ) [ k 1] 2 ( n 1 )

[ k 1]

Cambiando la variable k + 1 = m, obtenemos

h[n] 2

( n 1 )

n 1

[ m ] 2

( n 1 )

u [ n 1]

En esta ltima ecuacin tenemos que h [ 1] u [0] 1 0 y, por lo tanto, el sistema no es causal.

2.5.3 Estabilidad
Recuerde de la Seccin 1.10.5 que, para nuestros propsitos, un sistema es estable si pequeas
excitaciones producen respuestas que no divergen (no aumentan sin lmite); o dicho de otra forma, el
sistema es estable si toda entrada acotada produce una salida acotada. Para determinar las condiciones
bajo las cuales un sistema LIT de tiempo discreto es estable, considere una excitacin x[n] acotada en
magnitud, es decir,

x[ n]

para toda n

donde es una constante (finita). Si aplicamos esta excitacin a un sistema LIT cuya respuesta al
impulso unitario es h[n], la suma de convolucin nos dar una rplica para la magnitud de la respuesta:

88

y[n]

h [k ] x [n k ]

h[k ]

x [n k

(2.51)

Pero x [ n k ] para todos los valores de k y n, por lo que esta condicin y la Ec. (2.51) implican
que

y[n]

h[k ]

(2.52)

para toda n

De la relacin (2.52) se puede concluir que si la respuesta al impulso es absolutamente sumable, es


decir, si

h [k ]

(2.53)

entonces y[n] est acotada en magnitud y, en consecuencia, el sistema es estable. Por consiguiente, la
Ec. (2.53) es una condicin suficiente para garantizar la estabilidad de un sistema LIT de tiempo
discreto. De hecho, esta condicin tambin es necesaria, ya que si ella no se cumple, existiran entradas
acotadas cuyas salidas no estaran acotadas.
Siguiendo un procedimiento similar para los sistemas LIT de tiempo continuo, se obtiene que el
sistema es estable si su respuesta al impulso, h(t) es absolutamente integrable, vale decir,

h ( t ) dt

(2.54)

Ejemplo 13. Considere un sistema LIT de tiempo discreto cuya respuesta al impulso h[n] est dada por
h [ n ] n u [ n ]

Determine si el sistema es estable.


Solucin: Tenemos que

h [k ]

k u [ k ]

k 0

1
1

Por lo tanto, el sistema es estable si 1 .

Ejemplo 14. Para el acumulador en tiempo discreto, su respuesta al impulso es el escaln unitario u[n].
Este sistema es inestable porque

u[k ]

Es decir, la respuesta al impulso del sumador no es absolutamente sumable. Para el integrador,


contraparte en tiempo continuo del acumulador, se obtiene una relacin similar:

89

u ( ) d d

por lo que ambos sistemas son inestables.

2.5.4 Invertibilidad
Considere un sistema LIT de tiempo continuo cuya respuesta al impulso es h(t). Como ya vimos, este
sistema es invertible solamente si existe un sistema inverso que, al ser conectado en serie (cascada) con
el sistema original, produce una respuesta igual a la entrada al primer sistema. Tambin, si un sistema
LIT es invertible, entonces tiene un inverso. Esta cualidad se ilustra en la Fig. 2.20. En la Fig. 2.20a, el
sistema original tiene una respuesta al impulso h(t) y su respuesta a una entrada x(t) es y(t). El sistema
inverso, con respuesta al impulso h1(t), produce una salida que es igual a w(t) = x(t), lo que indica que
la interconexin en la Fig. 2.20a produce el sistema identidad de la Fig. 2.20b.

x(t)

h(t)

h1 (t)

w(t) = x(t)

(a)

x(t)

Sistema identidad

(t)

y(t)

(b)

Figura 2.20

La respuesta del sistema combinado en la Fig. 2.20a es h ( t ) h1 ( t ) y, por ello, para que h1(t) sea la
respuesta al impulso del sistema inverso debe satisfacer la condicin
h ( t ) h1 ( t ) ( t )

(2.55)

En tiempo discreto, la respuesta al impulso h1[n] del sistema inverso de un sistema LIT cuya respuesta
al impulso es h[n] debe cumplir con una condicin similar a la dada por la Ec. (2.55) y ella es
h [ n ] h1 [ n ] [ n ]

(2.56)

Ejemplo 15. Considere un sistema LIT cuya respuesta al impulso es

h[ n ] u [ n ]
La respuesta de este sistema a una entrada arbitraria x[n] es

y[n]

x [ k ]u [ n k ]

(2.57)

90

Puesto que u [ n k ] 0 para n k 0, esta ltima ecuacin se puede escribir como


n

y [n]

x[k ]

(2.58)

Es decir, el sistema es un sumador. Esta ecuacin se puede escribir como


n 1

y[n]

x [ k ] x n y n 1 x n

x [ n ] y [ n ] y [ n 1]

Este sistema es invertible y su inverso est dado por


y [ n ] x [ n ] x [ n 1]

(2.59)

Tomando x[n] = [n], la respuesta al impulso del sistema inverso es


h1 [ n ] [ n ] [ n 1]

(2.60)

Mediante clculo directo, se obtiene

h [ n ] h1 [ n ] u [ n ] [ n ] [ n 1]
u [ n ] [ n ] u [ n ] [ n 1] u [ n ] u [ n 1]

[ n ]
lo que verifica que los sistemas especificados por las Ecs. (2.57) y (2.59) son inversos.
2.6 Funciones Propias de Sistemas LIT de Tiempo Continuo
Sea y(t) la salida de un sistema LIT de tiempo continuo cuando la entrada es x ( t ) est , donde s es una
variable compleja. Entonces
est y ( t )

(2.61)

en la cual representa la accin del sistema. Puesto que el sistema no vara con el tiempo, tenemos
que

es ( t t0 ) y ( t t0 )

para cualquier t 0 real y arbitrario. Como el sistema es lineal, se tiene tambin que

es ( t t0 ) est est0 est9 est est0 y ( t )

Por lo tanto,
y ( t t0 ) est0 y ( t )

Haciendo t = 0, obtenemos

91

y ( t0 ) y (0) est0

(2.62)

Puesto que t0 es arbitrario, cambiando t0 a t, podemos reescribir la Ec. (2.62) como


y ( t ) y (0) est est

est est

(2.63)

En lenguaje matemtico, una funcin x() que satisface la ecuacin

x ( ) x ( )

(2.64)

se denomina una funcin propia (o funcin caracterstica) del operador , y la constante se llama un
valor propio (o valor caracterstico) correspondiente a la funcin propia x().
Si ahora hacemos x ( t ) est en la integral de convolucin, hallamos que

y ( t ) e

st

h ( ) e

s ( t )

d h ( ) e s d e st

H ( s ) est est

(2.65)

donde

H (s)

h ( ) e

(2.66)

Es decir, el valor propio de un sistema LIT de tiempo continuo asociado con la funcin propia e st est
dado por H(s), la cual es una constante compleja cuyo valor es determinado por el valor de s dado por
la Ec. (2.66). Observe en la Ec. (2.64) que y(0) = H(s).
2.7 Funciones Propias de Sistemas LIT de Tiempo Discreto
Para sistemas LIT de tiempo discreto representados por , las funciones propias son las exponenciales
complejas z n , donde z es una variable compleja. Es decir,
z n z n

(2.67)

Siguiendo un procedimiento similar al de la Seccin 2.6 para sistemas LIT de tiempo continuo, se
determina que, para una entrada x [ n ] z n , la respuesta y[n] est dada por
y[ n] H ( z ) zn zn

(2.68)

donde

H ( z)

h[k ] z

(2.69)

As que los valores propios de un sistema LIT de tiempo discreto asociados con las funciones propias
z n estn dados por H(z), la cual es una constante compleja cuyo valor lo determina el valor de z usando
la Ec. (2.69).

92

Ejemplo 16. Considere el sistema LIT de tiempo continuo descrito por la relacin
t T 2

y (t )

x ( ) d

(2.70)

t T 2

Se quiere determinar el valor propio del sistema correspondiente a la funcin propia e st .


Solucin: Sustituyendo el valor x ( ) es en la Ec. (2.70), se obtiene

y (t )

t T 2

e s d

t T 2

e st
sT

sT 2

e sT 2

est
y el valor propio correspondiente a e st es

e
sT
1

sT 2

e sT 2

2.8 Sistemas Descritos por Ecuaciones Diferenciales


Considere el circuito RC mostrado en la Fig. 2.21. Este circuito puede considerarse como un sistema de
tiempo continuo cuya entrada x(t) es igual a la fuente de corriente i(t) y cuya salida y(t) es igual al
voltaje en el capacitor.

+
x (t ) i (t )

y (t ) vC (t )

Figura 2.21

La relacin entre la entrada y la salida es descrita por la ecuacin diferencial

dy ( t )
dt

1
R

y (t ) x (t )

(2.71)

En general, la respuesta de muchos sistemas fsicos puede describirse mediante una ecuacin
diferencial. En esta seccin solamente trataremos sistemas lineales descritos por ecuaciones

93

diferenciales con coeficientes constantes, su realizacin o simulacin usando sumadores,


multiplicadores e integradores y demostraremos cmo se determina la respuesta al impulso de sistemas
LIT.
2.8.1

Ecuaciones Diferenciales Lineales con Coeficientes Constantes

La forma general de una ecuacin diferencial lineal de coeficientes constantes de N-simo orden est
dada por

d k y (t )

dt k

k 0

d k x (t )

k 0

dt k

(2.72)

donde los coeficientes ai, i = 1, 2, , N y bj, j = 1, 2, , , M, son constantes reales. El orden N se


refiere a la mayor derivada de y(t) en la Ec. (2.72). Estas ecuaciones juegan un papel primordial en la
descripcin de las relaciones de entrada-salida de una amplia variedad de sistemas fsicos. Por ejemplo,
en el circuito RC de la Fig. 2.21, la entrada y la salida estn relacionadas por una ecuacin diferencial
de primer orden con coeficientes constantes, Ec. (2.71).
La solucin general de la Ec. (2.72) para una entrada especfica x(t) est dada por
y ( t ) y p ( t ) yh ( t )

(2.73)

donde yp(t) es una solucin particular que satisface la Ec. (2.71) y yh(t) es una solucin homognea (o
solucin complementaria) que satisface la ecuacin diferencial homognea
N

ak

k 0

d k yh ( t )
dt k

(2.74)

La forma exacta de y(t) se determina mediante los valores de N condiciones auxiliares especificadas en
algn punto en el tiempo, digamos, t0:
y ( t0 ), y ( t0 ),, y ( N 1) ( t0 )

(2.75)

Ejemplo 17. Como un ejemplo, considrese la ecuacin diferencial de primer orden

d y (t )
dt

a y ( t ) bx ( t )

(2.76)

donde a y b son constantes arbitrarias y x(t) es una funcin continua de t. Multiplicando ambos lados
de la ecuacin por e at , se tiene que
e at

d y (t )

e at

d y (t )

dt

ae at y ( t ) be at x ( t )

o tambin

dt

a at y ( t ) be at x ( t )

94

la cual puede escribirse en la forma

e at y ( t ) b e at x ( t )
dt
e integrando desde t 0 hasta t,
t

e at y ( t )

t
t0

b e a x ( ) d
t0

e at y ( t ) e at0 y ( t0 ) b e a x ( ) d
t0

Despejando a y(t) en la ecuacin anterior se obtiene

y (t ) e

a t t0

y ( t0 ) be

a t

x ( ) d

(2.77)

t0

y cuando t 0 = 0,
t

y ( t ) e y (0) b e
at

a t

x ( ) d

(2.78)

En la Ec. (2.77), la parte correspondiente a la solucin homognea [x(t) = 0] es


yh ( t ) ea ( t t0 ) y ( t0 )

2.8.2

Linealidad

El sistema especificado por la Ec. (2.72) es lineal solamente si todas las condiciones auxiliares son
idnticamente iguales a cero (por qu?). Si no lo son, entonces la respuesta y(t) de un sistema puede
expresarse como
y ( t ) yenc ( t ) yesc ( t )

(2.79)

donde yenc(t) se denomina la respuesta de entrada cero y es la respuesta a las condiciones auxiliares;
yesc(t) se llama la respuesta de estado cero, y es la respuesta del sistema cuando las condiciones
iniciales son iguales a cero. Esto se ilustra en la Fig. 2.22 (ver Sec. 1.10.7).
yenc(t)
x(t)

Sistema
lineal

yesc(t)

Figura 2.22

y(t)

95

2.8.3

Causalidad

Para que el sistema lineal descrito por la Ec. (2.72) sea causal debemos suponer que el sistema est
inicialmente en reposo. Es decir, si x(t) = 0 para t t0, entonces suponemos que y(t) = 0 para t t0 . En
consecuencia, la respuesta para t > t0 puede determinarse a partir de la Ec. (2.72) con las condiciones
iniciales

y (t )

t t0

d y (t )
dt

t t0

d N 1 y ( t )
dt n1

0
t t0

Claramente, si el sistema est en reposo inicial, yenc(t) = 0.


2.8.4

Invariabilidad en el Tiempo

Para que un sistema lineal sea causal, el estado de reposo inicial tambin implica que el sistema no
vara con el tiempo. Esto se ilustrar mejor mediante un ejemplo.

Ejemplo 18. Considere el sistema cuya entrada x(t) y salida y(t) estn relacionadas por la ecuacin
diferencial
dy ( t )
dt

a y (t ) x (t )

donde a es una constante y y(0) = 0. Sea y1(t) la respuesta a una entrada x1(t) y x1(t) = 0 para t 0 .
Entonces

dy1 ( t )
dt

a y1 ( t ) x1 ( t )

(2.80)

y1 (0) 0
Ahora, sea y2(t) la respuesta a la entrada desplazada x2 ( t ) x1 ( t ) . Puesto que x1(t) = 0 para t 0 ,
tenemos que
x2 ( t ) 0,

Entonces y2(t) debe satisfacer la relacin


dy2 ( t )
dt

a y2 ( t ) x2 ( t )

(2.81)

y2 ( ) 0

(2.82)

96

Ahora bien, de la Ec. (2.80) se tiene que

dy1 ( t )

a y1 ( t ) x1 ( t ) x2 ( )

dt

Si hacemos y2 ( t ) y1 ( t ) , entonces, puesto que y1 (0) 0 , se obtiene


y2 ( ) y1 ( t ) y1 (0) 0

Por lo tanto, se satisfacen las Ecs. (2.81) y (2.82) y se concluye que el sistema no vara con el tiempo.

2.8.5

Respuesta al Impulso

De la discusin sobre la integral de convolucin se sabe que si conocemos la respuesta de un sistema a


un impulso unitario, podemos determinar la respuesta del sistema a una entrada arbitraria. La respuesta
al impulso de un sistema puede determinarse a partir de la ecuacin diferencial que describe al sistema,
Ec. (2.72). Ella, h(t), se defini como la respuesta y(t) cuando x(t) = (t) y y ( t ) 0, t 0 , es
decir, la respuesta al impulso satisface la ecuacin diferencial

d k h (t )

k 0

dt k

d k (t )

k 0

(2.83)

dt k

con el sistema inicialmente en reposo.


Ahora estudiaremos un mtodo para determinar la respuesta h(t) de un sistema LIT de tiempo
continuo. Para ilustrar una forma de determinar la respuesta al impulso, considere un sistema definido
por la ecuacin diferencial

L { y ( t )} x ( t )

(2.84)

donde L es el operador definido por

L an

dn
dt n

an1

d n1
dt n1

a1

d
dt

a0

(2.85)

La respuesta s(t) al escaln unitario de la Ec. (2.83) se puede calcular a partir de la ecuacin
1, t 0
0, t 0

L { s ( t )}

con las condiciones iniciales apropiadas. Entonces, la respuesta al impulso, h(t), se puede obtener a
partir de
h (t )

ds ( t )
dt

Un mtodo ms poderoso se basa en el conocimiento de las soluciones homogneas de la Ec. (2.84).


Para desarrollar este mtodo, supngase que se tiene un sistema de segundo orden de la forma

97

L { y ( t )} ( D2 a1 D a0 ){ y ( t )} x ( t ),

d
dt

(2.86)

Si se supone que el sistema est inicialmente en reposo, las condiciones iniciales sern
y (0) 0
y (0) 0

(2.87)

Entonces, si la funcin de respuesta al impulso es h(t), la salida y(t) estar dada por la integral de
convolucin; es decir,
t

y (t ) x ( ) h (t ) d

(2.88)

Las Ecs. (2.86) y (2.88) representan dos mtodos de clculo de la respuesta de salida y(t). Empleando
ambas ecuaciones como punto de partida, considrense las condiciones impuestas por las Ecs. (2.86) y
(2.87) a la funcin de respuesta al impulso. Diferenciando la Ec. (2.88) con respecto a t, se tiene que
t

y ( t ) h ( t ) x ( )

h ( t ) x ( ) d
0

h (0) x ( t ) h ( t ) x ( ) d

(2.89)

Las condiciones en la Ec. (2.87) requieren que y'(0) = 0, lo que implica que h(0) = 0 en la Ec. (2.89).
Diferenciando de nuevo, se obtiene
t

y ( t ) h (0) x ( t ) h ( t ) x ( ) d

(2.90)

Las Ecs. (2.88), (2.89) y (2.90) son expresiones para y(t), y ( t ) y y ( t ) . Consideremos ahora el
resultado de la suma y ( t ) a1 y ( t ) a0 y ( t ) . ste es
t

h (0) x ( t ) h ( t ) x ( ) d a1 h ( t ) x ( ) d a0 h ( t ) x ( ) d
0

(2.91)

h (0) x ( t )

h ( t ) a h ( t ) a h ( t ) x ( ) d
1

Se observa que si
(a)

h (0) 1

(2.92)

(b)

[ h ( t ) a h ( t ) a
1

h (t ) ] x ( ) d 0

(2.93)

98

entonces la Ec. (2.88) ser una solucin de la Ec. (2.86). La Ec. (2.93) implica que el integrando del
primer miembro en la integral del lado derecho de la Ec. (2.91) es igual a cero, puesto que si x (t ) 0 se
obtiene la solucin trivial. Si x ( t ) 0 , entonces el trmino entre corchetes es cero; es decir,
h ( t ) a1 h ( t ) a0 h ( t ) 0

o
h ( t ) a1 h ( t ) a0 h ( t ) 0

(2.94)

ya que el sistema no vara con el tiempo.


La Ec. (2.94) es la ecuacin diferencial homognea original. As que la respuesta al impulso puede
obtenerse calculando las soluciones homogneas de la ecuacin diferencial original sujeta a las
condiciones iniciales

h (0) 0
h (0) 1

(2.95)

Ejemplo 19. Considere el sistema representado por la ecuacin diferencial


y ( t ) y ( t ) x ( t )

(2.96)

La solucin homognea de (2.96) es


h ( t ) ( c1 sen t c2 cos t ) u ( t )

con condiciones iniciales


h (0) 0,

h (0) 1

Por lo tanto,
h (0) 0 c2
h (0) 1 c1

y, en consecuencia, la respuesta al impulso del sistema modelado por la Ec. (2.96) es


h ( t ) sen t u ( t )

Para verificar este resultado se sustituye la Ec. (2.97) en la Ec. (2.96) con
h ( t ) cos t u ( t ) sen t ( t ) cos t u ( t )
h ( t ) sen t u ( t ) cos t ( t ) sen t u ( t ) ( t )

para obtener
h ( t ) h ( t ) sen t u ( t ) ( t ) sen t u ( t ) ( t )

Ejemplo 20. Considere un sistema modelado por la ecuacin diferencial


y ( t ) 2 y ( t ) 2 y ( t ) x ( t )

(2.97)

99

La solucin homognea de esta ecuacin es


h ( t ) c1 e t sen t c2 e t cos t u ( t )

Las constantes c1 y c2 se obtienen aplicando las condiciones iniciales:


h (0) 0 c2
h (0) 1 c1

y la respuesta al impulso est dada por


h ( t ) e t sen t u ( t )

Este mtodo se puede generalizar de manera directa para sistemas de orden n. Para el caso general, la
ecuacin que describe el sistema es
L { y ( t )} Dn an1 Dn1 a1 D a0 [ y ( t ) ] x ( t )

(2.98)

sujeta a las condiciones iniciales dadas por


y (0) y (0) y ( n1) (0) 0

La respuesta se expresa como


t

y (t ) h (t ) x ( ) d

(2.99)

Igualando a cero las derivadas sucesivas de y(t) en la Ec. (2.99), se obtiene


h (0) h (0) h( n2 ) (0) 0

(2.100)

Para la derivada n-sima, obtenemos


t

(n)

(t ) h

( n 1 )

(0) x ( t ) h ( n ) ( t ) x ( ) d
0

Usando el mismo argumento empleado para el caso de segundo orden ya analizado, se encuentra que la
funcin de respuesta al impulso para el sistema de la Ec. (2.98) debe satisfacer la ecuacin homognea
L { h ( t )} 0

sujeta a las condiciones iniciales h (0) h (0) h( n2 ) (0) 0 y h( n1) (0) 1 .

Ejemplo 21. Considere un sistema modelado por la ecuacin diferencial


L { y ( t )} ( D2 1) ( D2 1)[ y ( t ) ] x ( t )

La solucin de la ecuacin homognea es


h ( t ) c1 et c2 e t c3 t et c4 t e t

u (t )

100

Aplicando las condiciones iniciales se obtiene

h (0) 0 c1 c2
h (0) 0 c1 c2 c3 c4
h (0) 0 c1 c2 2 c3 2 c4
h (0) 1 c1 c2 3 c3 3 c4
De estas ecuaciones se obtiene que c1 12 , c2 12 , c3 12 , c4 12 y la respuesta al impulso es

h (t )

e
2

e t t et t e t

u (t )

Para completar esta seccin, se extender el mtodo a sistemas excitados por una seal de la forma
LD { x ( t )} en lugar de x(t) y donde LD es un operador diferencial de la forma dada por la Ec. (2.84) y
de menor orden que L. Sea un sistema descrito por una ecuacin de la forma
L { y ( t )} LD { x ( t )}

(2.101)

Si el sistema L { y ( t )} x ( t ) tiene una respuesta al impulso h ( t ) , la respuesta del sistema modelado


por L { y ( t )} x ( t ) est dada por
t

y ( t ) h ( t ) x ( ) d

(2.102)

La respuesta al impulso h ( t ) se calcula empleando los mtodos descritos anteriormente en esta


seccin. Sin embargo, el sistema est siendo excitado ahora no por x(t), sino por LD { x ( t )} . Suponga
que aplicamos el operador LD a ambos lados de la ecuacin
L { y ( t )} x ( t )

Se obtiene entonces que


LD {L { y ( t )}} LD { x ( t )}

(2.103)

Empleando la propiedad conmutativa de los operadores diferenciales LIT, la Ec. (2.103) se puede
escribir como
L {LD { y ( t )}} LD { x ( t )}

(2.104)

Comparando las Ecs. (2.101) y (2.103) vemos que LD { y ( t )} y ( t ) . Se tiene entonces que la salida
del sistema original es simplemente el operador LD operando sobre y ( t ) As que la respuesta al
impulso h(t) para el sistema descrito por la Ec. (2.101) debe ser

h ( t ) LD { h ( t )}

(2.105)

101

Ejemplo 22. Considere el circuito de la Fig. 2.23 en el que se utiliza una funcin x(t) cualquiera como
excitacin.

x (t )

y (t )

1F

Figura 2.23

La ecuacin diferencial que relaciona la salida con la entrada es

2 D 2 { y ( t )} ( D 1){ x ( t )}

(2.106)

El primer paso es determinar la respuesta al impulso h ( t ) del sistema

2 D 2 { h ( t )} x ( t )

Este problema ya se resolvi en el Ejemplo 20 y su respuesta al impulso es


h ( t ) e t sen t u ( t )

Entonces, la respuesta al impulso de la Ec. (2.106) est dada por


h ( t ) ( D 1){ h ( t )} ( D 1){ e t sen t u ( t )}
e t sen t u ( t ) e t cos t u ( t ) e t sen t ( t ) e t sen t u ( t )
e t cos t u ( t )

y la salida y(t) ser


t

y ( t ) e ( t ) cos( t ) x ( ) d ,

t0

Si, por ejemplo, x(t) = u(t), la salida ser


t

y (t ) e
0

( t )

t
t
1

2 (1 e sen t e cos t ), t 0
cos( t ) d
t0

0,

2.9 Sistemas Descritos por Ecuaciones en Diferencias


Anteriormente vimos que un sistema de tiempo continuo puede caracterizarse en funcin de una
ecuacin diferencial que relaciona la salida y sus derivadas con la entrada y sus derivadas. La

102

contraparte en tiempo discreto de esta caracterizacin es la ecuacin en diferencias, la cual, para


sistemas lineales e invariables en el tiempo, toma la forma
N

ak y [ n k ]

k 0

b x [ n k ],
k

n0

(2.107)

k 0

donde ak y bk son constantes conocidas. El orden N se refiere al mayor retardo de y[n] en la Ec. (2.107).
En una forma anloga al caso en tiempo continuo, la solucin de la Ec. (2.107) y todas las propiedades
de los sistemas, tales como linealidad, causalidad e invariabilidad en el tiempo, pueden desarrollarse
siguiendo un mtodo de discusin similar al usado para las ecuaciones diferenciales.
Definiendo el operador
Dk y [ n ] y [ n k ]

(2.108)

podemos escribir la Ec. (2.107) en notacin operacional como


N

ak D k y [ n ]

k 0

b D x [ n ]
k

(2.109)

k 0

Una forma alterna de la ecuacin en diferencias, Ec. (2.107), se da algunas veces como
N

ak y [ n k }

k 0

b x [ n k ],
k

n0

(2.110)

k 0

En esta forma, si el sistema es causal, debemos tener M N.


La solucin a cualquiera de las Ecs. (2.105) o (2.110) puede determinarse, en analoga con una
ecuacin diferencial, como la suma de dos componentes: (a) la solucin homognea, que depende de
las condiciones iniciales que se suponen conocidas, y (b) la solucin particular, la cual depende de la
entrada.
Antes de explorar este enfoque para determinar la solucin a la Ec. (2.107), consideremos un mtodo
alterno escribiendo de nuevo la Ec. (2.107) como
y [n]

1 M
bk x [ n k ]
a0 k 0

a y [ n k ]
k

(2.111)

k 1

En esta ecuacin, los valores x[n k] son conocidos. Si tambin se conocen los valores y[n k],
entonces y[n] puede determinarse. Haciendo n = 0 en la Ec. (2.111) da
1 M
y (0) bk x [ k ]
a0 k 0

ak y [ k ]
k 1

(2.112)

Las cantidades y[k], para k = 1, 2, , N, representan las condiciones iniciales para la ecuacin en
diferencias y por tanto supuestas conocidas. Entonces, como todos los trminos en el lado derecho son
conocidos, podemos determinar y[0].
Ahora hacemos n = 1 en la Ec. (2.111) para obtener

103

y (1)

1
a0

bk x [1 k ]

k 0

ak y [1 k ]

k 1

y usamos el valor de y[0] determinado anteriormente para resolver por valores sucesivos de n y obtener
y[n] por iteracin.
Usando un argumento similar al anterior, se puede ver que las condiciones necesarias para resolver la
Ec. (2.111) son las condiciones iniciales y[0], y[1], , y[N 1]. Comenzando con estas condiciones
iniciales, la Ec. (2.111) puede resolverse iterativamente en igual forma. sta es la formulacin
recursiva y la Ec. (2.111) se conoce como una ecuacin recursiva ya que ella especifica un
procedimiento recursivo para determinar la salida en funcin de la entrada y salidas previas.

Ejemplo 23. Considere la ecuacin en diferencias

y [ n ] 34 y [ n 1] 81 y [ n 2] 12

con condiciones iniciales y[1] = 1 y y[2] = 0.


Entonces

y [ n ] 34 y [ n 1] 81 y [ n 2] 12

de modo que
y [0] 34 y [ 1] 18 y [ 2] 1
y [1] 34 y [0] 81 y [ 1]

y [2] 34 y [1] 81 y [0]

7
4

27
16

83
64

. . . . . . . . . . .

En el caso especial cuando N = 0, de la Ec. (2.111) tenemos


y[n]

1 M
bk x [ n k ]
a0 k 0

la cual es una ecuacin no-recursiva ya que no se requieren los valores previos de la salida para
calcular la salida presente. Por ello, en este caso, no se necesitan condiciones auxiliares para determinar
y[n].
Aun cuando el procedimiento iterativo descrito anteriormente puede usarse para obtener y[n] para
varios valores de n, l, en general, el mtodo no produce una expresin analtica para evaluar y[n] para
cualquier n arbitraria. Ahora consideraremos la solucin analtica de la ecuacin en diferencias
determinando las soluciones homognea y particular de la Ec. (2.107)

104

2.9.1

Solucin Homognea de la Ecuacin en Diferencias

La ecuacin homognea correspondiente a la Ec. (2.107) est dada por


N

a y [n k ] 0
k

(2.113)

k 0

En analoga con nuestra discusin del caso en tiempo continuo, suponemos que la solucin a esta
ecuacin viene dada por una funcin exponencial de la forma
yh [ n ] A n

Sustituyendo esta relacin en la ecuacin en diferencias, se obtiene


N

a A

n k

k 0

Entonces, cualquier solucin homognea debe satisfacer la ecuacin algebraica


N

a
k

(2.114)

k 0

La Ec. (2.114) es la ecuacin caracterstica para la ecuacin en diferencias y los valores de que
satisfacen esta ecuacin son los valores caractersticos. Es evidente que hay N races caractersticas
1 , 2 , , N , y que estas races pueden ser distintas o no. Si las races son distintas, las soluciones
caractersticas correspondientes son independientes y podemos obtener la solucin homognea yh[n]
como una combinacin lineal de trminos del tipo in , es decir,
yh [ n ] A1 1n A2 2n AN nN

(2.115)

Si cualesquiera races son repetidas, entonces generamos N soluciones independientes multiplicando la


solucin caracterstica correspondiente por la potencia apropiada de n. Por ejemplo, si 1 tiene una
multiplicidad de P1, mientras que las otras N P1 races son distintas, suponemos una solucin
homognea de la forma

yh [ n ] A1 1n A2 n 1n AP1 n P1 1 1n AP1 1 nP1 1 AN nN

Ejemplo 24. Considere la ecuacin


y [ n ] 56 y [ n 1] 16 y [ n 2] 0

con las condiciones iniciales


y[1] = 2,
La ecuacin caracterstica es

y[2] = 0

(2.116)

105

1 56 1 16 2 0
o
2 56 16 0

la cual puede factorizarse como

12 13 0
y las races caractersticas son
1
,
2

1
3

Puesto que estas races son distintas, la solucin homognea es de la forma


n

1
1
yh [ n ] A1 A2
2
3

La sustitucin de las condiciones iniciales da entonces las siguientes ecuaciones para las constantes
incgnitas A1, y A2:
2 A1 3 A2 2
4 A1 9 A2 0

cuya solucin es
A1 3,

A2

4
3

y la solucin homognea de la ecuacin es igual a


n

1 4 1
yh [ n ] 3
2 3 3

Ejemplo 25. Considere la ecuacin


y [ n ] 54 y [ n 1] 12 y [ n 2] 161 y [ n 3] 0

con las condiciones iniciales


y [ 1] 6,

y [ 2] 6,

y [ 3] 2

La ecuacin caracterstica es
1 54 1 12 2 161 3 0

y sus races son

1
1 ,
2

1
2 ,
2

1
4

106

Aqu se tiene una raz repetida. Por consiguiente, escribimos la solucin homognea como
n

1
1
1
yh [ n ] A1 A2 n A3
2
2
4

Sustituyendo las condiciones iniciales y resolviendo las ecuaciones resultantes, obtenemos

9
A1 ,
2

5
A2 ,
4

A3

1
8

y la solucin homognea es
n

91 5 1 11
yh [ n ] n
2 2 4 2 8 4
2.9.2

La Solucin Particular

Ahora consideraremos la determinacin de la solucin particular para la ecuacin de diferencias


N

a y [ n k ] b x [ n k ]
k

k 0

(2.117)

k 0

Observamos que el lado derecho de esta ecuacin es la suma ponderada de la entrada x[n] y sus
versiones retardadas. Por lo tanto, podemos obtener yp[n], la solucin particular de la Ec. (2.117),
determinando primero la solucin particular de la ecuacin
N

a y [ n k ] x [ n ]
k

(2.118)

k 0

El uso del principio de superposicin nos permite entonces escribir


N

yp [n]

b y [ n k ]
k

(2.119)

k 0

Para hallar ~y [n] , suponemos que ella es una combinacin lineal de x[n] y sus versiones retardadas x[n
1], x[n 2], etc. Por ejemplo, si x[n] es una constante, tambin lo es x[n k] para cualquier k. Por
consiguiente, y [ n ] tambin es una constante. Similarmente, si x[n] es una funcin exponencial de la
forma n , y [ n ] es tambin una exponencial de la misma forma. Si
x [ n ] sen 0 n

entonces
x [ n k ] sen 0 ( n k ) cos 0 k sen 0 n sen 0 k cos 0 n

y, como corresponde, tenemos


y [ n ] A sen 0 n B cos 0 n

107

Se obtiene la misma forma para y [ n ] cuando


x [ n ] cos 0 n

Las constantes incgnitas en la solucin supuesta se pueden determinar sustituyendo en la ecuacin en


diferencias e igualando los trminos semejantes.

Ejemplo 26. Considere la ecuacin en diferencias

1
n
y [ n ] 34 y [ n 1] y [ n 2] 2sen
8
2
con condiciones iniciales
y[1] = 2

y y[2] = 4

De acuerdo con el procedimiento indicado, suponemos entonces que la solucin particular es de la


forma

y p [ n ] A sen

n
2

B cos

Entonces

( n 1)

y p [ n 1] A sen

B cos

( n 1)
2

Usando identidades trigonomtricas se puede verificar fcilmente que


sen

( n 1)
2

cos

n
2

y cos

( n 1)

sen

n
2

de modo que
y p [ n 1] A cos

n
2

B sen

n
2

En forma similar se puede demostrar que y p [ n 2] es


y p [ n 2] A sen

n
2

B cos

n
2

Sustituyendo ahora en la ecuacin en diferencias da

3
4

B 81 A sen

n
2

B 43 A 81 B cos

n
2

2sen

n
2

Igualando los coeficientes de los trminos semejantes, se obtienen los valores de las constantes A y B:
A

y la solucin particular es

112
85

96
85

108

yp [ n]

112
85

sen

n 96
n
cos
2 85
2

Para determinar la solucin homognea, escribimos la ecuacin caracterstica para la ecuacin en


diferencias como
1 34 1 18 2 0

cuyas races caractersticas son


1
1 ,
4

1
2

y la solucin homognea est dada por

yh [ n ] A1 14 A2 12
n

de manera que la solucin completa est dada por

n 96
n
1
1 112
y [ n ] A1 A2
sen
cos
85
2 85
2
4
2
n

Ahora podemos sustituir las condiciones iniciales dadas para resolver por las constantes A1 y A2 y se
obtiene

A1

8
17

A2

13
5

de modo que

8 1 13 1 112
n 96
n
y[n]
sen
cos
17 4
5 2
85
2 85
2
n

Ejemplo 27. Considere el sistema descrito por la ecuacin en diferencias


y [ n ] a y [ n 1] Kbn u [ n ]

donde a, b y K son constantes y y [ 1] y1 .


La solucin que satisface la ecuacin homognea
yh [ n ] a yh [ n 1] 0

es dada por

yh [ n ] Aa n
Para determinar la solucin particular, suponemos que

y p [ n ] Bbn ,
y sustituyendo sta en la ecuacin original, se obtiene

n0

109

Bbn a Bbn1 K bn

a partir de la cual se obtiene que

Kb
ba

y
yp [ n]

K
ba

bn 1

Combinando ahora yh[n] y yp[n], da

y [ n ] Aa n

K
ba

b n1 ,

n0

Para determinar A, aplicamos la condicin dada:


y [ 1] y1 Aa 1

K
ba

de donde

A ay1 K

a
ba

y la solucin buscada es
y [ n ] y1 a

n 1

bn1 a n1
ba

n0

Para n < 0, tenemos x[n] = 0 y, en este caso,


y [ n ] Aa n

Aplicando la condicin y [ 1] y1 , se obtiene que A y1 a y


y [ n ] y1 a n1

n0

y la solucin completa para toda n es


y [ n ] y1 a n1 K

2.9.3

bn 1 a n 1
ba

u[n]

Determinacin de la Respuesta al Impulso

Concluimos esta seccin considerando la determinacin de la respuesta al impulso de sistemas


descritos por la ecuacin en diferencias de la Ec. (2.107). Recuerde que la respuesta al impulso es la
respuesta del sistema a una entrada de muestra unitaria con cero condiciones iniciales; es decir, la
respuesta al impulso no es sino la solucin particular de la ecuacin en diferencias cuando la entrada

110

x[n] es una funcin impulso unitario [n]. A diferencia del caso continuo, la respuesta al impulso h[n]
de un sistema de tiempo discreto descrito por la Ec. (2.107) puede determinarse a partir de la relacin
1 M
h [ n ] bk [ n k ]
a0 k 0

ak h [ n k ]
k 1

(2.120)

Para el caso especial cuando N = 0, la respuesta al impulso h[n] est dada por

bn
,
h[n]
bk [ n k ] a0
a0 k 0
0

0n M

(2.121)

otros valores de n

Observe que la respuesta al impulso para este sistema tiene trminos finitos; es decir, es diferente de
cero solamente para una duracin finita.

Ejemplo 28. Determine la respuesta al impulso para cada uno de los sistemas causales descritos por las
ecuaciones en diferencias siguientes:
(a) y [ n ] x [ n ] 2 x [ n 1] 3 x [ n 3]
(b) y [ n ] 12 y [ n 2] 2 x [ n ] x[ n 2]
Solucin:
(a) Por la definicin (2.120)

h [ n ] [ n ] 2 [ n 1] 3[ n 3]
(b) h [ n ] 12 h [ n 2] 2 [ n ] [ n 2]
Puesto que el sistema es causal, h[2] = h[1} = 0. Entonces,

h [0] 12 h [ 2] 2 [0] [ 2] 2 [0] 2


h [1] 12 h [ 1] 2 [1] [ 1] 0
h [2] 12 h [0] 2 [2] [0] 12 (2) 1 0
h [3] 12 h [1] 2 [3] [1] 0

y, por tanto,
h [ n ] 2 [ n ]

Consideremos ahora de nuevo la Ec. (2.107), con x [ n ] [ n ] y y[n] = h[n]:


N

k 0

ak h [ n k ]

b [ n k ],
k

k 0

n0

(2.122)

111

con h[1], h[2], etc. iguales a cero.


Claramente, para n > M, el lado derecho de la Ec. (2.122) es cero, de modo que tenemos una ecuacin
homognea. Las N condiciones iniciales requeridas para resolver esta ecuacin son h[M], h[M 1] ,
, h [ M N 1] . Puesto que N M
para un sistema causal, slo tenemos que
determinar y[0], y[1], , y[M]. Haciendo que n tome sucesivamente los valores 0, 1, 2, , M en la
Ec. (2.122) y usando el hecho de que y[k] es cero para k < 0, obtenemos el siguiente conjunto de M + 1
ecuaciones:
j

a y [ n k ] b ,
k

j 0,1, 2, M

(2.123)

k 0

o, equivalentemente, en forma matricial

a0

a1

a0

a2

a1

a0

aM

aM 1

0 y [0] b0
0 y [1] b1

0 y [2] b2



a0 y [ M ] bM

(2.124)

Las condiciones iniciales obtenidas al resolver estas ecuaciones se usan ahora para determinar la
respuesta al impulso como la solucin de la ecuacin homognea:
N

a h [ n k ] 0,
k

nM

(2.125)

k 0

Ejemplo 29. Considrese la ecuacin en diferencias del Ejemplo 26, pero con una excitacin diferente,
es decir,

1
y [ n ] 34 y [ n 1] y [ n 2] x [ n ] 12 x [ n 1]
8
tal que N = 2 y M = 1. Se deduce que la respuesta al impulso se determina como la solucin de la
ecuacin
1
y [ n ] 34 y [ n 1] y [ n 2] 0,
8

n2

De la Ec. (2.123), encontramos la ecuacin para determinar las condiciones iniciales como

1
3
4

0 y [0] 1

1 y [1] 12

y [0] 1,

y [1]

5
4

112

Utilizando estas condiciones iniciales produce la respuesta al impulso:


n

1
1
h[ n ] 4 3
2
4

2.10 Simulacin de Sistemas


2.10.1 Componentes Bsicas: Sistemas de Tiempo Continuo
Cualquier sistema descrito por la ecuacin diferencial

d k y (t )

dt k

k 0

d k x (t )

(2.126)

dt k

k 0

o, tomando aN 1 , por la ecuacin

d N y (t )
dt N

N 1

a
k 0

d k y (t )
k

dt k

d k x (t )

dt k

k 0

con M N puede simularse usando sumadores, multiplicadores por escalares e integradores.


El Integrador Un elemento bsico en la teora y prctica de la ingeniera de sistemas es el integrador.
Matemticamente, la relacin de entrada-salida que describe el integrador, cuyo smbolo se muestra en
la Fig. 2.24, es
t

y ( t ) y ( t0 ) x ( ) d ),

t t0

t0

y la ecuacin diferencial de entrada-salida es


d y (t )
x (t )
dt

x(t)

y(t)

Figura 2.24

Sumadores y Multiplicadores por Escalares En la Fig. 2.25 se ilustran las operaciones de suma y
multiplicacin por un escalar y los smbolos que las identifican.

Ejemplo 30. Considere el sistema mostrado en la Fig. 2.26.

113

x2(t)
x1(t)

x2(t)
x1(t) + x2(t)

x1(t)

x1(t) x2(t)

x(t)

y(t) = Kx(t)
K

Figura 2.25

Denote la salida del primer integrador en la figura por v(t); entonces, la entrada a este integrador es
dv ( t )
dt

a1 v ( t ) a0 y ( t ) b0 x ( t )

(2.127)

La entrada al segundo integrador es dy ( t ) dt , por lo que se puede escribir


dy ( t )

v (t )

dt

Diferenciando ambos lados de esta ltima ecuacin y usando la Ec. (2.127), se obtiene

d 2 y (t )
dt

dv ( t )
dt

a1

dy ( t )
dt

a0 y ( t ) b0 x ( t )

d 2 y (t )
dt 2

a1

dy ( t )
dt

a0 y ( t ) b0 x ( t )

que es la ecuacin diferencial que relaciona la entrada y la salida en la Fig. 2.26.

a1

x(t)
b0

a0

Figura 2.26

y(t)

114

2.10.2 Diagramas de Simulacin: Sistemas de Tiempo Continuo


Utilizando notacin de operadores (D = (d/dt), la ecuacin diferencial para un sistema LIT puede
escribirse en la forma
N
D

N 1

i 0

ai D y ( t )

b D
i

i 0

x ( t ),

aN 1

(2.128)

En esta seccin se derivarn dos simulaciones cannicas diferentes para la Ec. (2.128). Para derivar la
primera forma, se supone N = M y escribimos de nuevo la ecuacin como
D N ( y bN x ) D N 1 ( an1 y bN 1 x ) D ( a1 y b1 x ) a0 y b0 x 0

Multiplicando la ecuacin por D N y reacomodando los trminos, se obtiene la relacin


y bN x D1 ( bN 1 x aN 1 y ) D ( N 1) ( b1 x a1 y ) D N ( b0 x a0 y )

(2.129)

a partir de la cual se puede dibujar el diagrama de la Fig. 2.27, comenzando por la salida y(t) en la
derecha y trabajando hacia la izquierda. El operador D k representa k integraciones y el diagrama de la
Fig. 2.27 es la primera forma cannica.
Se puede obtener otro diagrama til convirtiendo la ecuacin diferencial de orden N en dos
ecuaciones de orden menor. Para obtenerlas, sea

N
D

a j D j v (t ) x (t )

j 0

N 1

(2.130)

Entonces,

y (t )

b D v ( t )
i

(2.131)

i 0

x(t)
b0

b1

a0

bN1

a1

bN

aN1

Figura 2.27 Primera forma cannica.

y(t)

115

Para verificar que estas dos ltimas ecuaciones son equivalentes a la ecuacin diferencial original,
sustituimos (2.130) en (2.129) para obtener

N
D

a j D y (t )

j 0

N 1

i 0

bi D D N

bi D

( iN )

a j D j v (t )

j 0

N 1

N 1

i 0


aj

j 0

N iN
bi D
i 0

i 0

N 1

a D
j

j 0

bi D ( i j ) v ( t )

i j

v (t )

b D x ( t )
i

i 0

y as queda demostrada la equivalencia. La segunda forma cannica se muestra en la Fig. 2.28. Las
variables v ( N 1) (t ), , v (t ) que se usan en la construccin de y(t) y x(t) en las Ecs. (2.130) y (2.131),
respectivamente, son obtenidas integrando sucesivamente a v ( N ) (t ) . Observe que en esta
representacin, la entrada a cualquier integrador es exactamente la misma que la salida del integrador
precedente.

y(t)

bN
x(t)

b N1

b N2

b1

a N1

b0

a N2

a 1

a 0

y (t )

Figura 2.28 Segunda forma cannica.

Ejemplo 31. Obtenga un diagrama de simulacin para el sistema LIT descrito por la siguiente ecuacin
diferencial:
y ( t ) 5 y ( t ) 4 y ( t ) 2 x ( t ) x ( t )

Primero escribimos de nuevo la ecuacin como

116

D2 y ( t ) D 2 x ( t ) 5 y ( t ) [ x ( t ) 4 y ( t )]
y ahora integramos dos veces para obtener
y ( t ) D1 [2 x ( t ) 5 y ( t )] D2 [ x ( t ) 4 y ( t )]

Los diagramas de simulacin correspondientes se muestran en la Fig. 2.29a y b para la primera y


segunda forma, respectivamente.
x(t)

y(t)

.
(a)
y(t)

2
x(t)
v''(t)

v'(t)
5

.
(b)
Figura 2.29 Diagramas para el Ejemplo 31.

2.10.3 Componentes Bsicas: Sistemas de Tiempo Discreto


Para simular mediante diagramas a los sistemas LIT de tiempo discreto descritos por ecuaciones en
diferencias, se definirn tres elementos bsicos: El sumador, el multiplicador por una constante y el
elemento de retardo. Los tres se muestran en la Fig. 2.30. Estos elementos se pueden utilizar para
obtener diagramas de simulacin usando un desarrollo similar al del caso de sistemas en tiempo
continuo. Igual que en este caso, podemos obtener varios diagramas de simulacin diferentes para el
mismo sistema. Esto se ilustra considerando dos enfoques para obtener los diagramas.

117

x2[n]
x1[n]

x[n]

ax[n]

x[n 1]

x[n]

(b)

(c)

x1[n] + x2[n]

(a)

Figura 2.30

Ejemplo 32. Ahora se obtendr un diagrama de simulacin para el sistema descrito por la ecuacin de
diferencias
y [ n ] y [ n 1] y [ n 2] 0.25 y [ n 3] x [ n ] 2 x [ n 1] x [ n 2]

(2.132)

usando un mtodo similar al usado para sistemas en tiempo continuo.


Primero resolvemos por y[n] y agrupando trminos semejantes, podemos escribir
y [ n ] x [ n ] D [2 x [ n ] y [ n ]] D2 [ x [ n ] y [ n ]] D3 [ 0.25 y [ n ]]

donde D representa el operador de retardo unitario. Para obtener el diagrama de simulacin para este
sistema, suponemos que y[n] est disponible y primero formamos la seal
v4 [ n ] 0.25 y [ n ]

x[n]
1

v4[n]

v3[n]

+
0.2
5

+ + v2[n
]
+

+ v1[n] = y[n]

+ v1[n] = y[n]

Figura 2.31

x[n]
1

v4[n]

v3[n]

+
0.2
5

+ + v2[n
]
+
1

Figura 2.31

118

Esta seal la pasamos por un retardo unitario y le aadimos x [ n ] y [ n ] para formar


v3 [ n ] D{0.25 y [ n ]} { x [ n ] y [ n ]}

Ahora retrasamos esta seal y le aadimos 2 x [ n ] y [ n ] para obtener


v2 [ n ] D2 {0.25 y [ n ]} D{ x [ n ] y [ n ]} {2 x [ n ] y [ n ]}

Si ahora pasamos v2[n] a travs de un retardo unitario y le aadimos x[n], obtenemos


v1 [ n ] D3 {0.25 y [ n ]} D2 { x [ n ] y [ n ]} D{2 x [ n ] y [ n ]} x[ n ]

Claramente, v1[n] es igual a y[n], de modo que podemos completar el diagrama de simulacin
igualando v1[n] con y[n]. El diagrama de simulacin se muestra en la Fig. 2.31.
Considere la ecuacin de diferencias de orden N-simo
y [ n ] a1 y [ n 1] aN y [ n N ] b0 x [ n ] b1 x [ n 1} bN x [ n N ]

(2.133)

Siguiendo el enfoque dado en el ltimo ejemplo, similar el mtodo usado para sistemas de tiempo
continuo, podemos construir el diagrama de simulacin mostrado en la Fig. 2.32.

x[n]
bN

bN1

b1

b0

y[n]
D

aN

aN1

a1

Figura 2.32

Para derivar un diagrama de simulacin alterno para el sistema en la Ec. (2.132), escribimos la
ecuacin en funcin de una nueva variable v[n] como
N

v[n]

a v [ n j ] x [ n ]
j

j 1

(2.134)

119

y[n]

b v [ n m ]

(2.135)

m 0

Observe que el lado izquierdo de la Ec. (2.134) es de la misma forma que el lado izquierdo de la Ec.
(2.132) y el lado derecho de la Ec. (2.135) es de la forma del lado derecho de la Ec. (2.132).
Para verificar que estas dos ecuaciones son equivalentes a la Ec. (2.132), sustituimos la Ec. (2.135) en
el lado izquierdo de la Ec. (2.117) para obtener
N

y[n]
a j y [n j ]
bm v [ n m ]
a j bm v [ n m j ]
j 1
m 0
j 1
m 0

bm v [ n m ]

m 0
N

a v [ n m j ]
j

j 1

b x[ n m ]
m

m 0

donde el ltimo paso se obtiene a partir de la Ec. (2.134).


Para generar el diagrama de simulacin, primero determinamos el diagrama para la Ec. (2.134). Si
tenemos disponible a v[n], podemos generar v[n 1], v[n 2], etc., pasando sucesivamente a v[n] a
travs de unidades de retardo. Para generar a v[n], de la Ec. (2.135) observamos que
N

v[n] x[n]

a v [ n j ]

(2.136)

j 1

b0

b1
b2

+
+

b N-1
x[n]

+
_

v[n - 1]

_ _

v[n]

v[n - N - 1]
D

+
bN

v[n - 2]

v[n - N ]

a1

a N-1
aN

Figura 2.33

y[n]

120

Para completar el diagrama de simulacin, generamos y[n] como en la Ec. (2.135) mediante una
combinacin adecuada de v[n], v[n 1], etc. El diagrama completo se muestra en la Fig. 2.33.
Observe que ambos diagramas de simulacin pueden obtenerse en una forma directa a partir de la
ecuacin de diferencias correspondiente.

Ejemplo 33. El diagrama de simulacin alterno para el sistema del Ejemplo 32, Ec. (2-131), es

v [ n ] v [ n 1] v [ n 2] 0.25v [ n 3] x [ n ]
y
y [ n ] v [ n ] 2 v [ n 1] v [ n 2]

se muestra en la Fig. 2.34 usando estas dos ecuaciones.

2
v[n 1]
x[n]

y[n]
v[n 2]
v[n 3]

v[n]
D

0.25

Figura 2.34

2.11 Representacin Mediante Variables de Estado: Tiempo Continuo


En esta seccin se analizar la caracterizacin de sistemas en el dominio del tiempo (continuo) usando
la descripcin de la ecuacin de estado y las variables de estado. El mtodo permite estudiar el sistema
como un todo, tomando en cuenta tanto sus variables internas como las variables de entrada y salida
(excitacinrespuesta). El mtodo ha sido utilizado durante muchos aos en la descripcin y estudio de
sistemas dinmicos y tambin es de mucha utilidad en la resolucin de redes elctricas.
La descripcin mediante variables de estado utiliza un sistema de ecuaciones diferenciales (en forma
matricial) de primer orden y es aplicable a sistemas lineales o no, variables o invariables en el tiempo.
Esta descripcin con matrices que se emplea en la representacin mediante variables de estado es
independiente de la complejidad del sistema y, en consecuencia, puede facilitar grandemente el estudio
de sistemas complejos. Adems, la formulacin con variables de estado proporciona un mtodo
apropiado para el proceso de solucin de las ecuaciones por computadora.

121

2.11.1 Definiciones
Desde el punto de vista del anlisis y sntesis de sistemas, es conveniente clasificar las variables que
caracterizan o estn asociadas con el sistema en la forma siguiente: (1) variables de entrada o de
excitacin, ui, las cuales representan los estmulos generados por sistemas diferentes del sistema bajo
estudio y que influyen en su conducta; (2) variables de salida o de respuesta, yj, las cuales describen
aquellos aspectos de la conducta del sistema que son de inters; y (3) variables de estado o
intermedias, xk, las cuales caracterizan la conducta dinmica del sistema bajo investigacin.
El estado de un sistema es un resumen completo de cmo se encuentra el sistema en un punto
particular en el tiempo, es decir, el estado de un sistema se refiere a sus condiciones pasadas, presentes
y futuras. El conocimiento del estado en algn punto inicial, t0, ms el conocimiento de las entradas al
sistema despus de t 0 , permiten la determinacin del estado en un tiempo posterior t1. As que el estado
en t0 constituye una historia completa del sistema antes de t 0 , en la medida que esa historia afecta la
conducta futura. El conocimiento del estado presente permite una separacin bien definida entre el
pasado y el futuro.
En cualquier instante fijo, el estado del sistema puede describirse mediante los valores de un conjunto
de variables xi, denominadas variables de estado. Las variables de estado pueden tomar cualquier valor
escalar, real o complejo y se definen como un conjunto mnimo de variables x1 , x2 ,, xn cuyo
conocimiento en cualquier tiempo t0 y el conocimiento de la excitacin que se aplique posteriormente,
son suficientes para determinar el estado del sistema en cualquier tiempo t > t0.
Cuando un grupo de ecuaciones diferenciales ordinarias que representan un sistema fsico dinmico
est expresado en la forma

xi fi x1 , x2 , ,xn ; u1 , u2 , ,um ,

i 1, 2, ,n ,

se dice que el grupo de ecuaciones est en la forma normal. Las variables xi (i = 1, 2, , n) son las
variables de estado y las variables uj (i = 1, 2, , m) son las funciones de entrada o de excitacin. Si
el sistema es lineal, las ecuaciones pueden escribirse en la forma
n

xi

j 1

aij x j

b u
ik

i 1, 2, , n

k 1

o en forma matricial
x ( t ) Ax ( t ) Bu ( t )

(2.137)

en donde el conjunto de variables de estado se describe mediante un vector de estado

x1 ( t )
x (t )
x (t ) 2

xn ( t )

(2.138)

Este vector pertenece a un espacio n-dimensional, el espacio de estados, y el conjunto de variables de


entrada o de excitacin se describe mediante un vector de excitacin o de entrada

122

u1 ( t )
u ( t )

u (t ) 2

um ( t )

(2.139)

A es una matriz de dimensin n n y se denomina la matriz de los coeficientes, B es una matriz de


dimensin n m y se conoce como la matriz de distribucin, x es simplemente la derivada de x con
respecto al tiempo t, es decir, x dx dt . Todos los vectores y matrices que aparecen en la Ec. (2.137)
pueden depender del tiempo (sistemas variables en el tiempo). En este libro slo se tratarn sistemas
que no varan con el tiempo y, por tanto, las matrices A y B se tomarn siempre constantes y de la
forma

a11
a
A 21

an 1

a12 a1 n
a22 a2 n
,

an 2 ann

b11 b12 b1 n
b b b
2n
B 21 22

bn 1 bn 2 bnn

(2.140)

2.11.2 Solucin General de la Ecuacin de Estado


Considrese ahora la ecuacin diferencial escalar de primer orden

dx ( t )

ax ( t ) bu ( t )

dt

(2.141)

donde a y b son constantes arbitrarias y u(t) es una funcin continua de t (no confundir con la funcin
escaln definida en el captulo anterior). Multiplicando ambos lados de la ecuacin por eat , se tiene
que
e at

dx(t )
ae at x(t ) be at u (t )
dt

e at

dx(t )
a at x(t ) be at u (t )
dt

o tambin

la cual puede escribirse en la forma

d
e at x(t ) b e at u (t )
dt
e integrando desde t0 hasta t,
t

e at x(t ) b e au () d
t

t0

at

e x(t ) e

t0
at0

x(t0 ) b e au () d
t0

123

Despejando a x(t) en la ecuacin anterior se obtiene

x(t ) e

a t t0

x ( t0 ) be

a t

u ( ) d

(2.142)

t0

y cuando t0 = 0,
t

x ( t ) e x (0) b e
at

a t

(2.143)

Ejemplo 34
Resolver la ecuacin diferencial
dx
2x 5
dt

sujeta a la condicin inicial x(0) = 3.


Solucin. Esta ecuacin puede escribirse en la forma
dx
2 x 5
dt

de donde a = 2 y u(t) = 1. Aplicando la Ec. (2.141) se obtiene


t

x(t ) e2t 3 5e2(t ) d 3e2t 5e2t e 2 d


0

3e2t 2.5e2t e

2 t
0

2.5 0.5e2t

Obsrvese en (2.141) que u(t) = 0 corresponde a la ecuacin diferencial homognea


dx ( t )
dt

ax ( t )

(2.144)

cuya solucin es

x (t ) e

a t t0

x ( t0 )

(2.145)

Considrese ahora el conjunto homogneo de n ecuaciones de estado


x Ax ,

x ( t0 ) dado, A constante

(2.146)

La matriz de transicin de estados se define como una matriz que satisface la ecuacin de estado
lineal homognea

dx ( t )
dt

A x (t )

(2.147)

124

Sea (t) una matriz de n n que representa la matriz de transicin de estados; entonces, por definicin,
ella debe satisfacer la ecuacin

d ( t )
dt

A (t )

(2.148)

An ms, sea x(0) el estado inicial en t = 0; entonces (t) tambin se define mediante la ecuacin
matricial
x ( t ) ( t ) x (0)

(2.149)

la cual es la solucin de la ecuacin de estado homognea para t 0 .


Una forma alterna de resolver la ecuacin de estado homognea es suponer una solucin, igual que en
el mtodo clsico de solucin de las ecuaciones diferenciales lineales. Comparando las ecuaciones de
estado y la ecuacin escalar correspondiente muestra que la solucin de la Ec. (2.147) es anloga a la
de la Ec.(2.143); ella es

x ( t ) eA t x (0)

(2.150)

para t 0 , donde la funcin exponencial e A t representa la siguiente serie de potencias para la matriz
At:

e At I At

1
2!

A2 t 2

1
3!

A3 t 3

(2.151)

Aqu I es la matriz identidad de n n. Es fcil demostrar que la Ec. (2.150) es una solucin de la
ecuacin de estado homognea ya que, de la Ec. (2.151), tenemos que
de At
dt

A e At

(2.152)

Por tanto, adems de la Ec. (3.17), se obtuvo otra expresin para la matriz de transicin de estados:

( t ) e At I At

1
2!

A2 t 2

1
3!

A3 t 3

(2.153)

La Ec. (2.153) tambin se puede obtener directamente a partir de la Ec. (3.17). Esto se deja como un
ejercicio para el lector.
Ahora se considerar el conjunto no homogneo de las ecuaciones de estado. La matriz A todava se
considera una constante, pero B puede ser una funcin del tiempo, es decir, B = B(t). Se supone que
las componentes de Bu(t) son seccionalmente continuas para garantizar una solucin nica de la
ecuacin
x Ax B ( t ) u ( t ),

x ( t0 ) dado

(2.154)

Observe que aqu el tiempo inicial es t0 y no t = 0. Se repite la tcnica usada para resolver la ecuacin
escalar con slo modificaciones menores. Sea K(t) una matriz de n n. Premultiplicando la Ec. (2.154)
por K(t) y reagrupando, se obtiene

K t x (t ) K(t )Ax(t ) K(t )B(t )u(t )

125

, el lado izquierdo puede escribirse como una diferencial total


Puesto que d [ K ( t ) x ( t )] dt Kx Kx
K ( t ) A . Una matriz as es K e A ( t t0 ) . Aceptando que sta es la matriz
(vectorial) con tal que K
que debe usarse, la ecuacin diferencial puede escribirse en la forma

d K ( t ) x ( t ) K ( t ) B ( t ) u ( t ) dt
e integrando da
t

K ( t ) x ( t ) K ( t0 ) x ( t0 ) K ( ) B ( ) u ( ) d
t0

La forma de K seleccionada siempre tiene una inversa, de modo que


t

x ( t ) K ( t ) K ( t0 ) x ( t0 ) K 1 ( t ) K ( ) B ( ) u ( ) d
t0

o
t

x(t ) e

A ( t t0 )

x ( t0 ) e A ( t ) B ( ) u ( ) d

(2.155)

t0

sta representa la solucin para cualquiera ecuacin del sistema en la forma de la Ec. (2.154).
Obsrvese que est compuesta de un trmino que depende solamente del estado inicial y una integral de
convolucin que incluye la entrada pero no el estado inicial. Estos dos trminos se conocen por
diferentes nombres, tales como la solucin homognea y la integral particular, la respuesta libre de
excitacin y la respuesta forzada, la respuesta de entrada cero y la respuesta de estado cero, etc.
A continuacin se estudiarn varios mtodos para determinar la solucin de la ecuacin de estado
(2.146) cuando la matriz A es constante (sistemas invariables en el tiempo).
2.11.3 Solucin de la Ecuacin de Estado Mediante Integracin
Si la matriz A en la Ec. (2.146) es diagonal (valores diferentes de cero solamente en la diagonal
principal), la solucin para x se obtiene fcilmente por integracin separada de cada una de las
variables.

Ejemplo 35
Resolver el siguiente sistema de ecuaciones:

x1 1 0 x1 2
x 0 2 x 3 ,
2
2

5
x (0)
1

A partir del sistema se obtiene el par de ecuaciones escalares desacopladas (en este caso)
x1 x1 2, x1 (0) 5
x2 2 x2 3, x2 (0) 1

126

Usando ahora la Ec. (2.141) se obtienen las soluciones


t

x1 ( t ) 5 e

2e

( t )

d 5e

2e

5e

e d

2e e

t
0

2 3e

x2 ( t ) e

2 t

3 e2 ( t ) d e2 t 15 e2 t e2

t
0

1.5 0.5 e2 t

Ahora se estudiarn algunas propiedades de la matriz de transicin de estados. Puesto que la matriz de
transicin de estados satisface la ecuacin de estado homognea, ella representa la respuesta libre o
natural de la red. En otras palabras, ella rige la respuesta producida por las condiciones iniciales
solamente. De las Ecs. (3.17) y (2.153), se observa que la matriz de transicin de estados depende
solamente de la matriz A, por lo que en ocasiones tambin se conoce como la matriz de transicin de
estados A. Como el nombre lo indica, la matriz de transicin de estados (t) define por completo la
transicin de estados desde el tiempo inicial t = 0 hasta cualquier tiempo t cuando las entradas son
iguales a cero.
La matriz de transicin de estados (t) posee las siguientes propiedades:
1. (0) I (matriz identidad)

(2.156)

Demostracin La Ec. (2.154) se deduce directamente de la Ec. (2.153) al hacer t = 0.


2. 1 t t

(2.157)

Demostracin Posmultiplicando ambos lados de la Ec. (2.153) por e At , se obtiene


( t ) e At eAt e At I

(2.158)

Premultiplicando ahora ambos miembros de la Ec. (2.153) por 1 (t ) , se obtiene


e At 1 ( t )

(2.159)

( t ) 1 ( t ) e At

(2.160)

Por lo que

Un resultado interesante de esta propiedad de (t) es que la Ec. (2.150) se puede escribir como

x (0) ( t ) x ( t )

(2.161)

lo que significa que el proceso de transicin entre estados se puede considerar como bilateral en el
tiempo. Es decir, la transicin en el tiempo se puede dar en cualquier direccin.
3. ( t2 t1 ) ( t1 t0 ) ( t2 t0 ) para cualquier t0, t1 y t2.

127

Demostracin:

( t2 t1 ) ( t1 t0 ) e A ( t2 t1 ) e A ( t1 t0 )

(2.162)

eA ( t2 t0 ) ( t2 t0 )

Esta propiedad de la matriz de transicin de estados es muy importante, ya que ella implica que un
proceso de transicin de estados se puede dividir en un nmero de transiciones esenciales. La Fig.
2.1 ilustra que la transicin de t = t0 a t = t = t2 es igual a la transicin de t0 a t1 y luego de t1 a t2. En
general, por supuesto, el proceso de transicin de estados se puede dividir en cualquier nmero de
etapas.
4. ( t ) ( kt ) para k entero y positivo.
k

Demostracin:

( t )

e At e At e At
e

kAt

(k trminos)

(2.163)

( kt )

(t2 t0)
x(t0)

x(t2)

x(t1)
(t1 t0)
t0

(t2 t1)
t1

t2

Figura 2.1

2.11.4 Mtodo de los Valores y Vectores Caractersticos


Ahora se estudiar un mtodo muy poderoso para determinar la solucin de un sistema de ecuaciones
diferenciales lineales de primer orden, homogneo y con coeficientes constantes. El sistema a resolver
es

x1 a11 x1 a12 x2 + a1 n xn
x2 a21 x1 a22 x2 + a2 n xn

xn an 1 x1 an 2 x2 + ann xn

(2.164)

o, en forma vectorial,
x ( t ) Ax ( t )

(2.165)

128

De la teora de ecuaciones diferenciales se sabe que si x1, x2, , xn son n soluciones independientes
de la ecuacin lineal homognea x A x en algn intervalo abierto I donde los elementos aij de A son
continuos, entonces una solucin cualquiera de la ecuacin en I puede escribirse en la forma
x ( t ) c1 x1 ( t ) c2 x2 ( t ) + cn xn ( t )

(2.166)

para toda t en I; las ci, i = 1, 2, , n, son constantes. Esto quiere decir que basta obtener n vectores
solucin linealmente independientes x1, x2, , xn y entonces la Ec. (2.166) ser una solucin general
del sistema dado por la Ec. (2.164).
El procedimiento para obtener las n soluciones vectoriales linealmente independientes es anlogo al
mtodo de las races caractersticas usado para resolver una ecuacin lineal homognea con
coeficientes constantes. Es decir, se anticipan vectores solucin de la forma
t
x1 v1 e v1
x t v
ve
x ( t ) 2 2 2 e t v e t



t
xn vn e vn

(2.167)

donde , v1, v2, , vn son constantes. Al sustituir

xi vi et ,

x i vi et ,

i 1, 2, , n

en la Ec. (2.164), el factor e t se cancelar y quedarn n ecuaciones lineales en las que (para valores
apropiados de ) se espera obtener los coeficientes v1, v2, , vn en (2.167), de modo que x(t ) ve t
sea una solucin del sistema (2.162).
Para explorar esta posibilidad ms eficazmente, se usa la forma vectorial compacta

x Ax

(2.168)

donde A = [aij] y se sustituye la solucin tentativa x = vet con su derivada x vet . El resultado es
v et A v et

El factor no nulo et se cancela y se obtiene


Av v

(2.169)

Esto significa que x = ve t ser una solucin no trivial de la Ec. (2.168) siempre que v sea un valor no
nulo y una constante para que la Ec. (2.169) se cumpla; es decir, que el producto matricial Av sea un
mltiplo escalar del vector v.
Ahora se proceder a determinar y v. Primero se escribe la Ec. (2.169) en la forma
( I A ) v 0

(2.170)

donde I es la matriz identidad. Dado , ste es un sistema de n ecuaciones lineales homogneas en las
incgnitas v1, v2, , vn. Del lgebra lineal se sabe que la condicin necesaria y suficiente para que el
sistema tenga una solucin no trivial es que el determinante de los coeficientes de la matriz se haga
cero; es decir, que

129

det I A I A 0

(2.171)

Los nmeros (sean iguales a cero o no) obtenidos como soluciones de (2.171) se denominan
valores caractersticos o propios de la matriz A.y los vectores asociados con los valores caractersticos
tales que Av = v, v diferente de cero, se conocen como vectores caractersticos o propios. La
ecuacin

I A

a11

a12

a1 n

a21

a22

a2 n

an 1

an 2

(2.172)

ann

se conoce como la ecuacin caracterstica de la matriz A.


La Ec. (2.172) tiene n races (es un polinomio en de grado n) por lo que una matriz de n n posee
n valores caractersticos (contando la multiplicidad), los cuales pueden ser distintos o repetidos, reales
o complejos. Los casos se estudiarn por separado.
Valores Caractersticos Reales y Distintos
Si los valores caractersticos son reales y distintos, se sustituye cada uno de ellos sucesivamente en la
Ec. (2.171) y se determinan los vectores caractersticos asociados v1, v2, , vn, los cuales darn las
soluciones
x1 ( t ) v1 e1t , x2 ( t ) v2 e2 t , , x n ( t ) v n en t

(2.173)

Se puede demostrar que estos vectores solucin siempre son linealmente independientes. El
procedimiento para otenerlos se ilustrar mediante ejemplos.

Ejemplo 36
Encuntrese una solucin general del sistema
x1 4 x1 2 x2
x2 3 x1 x2

Solucin. La forma matricial del sistema es


4
x
3

2
x
1

La ecuacin caracterstica de la matriz de los coeficientes es

I A

+1

( 4)( 1) 6 2 3 10

( 2)( 5) 0
y as se obtienen los valores caractersticos reales y distintos 1 = 2 y 2 = 5.

130

Para la matriz de los coeficientes A del sistema, la ecuacin para los vectores caractersticos toma la
forma
4
3

2 a 0

1 b 0

(2.174)

donde el vector caracterstico asociado es v = [a b]T (la T indica la matriz transpuesta).


(a) 1 =2:
La sustitucin = 2 en (2.172) produce el sistema
6 2 a 0
3 1 b 0

o las dos ecuaciones escalares

6a 2b 0
3a b 0
Obviamente, estas dos ecuaciones escalares son equivalentes y, por lo tanto, tienen una infinidad de
soluciones no nulas; por ejemplo a se puede escoger arbitrariamente (diferente de cero) y entonces
despejar b. Normalmente buscamos una solucin sencilla con valores enteros pequeos (si ello es
posible). En este caso tomaremos a = 1, lo cual produce b = 3, y entonces

1
v1
3
Observacin: Si en lugar de a = 1 se hubiese tomado a = c, por ejemplo, se obtendra el vector
caracterstico
c
1
v1
c

3c
3

Puesto que ste es un mltiplo constante del resultado previo, cualquier seleccin que se haga ser
un mltiplo constante de la misma solucin.
(b) = 2:
La sustitucin de este valor en (2.174) produce el par de ecuaciones
a 2b 0
3a 6b 0

Las cuales son equivalentes. Se escoge b = 1 y en consecuencia a = 2, de modo que


2
v2
1

Estos dos valores caractersticos con sus vectores caractersticos asociados producen las dos
soluciones

131

1
x1 ( t ) e 2 t
3

2
y x 2 ( t ) e5 t
1

Es fcil demostrar que estas soluciones son linealmente independientes. En consecuencia, la


solucin general del sistema dado es
1
2
x ( t ) c1 x1 ( t ) c2 x2 ( t ) c1 e2 t c2 e5 t
3
1

Ejemplo 37
Determnese una solucin general del sistema
0 6
x
x
1 5

El polinomio caracterstico es

6
I A
2 5 6

1 5
( 2)( 3) 0
y as se obtienen los valores caractersticos 1 = 2 y 2 = 3, y la ecuacin para los vectores
caractersticos toma la forma
6 a 0
1 5 b 0

(2.175)

siendo v = [a b]T el vector caracterstico asociado.


(a) 1 = 2:
La sustitucin de = 2 en (2.173) produce el sistema
2
1

6 a 0

3 b 0

o las dos ecuaciones escalares

2 a 6 b 0
a 3b 0
Igual que en el Ejemplo 36, este sistema tiene infinidad de soluciones. Se escoge b = 1, lo cual produce
a 3 y entonces

3
v1
1
(b) = 3:
La sustitucin de este valor en la Ec. (2.173) produce el par de ecuaciones

132

3a 6 b 0
a 2b 0
las cuales son equivalentes. Se escoge b = 1 y entonces a = 2, de manera que
2
v2
1

y los dos vectores solucin asociados son


3
x1 ( t ) e 2 t
1

2
y x 2 ( t ) e 3 t
1

En consecuencia, la solucin general del sistema es


3
2
x ( t ) c1 e 2 t c2 e 3 t
1
1

Valores Propios Complejos y Distintos


Si los valores propios son complejos pero distintos, el mtodo ya descrito producir las n soluciones
independientes. La nica complicacin consiste en que los vectores propios asociados con valores
propios complejos en general tomarn tambin valores complejos.
Puesto que se est suponiendo que los elementos de la matriz A son reales, los coeficientes de la
ecuacin caracterstica (2.173) sern reales. Por lo tanto, los valores propios complejos debern
aparecer en pares de complejos conjugados. Supngase que = p + jq y * = p jq constituyen un par
de esos valores propios. Si v es un vector propio asociado con , es decir,
( I A ) v 0

entonces, al tomar el conjugado de esta ecuacin se obtiene


( * I A ) v* 0

lo que significa que v*, el conjugado de v, es un vector propio asociado con *. La solucin compleja
asociada con y v es entonces v = a + jb y, por tanto,

x ( t ) v et ( a jb ) e( p jq ) t
( a jb ) e pt (cos qt j sen qt )
es decir,
x ( t ) e pt ( a cos qt b sen qt ) je pt ( b cos qt a sen qt )

Puesto que las partes real e imaginaria de una solucin con valores complejos son, a su vez, soluciones
del sistema, entonces se obtienen dos soluciones con valores reales

x1 ( t ) Re{ x ( t )} e pt ( a cos qt b sen qt )


x2 ( t ) Im{ x ( t )} e pt ( b cos qt a sen qt )

(2.176)

133

asociadas con los valores propios complejos p jq.


No hay necesidad de memorizar las frmulas (2.176) y esto se ver fcilmente en los ejemplos,

Ejemplo 38
Encuntrese una solucin general del sistema
x1 4 x1 3 x2
x2 3 x1 4 x2

(2.177)

La matriz de los coeficientes


4
A
3

3
4

tiene la ecuacin caracterstica


I A

2 8 25 0

y por consiguiente los valores propios conjugados son = 4 j3 y * = 4 + j3. Sustituyendo = 4 j3


en la ecuacin para el vector propio (I A)v = 0, se obtiene
j 3 3 a 0
[(4 j 3) I A ] v

3 j 3 b 0

para un vector propio asociado v = [a b]T. La divisin de cada fila entre 3 produce las dos ecuaciones
escalares
ja b 0
a jb 0

cada una de las cuales se satisface con a = 1 y b = j. As v = [1 j]T es un vector complejo asociado con
el valor propio complejo = 4 j3.
La solucin correspondiente para los valores complejos x(t) v et de x Ax es entonces

1
1
x ( t ) e( 43 j ) t e4 t (cos 3t j sen 3t )
j
j
cos 3t j sen 3t
e4 t

j cos 3t sen 3t
Las partes real e imaginaria de x(t) son las soluciones con valores reales:
cos3t
x1 ( t ) e4 t

sen 3t

sen 3t
y x 2 ( t ) e4 t

cos3t

y entonces una solucin general con valores reales viene dada por

134

c1 cos3t c2 sen 3t
x ( t ) c1 x1 ( t ) c2 x 2 ( t ) e4 t

c1 sen 3t c2 cos3t
o, en forma escalar,

x1 ( t ) e4 t ( c1 cos3t c2 sen 3t )
x2 ( t ) e4 t ( c1 sen 3t c2 cos3t )
Si se hubiese utilizado el otro valor caracterstico = 4 + j3, el vector propio asociado obtenido sera
v* = [1 j]T

Ejemplo 39
Determine la solucin general del sistema
4 2
x
x
1 2

La ecuacin caracterstica es

I A

2 6 10

=( +3 j )( +3+j ) 0
Por lo que los valores caractersticos son = 3 + j y * = 3 j. Para = 3 + j se tiene que
1 j 2 a 0
1 1 j b 0

lo cual produce las ecuaciones escalares


(1 j ) a 2 b 0
a ( 1 j ) b 0

Las cuales se satisfacen con b = 1 y a = 1j. As que v = [1j1 1]T es un vector caracterstico
complejo asociado con = 3+j. El vector caracterstico asociado con = 3j es v* = [1+j1 1]T.
La solucin correspondiente de vectores complejos x(t) es entonces

1 j ( 3 j ) t 1 j 3 t
x(t )

e
e (cos t sen t )
1
1
(1 j )(cos t sen t )
cos t sen t j (sen t cos t )
e 3 t
e 3 t

cos t jsen t
cos t j sen t

Las partes real e imaginaria de x(t) son las soluciones con valores reales:
cos t sen t
x1 ( t ) e 3 t

cos t

sen t cos t
y x 2 ( t ) e 3 t

sen t

135

y la solucin general con valores reales est dada por

c1 (cos t sen t ) c2 (sen t cos t )


x ( t ) c1 x1 ( t ) c2 x 2 ( t ) e 3 t

c1 cos t c2 sen t

2.11.5 Solucin Mediante Diagonalizacin de Matrices


Se dice que una matriz A = [ai j] de n n es una matriz diagonal si ai j = 0 para i j. Por lo tanto, en
una matriz diagonal, todos los elementos fuera de la diagonal principal son iguales a cero.
Si A y B son matrices de n n, decimos que B es semejante a A si existe una matriz S no singular tal
que B = S1AS. Del lgebra Lineal se sabe que si A es una matriz de n n que es semejante a una
matriz diagonal S 1AS y si las columnas de S son los vectores caractersticos de A, entonces los
elementos de la diagonal principal de son los valores caractersticos de A (A no tiene valores
caractersticos repetidos), es decir,

1 0 0
0 0
2
1

diagonal ( 1 , , n ) S AS

0 0 n

(2.178)

Ejemplo 40
Diagonalizar la matriz
0 6
A

1 5

Del Ejemplo 37 se tiene


3
S [ v1 v 2 ]
1

2
,
1

1 2
S 1
3
1

por lo que
6 3
1 2 0
S 1 AS

3 1 5 1
1

2 2 0

1 0 3

Considrese ahora la ecuacin de estado


x Ax Bu

y defnase la transformacin x = Sz; entonces, sustituyendo en la Ec. (2.179), se obtiene


x Sz ASz Bu

(2.179)

136

y despejando a z ,

z S1 ASz S1 Bu

(2.180)

Si S es la matriz cuyas columnas son los vectores caractersticos de A, entonces el producto S 1AS
es una matriz diagonal y (2.180) se puede escribir como
z z S1 Bu

(2.181)

Es evidente que la transformacin lineal aplicada a x convierte al sistema original (2.179) con variables
de estado x1, x2, , xn en un nuevo sistema en el cual las nuevas variables de estado z1, z2 , zn estn
completamente desacopladas. Estas nuevas variables de estado se consiguen fcilmente mediante el
mtodo aplicado en el Ejemplo 35 y luego, a estas variables, se les aplica la transformacin x Sz para
obtener las variables originales.

Ejemplo 41. Resolver el sistema


0 6
0
x
x ,

1 5
1

1
x (0)
2

Usando el resultado obtenido en el Ejemplo 40,


z z S 1 Bu
0
2
1 2 0


0 3
1 3 1
0
2
2

3
0 3

1 2 1 5
z (0) S 1 x

1 3 2 7

Por lo que
z1 2 z1 2,
z2 3 z2 7,

z1 (0) 5
z2 (0) 7

y usando la Ec. (2.142),


t

z1 ( t ) e

2 t

( 5) ( 2) e

2 ( t )

d 5 e

2 t

2e

2 t

e
0

4 e 2 t 1
t

z2 ( t ) e

3 t

(7) 3 e
0

6e 1
3t

de donde

3 ( t )

d 7e

3 t

3e

3 t

e
0

137

3
x ( t ) Sz
1

2 4 e 2 t 1

1 6 e 3 t 1

1+12e 2 t 12 e 3 t
=
2 t
3 t
4e 6e

Ejemplo 43
Resolver el sistema
4 2
0
x
x ,

1 2
2

3
x (0)
1

Usando los resultados del Ejemplo 39, se tiene que

3 j
z
0
3 j
=
0

1 j 0
1 j 2

0
1 1
z

3 j
2 j 1
0
1
z

3 j
1

1 1
z (0) S 1 x (0)
2 j 1

j
j
1

1 j 3 2

1 j 1 1

por lo que
z1 ( 3 j ) z1 (1 j ),

z1 (0)

z2 ( 3 j ) z2 (1 j ),

z2 (0)

1
2
1
2

j
j

Entonces
1
z1
2

j e( 3 j ) t (1 j ) e( 3 j ) t e ( 3 j ) d

1 j 1 j ( 3 j ) t
1

j e( 3 j ) t

e
3 j 3 j
2

4 2 j 1 12 j ( 3 j ) t

e
10
10

138

1 j 1 j ( 3 j ) t
1

z2 j e ( 3 j ) t

e
3 j 3 j
2

4 2 j 1 12 j ( 3 j ) t

e
10
10
y por ltimo,
4 2 j 1 12 j ( 3 j ) t

1 j 1 j 10
10
x

1 4 2 j 1 12 j ( 3 j ) t
1
10 10 e

0.4 e 3 t (2.6cos t 2.2sen t ) 0.4 3.406 e 3 t sen( t 130.24 )

3 t
3 t
0.8 e (0.2 cos t 2.4sen t ) 0.8 2.408 e sen( t 175.24 )

2.11.6 Solucin por Reduccin a la Forma Cannica de Jordan


En la Seccin 2.7 se ilustr que una matriz cuadrada A con valores caractersticos distintos puede ser
siempre reducida a una matriz diagonal mediante una transformacin lineal. En el caso en que la
ecuacin caracterstica de la matriz A (n n) no posea n races distintas, entonces no siempre se puede
obtener una matriz diagonal, pero se puede reducir a la forma cannica de Jordan (sta se define ms
adelante).
Un valor propio es de multiplicidad k si es una raz de multiplicidad k de la ecuacin |I A| = 0.
Para cada valor caracterstico la ecuacin para el vector caracterstico asociado

( A I ) v 0

(2.182)

posee al menos una solucin no nula, de modo que hay por lo menos un vector caracterstico asociado
con . Pero un valor caracterstico de multiplicidad k > 1 puede tener menos de k vectores
caractersticos asociados linealmente independientes. En este caso no se puede determinar un conjunto
completo de los n vectores caractersticos linealmente independientes de A que se necesitan para
formar la solucin de la ecuacin x Ax . Considrese el ejemplo siguiente:

Ejemplo 44
La matriz
1
0
A

4 4

tiene la ecuacin caracterstica


g ( ) I A

4 4

( 2)2 0

139

De aqu resulta que A tiene el valor propio 1 = 2 con multiplicidad 2. La ecuacin para el vector
caracterstico es
2
( I A ) v
4

1 a 0

2 b 0

o en forma escalar,

2 a b 0
4a 2b 0
Por tanto, b = 2a si v = [a b] es un vector caracterstico de A y cualquier vector caracterstico
asociado con 1 = 2 de multiplicidad 2 tiene solamente un vector caracterstico independiente y es,
por consiguiente, incompleto.
T

Si un valor caracterstico de multiplicidad k > 1 no es completo se denomina defectuoso. Cuando


tiene solamente p < k vectores caractersticos linealmente independientes, entonces el nmero
d=kp
de los vectores caractersticos faltantes se llama el defecto del valor caracterstico defectuoso. En el
Ejemplo 44, el valor caracterstico defectuoso = 2 tiene una multiplicidad k = 2 y un defecto d = 1
porque solamente tiene un vector caracterstico asociado (p = 1).
Para este caso de valores caractersticos defectuosos, el mtodo descrito en la Seccin 2.7 producir
menos de las n soluciones linealmente independientes necesarias del sistema x Ax y por ello se
necesita un mtodo para encontrar las soluciones faltantes correspondientes a un valor propio
defectuoso de multiplicidad k > 1. Considrese el caso k = 2 y supngase que hay solamente un
vector v1 asociado con y la solucin
x1 ( t ) v1 et

(2.183)

Por analoga con el caso de una raz caracterstica repetida para una sola ecuacin diferencial, se
debera esperar una segunda solucin de la forma
x 2 ( t ) w t et

(2.184)

Al sustituir (2.184) en la ecuacin x Ax , se obtiene la relacin

wet wtet Awtet


de la cual se deduce que w = 0 y entonces no existe una solucin no trivial de la forma (2.183).
Ahora se intentar una solucin de la forma
x2 ( t ) v t et w et

Cuando se sustituye la Ec. (2.185) en la relacin x Ax , se obtiene la ecuacin


vet vtet wet Avtet Awet

e igualando los coeficientes de las potencias de t iguales, se obtienen las dos ecuaciones

(2.185)

140

[ I A ] v 0

(2.186)

[ I A ] w v

(2.187)

Los vectores v y w deben satisfacer las Ecs. (2.186) y (2.187) para que la Ec. (2.185) sea una
solucin de x Ax . Obsrvese que la Ec. (2.185) significa solamente que v1 = v es un vector
caracterstico asociado con , y entonces la Ec. (2.186) implica que
[ I A ]2 w [ I A ] v 0

En consecuencia, para el caso de un valor caracterstico defectuoso de multiplicidad 2, el mtodo


consiste en lo siguiente:
1. Encontrar una solucin no nula de la ecuacin
[ I A ]2 v 0

(2.188)

[ I A ] v 2 v1

(2.189)

tal que

no se anule y
2. Formar las dos soluciones independientes
x1 ( t ) v1 et

(2.190)

x2 ( t ) ( v1 t v 2 ) et

(2.191)

Ejemplo 45
Encuntrese una solucin general del sistema
1
0
x
x
4 4

(2.192)

En el Ejemplo 44 se encontr que la matriz de los coeficientes A en la Ec. (2.192) tiene el valor propio
defectuoso = 2 de multiplicidad 2. Entonces se calcula
2 1 2 1 0 0
[ I A ]2

2 4
2 0 0
4

y la Ec. (2.188) en este caso se convierte en


0 0
0 0 v 2 0

y en consecuencia es satisfecha por cualquier seleccin de v2. Usando ahora la Ec. (2.189), se obtiene
2 1 a
1
[ I A ] v 2

1
2
2 b
4

de donde se obtienen las ecuaciones escalares

141

2 a b 1
4a 2b 2
y tomando b = 1 da a = 0; en consecuencia, v2 = [0 1]T. Las dos soluciones de (2.192) son
1
x1 ( t ) v1 e 2 t e 2 t
2
t 2 t
x 2 ( t ) ( v1 t v 2 ) e 2 t
e
1 2 t

y la solucin general resultante es


x ( t ) c1 x1 ( t ) c2 x 2 ( t )
1
t 2 t
c1 e 2 t c2
e
2
1 2 t
c1 c2 t

2 t

e
2 c1 c2 2 c2 t

El vector v2 en la Ec. (2.189) es un ejemplo de un vector propio generalizado. Si es un valor


caracterstico de la matriz A, entonces un vector caracterstico generalizado de rango r asociado con
es un vector v tal que
[ I A ]r v 0

pero

[ I A ]r1 v 0

(2.193)

Si r = 1, entonces la Ec. (2.193) significa sencillamente que v es un vector caracterstico asociado con
. As, un vector caracterstico generalizado de rango 1 es un vector caracterstico ordinario. El vector
v2 en la Ec. (2.189) es un vector caracterstico generalizado de rango 2.
El mtodo para multiplicidad 2 descrito anteriormente consisti en determinar un par de vectores
caractersticos generalizados {v1, v2} tales que [I A]v 2 v1 . Cuando la multiplicidad es superior, se
obtienen cadenas ms largas de vectores caractersticos generalizados. Una cadena de longitud k de
vectores caractersticos generalizados basados en el vector caracterstico v1 es un conjunto {v1, v2,
, vk} de k vectores caractersticos generalizados tales que

[ I A ] v k v k 1
[ I A ] v k 1 v k 2

(2.194)

[ I A ] v 2 v1
ya que v1 es un vector caracterstico ordinario, [I A]v1 = 0. Por consiguiente, de la Ec. (2.193) se
deduce que
[ I A ]k v k 0

Las Ecs. (2.194) se pueden escribir en forma compacta como

(2.195)

142

[ I A ] v1 0
[ I A ] vi 1 v1 ,

i 1, 2, , k 1

(2.196)

donde k es la multiplicidad (rango) del valor caracterstico .


Al comienzo de esta seccin se dijo que cuando la matriz cuadrada A (nn) posea valores
caractersticos repetidos, entonces no poda ser diagonalizada. En los cursos de lgebra Lineal se
demuestra que bajo la transformacin S1AS siempre hay una seleccin de la matriz S tal que la matriz
S1AS tenga la forma cannica de Jordan, en la cual aparecen bloques de Jordan
J1 , J 2 , , J k (1 k n ) en la diagonal principal y todos los otros elementos son iguales a cero:

J1 0 0 0
0 J 0 0
2

J S 1 AS
. . . . . . . . . .

0 0 0 J n

(2.197)

Cada bloque Jj es una matriz de orden nj (1 nj n) de la forma

0 0
j 1

0 j 1 0
J j

0 0 j 1

0 0 0 j

(2.198)

donde una de las races j de la ecuacin caracterstica |I A| = 0 aparece en la diagonal principal, el


nmero 1 aparece en la diagonal justo encima de la diagonal principal y todos los otros elementos de la
matriz son iguales a cero.
Las columnas de la matriz S en la Ec. (2.197) se forman con los vectores caractersticos dados por las
Ecs. (2.196).

Ejemplo 46
Resolver el sistema
1
0
0
x
x

3 ,
4 4

1
x (0)
2

En este caso, la matriz A de los coeficientes es la misma de los Ejemplos 14 y 15. All se determin
que la ecuacin caracterstica |I A| = 0 produce el valor propio = 2 de multiplicidad 2 y que los
vectores caractersticos asociados son v1 = [1 2]T y v2 = [0 1]T. Por lo tanto,
1 0
S [ v1 v 2 ]
,
2 1

1 0
S 1

2 1

Bajo la transformacin x = Sz, la ecuacin original se convierte en

143

1 1 0
1 0 0
1 0 0 2
z
z

3 0
2
1

2
1
2
1

1
0
z

2
3

1 0 1 1
z (0) S 1 x (0)

2 1 2 4

o, en forma escalar,
z1 2 z1 z2 ,
z2 2 z2 3,

z1 (0) 1
z2 (0) 4

Resolviendo primero por z2:


t

z2 4 e

2 t

3e

2 t

d 4e

2 t

e
2

2 t

5
e 2 t
2 2

Sustituyendo ahora a z2 en la ecuacin para z1, se obtiene


z1 2 z1

5
e 2 t ,
2 2

z1 (0) 1

y resolviendo,
t

z1 e

2 t

2 t

e
2

3
2

25 e 2 d

5
e 2 t t e 2 t
4 4
2

Por lo tanto,
2 t
2 t
3
5
1 2 t
3
5
1 2 t
x1
1 0 4 4 e 2 te 4 4 e 2 t e
x Sz

2 t
2 t
3
5 2 t

e
2
e

5
te

2 1
x2

2
2

Ejemplo 47
Resolver el sistema

2
x 0

1
La ecuacin caracterstica es

0
0
0

1
1
1 x 0 ,


0
0

1
x (0) 1

2

144

2
g ( ) I A 0

1
1 3 2 2 ( 1) 2

De aqu resulta 1 = 0 (multiplicidad 1) y 2 = 1 (multiplicidad 2).


Para 1 = 0:

2
[ I A ] v1 0

1 a 0
0 1 b 0

0 1 c 0

y se obtienen las tres ecuaciones escalares


2a c 0
c 0
a 0

As que a = c = 0 y b puede tener cualquier valor. Tomando b = 1 se tiene que v1 = [0 1 0]T.


Para = 1:

1 0 1 a 0
[ I A ] v 2 0 1 1 b 0


1 0 1 c 0
lo que produce las tres ecuaciones escalares
ac0
b c 0
a c 0

Si tomamos c = 1, entonces a = 1, b = 1 y v2 = [1 1 1]T:


Para determinar el otro vector caracterstico asociado con 2, se usa la Ec. (2.196):
[I A]v3 = v2
es decir,

1 0 1 a
1
0 1 1 b 1



1 0 1 c
0
o en forma escalar,
a c 1
bc 1
a c 1

145

Tomando c = 1, se obtiene a = 0, b = 2 y entonces v3 = [0 2 1]T.


Ahora se forma la matriz S =[v1 v2 v3]:

0 1 0
S 1 1 2

1 1
0
de donde

1
1

1
0
0

2
0

Bajo la transformacin x = Sz, el sistema original se transforma en

0 0 0
1

z 0 1 1 z 1


0 0 1
1
con

1
z (0) 1

2 1 6
0 1 1

1 2 3

1
0
0

o en forma escalar,

z1 1,
z2 z2 z3 1,
z3 z3 +1,

z1 (0) 6
z2 (0)= 1
z3 (0)=3

Resolviendo,
t

z1 6 d 6 t
0

z3 3 e

e d 1 2e

Entonces,
z2 z2 1 2 e t 1,

z2 (0) 1

o
t

z2 e

e 2e d e

2 t e t

146

y por consiguiente,

x1
0 1 0 6 t
t

x x2 S z 1 1 1 e 2 te

x3
0 1 1 1 2 e t

e t 2 t e t

t
t
4 t 3 e 2 t e
1 e t 2 t e t

147

Problemas
2.1 Tales conceptos como memoria, invariabilidad en el tiempo, linealidad y causalidad tambin son
vlidos para sistemas de tiempo discreto. En lo que sigue, x[n] se refiere a la entrada a un sistema y
y[n] a la salida. Determine si los sistemas son (i) lineales, (ii) sin memoria, (iii) invariables en el
tiempo y (iv) causales. Justifique su respuesta en cada caso
(a) y[n] = log{[ x[n]}
(b) y[n] = x[n] x[n 2]
(c) y [n] n x [n] 3
(d) y [n] x [n] 2 x [n 1]

(e) y [ n ]

x[k ]
k 0

(f) y [ n ]

1
N

N 1

x[n k ]
k 0

(g) y [ n ] mediana de x [ n 1], x [ n ], x [ n 1]


x [ n ], n 0
(h) y [ n ]
x [ n ], n 0

2.2 Evale las siguientes convoluciones:


(a) rect( t a ) ( t a )
(b) rect ( t a ) rect ( t a )
(c) rect ( t a ) rect ( t 2 a )
(d) rect ( t a ) u ( t )

d
dt

(t )

(e) rect ( t a ) ( t ) ( t a )
(f)

rect ( t a ) 2 rect ( t 3a a ( t a )

(g) sgn ( t ) rect ( t a )


2.3 Para las seales x[n] y h[n] dadas, determine la convolucin y[n] = h[n]x[n]:
(a) x[n] = 1, 5 n 5 ,
(b) x [ n ] 3n ,

n 0,

h[n]=
h [ n ] 1,

12

u[n]

0n9

148

(c) x [n] u [n]

h [ n ] 13 u [ n ]
n

(d) x [ n ] [ n ] 2 [ n 1] 12 u [ n ]
n

h [ n ] 12 u [ n ]
n

1 0 n 5
(e) x [ n ]
1 6 n 10

h [ n ] 12 u [ n ] 13 u [ n ]
n

(f) x [n] nu [n]


h [n] u [n] u [n N ]

2.4 Halle la convolucin y[n] = h[n]x[n] para cada uno de los dos pares de secuencias finitas dadas:
(a)

x [ n ] 1, 12 , 14 , 81 , 161 ,

h [ n ] 1, 1,1, 1

(b) x [ n ] 1,2,3,0, 1,

h [ n ] 2, 1,3,1, 2

(c) x [ n ] 1, 12 , 43 , 15 ,1 ,

h [ n ] 1,1,1,1,1

2.5 Determine grficamente la convolucin de los pares de seales mostrados en la Fig. P2.5.
x(t)

x(t)

h(t)

h(t)

1
-1

-1

-1

(b)

(a)
x(t)

h(t)

x(t)

h(t)

1
-1

2 t

-1

-1

-1
(d)

(c)

Figura P.2.5

149

2.6 Use la integral de convolucin para hallar la respuesta y(t) del sistema LIT con respuesta al
impulso h(t) a la entrada x(t).
(a) x(t ) 2exp(2t ) u (t ),

h (t) exp(t) u (t)

(b) x(t ) t exp(2t ) u (t ),

h (t) exp(t) u (t)

(c) x(t ) t exp(t ) u(t ),

h (t) exp(t) u(t)

(d) x (t ) exp(3t ) u (t ),

h (t) rect (t 2)

(e) x (t ) (2 t )exp(2t ) u (t ),

h (t ) exp(t ) u (t )

2.7 La correlacin cruzada de dos seales diferentes se define como


Rxy ( t )

x ( ) y ( t ) d x ( t ) y ( ) d

(a) Demuestre que

Rxy ( t ) x ( t ) y ( t )
(b) Demuestre que la correlacin cruzada no obedece la ley conmutativa.
(c) Demuestre que Rxy(t) es simtrica [Rxy(t) = Ryx(t)].
2.8 Determine la correlacin cruzada entre una seal x(t) y la seal y(t) = x(t1) + n(t) para
B A 0.01, y 1, donde x(t) y n(t) son como se muestra en la Fig. P2.8.

x(t)

n(t)

B
3/2

Figura P2.8

2.9 La autocorrelacin es un caso especial de la correlacin cruzada con y(t) = x(t). En este caso,

Rx ( t ) Rxx ( t )

x ( ) x ( t ) d

(a) Demuestre que


Rx (0) E ,

(b) Demuestre que

la energa de x ( t )

150

Rx ( t ) Rx (0) (use la desigualdad de Schwartz)


(c) Demuestre que la autocorrelacin de z (t ) x (t ) y (t ) es

Rz ( t ) Rx ( t ) Ry ( t ) Ryx ( t )
2.10 Considere un sistema LIT cuya respuesta al impulso es h(t). Sean x(t) y y(t) la entrada y salida del
sistema, respectivamente. Demuestre que
Ry ( t ) Rx ( t ) h ( t ) h ( t )

2.11 La entrada a un sistema LIT con respuesta al impulso h(t) es la exponencial compleja exp(jt).
Demuestre que la salida correspondiente es
y ( t ) exp( jt ) H ( )

donde

H ( )

h ( t ) exp( jt ) dt

2.12 Determine si los siguientes sistemas LIT de tiempo continuo son causales o no causales, estables
o inestables. Justifique sus respuestas.
(a) h (t ) exp(2t )sen 3t u (t )
(b) h (t ) exp(2t ) u (t )
(c) h (t ) t exp(3t ) u (t )
(d) h (t ) t exp(3t ) u (t )
(e) h (t ) exp t

(f) h (t ) rect (t 2)
(g) h (t ) (t )
(h) h (t ) u (t )
(i) h ( t ) 1 t

rect ( t 2)

2.13 Determine si cada una de los siguientes sistemas es invertible. Para aquellos que lo son, halle el
sistema inverso.
(a) h (t ) (t 2)
(b) h (t ) u (t )
(c) h (t ) (t 3)
(d) h (t ) rect (t 4)
(e) h (t ) exp(t ) u (t )

151

2.14 Considere los dos sistemas mostrados en las Figs. P2.14(a) y P2.14(b). El sistema 1 opera sobre
x(t) para producir una salida y1(t) que es ptima acorde con algn criterio especificado. El sistema
II primero opera sobre x(t) con una operacin invertible (subsistema I) para obtener z(t) y
entonces opera sobre z(t) para producir una salida y2(t) mediante una operacin que es ptima de
acuerdo al mismo criterio que en el sistema I.
(a) Puede el sistema II comportarse mejor que el sistema I? (Recuerde la suposicin de que el
sistema I es la operacin ptima sobre x(t)).
(b) Reemplace la operacin ptima sobre z(t) por dos subsistemas, como lo muestra la Fig.
P2.14(c). Ahora el sistema completo trabaja tan bien como el sistema I. Puede el nuevo
sistema ser mejor que el sistema II? (Recuerde que el sistema II ejecuta la operacin ptima
sobre z(t)).
(c) Qu concluye de las partes (a) y (b)?
(d) Tiene el sistema que ser lineal para que la parte (c) sea verdad?

Sistema I

x(t)

Sistema II

Operacin
ptima
sobre x(t)

y1(t)

x(t)

Preporcesamiento
subsistema I

(a)

z(t)

Operacin ptima
sobre z(y)

(b)

z(t)

Inverso del
subsistema I

x(t)

Sistema I

y1(t)

(c)

Figura P2.14

2.15

Determine si el sistema en la Fig. P2.15 es estable (entrada acotada salida acotada)


h1 ( t ) exp( 2 t ) u ( t )

h2 ( t ) 2exp( t ) u ( t )

h3 (t ) 3 exp(t ) u (t )

h1(t)

h4 (t ) 4 (t )

h2(t)

x(t)

y(t)
h3(t)

h4(t)

Figura P2.15

y2(t)

152

2.16 Las seales en la Fig. P2.16 son la entrada y la salida de un cierto sistema LIT. Grafique las
respuestas a las entradas siguientes:
(a)

x(t 3)

x(t)

(b) 2x(t)

y(t)

(c) x(t)
(d) x(t 2) + 3x(t)
(e)

dx (t )
dt

Figura P2.16

2.17 Determine la respuesta al impulso del sistema inicialmente en reposo mostrado en la Fig. P2.17.
L
+

x(t)
x(t)

y(t)
Sistema

y(t)

Figura P2.17

2.18 Determine la respuesta al impulso del sistema inicialmente en reposo mostrado en la Fig. P2.18.
Use este resultado para hallar la salida del sistema cuando la entrada es
(a) u t 2
(b) u t 2
(c) rect ( t ), donde 1 RC

x(t ) e(t )

x(t)
C

y (t )

Figura P2.18

Sistema

y(t)

153

2.19 Repita el Problema 2.18 para el circuito mostrado en la Figura P2.19.

x(t)

x(t ) e(t )

y (t )

y(t)

Sistema

Figura P2.19

2.20 Resuelva las siguientes ecuaciones en diferencias por iteracin:


(a) y [ n ] y [ n 1] y [ n 2] x [ n ], n 0

y [ 1] 0, y [ 2] 1, x [ n ] 12

(b) y [ n ] 14 y [ n 1] 81 y [ n 2] x [ n ] 12 x [ n 1], n 0
y [ 1] 1, y [ 2] 0, x [ n ] 12

y [ n 2] x [ n ], n 0
(c) y [ n ] y [ n 1] 15
64
y [ 1] 1, y [ 2] 1, x [ n ] 2n

(d) y [ n 2] 23 y [ n 1] 19 y [ n ] x [ n ], n 0

y [1] 0, y [0] 1, x [ n ] 12

(e) y [ n ] x [ n ] 3 x [ n 1] 2 x [ n 2] x [ n 3]
x[n] u[n]

2.21 (a) Determine la respuesta al impulso del sistema mostrado en la Fig. P2.21. Suponga que

h1 [ n ] 12 u [ n ] , h2 [ n ] [ n ] 12 [ n 1] , h3 [ n ] u [ n ] u [ n 5] y h4 [ n ] 13 u [ n ] .
n

(b) Determine la respuesta del sistema a una entrada igual a un escaln unitario.
2.22 (a) Repita el Problema 2.21 si

h1 [ n ] 12 u [ n ]
n

h2 [ n ] [ n ]

h3 [ n ] h4 [ n ] 13 u [ n ]

(b) Determine la respuesta del sistema a un escaln unitario.

154

h1[n]

h2[n]

h3[n]

h4[n]

Figura P2.21

2.23 Determine las races caractersticas y las soluciones homogneas de las siguientes ecuaciones en
diferencias:
(a) y [ n ] 85 y [ n 1] 323 y [ n 2] x [ n ] x [ n 1], n 0
y [ 1] 1, y [ 2] 0

(b) y [ n ] y [ n 1] 14 y [ n 2] x [ n ], n 0
y [1] y [2] 1

(c) y [ n ] y [ n 1] 18 y [ n 2] x [ n ], n 0
y[1] = 1, y[2] = 0
(d) y [n] 3 y [n 1] 2 y [n 2] x[n], n 0
y [1] 1, y [2] 1

(e) y [ n 2] 121 y [ n 1] 121 y [ n ] x [ n ] 12 x [ n 1], n 0


y [1] 0, y [0] 1

2.24 Halle las respuestas al impulso de los sistemas en los Problemas 2.22 y 2.23.
2.25 Demuestre que cualquier sistema que pueda describirse por una ecuacin diferencial de la forma

d N y (t )
dt N

N 1

k 0

ak ( t )

d k y (t )
dt k

bk ( t )

d k x(t )

k 0

dt k

es lineal (suponga que el sistema est inicialmente en reposo).


2.26 Demuestre que cualquier sistema que pueda describirse por la ecuacin diferencial en el
Problema 2.25 es invariable en el tiempo. Suponga que todos los coeficientes son constantes.
2.27 Considere un pndulo de longitud y masa M como se muestra en la Fig. P2.27. El
desplazamiento desde la posicin de equilibrio es , por lo tanto la aceleracin es . La
entrada x(t) es la fuerza aplicada a la masa M tangencial a la direccin de movimiento de la masa.
La fuerza restauradora es la componente tangencial Mg sen . Desprecie la masa de la barra y la
resistencia del aire. Use la segunda ley del movimiento de Newton para escribir la ecuacin
diferencial que describe al sistema. Es este sistema lineal? Como una aproximacin, suponga
que es lo suficientemente pequea para la aproximacin sen 0. Es lineal este ltimo
sistema?

155

Masa M

Figura P2.27

2.28 (a) Al resolver ecuaciones diferenciales en una computadora, podemos aproximar las derivadas
de orden sucesivo con las diferencias correspondientes en incrementos del tiempo discretos, T. Es
decir, reemplazamos

y (t )

d x (t )
dt

por
x ( nT ) x (( n 1) T )

y ( nT )

y
d y (t )

z (t )

dt

d 2 y (t )
d t2

por
z ( nT )

y ( nT ) y (( n 1) T )
T

x ( nT ) 2 x (( n 1) T ) x (( n 2) T )

T2

, etc.

Use esta aproximacin para derivar la ecuacin que se usara para resolver la ecuacin
diferencial

d y (t )
dt

y (t ) x (t )

(b) Repita la parte (a) usando la aproximacin de las diferencias directas

d x (t )
dt

x (( n 1) T ) x ( nT )
T

2.29 Verifique que el sistema descrito por la ecuacin diferencial


d 2 y (t )
dt

dy ( t )
dt

b y (t ) c x (t )

156

es realizado por la interconexin mostrada en la Fig. P2.29.

x(t)

y(t)

-a

-b

Figura P2.29

2.30 Para el sistema simulado por el diagrama mostrado en la Fig. P2.30, determine las ecuaciones
diferenciales que describen el sistema.
x(t)
2

y(t)

Figura P2.30

2.31 Considere el circuito RLC en serie mostrado en la Fig. P2.31.


(a) Derive la ecuacin diferencial de segundo orden que describe el sistema.
(b) Determine los diagramas de simulacin de la primera y segunda forma.
R

L
+

x(t) +_

y(t)
_

Figura P.2.31

2.32 Dado un sistema LIT descrito por

157

y ( t ) 3 y ( t ) y ( t ) 2 y ( t ) 3 x ( t ) x ( t )
Halle los diagramas de simulacin de la primera y segunda formas cannicas.
2.33 Determine la respuesta al impulso del sistema inicialmente en reposo mostrado en la Fig. P2.33.
C
+
x( t) v( t) _+

y( t) vR ( t)

x( t)

Sistema

y( t)

Figura P2.33

2.34 Halle los dos diagramas cannicos de simulacin para los sistemas de los Problemas 2.22 y 2.23.
2.35 Resuelva las ecuaciones de estado siguientes cualquiera de los mtodos estudiados en este
captulo.
1.

2.

0
x 0

0
0
0
1 x 0 .


1 2
1
1

1
x (0) 2

3

3.

0
x 1

6 5
1
0 2 x 1 ,


2 4
2

1
x (0) 3

1

4.

1
0

x 6 11

6 11

1
2
3

6 x 1 , x (0) 2



5
0
0

5.

1
0
0
0
0

x
0
0
1 x 0 , x (0) 1



25 35 11
1
3
6.

0
3 2
1

x 1 1 1 x 1 ,


5 2 1
2

3
x (0) 2

0

7.

9
x 26

24

1
0
0

0
2
3

1 x 5 , x (0) 4



0
0
0

8.

0 1 0
0

x 2 0 1 x 0 ,


0 2 3
1

3
x (0) 2

0

0
1 10
0
3

x
0
0
1 x 0 , x (0) 4



0 20 10
5
0

158

9.

10.

0 1 0
0
x 2 0 1 x 1 ,


0 2 3
1

1 2
x 0 2

1 0

0
x (0) 5

1

11.

1
0
0 x 0 ,


1
1

1
x (0) 1

1

1
0

0 1 x 0 ,


0 3
2

3
x (0) 0

2

12.

1 1 0
1

x 0 1 0 x 0 ,


0 0 1
1

1
x (0) 1

2

0
x 1

2.36 Resuelva las ecuaciones diferenciales siguientes reducindolas primero a una ecuacin de estado.
1.

2.

3.

4.

5.

6.

7.

8.

9.

10.

d2x
dt 2
d3x
dt

d2x
dt

d4x
dt

d3x
dt

d2x
dt

d3x
dt

d2x
dt 2
d2x
dt 2
d2x
dt 2

dx
dt

d2x
dt

dx
dt

d2x
dt

dx
dt

dt

d2x
dt

19

dx
dt

dt

dx
dt

dx
dt

dt

x t 2,

dt

3,

dt

t 0

1,
d3x
dt

dt

dt

2,

12,

t 0

dx
dt

t 0

dx
dt

d2x

t 0

d2x

dt

2,

x (0) 1

5,

dx

2
t 0

3,
dx

4.25 x t 2 1,

2,

dx
dt

1,
t 0

x (0) 0

t 0

t 0

dt

1,
t 0

x (0) 2

t 0

dt

x (0) 1

t 0

x (0) 1

6,

d2x

dx

dx

5,

t 0

13 x 5,

x 10 e 2 t ,
2

dt

t,

x (0) 1.

d2x

dx

20,

dx

2,
t 0

4 x 4,

5 x 8sen t ,

d2x
dt

dx

dt

4 x 5t ,

d3x
dt

dx

2 x 4,

1,

x (0) 0

dx

1,

t 0

dt

t 0

x (0) 0

x (0) 2

x (0) 1

159

11.

12.

d3x
dt

d3x
dt 3

d2x
dt

dx
dt

5x t ,

x e 3 t ,

d2x
dt
d2x
dt 2

dt

t 0

2,
t 0

dx

2,

dx
dt

5,

6,
t 0

x (0) 3

t 0

x (0) 5

CAPTULO TRES

ANLISIS DE FOURIER
TIEMPO CONTINUO

Introduccin

La representacin de la seal de entrada a un sistema (entendiendo como sistema un conjunto de


elementos o bloques funcionales conectados para alcanzar un objetivo deseado) de tiempo continuo
lineal e invariable en el tiempo (LIT), como una integral ponderada de impulsos desplazados conduce a
la integral de convolucin. Esta representacin de sistemas LIT de tiempo continuo indica cmo la
respuesta de tales sistemas, para una entrada arbitraria se construye a partir de las respuestas a los
impulsos unitarios desplazados. Entonces, la integral de convolucin no slo proporciona una manera
conveniente de calcular la respuesta de un sistema LIT, suponiendo conocida su respuesta al impulso
unitario, sino que tambin indica que las caractersticas de un sistema LIT son especificadas
completamente por su respuesta al impulso unitario. A partir de este hecho se pueden analizar en
detalle muchas de las propiedades de los sistemas LIT y relacionar estas propiedades con las
caractersticas equivalentes de las respuestas al impulso de tales sistemas.
En este trabajo se desarrollar una representacin alterna para las seales y los sistemas LIT. El punto
de partida de esta discusin es el desarrollo de una representacin de seales como sumas ponderadas
de un conjunto de seales bsicas, las seales exponenciales complejas. Las representaciones
resultantes son conocidas como la serie y la transformada de Fourier y, como se ver, estas
representaciones se pueden emplear para construir diferentes clases de seales. La idea central de la
representacin es la siguiente: Dada una secuencia de funciones u1(t), u2(t), u3(t), que tienen, en un
intervalo (a, b), la propiedad de ortogonalidad:
b

( t ) un ( t ) dt 0

siempre que n m (el asterisco indica el conjugado complejo), se desea expandir una funcin
arbitraria f (t) en una serie infinita de la forma
f ( t ) C1u1 ( t ) C2 ( t ) u2 ( t ) C3 ( t ) u3 ( t )

A primera vista, esto parece bastante sencillo. Para determinar Cn para cualquier valor fijo de n,
multiplicamos ambos lados de esta ecuacin por un ( t ) e integramos en el intervalo (a, b):

162

f ( t ) u ( t ) dt C u ( t ) u ( t ) dt C u ( t ) u ( t ) dt

Debido a la propiedad de ortogonalidad, todos los trminos en el lado derecho se anulan excepto el nsimo y se obtiene
b

f ( t ) un ( t ) dx Cn

u (t )
n

dt

y esta relacin se puede resolver para obtener Cn.


Las seales con las cuales se trabaja normalmente son magnitudes variables en el tiempo; por
ejemplo, en las comunicaciones elctricas, ellas son el voltaje y la corriente. La descripcin de una
seal x(t) usualmente existe en el dominio del tiempo, donde la variable independiente es el tiempo t.
Para muchas aplicaciones, con frecuencia es ms conveniente describir las seales en el dominio de la
frecuencia f, donde la variable independiente es la frecuencia f. As, si la seal existe fsicamente en el
dominio del tiempo, entonces se puede decir que ella consiste de diferentes componentes en el dominio
de la frecuencia y esas componentes en conjunto se denominan el espectro.
El anlisis espectral basado en la serie y la transformada de Fourier es una herramienta poderosa en
muchas ramas de la ingeniera. Como consecuencia, la atencin en este trabajo se centrar en la teora
de Fourier antes que en otras tcnicas como, por ejemplo, la transformacin de Laplace y el anlisis en
el dominio del tiempo. Primero, el dominio de la frecuencia es esencialmente un punto de vista de
rgimen permanente y, para muchos propsitos, es razonable restringir nuestra atencin al
comportamiento en rgimen permanente de los sistemas bajo estudio. En realidad, teniendo en cuenta
la multitud de seales posibles que un sistema puede manejar, sera imposible encontrar soluciones
detalladas de las respuestas transitorias para cada una de ellas. Segundo, el anlisis espectral permite
considerar clases completas de seales que poseen propiedades similares en el dominio de la
frecuencia. Esto no slo conduce a un conocimiento ms profundo en el anlisis, sino que es de gran
valor para el diseo. Tercero, muchas de las componentes de un sistema se pueden clasificar como
dispositivos lineales e invariables en el tiempo; siendo as, se pueden describir por sus caractersticas
de respuesta en frecuencia, las cuales, a su vez, facilitan an ms el anlisis y el trabajo de diseo.
Por lo tanto, este captulo est dedicado al anlisis de seales y sus respectivos espectros, poniendo
atencin especial a la interpretacin de las propiedades de esas seales en el dominio de las
frecuencias. Se examinarn los espectros de lneas basados en la expansin en serie de Fourier para
seales peridicas y en los espectros continuos basados en la transformada de Fourier de seales
aperidicas. Finalmente, estos dos espectros se conjugarn con la ayuda del concepto de la respuesta al
impulso.
Como primer paso, se deben escribir las ecuaciones que representan las seales en funcin del tiempo,
pero sin olvidar que esas ecuaciones son slo modelos matemticos del mundo real y, por lo general,
son modelos imperfectos. En efecto, una descripcin completamente fiel de la seal fsica ms sencilla
sera demasiado complicada e imprctica para los propsitos de la ingeniera. Por lo tanto, se tratar de
idear modelos que representen con una complejidad mnima las propiedades significativas de las
seales fsicas. El estudio de muchos modelos diferentes para las seales proporciona la informacin
necesaria para seleccionar modelos apropiados para aplicaciones especficas.

163

3.1 Respuesta de Sistemas LIT a Exponenciales Complejas

En el estudio de sistemas LIT es ventajoso representar las seales como combinaciones lineales de
seales bsicas que posean las propiedades siguientes:
1. El conjunto de seales bsicas se puede usar como base para construir una clase de seales amplia y
de mucha utilidad.
2. La estructura de la respuesta de un sistema LIT a cada seal bsica debe ser lo suficientemente
sencilla como para proporcionar una representacin conveniente de la respuesta del sistema a
cualquier seal construida como una combinacin lineal de seales bsicas.
En sistemas LIT de tiempo continuo, estas dos ventajas son proporcionadas por el conjunto de
exponenciales complejas de la forma est, en donde s es una variable compleja. Considrese, por
ejemplo, un sistema LIT con entrada x(t) y cuya funcin de transferencia H(j), s = j en este caso, se
define de tal forma que cuando x ( t ) e jt , la salida es igual a H ( j) e jt ; es decir,

H ( j )

y(t )
x(t )

(3.1)
x ( t ) e

jt

Combinando la Ec. (3.1) con el principio de superposicin, se deduce que si x(t) es una combinacin
lineal de seales exponenciales, digamos
x ( t ) a1 e j1t a2 e j2 t

(3.2)

donde a1, a2,, son constantes, entonces


y ( t ) a1 H ( j1 ) e j1t a2 H ( j2 ) e j2 t

(3.3)

y H(j1) representa la funcin H(j) evaluada en 1, etc.


Generalizando a la seal compleja est, la respuesta de un sistema LIT a este tipo de seal, como se
ver ms adelante, es la misma exponencial compleja modificada por un factor de multiplicacin, es
decir,

est H s e st

(3.4)

donde el factor complejo H(s), llamado la funcin de transferencia, ser en general una funcin de la
variable compleja s. Una seal para la cual la respuesta del sistema es igual a la entrada multiplicada
por una constante (posiblemente compleja) se conoce como una funcin caracterstica del sistema, y el
factor de amplitud se conoce como un valor caracterstico. O sea que el valor caracterstico de un
sistema LIT de tiempo continuo asociado con la funcin caracterstica e st est dado por H(s) cuyo
valor lo determina el valor de s a travs de la Ec. (3.7) (ms adelante).
Para mostrar que las exponenciales complejas son en efecto funciones caractersticas de los sistemas
LIT, considrese uno cuya respuesta al impulso es h(t). Para una entrada x(t), se puede determinar la
salida empleando la integral de convolucin, de manera que si x ( t ) est , se tiene que

164

y(t )

h( ) x(t ) d

h( ) e

s ( t )

(3.5)

st

h( ) e

Entonces la respuesta a la excitacin exponencial est es de la forma

y t H s est

(3.6)

donde H(s) es una respuesta compleja cuyo valor depende de s y que est relacionada con la
respuesta al impulso del sistema por

H (s)

h( )e

(3.7)

Como ya se vio, si x(t) es una combinacin lineal de exponenciales complejas aplicada a un sistema
LIT, la respuesta es la suma de las respuestas a cada una de las exponenciales por separado. En forma
general,

a e
k

sk t

a H ( s
k

) e sk t

(3.8)

Entonces, para un sistema LIT, si se conocen los valores caractersticos H(sk), la respuesta a una
combinacin lineal de exponenciales complejas se puede obtener de manera directa.

3.2

Representacin de Seales Usando Series de Fourier

3.2.1

Seales Peridicas y Combinaciones Lineales de Exponenciales Complejas

Una seal es peridica si para algn valor de T diferente de cero la seal obedece la relacin
x ( t ) x ( t T ),

(3.9)

El menor valor de T > 0 que satisface la Ec. (3.9) se denomina el perodo fundamental de x(t), T0, o
simplemente el perodo de x(t). Observe que la definicin dada en (3.9) tambin puede escribirse en la
forma
x ( t ) x ( t mT0 ),

(3.10)

donde m es un entero. Esta ltima ecuacin simplemente dice que desplazando la seal un nmero
entero de perodos hacia la izquierda o hacia la derecha en el eje del tiempo no produce cambios en la
onda. Como consecuencia, una seal peridica se describe completamente especificando su conducta

165

en cualquier perodo. Una seal para la cual no existe ningn valor de T que satisfaga la Ec. (3.9) se
denomina no-peridica o aperidica.
El valor

(3.11)

T0

se conoce como la frecuencia angular fundamental (en rad/s).


Dos seales bsicas conocidas son la sinusoide
x ( t ) cos 0 t

(3.12)

x ( t ) e j0 t

(3.13)

y la exponencial compleja peridica

Estas dos seales son peridicas con frecuencia fundamental 0 y perodo fundamental
T0 2 0 1 f0 , donde f0 es la frecuencia fundamental (en Hz). Para la funcin x ( t ) cos 0 t y
cualquier valor de t, se tiene que
2
cos 0 t T0 cos 0 t
cos 0 t
0

lo que muestra que su perodo fundamental es T0 2 0 .


Con la seal de la Ec. (3.13) se encuentra asociado el conjunto de funciones exponenciales
complejas relacionadas armnicamente,

k e

jk 0 t

e j 2 kf0 t ,

k 0, 1, 2,

(3.14)

Cada una de estas seales tiene una frecuencia fundamental que es un mltiplo de 0 y, por lo tanto,
cada una de ellas es peridica con perodo fundamental T0 [aunque para | k | 2 el perodo fundamental
de k(t) es una fraccin de T0]. As que una combinacin lineal de exponenciales complejas
relacionadas armnicamente de la forma

x (t )

c e
k

jk 0 t

c e
k

j 2 kf 0 t

(3.15)

es tambin peridica con perodo T0 . En la Ec. (3.15), el trmino para k = 0 es un trmino constante o
CD. Los dos trminos k = +1 y k = 1 tienen ambos un perodo fundamental igual a T0 y se conocen
como las componentes fundamentales o como las primeras componentes armnicas. Los dos trminos
para k = +2 y k = 2 son peridicos con la mitad del perodo (o, equivalentemente, el doble de la
frecuencia) de las componentes fundamentales y se les conoce como las componentes de la segunda
armnica. Ms generalmente, las componentes para k N y k = N se conocen como las
componentes de la N-sima armnica.
La representacin de una seal peridica es un espectro de lneas obtenido mediante una expansin
en serie de Fourier como la de la Ec. (3.15). La expansin requiere que la seal tenga potencia

166

promedio finita. Como la potencia promedio y otros promedios temporales son propiedades
importantes de las seales, ahora procederemos a formalizar estos conceptos.
Dada cualquier funcin x(t), su valor promedio para todo el tiempo se define como
x ( t ) lm

1
T

T /2

(3.16)

x ( t ) dt

T /2

La notacin x(t) representa la operacin de promediar, la cual comprende tres pasos: (i) integrar x(t)
para obtener el rea bajo la curva en el intervalo T/2 t T/2; (ii) dividir esa rea por la duracin T
del intervalo de tiempo, y (iii) hacer que T para cubrir todo el tiempo. En el caso de una seal
peridica, la Ec. (3.16) se reduce al promedio durante cualquier intervalo de duracin T0, vale decir,

x(t )

T0

t1 T0

x ( t ) dt

T0

t1

x ( t ) dt

(3.17)

T0

donde t1 es un tiempo arbitrario y la notacin To representa una integracin desde cualquier valor
arbitrario t1 hasta t1+T0, es decir, integracin por un perodo completo.
Si x(t) es el voltaje entre las terminales de una resistencia R, se produce la corriente i(t) = x(t)/R y se
puede calcular la potencia promedio resultante, promediando la potencia instantnea
x ( t ) i ( t )x 2 ( t ) R Ri 2 ( t ) . Pero no necesariamente se sabe si una seal dada es un voltaje o una
corriente, as que se normalizar la potencia suponiendo de aqu en adelante que R = 1 . La definicin
de la potencia promedio asociada con una seal peridica arbitraria se convierte entonces en

x(t )

1
T0

x(t )

dt

(3.18)

T0

donde se ha escrito x(t) 2 en lugar de x2(t) para permitir la posibilidad de modelos de seales
complejas. En cualquier caso, el valor de P ser real y no negativo.
Cuando la seal en la Ec. (3.18) existe y da como resultado que 0 < P < , se dice que la seal x(t)
tiene una potencia promedio bien definida y se denominar una seal de potencia peridica. Casi todas
las seales peridicas de inters prctico caen en esta categora. El valor promedio de una seal de
potencia puede ser positivo, negativo o cero, pero est acotado por

x ( t ) PT0
valor que proviene de la relacin integral

x ( t ) dt

x ( t ) dt

Algunos promedios de seales pueden determinarse por inspeccin, usando la interpretacin fsica
del promedio. Como un ejemplo especfico, considrese la sinusoide
x ( t ) A cos ( 0 t )

para la cual

167

x ( t ) 0,

A2
2

La energa disipada por la seal x ( t ) en el intervalo de tiempo (T/2, T/2) est dada por
T 2

x(t )

(3.19)

dt

T 2

Se dice que una seal x(t) cualquiera es una seal de energa si y slo si 0 < E < , donde
T 2

E lm

3.2.2

x(t )

(3.20)

dt

T 2

Series de Fourier

La representacin de una seal peridica en la forma de la Ec. (3.15) se conoce como la


representacin en serie de Fourier. Especficamente, sea x(t) una seal de potencia con perodo
T0 2 0 1 f0 . Su expansin en una serie de Fourier exponencial es
x (t )

c e

jk 0 t

c e

j 2 kf 0 t

(3.21)

Si x(t) es una funcin real, entonces x*(t) = x(t) y, por tanto,


x *( t ) x ( t )

c e

jk 0 t
k

Reemplazando k por k en la sumatoria, se tiene que

x (t )

e jk 0 t

la cual, al compararla con la Ec. (3.21), requiere que ck c k , o, en forma equivalente, que
ck c k

(3.22)

Ahora se determinarn los coeficientes ak. Multiplicando ambos lados de (3.21) por e jn0 t , se obtiene
x ( t ) e jn 0 t

c e
k

jk 0 t

e jn 0 t

Ahora se integra ambos lados de esta relacin desde 0 hasta T0 = 2/0 y se obtiene

(3.23)

168

T0

T0

x (t ) e

j n 0 t

dt

c e

jk 0 t

e jn 0 t dt

0 k

donde T0 es el perodo fundamental de x(t). Intercambiando el orden de la integracin y la sumatoria,


da
T0

x(t ) e

j n o t

T0

dt
ak e j ( k n ) 0 t dt
0

(3.24)

Aqu se est suponiendo que las condiciones sobre la integracin y la serie son tales que permiten el
intercambio. Para k n,
T0

e j ( k n ) 0 t dt

1
j ( k n ) 0

j ( k n ) 0 t T0

e j ( k n ) 2 1 0
j ( k n ) 0

puesto que e j ( k n ) 2 1 (k y n son enteros). Si k = n, entonces


T0

dt T

y, por tanto, el lado derecho de la Ec. (3.24) se reduce a T0 c n. Por consiguiente,

cn

1
T0

T0

x (t )e
0

jn0 t

dt

1
T0

T0

x (t ) e

j 2 nf 0 t

dt

k n

(3.25)

la cual es la relacin buscada para determinar los coeficientes. La integracin en (3.25) es para
cualquier intervalo de longitud T0.
A la Ec. (3.21) a menudo se le refiere como la ecuacin de sntesis y a la Ec. (3.25) como la ecuacin
de anlisis. Los coeficientes {ck} se conocen como los coeficientes de la serie de Fourier de x(t) o los
coeficientes espectrales de x(t). Puesto que, en general, los coeficientes son cantidades complejas, se
pueden expresar en la forma polar

ck ck e j arg ak
As que la Ec. (3.21) expande una seal peridica como una suma infinita de fasores, siendo

ck e jk0 t ck e j arg ak e jk0 t


el trmino k-simo.
Obsrvese que x(t) en la Ec. (3.21) consiste de fasores con amplitud ck y ngulo arg(ck) en las
frecuencias k0 = 0, 0, 20, . De aqu que el grfico correspondiente en el dominio de la
frecuencia sea un espectro de lneas bilateral definido por los coeficientes de la serie. Se da un mayor
nfasis a la interpretacin espectral escribiendo
c(k0) = ck

169

tal que a k0 representa el espectro de amplitud en funcin de f u 0 y arg[c(k0)] representa el


espectro de fase. La Ec. (3.22) da una propiedad espectral importante para seales de potencia
peridicas reales. Otras dos propiedades importantes para seales de potencia peridica son:
1. Todas las frecuencias son nmeros enteros mltiplos o armnicos de la frecuencia fundamental
0 2 / T0 2 f0 . As que las lneas espectrales tienen una separacin uniforme 0 (o f0).
2. La componente CD es igual al valor promedio de la seal, ya que al hacer k = 0 en la Ec. (3.25) da

c0

T0

x ( t ) dt

(3.26)

x (t )

T0

Tambin, de la Ec. (3.22) se deduce que para x(t) real, entonces

c k ck* ck e j arg ak
y as se obtiene una tercera propiedad:
3.

c k 0 c k 0 arg c k 0 arg c k 0

(3.27)

lo cual significa que el espectro de amplitud tiene simetra par y el de fase simetra impar.
La propiedad dada por la Ec. (3.22) para seales de valores reales permite reagrupar la serie
exponencial en pares de conjugados complejos, excepto por a0, en la forma siguiente:
m 1

x ( t ) c0

c e

jm0 t

c0

c n e jn0 t

c e

jn0 t

n 1

e jn0 t cn e jn0 t

n 1

c0

jm0 t

m 1

n 1

c0

c e

2 Re c e
jn0 t

n 1

c0

2 Re c cos n t 2 Im c sen t
n

n 1

Esta ltima ecuacin puede escribirse como

x ( t ) a0

cos n0 t bn sen n0 t

(3.28)

n 1

La expresin para x(t) en la Ec. (3.28) se conoce como la serie trigonomtrica de Fourier para la
seal peridica x(t). Los coeficientes cn y dn estn dados por

170

a0 c0

1
T0

x (t ) dt

T0

an 2 Re cn

2
T0

bn 2 Im cn

x ( t ) cos n t dt
0

(3.29)

T0

2
T0

x ( t )sen n t dt
0

T0

En funcin de la magnitud y la fase de cn, la seal de valores reales x(t) puede expresarse como

x ( t ) c0

2 c

cos n0 t arg an

n 1

c0

A cos n t
n

(3.30)

n 1

donde

An 2 cn

(3.31)

n arg cn

(3.32)

La Ec. (3.30) representa una forma alterna de la serie de Fourier que es ms compacta y ms clara que
la Ec. (3.28) y se conoce como la forma armnica de la serie de Fourier de x(t). Cada trmino en la
serie representa un oscilador necesario para generar la seal peridica x(t).
Los coeficientes de la serie de Fourier de una seal se muestran en un conjunto de dos grficas en el
dominio de la frecuencia, los espectros de lneas. Una grfica de | cn | y arg(cn), lneas, versus n o n0
(nf0) para valores positivos y negativos de n o n0 (nf0) se denomina un espectro bilateral de amplitud
o de fase. La grfica de An y n versus n o n0 (nf0) para n positiva se denomina un espectro unilateral.
Se debe sealar que la existencia de una lnea en una frecuencia negativa no implica que la seal est
formada por componentes de frecuencias negativas, ya que con cada componente cnexp(jn0t) est
asociada una correspondiente de la forma c n exp ( jn0 t ) . Estas seales complejas se combinan para
crear la componente real an cos n0 t bn sen n0 t .

Ejemplo 1. Considere la seal


x ( t ) 1 sen 0 t 2cos 0 t cos(2 0 t 4)

Aqu se podra aplicar la Ec. (3.28) para obtener los coeficientes de la serie de Fourier. Sin embargo,
para este caso es ms sencillo expandir las funciones sinusoidales como una combinacin lineal de
exponenciales complejas e identificar por inspeccin los coeficientes de la serie; as tenemos que
x ( t ) 1 21j e j0 t e j0 t e j0t e j0t 12 e j ( 2 0t / 4 ) e j ( 0t / 4 )

171

Agrupando trminos se obtiene

x ( t ) 1 1 21j e j0 t 1 21j e j0 t 12 e j / 4 e j 2 0 t 12 e j / 4 e j 2 0 t
y los coeficientes de Fourier para este ejemplo son
c0 1

c1 1 21j 1 12 j

c1 1 21j 1 12 j
c2 12 e j / 4

2
4

c2 12 e j / 4
ck 0,

(1 j )
2
4

(1 j )

k 2

En la Fig. 3.1 se grafican la magnitud y la fase de ck.

ck

ck
...

...

...

2 1 0 1 2

1 0

...

Figura 3.1

Ejemplo 2. Tren de Pulsos Rectangulares


Considrese el tren peridico de pulsos rectangulares en la Fig. 3.2. Cada pulso tiene una altura o
amplitud A y una anchura o duracin . Hay discontinuidades escalonadas al comienzo y al final de
cada pulso en t 2 , etc., de manera que los valores de x(t) no estn definidos en estos puntos de
discontinuidad. Esto pone de manifiesto otra posible diferencia entre una seal fsica y su modelo
matemtico, ya que una seal fsica nunca hace una transicin escalonada perfecta. Sin embargo, el
modelo todava puede ser razonable si los tiempos de transicin efectivos son pequeos en
comparacin con la duracin del pulso.
Para calcular los coeficientes de Fourier, se toma como intervalo de integracin en la Ec. (3.25)
el perodo central T0 2 t T0 2 , donde

A,

x(t )

0,

t /2
t /2

172

x(t)
A
...

...
0

T0

T0

Figura 3.2

Entonces,

ck

1
T0

T0 2

x (t ) e

jk 0 t

dt

T0 2

jk 0 T0

jk 0 2

1
T0

Ae

jk 0 t

dt

e jk 0 2

(3.33)

A sen ( k 0 2)
T0

k 0 2

Antes de continuar con este ejemplo, se derivar una expresin que aparece repetidamente en el anlisis
espectral. Esta expresin es la funcin sinc definida por
sinc

sen

(3.34)

La Fig. 3.3 muestra que la funcin sinc es una funcin de que tiene su valor pico en = 0 y sus
cruces con cero estn en todos los otros valores enteros de , as que
0

1
sinc
0

1, 2,

En trminos de la funcin definida por la Ec. (3.34), la ltima igualdad en la Ec. (3.33) se convierte
en

ck

A
T0

donde 0 = 2f0.

sinc k 0 2

A
T0

sinc kf 0

173

sinc
1.0

3 2

Figura 3.3

El espectro de amplitudes para c ( kf0 ) ck Af0 sinc ( f ) se muestra en la Fig. 3.4a para el
caso /T0 = f0 = 1/4.
c(kf 0 )

Af 0

0 f0

(a
)

arg[c(kf0 )]
180

180
(b)

Figura 3.4. Espectro del tren de pulsos rectangulares con f c 1 4 .


(a) Amplitud; (b) Fase.

Esta grfica se construye dibujando la funcin continua Af 0 sinc f como una curva punteada, la
cual se convierte en la envolvente de las lneas. Las lneas espectrales en 4f0, 8f0, etc., no aparecen,
ya que ellas caen a cero precisamente en mltiplos de 1/ donde la envolvente es igual a cero. La
componente CD tiene amplitud c(0) = A/T0, lo que debe reconocerse como el valor promedio de x(t) a
partir una inspeccin de la Fig. 3.2. Incidentalmente, /T0 es la relacin entre el tiempo cuando la onda
es diferente de cero y su perodo, y frecuentemente se designa como el ciclo de trabajo en la
electrnica de pulsos.

174

El espectro de fase en la Fig. 3.4b se obtiene observando que ak es siempre real pero algunas veces
negativa. Por lo tanto, arg[c(kf 0 )] toma los valores 0 y 180, dependiendo del signo de sinc(kf0. Se
us +180 y 180 para resaltar la simetra impar de la fase.

Ejemplo 3. El Rectificador de Media Onda


Un voltaje sinusoidal E sen 0 t se pasa por un rectificador de media onda que elimina la porcin
negativa de la onda, como se muestra en la Fig. 3.5. Este tipo de seales puede encontrarse en
problemas de diseo de rectificadores.
x(t)

E sen 0t

E
...

...

Figura 3.5

La representacin analtica de x(t) es

0,

x (t )
E sen t ,
0

cuando

t 0

cuando 0 < t <

y x(t + 2/0) = x(t). Como x(t) = 0 cuando /0 < < 0, de la Ec. (3.25) obtenemos
T0

cn

1
T0

E sen t exp j
0

/ 0

E 0

E 0
2

2 nt
dt
T0

exp j0 t exp j0 t exp jn0 t dt


2j

/ 0

exp j n 1 t exp j n 1 t dt
0

E exp jn 2
jn
jn
exp
exp

2
2 (1 n )
2
2

E
(1 n2 )

cos n / 2 exp jn / 2 ,

n 1

(3.35)

175

(1 n 2 )
0,

n par

(3.36)

n impar, n 1

Haciendo n = 0, se obtiene la componente CD o el valor promedio de la seal peridica como


c0 E . El resultado puede verificarse calculando el rea bajo medio ciclo de una onda seno y
dividiendo por T0. Para determinar los coeficientes a1 y a 1 que corresponden a la primera armnica,
observe que no se puede sustituir n = 1 en la Ec. (3.36), puesto que ello produce una cantidad
indeterminada. En su lugar usamos la Ec. (3.35) con n = 1 lo cual resulta en
c1

y c1

4j

E
4j

Los espectros de lneas de x(t) se muestran en la Fig. 3.6.


E/
E/
E/3

E/15
4

cn

E/
E/3

(a)

cn

E/15

(b)

Figura 3.6

3.2.3

Condiciones para la Convergencia de las Series de Fourier

Hemos visto que una seal peridica puede ser aproximada por un nmero finito de trminos de su
serie de Fourier. Pero converge la serie infinita a x(t)? Para entender mejor la cuestin de la validez de
las representaciones mediante series de Fourier, considrese primero el problema de aproximar una
seal peridica x(t) dada, por una combinacin lineal de un nmero finito de exponenciales complejas
relacionadas armnicamente, es decir, por una serie finita de la forma
N

xN ( t )

c e

jk 0 t

k N

Denote por eN(t) el error de la aproximacin, el cual est dado por

(3.37)

176

eN ( t ) x ( t ) xN ( t ) x ( t )

c e

jk 0 t

(3.38)

k N

Para determinar la bondad de cualquier aproximacin en particular, es necesario especificar una


medida cuantitativa del tamao del error de la aproximacin. El criterio que se usar es el de la
magnitud total del error al cuadrado en un perodo:

EN

( t ) 2 dt eN ( t ) eN* ( t ) dt

T0

(3.39)

T0

En general, para cualquier seal z(t), la cantidad, definida anteriormente,


b

z(t )

dt

es la energa en z(t) en el intervalo de tiempo a t b. Esta terminologa es motivada por el hecho de


que si z(t) corresponde, por ejemplo, a la corriente que fluye en un resistor de 1 , entonces E es la
energa total disipada en el resistor durante el intervalo de tiempo a t b. Con respecto a la Ec. (3.39)
, EN representa entonces la energa en el error de aproximacin durante un perodo.
Ahora se proceder a demostrar que la escogencia particular para los coeficientes ck en la Ec. (3.37)
minimiza la energa en el error (error cuadrtico) y da para los coeficientes:

ck

1
T0

x (t ) e

jk 0 t

dt

(3.40)

T0

Suponga que se tiene la funcin f(t) y que se desea representarla mediante un conjunto de funciones en
el intervalo finito [t1, t2]. Suponga tambin que estas funciones 1(t), 2(t), , n(t) son ortogonales en
el intervalo [t1, t2], es decir,
t2

(t )
i

t1

0, i j
( t ) dt
ki , i j

(3.41)

La representacin de f(t) en [t1, t2] es una combinacin lineal de las funciones i(t), i 1, 2, , n , es
decir,
f ( t ) c1 1 ( t ) c2 2 ( t ) cn n ( t )

(3.42)

En la Ec. (3.42) no aparece el signo de igualdad debido a que, en general, la representacin


n

c (t )
i

(3.43)

i 1

contiene algn error. Se desea que la representacin o aproximacin est cerca de f(t). Uno de los
criterios ms utilizados para elegir una aproximacin es el de minimizar el error cuadrtico entre el
valor real de f(t) y la aproximacin (3.43). Es decir, las c i , i = 1, 2, , n, se eligen para minimizar la
cantidad

177

t2

EC

f (t )

i 1

ci i ( t ) dt

(3.44)

t1

Evidentemente, el integrando de (3.44) es el error cuadrtico. La integral da el error cuadrtico en el


intervalo [t1, t2]. La Ec. (3.44) se puede escribir como
t2

EC f ( t ) c11 ( t ) c2 2 ( t ) cn n (t ) 2 dt
t1

t2

f 2 ( t ) c12 12 ( t ) c22 22 ( t ) cn2 2n ( t ) 2 c11 ( t ) f ( t ) 2 c2 2 ( t ) f ( t ) 2 cn n ( t ) dt

(3.45)

t1

t2

(t ) dt c12 k1 c22 k 2 cn2 k n 2c11 2c2 2 2cn n

t1

donde i, i =1, 2, , n, se define como


t2

f ( t ) ( t ) dt

(3.46)

t1

c k

2
i i

Ahora se completa el cuadrado de cada uno de los trminos

2 ci i , sumando y restando

i2 ki , es decir,

c k 2 ci i ci ki i
2
i i

ki

i2

(3.47)

ki

y la expresin dada por (3.44) se puede escribir como


t2

EC

f 2 ( t ) dt

ci ki i

i 1

t1

i2

k
2

ki

i 1

(3.48)

Segn la Ec. (3.44), es evidente que el EC es siempre mayor o igual a cero; es decir, EC 0. En la Ec.
(3.48), se observa que la relacin para el EC toma su menor valor cuando

ci ki

i 1, 2, , n

ki

Es decir, los coeficientes c i se deben elegir como


t2

ci

i
ki

f ( t ) ( t ) dt
i

t1

(3.49)

t2

( t ) ( t ) dt
i

t1

*
i

178

Para el caso de la expansin de Fourier, las funciones i(t) corresponderan a las exponenciales e jk 0 t
(cambiando i por k), el intervalo [t1, t2] correspondera al perodo T0 y c i correspondera a ck.
Comparando la Ec. (3.40) con la Ec. (3.25), vemos que la primera es idntica a la expresin usada
para determinar los coeficientes de la serie de Fourier. Entonces, si x(t) tiene una representacin en
serie de Fourier, la mejor aproximacin usando slo un nmero finito de exponenciales complejas
relacionadas armnicamente se obtiene truncando la serie de Fourier en el nmero deseado de
trminos. Conforme N aumenta, se aaden nuevos trminos pero los anteriores permanecen inalterados
y EN disminuye. Si, efectivamente, x(t) tiene una representacin en serie de Fourier, entonces el lmite
de EN conforme N es cero.
Pongamos atencin ahora al problema de la validez de la representacin mediante series de Fourier
de seales peridicas. Para cualquiera de estas seales se puede intentar obtener un conjunto de
coeficientes de Fourier mediante el uso de la Ec. (3.25). Sin embargo, en algunos casos la integral en la
Ec. (3.25) puede divergir; es decir, el valor de las ck puede ser infinito. An ms, si todos los
coeficientes de la Ec. (3.25) son finitos, cuando estos coeficientes se sustituyen en la ecuacin de
sntesis (3.21), la serie infinita resultante puede no converger hacia la seal original x(t). Sin embargo,
sucede que no hay dificultades de convergencia si x(t) es continua. Es decir, toda seal peridica
continua tiene una representacin en serie de Fourier tal que la energa EN en el error de aproximacin
tiende a cero conforme N . Esto tambin es vlido para muchas seales discontinuas. Puesto que
ser de mucha utilidad usar seales discontinuas, tales como la onda cuadrada del Ejemplo 2, es
importante considerar con ms detalle el problema de la convergencia. Se discutirn dos condiciones
algo diferentes que debe cumplir una seal peridica para garantizar que pueda ser representada por
una serie de Fourier.
En muchos problemas prcticos se nos da la funcin x(t) y la usamos para construir una serie de
Fourier. Por consiguiente, estaremos interesados en teoremas que nos digan algo bueno sobre la
expansin en serie de x, con tal que x a su vez sea buena en algn sentido. Un teorema tpico de esta
clase es el siguiente: Si x(t) integrable en el intervalo (0, T0), su serie de Fourier converger a x(t) en
cualquier punto t (0 < t < T0) donde x(t) sea diferenciable. Observe que el teorema no dice nada sobre
qu sucede en los puntos extremos. Todo lo que dice es que si 0 < t < T0 y si x'(t0) existe cuando t = t0,
entonces la serie converge cuando t = t0 y su suma es x(t0).
Una clase de funciones peridicas representable mediante series de Fourier es aquella que incluye
seales cuyo cuadrado es integrable sobre un perodo. Es decir, cualquier seal x(t) en esta clase tiene
energa finita en un solo perodo:

x ( t ) 2 dt

(3.50)

T0

Cuando se cumple esta condicin, se garantiza que los coeficientes a k obtenidos a partir de la Ec.
(3.40) son finitos. Adicionalmente, sea xN(t) la aproximacin a x(t) usando estos coeficientes para | k|
N, es decir,
N

xN ( t )

c e

jk 0 t

k N

Entonces, se cumple que lm EN 0, donde EN se define en la Ec. (3.39). Es decir, si definimos


N

(3.51)

179

e(t ) x (t )

c e

jk 0 t

(3.52)

se obtiene que

e ( t ) dt 0
2

(3.53)

T0

Como se ver en un ejemplo ms adelante, la Ec. (3.53) no implica que la seal x(t) y su representacin
en serie de Fourier

c e

jk 0 t

(3.54)

sean iguales para todo valor de t. Lo que ella dice es que su diferencia no contiene energa.
Un conjunto alterno de condiciones, desarrolladas por Dirichlet, y tambin cumplidas por
esencialmente todas las seales que nos interesan, garantiza que x(t) ser efectivamente igual a su
expansin, excepto en valores aislados para los cuales x(t) es discontinua. En estos valores de t, la serie
infinita de (3.54) converge al valor promedio de la discontinuidad; es decir, si x(t ) tiene una
discontinuidad en t0, la serie converge al valor dado por
lm

x ( t0 ) x ( t 0 )

Las condiciones de Dirichlet para la expansin en serie de Fourier son: Si una funcin peridica x(t)
es acotada, tiene un nmero finito de mximos, mnimos y discontinuidades por perodo, y si x(t) es
absolutamente integrable en cualquier perodo, es decir,

x ( t ) dt

(3.55)

T0

entonces la serie de Fourier existe y converge uniformemente dondequiera que x(t) sea continua. Dicho
de otra forma, si una funcin peridica x(t) es continua por tramos, entonces es integrable en el sentido
dado por la Ec. (3.55) en cualquier intervalo de longitud finita y, en especial, en uno de longitud T0, y
converge a x(t) dondequiera que la funcin sea continua y a x t x t 2 en todo punto t donde
posea ambas derivadas por la derecha y por la izquierda. Vase teorema ms adelante.
Ahora bien, si la seal x(t) es absolutamente integrable o cuadrado integrable, la serie exhibe una
conducta conocida como el fenmeno de Gibbs en los puntos de discontinuidad. La Fig. 3.7 ilustra esta
conducta para una discontinuidad de tipo escaln en t = t0. La suma parcial xN(t) converge al punto
medio de la discontinuidad, lo cual parece muy razonable. Sin embargo, a cada lado de la
discontinuidad, xN(t) tiene sobrepasos oscilatorios con perodo T0/2N y valor pico de aproximadamente
18% de la altura del escaln e independiente de N. As que, conforme N , las oscilaciones colapsan
formando picos denominados lbulos de Gibbs por encima y por debajo de la discontinuidad.
Puesto que una seal real debe ser continua, el fenmeno de Gibbs no ocurre y tenemos justificacin
para tratar a x(t) y su representacin en serie de Fourier como idnticas; pero modelos de seales

180

idealizadas como, por ejemplo, el tren de pulsos rectangulares, s tienen discontinuidades. Por lo tanto,
se debe tener cuidado con la convergencia cuando se trabaja con esos modelos.

xN(t)
0.09A
T0/2N
A
A/2

t0

Figura 3.7 Fenmeno de Gibbs en una discontinuidad de tipo escaln.

Las condiciones para la convergencia de una serie de Fourier se resumen en el teorema que se dar a
continuacin, sin demostracin, pero antes se definirn algunos trminos que se necesitan para su
exposicin.
Se dice que la funcin x(t) es suave en el intervalo [a, b] si posee una derivada continua en a , b .
En lenguaje geomtrico, esto significa que la direccin de la tangente cambia continuamente, sin saltos,
conforme se mueve a lo largo de la curva y = x(t).
La funcin x(t) es suave por tramos en el intervalo [a, b] si x(t) y su derivada son ambas continuas
en [a, b], o ellas tienen un nmero finito de discontinuidades de saltos en [a, b]. Se dice que una
funcin x(t) continua o discontinua definida en todo el eje t es suave por tramos si es suave por tramos
en todo intervalo de longitud finita. En particular, este concepto es aplicable a funciones peridicas.
Toda funcin x(t) suave por tramos (bien sea continua o discontinua) est acotada y tiene una derivada
acotada en todas partes, excepto en sus saltos y puntos de discontinuidad [en todos estos puntos, x'(t)
no existe].

TEOREMA Si x(t) es una funcin absolutamente integrable de perodo T0 y es suave (posee derivada
continua por tramos) por tramos en el intervalo [a, b], entonces para todo t en a < t < b, la serie de
Fourier de x ( t) converge a x(t) en los puntos de continuidad y al valor
x ( t 0) x ( t 0)
2

en los puntos de discontinuidades (la convergencia puede fallar en t = a y t = b).

181

3.3 Propiedades de las Series de Fourier

A continuacin se considerarn varias propiedades de las series de Fourier. Estas propiedades


proporcionan una mejor comprensin del concepto de espectro de frecuencias de una seal de tiempo
continuo y, adicionalmente, muchas de esas propiedades ayudan en la reduccin de la complejidad del
clculo de los coeficientes de las series.
3.3.1

Efectos de la Simetra

Los tipos ms importantes de simetra son:


1. Simetra par, x(t) = x(t),
2. Simetra impar, x(t) = x(t),

3. Simetra de media onda, x(t) = x t

T0
2

Cuando existe uno o ms de estos tipos de simetra, se simplifica bastante el clculo de los coeficientes
de Fourier. Por ejemplo, la serie de Fourier de una seal par x(t) con perodo T0 es una serie de Fourier
en cosenos:

x ( t ) a0

cos

2 n t
T0

n 1

con coeficientes
T0 2

T0 / 2

a0 T20

x ( t ) dt

an T40

2 nt

dt
x t cos
T0

en tanto que la serie de Fourier de una seal impar x(t) con perodo T0 es una serie de Fourier en senos:

2 n t
x (t )
bn sen
T0
n 1
con coeficientes dados por

T0 2

bn T40

x ( t )sen

2 n t
T0

dt

Si la funcin x(t) posee simetra de media onda, entonces


a0 0

a2 n 0

c2 n1 0

b2 n 0

b2 n1 0

La integracin es sobre medio perodo y los coeficientes se multiplican por 2.

182

EJEMPLO 4. Considere la seal mostrada en la Fig. 3.8


x(t)
A
T0/2

T0/2
0

T0

A
Figura 3.8

La seal est definida por

4A

A T t.

0
x (t )
4 A t 3 A,
T0

0 t T0 / 2
T0 / 2 t T0

Observe que x(t) tiene simetra par y tambin de media onda. Por lo tanto, bn = 0 y no hay armnicos
pares. Tambin
T0 2

an T40

4 At
2 n t

dt
A
cos
T0
T0

4A

1 cos n

n par
0,
8A

, n impar
n 2

Observe que a 0 , el cual corresponde al trmino CD, es cero porque el rea bajo un perodo de x(t) es
cero.

3.3.2

Linealidad

Suponga que x(t) y y(t) son peridicas y con el mismo perodo y sean

x(t )

e
n

jn0 t

y (t )

e
n

jn0 t

183

sus expansiones en series de Fourier y tambin sea


z ( t ) k1 x ( t ) k2 y ( t )

donde k1 y k2 son constantes arbitrarias. Entonces podemos escribir

z (t )

k2 n e jn0 t

jn0 t

La ltima ecuacin implica que los coeficientes de Fourier de z(t) estn dados por
n k1n k2 n

3.3.3

Diferenciacin

La derivada de x(t) se obtiene derivando cada trmino de su serie:

dx ( t )
dt

dt c e
d

jk 0 t

jk c e

jk 0 t

0 k

(3.56)

y se observa que los coeficientes de Fourier para la funcin dx(t)/dt son iguales a los coeficientes de
x(t) multiplicados por el factor jk0. La magnitud de cada armnico es ampliada por el factor k0, y el
espectro tiene un contenido de frecuencias mucho mayor.
Como una aplicacin de esta propiedad, considere el tren de pulsos de la Fig. 3.9a, cuya serie de
Fourier se obtuvo en el Ejemplo 2. Su derivada se muestra en la Fig. 3.9b y contiene slo impulsos. Los
coeficientes de Fourier ck para x'(t) estn dados por
T0 2

ck

1 T0
At
T0
2
T0 2

A
T0

jk 0 2

T0 jk o t
dt
A t e
2

e jk 0 2

2 jA
T0

sen k 0 2

y los coeficientes de Fourier correspondientes para la serie de pulsos en la Fig. 3.9a, de acuerdo con la
Ec. (3.56), son
ck

ck
jk 0

A sen k 0 2
T0

k 0 2

que es el mismo resultado obtenido en el Ejemplo 2, pero con un menor esfuerzo.

184

x(t)
A
...

...
0

T0

T0

(a)
x'(t)
...

A
...

T0

T0

(b)

Figura 3.9

3.3.4

Teorema de la Potencia de Parseval

El teorema de Parseval relaciona la potencia promedio P de una seal peridica con los coeficientes de
su serie de Fourier. Para derivar el teorema, comenzamos con la relacin

1
T0

x ( t ) dt
2

x ( t ) x *( t ) dt

1
T0

T0

T0

Ahora reemplazamos x*(t) por su serie exponencial

x *( t )
an e jn0 t
an e jn0 t
n
n

tal que
P T10

jn0 t
x (t )
cn e
dt
n

T0

T1 x ( t ) e jn0 t dt an
0

n
T0

y la expresin entre corchetes es igual a cn. Entonces

que es el teorema de Parseval.

c c c

n n

(3.57)

185

La interpretacin de este resultado es extraordinariamente sencilla: la potencia promedio se puede


determinar elevando al cuadrado y sumando las magnitudes | c n | = |c(n 0 )| de las lneas de amplitud.
Observe que la Ec. (3.57) no involucra el espectro de fase. Para una interpretacin adicional de la Ec.
(3.57), recuerde que la serie exponencial de Fourier expande x(t) como una suma de fasores de la forma
cn e jn0 t . Se puede demostrar fcilmente que la potencia promedio de cada fasor es
cn e jn0 t

cn

(3.58)

Por lo tanto, el teorema de Parseval implica la superposicin de la potencia promedio, puesto que la
potencia promedio total de x(t) es la suma de las potencias promedio de sus componentes fasoriales.
3.3.5

Integracin en el Tiempo

Suponga que y ( t ) x d . Debemos considerar dos casos por separado; c0 = 0 y c0 0. Si


t

c0 0 , entonces
t


y ( t ) x d cn e j 2 nf d
n

c e

j 2 nf 0

j 2 nf
cn

En consecuencia, si

y (t )

j 2 nf 0 t

se concluye que

cn
j 2 nf 0

cn
jn0

y, por tanto,
t

x d

j 2 nf

cn

e j 2 nf0 t
0

jn

cn

e jn0 t ,

c0 0

(3.59)

Si a0 0, la seal contiene un valor promedio diferente de cero. La integracin de esta seal produce
una componente que crece linealmente con el tiempo, y en este caso la seal resultante no es peridica
y la serie obtenida por integracin no converge.

186

3.3.6

Manipulacin de Seales

Cuando una seal peridica x(t) es trasladada, transpuesta (reflejada), diferenciada o integrada, los
coeficientes de Fourier de la seal resultante pueden obtenerse a partir de los de la seal original x(t).
En esta seccin derivaremos algunas de esas relaciones en funcin de los coeficientes de la serie de
Fourier exponencial.
Considere el efecto sobre el espectro de frecuencias de una seal peridica producida por un
desplazamiento en el tiempo de la seal. Sea x(t) una seal peridica de perodo T0, la cual tiene una
serie de Fourier dada por

x (t )

c e

jn0 t

(3.60)

Consideremos ahora la serie de Fourier de la seal peridica retardada x(t td) obtenida a partir de x(t)
por un desplazamiento en el tiempo igual a td. De (3.60) tenemos que

x ( t td )

c e

jn0 ( t td )

c e

jn0 td

ce

jn0 t

jn0 t

(3.61)

donde cn cn e jn0 td . Es decir, el n-simo coeficiente de Fourier de x(t td) es igual al n-simo
coeficiente de Fourier de x(t) multiplicado por exp ( jn0 td ) . El resultado muestra que el espectro de
amplitud no cambia pero el de fase es diferente. El n-simo armnico de la seal desplazada se retrasa
por una cantidad igual a n0 td radianes.
Suponga ahora que una seal peridica x(t) de perodo T0 es contrada en el tiempo por un factor ; la
seal resultante, x(t), tambin es peridica pero tiene un perodo contrado T/. La frecuencia
fundamental de x(t) es por tanto 0, donde 0 es la frecuencia fundamental de x(t). Para valores del
factor de escalamiento menores que la unidad, la seal es expandida en el tiempo y los armnicos son
escalados hacia abajo. Los coeficientes de Fourier an de x(t) pueden ser calculados por

cn

T0

T0

x ( t ) e
0

jn0 t

dt

T0

x () e
T

jn0

donde = t. Esta ltima relacin nos dice que los coeficientes de Fourier de una seal peridica no
cambian cuando se cambia la escala del tiempo. No obstante, las frecuencias de los armnicos
cambian por un factor igual al factor de escalamiento.
Otras propiedades producidas por manipulacin de la seal se dejan como ejercicios.

187

3.4 Transformadas de Fourier y Espectros Continuos


Pasemos ahora de seales peridicas a seales no peridicas concentradas en intervalos de tiempo muy
cortos. Si una seal no peridica tiene energa total finita, su representacin en el dominio de la
frecuencia ser un espectro continuo obtenido a partir de la transformada de Fourier. Consideramos la
transformada de Fourier como un lmite de la serie de Fourier haciendo que el perodo se torne muy
largo. De esta manera el espectro de lneas tiende a un espectro continuo.
3.4.1

La Transformada de Fourier

Las series de Fourier por su propia naturaleza, estn limitadas a la representacin de funciones
peridicas que satisfacen las condiciones de Dirichlet.. Por otra parte, muchas funciones importantes
son no peridicas y con frecuencia se necesita una representacin efectiva para esas seales. La Fig. 3.9
muestra dos seales no peridicas tpicas. El pulso rectangular (Fig. 3.9a) est estrictamente limitado
en tiempo ya que x(t) es idnticamente igual a cero fuera de la duracin del pulso. La otra (Fig. 3.9b)
est asintticamente limitada en el tiempo en el sentido que x(t ) 0 conforme t . Tales seales
tambin se pueden describir como pulsos. En cualquier caso, si se intenta promediar x(t) o | x(t)|2 para
todo el tiempo, se encontrar que estos promedios son iguales a cero. Por consiguiente, en lugar de
hablar sobre potencia promedio, una propiedad ms significativa de una seal no-peridica es su
energa.

x(t)

x(t)
A

1/b

(a)

1/b

(b)

Figura 3.9

Como ya se dijo anteriormente, si x(t) es el voltaje en una resistencia, la energa total suministrada se
encontrara integrando la potencia instantnea x2(t)/R. Por lo tanto, repetimos aqu la frmula para la
energa normalizada de la seal

x ( t ) dt

(3.62)

Algunos clculos de la energa pueden hacerse por inspeccin ya que E es sencillamente el rea bajo la
curva de | x(t)| 2 . Por ejemplo, la energa de un pulso rectangular de duracin y amplitud A es E = A2.
Cuando la integral en la Ec. (3.62) existe y da como resultado 0 < E < , se dice que la seal x(t)
tiene energa bien definida y se denominar una seal de energa no peridica. Casi todas las seales

188

limitadas en tiempo de inters prctico caen en esta categora, la cual es la condicin esencial para el
anlisis de Fourier usando la transformada de Fourier.
Considere la seal peridica definida por

1,

x (t )
0,

t T1
T1 t

T0
2

donde T0 es el perodo. Los coeficientes de la serie de Fourier para esta onda son

ck

2sen k 0 T1
k 0 T0

(3.63)

T0

En la Fig. 3.10 se grafica T0ak en lugar de ak y tambin se modifica la separacin horizontal en cada
grfico. El significado de estos cambios se puede ver examinando la Ec. (3.63). Multiplicando ak por T0
se obtiene

T0 ck

2sen k 0 T1
k 0

2sen T1

(3.64)
k 0

T0 a k
2 0

2 0

(a)
T0 a k
4 0

4 0

(b)
T0 a k
8 0

8 0

(c)
Figura 3.10. Coeficientes de Fourier y sus envolventes para la onda peridica cuadrada:

(a). T0 = 4T1; (b) T0 = 8T1; (c) T0 = 16T1.

189

Entonces, con considerada como una variable continua, la funcin 2sen T1 / representa la
envolvente de T0ck y estos coeficientes son muestras igualmente espaciadas de esta envolvente.
Tambin, para T1 fijo, la envolvente de T0ak es independiente de T0. Sin embargo, de la Fig. 3.10 vemos
que conforme T0 aumenta (o, equivalentemente, 0 disminuye), la envolvente es muestreada con un
espaciamiento ms y ms corto. Conforme T0 se hace arbitrariamente grande, la onda cuadrada
peridica original se aproxima a un pulso rectangular, es decir, todo lo que queda en el dominio del
tiempo es una seal aperidica correspondiente a un perodo de la onda cuadrada. Tambin, los
coeficientes de la serie de Fourier, multiplicados por T0 se convierten en muestras de la envolvente
menos separadas, as que de alguna forma el conjunto de los coeficientes de la serie de Fourier tiende a
la funcin envolvente conforme T0 .
Este ejemplo ilustra la idea bsica detrs del desarrollo de Fourier de una representacin para seales
aperidicas. Especficamente, consideramos una seal aperidica como el lmite de una seal peridica
conforme el perodo se hace arbitrariamente grande y examinamos la conducta en el lmite de la
representacin en serie de Fourier de esta seal. Considere una seal aperidica general x(t) de
duracin finita. Es decir, para algn valor T1, x(t) = 0 si | t | > T1. En la Fig. 3.11a se muestra una seal
de este tipo. Partiendo de esta seal aperidica podemos construir una seal peridica x ( t ) para la
cual x(t) es un perodo, como se indica en la Fig. 3.11b. Conforme aumentamos el perodo T0, x ( t ) se
hace ms semejante a x(t) durante intervalos ms largos, y conforme T0 , x ( t ) es igual x(t) para
cualquier valor finito de x(t).

x(t)

T1

T1

(a)

~
x (t )
...

...
T0

T1

T1

T0

2T0

(b)

Figura 3.11. (a) Seal aperidica x(t): (b) seal peridica x ( t ),


construida para que sea igual a x(t) por un perodo.

Este ejemplo ilustra la idea bsica detrs del desarrollo de Fourier de una representacin para seales
aperidicas. Especficamente, consideramos una seal aperidica como el lmite de una seal peridica
conforme el perodo se hace arbitrariamente grande y examinamos la conducta en el lmite de la
representacin en serie de Fourier de esta seal. Considere una seal aperidica general x(t) de
duracin finita. Es decir, para algn valor T1, x(t) = 0 si | t | > T1. En la Fig. 3.11a se muestra una seal

190

de este tipo. Partiendo de esta seal aperidica podemos construir una seal peridica x ( t ) para la
cual x(t) es un perodo, como se indica en la Fig. 3.11b. Conforme aumentamos el perodo T0, x ( t ) se
hace ms semejante a x(t) durante intervalos ms largos, y conforme T0 , x ( t ) es igual x(t) para
cualquier valor finito de x(t).
Examinemos ahora qu efecto tiene esto sobre la representacin en serie de Fourier de la seal x ( t ) :

c e

x ( t )

jk 0 t

(3.65)

k
T0 2

ck

1
T0

x ( t ) e jk 0 t dt

(3.66)

T0 2

La separacin entre los coeficientes ck es (k 1)0 k 0 0 .


Puesto que x ( t ) x ( t ) para t T0 2 y tambin como x(t) = 0 fuera de este intervalo, la Ec. (3.66)
puede escribirse de nuevo como

T0 / 2

ck

1
T0

x (t ) e

jk 0 t

dt

1
T0

T0 / 2

x (t ) e

jk 0 t

dt

Por lo tanto, definiendo la envolvente X() de T0ak como


X

x (t ) e

jt

dt

(3.67)

tenemos que los coeficientes ck pueden expresarse como

ck

X k 0

T0

(3.68)

Combinando las Ecs. (3.68) y (3.65), x ( t ) se puede expresar en funcin de X() como

x ( t )

X k 0 e jk 0 t

o equivalentemente, como 2/T0 = 0, por


x ( t )

X k e
2
1

jk 0 t

X k e
2
1

jk 0 t

(3.69)

Conforme T0 , ~x (t ) tiende a x(t) y por consiguiente, la Ec. (3.67) se convierte en una


representacin de x(t). Adicionalmente, = 0 0 conforme T0 , y el lado derecho de la Ec.
(3.69) pasa a ser una integral. Por lo tanto, usando el hecho de que x ( t ) x ( t ) conforme T0 , las
Ecs. (3.69) y (3.67) se convierten en

191

x(t )

1
2

X ( ) e

j t

x(t ) e

j2 f t

df

(3.70)

X ( )

X ( f )e

jt

dt

x(t ) e

j2 f t

dt X ( f )

(3.71)

Las Ecs. (3.70) y (3.71) se conocen como el par de transformadas de Fourier; la funcin X() [o
X(f) ] dada por la Ec. (3.71) se conoce como la transformada de Fourier o integral de Fourier de x(t) y
la Ec. (3.70) como la ecuacin de la transformada de Fourier inversa. La ecuacin de sntesis (3.70)
juega un papel para las seales aperidicas semejante al de la Ec. (3.21) para seales peridicas, ya que
ambas corresponden a una descomposicin de una seal en una combinacin lineal de exponenciales
complejas. Para seales peridicas, estas exponenciales complejas tienen amplitudes {ak} dadas por la
Ec. (3.25) y ocurren en un conjunto discreto de frecuencias relacionadas armnicamente k0, k = 0, 1,
2, . Para seales aperidicas, estas exponenciales complejas son equivalentes a especificar x(t),
ocurren en un continuo de frecuencias y, de acuerdo con la ecuacin de sntesis (3.70), tienen
amplitud X()(d/). En analoga con la terminologa usada para los coeficientes de la serie de
Fourier para una seal peridica, la transformada X() [o X(f) ] de una seal aperidica x(t) se conoce
comnmente como el espectro de X() [o de X(f) ], ya que nos da la informacin concerniente de cmo
x(t) est compuesta de seales sinusoidales de diferentes frecuencias y proporciona una medida de la
intensidad de x(t) en el intervalo de frecuencias entre 0 y 0 + (d en el lmite); es decir, en el
dominio de la frecuencia, X() determina cunto del valor de x(t) en t es atribuible a los valores de
entre 0 y 0 + ,
3.4.2

Convergencia de las Transformadas de Fourier

Aunque el argumento usado para derivar el par de transformadas de Fourier supuso que x(t) era de
duracin arbitraria pero finita, las Ecs. (3.70) y (3.71) se mantienen vlidas para una clase
extremadamente amplia de seales de duracin infinita. De hecho, nuestra derivacin de la
transformada de Fourier sugiere que aqu tambin debe ser aplicable un conjunto de condiciones como
las requeridas para la convergencia de la serie de Fourier, y ciertamente se puede demostrar que ste es
el caso. Especficamente, considere la evaluacin de X() de acuerdo con la Ec. (3.71) y sea x ( t ) la
seal obtenida al usar X() en el lado derecho de la Ec. (3.70); es decir,

x ( t )

1
2

X ( ) e

j t

Lo que nos gustara saber es cundo es vlida la Ec. (3.70) [es decir, cundo x ( t ) es una
representacin vlida de la seal original x(t)]. Si x(t) es una seal cuyo cuadrado es integrable de modo
que

x ( t ) dt
2

(3.72)

192

entonces estamos garantizando que X() es finita [la Ec. (3.71) converge] y que, denotando por e(t) el
error entre x ( t ) y x(t) [es decir, e(t) = x ( t ) x ( t ) ], entonces

e ( t ) dt 0
2

(3.73)

Las Ecs. (3.72) y (3.73) son las contrapartes aperidicas de las Ecs. (3.50) y (3.53) para seales
peridicas. As, al igual que con seales peridicas, si x(t) es cuadrado integrable, entonces aunque x(t)
y su representacin de Fourier x ( t ) pueden diferir significativamente en valores individuales de t, en su
diferencia no hay energa.
Igual que con las seales peridicas, existe un conjunto alterno de condiciones que son suficientes
para asegurar que x ( t ) sea igual a x(t) para cualquier t excepto en alguna discontinuidad, donde es
igual al valor promedio de la discontinuidad. Estas condiciones, igualmente conocidas como las
condiciones de Dirichlet, requieren que:
1. x(t) sea absolutamente integrable, es decir,

x ( t ) dt

(3.74)

2. x(t) tenga un nmero finito de mximos y mnimos dentro de cualquier intervalo finito.
3. x(t) tenga un nmero finito de discontinuidades dentro de cualquier intervalo finito. Adicionalmente,
cada una de estas continuidades debe ser finita.
En consecuencia, las seales absolutamente integrables que son continuas o tienen un nmero finito de
discontinuidades tienen transformadas de Fourier.
Aunque los dos conjuntos alternos de condiciones que hemos dado son suficientes para garantizar
que una seal tiene una transformada de Fourier, en la prxima seccin veremos que seales
peridicas, que no son absolutamente integrables ni cuadrado integrables en un intervalo infinito,
pueden considerarse que poseen transformadas de Fourier si se permiten funciones impulso en la
transformada. Esto tiene la ventaja de que la serie y la transformada de Fourier pueden incorporarse en
un marco comn y esto ser muy conveniente para diferentes tipos de anlisis. Antes de examinar este
punto un poco ms en la prxima seccin, primero se debe sealar que la transformada de Fourier es
una transformacin lineal; es decir,
a1 x1 ( t ) a2 x2 ( t )

a1 X 1 ( ) a2 X 2 ( )

Se deja para el lector la demostracin de esta propiedad.


Por la frmula de Euler, la integral de Fourier, Ec. (3.71), puede escribirse en la forma

X ( )

x ( t ) (cos t j sen t ) dt

Escribiendo ahora
X ( ) Re[ X ( )] j Im{ X ( )

e igualando con las partes real e imaginaria de la Ec. (3.75), se obtiene

(3.75)

193

Re[ X ( )]

x ( t ) cos t dt

(3.76)

Im[ X ( )]

x ( t )sen t dt

(3.77)

Esto muestra que la parte real de X() es una funcin par de y la parte imaginaria de X() es una
funcin impar de , dando
(3.78)
X ( ) X *( )
La implicacin de la Ec. (3.76) es que si X() es conocida para > 0, entonces tambin es conocida
para < 0. Por esta razn, X() muchas veces slo se grafica para > 0.
Consideremos ahora algunos ejemplos de la transformada de Fourier.
3.4.3

Ejemplos de Transformadas de Fourier en Tiempo Continuo

Ejemplo 5. Considere la seal


x ( t ) e at u ( t )

Si a < 0, entonces x(t) no es absolutamente integrable y, por tanto, X() no existe. Para a > 0, X() se
obtiene a partir de la Ec. (3.71) como

X ( ) e a t e jt dt
0

1
a j

a j t

Es decir,
X ( )

1
a j

a0

Puesto que esta transformada de Fourier tiene partes real e imaginaria, para graficarla en funcin de
la expresamos en trminos de su magnitud y fase:
X ( )

1
a 2 2


X ()= tan 1
a

Cada una de estas componentes se grafica en la Fig. 3.12. Observe que si a es compleja, entonces x(t)
es absolutamente integrable siempre que Re{a} > 0, y en este caso el clculo precedente produce la
misma forma para X(); es decir,
X ( )

1
a j

Re a 0

194

X ()

1/a

(a)

X ()

2a

(b)

Figura 3.12 Transformada de Fourier de la seal x(t) = eatu(t).

Ejemplo 6. Sea
x (t ) e

a t

donde a > 0. Esta seal se grafica en la Fig. 3.13.


x(t)
1

Figura 3.13 Seal x(t) = ea| t | .

El espectro de esta seal es

X ( )

jt

dt

a t

1
a j

at

j t

1
a j

En este caso X() es real y se ilustra en la Fig. 3.14.

dt e a t e jt dt
0

2a
a 2
2

195

X()
2/a
1/a

Figura 3.14 Transformada de Fourier de la seal en la Fig. 3.13.

Ejemplo 7. Considere el pulso rectangular

1,
x (t )

0,

t T1

(3.79)

t T1

que se muestra en la Fig. 3.15a.


x(t)
1

T1

T1

(a)
2T1

X()

T1

(b)

Figura 3.15. El pulso y su transformada de Fourier.

Aplicando la definicin de la transformada, encontramos que sta est dada por


T1

X ( )

T1

e jt dt 2

sen T1

(3.80)

Como se explic al comienzo de esta seccin, la seal dada en la Ec. (3.79) puede considerarse como
la forma lmite de una onda cuadrada peridica conforme el perodo se hace arbitrariamente grande.
Por lo tanto, es de esperar que la convergencia de la ecuacin de sntesis para esta seal se comporte en
una forma similar a la observada para la onda cuadrada, y, de hecho, ste es el caso.

196

Ejemplo 8. Considere la seal x(t) cuya transformada de Fourier est dada por

1,
X ( )

0,

(3.81)

Esta transformada se ilustra en la Fig. 3.16b.


Usando la ecuacin de sntesis, podemos determinar x(t):
x(t )

e
2

j t

sen W t

(3.82)

la cual se muestra en la Fig. 3.16a.

W/

x(t)

(a)
X(
)

(b)

Figura 3.16. Par de transformadas de Fourier del Ejemplo 8.

Comparando las Figs. 3.15 y 3.16, o equivalentemente, las Ecs. (3.79) y (3.80) con las Ecs. (3.81) y
(3.82), se observa una relacin interesante. En cada caso, el par de transformadas de Fourier consiste de
una funcin sen x / x y un pulso rectangular. Sin embargo, en el Ejemplo 7 es la seal x(t) la que es
un pulso, mientras que en el Ejemplo 8, es la transformada. La relacin especial que aparece aqu es
una consecuencia directa de la propiedad de dualidad para las transformadas de Fourier, la cual se
discute ms adelante.
3.5 La Transformada de Seales Peridicas
En la seccin anterior se desarroll la transformada de Fourier para seales aperidicas considerando
el comportamiento de la serie de Fourier para seales peridicas conforme el perodo se hace
arbitrariamente grande. Como lo indican los resultados, las representaciones en series de Fourier y en
transformadas de Fourier estn ntimamente relacionadas y en esta seccin investigamos un poco ms
esa relacin y tambin desarrollamos una representacin en serie de Fourier para seales peridicas.

197

3.5.1

Los Coeficientes de la Serie de Fourier como Muestras de la Transformada

Como un primer paso, recuerde que en la derivacin de la transformada de Fourier se hizo la


observacin de que los coeficientes de Fourier de una seal peridica x ( t ) se podan obtener a partir
de muestras de una envolvente, que se determin era igual a la transformada de Fourier de una seal
aperidica x(t) y que a su vez era igual a un perodo de ~x (t ) . Especficamente, sea T0 el perodo
fundamental de x ( t ) , como se ilustra en la Fig. 3.17. Como ya se vio, si x(t) se toma como

x ( t ),
x (t )
0,

T0

T0

T0
2

o t

T0

(3.83)

entonces los coeficientes de Fourier c k de ~x (t ) pueden expresarse en funcin de las muestras de la


transformada de Fourier X() de x(t):
T0 / 2

ck

1
T0

T0 / 2

x ( t ) e

jk 0 t

dt

1
T0

T0 / 2

x ( t ) e jk 0 t dt

T0 / 2

1
T0

x ( t ) e jk0 t dt

T0

X ( k 0 )

(3.84)

x(t)
...

...
T0

T0 0
2

T0
2

T0

Figura 3.17. Seal Peridica.

Sin embargo, puesto que los coeficientes de Fourier, ck, pueden obtenerse integrando en cualquier
intervalo de longitud T0, podemos efectivamente obtener una expresin ms general que la dada en la
Ec. (3.84). Especficamente, sea s un punto arbitrario en el tiempo y definamos la seal x(t) como igual
a ~x (t ) en el intervalo s t s + T0 y cero para otros valores de s. Es decir,

s t s T0

x ( t ),
x (t )
0,

t s o t s T0

(3.85)

Entonces los coeficientes de la serie de Fourier de x ( t ) vienen dados por

ck

1
T0

X ( k 0 )

(3.86)

198

donde X() es la transformada de Fourier de x(t) en la forma definida en la Ec. (3.85). Observe que la
Ec. (3.86) es vlida para cualquier seleccin de s y no nicamente para s T0 / 2 . Sin embargo, esto
no significa que la transformada X() es la misma para todos los valores de s, pero s implica que el
conjunto de muestras X(k0) es independiente de s.
En lugar de dar una demostracin de la validez de la Ec. (3.86) en general, la ilustraremos mediante el
siguiente ejemplo.

Ejemplo 9. Sea x ( t ) la onda peridica cuadrada con perodo T0 ilustrada en la Fig. 3.18a, y sean x1(t)
y x2(t) como se muestran en las Figs. 3.18b y c. Estas seales son iguales a ~x (t ) en intervalos diferentes
de longitud T0. La transformada de Fourier de x1(t) ya se obtuvo y es

X1 ( )

2sen T1

(3.87)

La transformada de Fourier de x 2 (t) puede calcularse a partir la frmula de definicin:

X 2 ( )

x (t ) e
2

dt e

1 e
j

j t

j e
1

jT1

dt

e jt dt

T0 T1

jT0

jT1

e jT1 / 2 e jT1 / 2 e jT1 / 2

sen 1
j
2
2

T0

T1
jt

1
1
j

jT0 T1 / 2

jT1 / 2

e jT1 / 2

jT1 / 2
j T T / 2
e 0 1
e

(3.88)

Las transformadas X1() y X2() no son iguales. De hecho, X1() es real para todos los valores de
mientras que X2() no lo es. Sin embargo, para = k0, la Ec. (3.88) se convierte en
X 2 k 0

k T
sen 0 1
k 0
2
2

jk0T1 / 2
e jk 0T0 e jk 0T1 / 2
e

Puesto que 0T0 = 2, esta relacin se reduce a


X 2 ( k 0 )

k T
sen 0 1
k 0
2
2

jk 0T1 / 2
e jk0T1 / 2
e

k T
k T
sen 0 1 cos 0 1
k 0
2
2
2sen k 0 T1

X 1 ( k 0 )
k 0

199

x ( t )
1

T0

T1

T1

T0

(a
) x1(t)
1

T1

T1

(b)
x2(t)
1

T1 T0 T1

(c)
Figura 3.18

la cual confirma el resultado dado en la Ec. (3.86), a saber, que los coeficientes de Fourier de una seal
peridica pueden obtenerse a partir de muestras de la transformada de Fourier de una seal aperidica
que sea igual a la seal peridica original en cualquier intervalo arbitrario de longitud T0 y que sea
cero fuera de ese intervalo.
3.5.2

La Transformada de Fourier de Seales Peridicas

Consideremos una seal x(t) cuya transformada de Fourier X() es un solo pulso de rea 2 en =
0, es decir,

X ( ) 2 0

(3.89)

Para determinar la seal x(t) a la que corresponde esta transformada de Fourier, aplicamos la relacin
de la transformada de Fourier inversa para obtener
x(t )

2 e
2
0

j t

j t

Si X() est en la forma de una combinacin lineal de impulsos igualmente espaciados en frecuencia,
es decir,

200

X ( )

2 c k
k

(3.90)

entonces la aplicacin de la propiedad de linealidad produce

c e

x (t )

jk 0 t

(3.91)

y esta ltima corresponde exactamente a la representacin en serie de Fourier de una seal peridica.
As que la transformada de Fourier de una seal peridica con coeficientes de su serie de Fourier {ak},
puede interpretarse como un tren de impulsos que ocurren en las frecuencias relacionadas
armnicamente y donde el rea de la k-sima frecuencia armnica k0 es 2 veces el k-simo
coeficiente de la serie de Fourier ck.

Ejemplo 10. Considere de nuevo la onda cuadrada ilustrada en la Fig. 3.18a. Los coeficientes de la
serie de Fourier para esta seal son

ck

sen k 0 T1
k

y la transformada de Fourier es

X ( )

2sen k 0 T1
k

k 0

la cual se grafica en la Fig. 3.19 para T0 = 4T1.


X ()

Figura 3.19. Transformada de Fourier de una onda cuadrada peridica simtrica.

Ejemplo 11. Considere el tren de impulsos peridicos dado por

x (t )

t kT

y dibujado en la Fig. 3.20a. Esta seal es peridica con perodo fundamental T. Para determinar la
transformada de Fourier de esta seal, calculamos primero sus coeficientes de Fourier:

201

T /2

ck

1
T

( t ) e jk 0 t dt

T /2

1
T

Insertando sta en la Ec. (3.91) da

X ( )

2
T

2 k
T

La transformada de un tren de impulsos en el tiempo es entonces un tren de impulsos en frecuencia,


como se muestra en la Fig. 3.20b. Aqu se ve de nuevo una ilustracin de la relacin entre los dominios
del tiempo y de frecuencia. Conforme la separacin entre los impulsos en el tiempo se hace mayor, la
separacin entre los impulsos en frecuencia se hace menor.

x(t)
1

...

...
2T T

2T

(a)

X()
2
T

...

...

4T

2T

2
T

4
T

(b)

Figura 3.20. (a) Tren de impulsos peridicos; (b) su transformada de Fourier.

3.6 Propiedades Adicionales de la Transformada de Fourier


Ahora pasaremos a considerar varias propiedades de la transformada de Fourier que nos proporcionan
una cantidad significativa de comprensin de la transformada y de la relacin entre las descripciones de
la seal en los dominios del tiempo y la frecuencia. Adicionalmente, muchas de estas propiedades con
frecuencia son tiles en la reduccin de la complejidad en la evaluacin de las transformadas y para
estudiar mejor la relacin entre las representaciones en serie de Fourier y en transformada de Fourier de
una seal peridica. A travs de la presentacin tambin usaremos la notacin
x(t) X() = F { x ( t )}
para referirnos al par de transformadas x(t) y X().

202

3.6.1

Retardo en el Tiempo y Cambio de Escala

Dada una funcin del tiempo x(t), a partir de ella se pueden generar otras formas de ondas mediante
una modificacin del argumento de la funcin. Especficamente, reemplazando t por t td produce la
seal retardada en el tiempo x(t td). La seal retardada tiene la misma forma que x(t) pero esta
desplazada td unidades hacia la derecha en el eje del tiempo. Para establecer esta propiedad, suponga
que
x(t) X()
Considere entonces la transformada de la funcin retardada:

F { x ( t td }

x (t t

) e jt d

Haciendo = t td, se obtiene

F x ( t td )

x e

j td

d e j t d X ( )

esto es,
x ( t td )

e jtd X ( ) e j 2 ftd X ( f )

(3.92)

Antes de proceder con el siguiente ejemplo, definamos el pulso rectangular de la Fig. 3.21, el cual por
ser tan comn merece un smbolo propio. Se adoptar la notacin

1
( t / ) rect t

t /2
t /2

(3.93)

x(t)
A

Figura 3.21. La funcin rectangular.

que representa una funcin rectangular con amplitud unitaria y duracin centrada en t = 0. El pulso en
la figura se escribe entonces como

x ( t ) A ( t ) A rect t
y su transformada de Fourier es

(3.94)

203

/2

X ( )

Ae jt dt

/2

2A

sen

A sinc

(3.95)

El desplazamiento o corrimiento de fase en la transformada es una funcin lineal de . Si td es una


cantidad negativa, la seal es adelantada en el tiempo y la fase aadida tiene pendiente positiva.
Si el pulso rectangular no est centrado en el origen, entonces se denota como rect t T0 ; esto
indica que el pulso est centrado en T0 y su ancho es igual a .
Ejemplo 12. La seal en la Fig. 3.22 se construye usando dos pulsos rectangulares x(t) = At tales
que
za ( t ) x ( t td ) 1 x t td T

donde t0 = td + T/2. Aplicando los teoremas de superposicin (linealidad) y de retardo, se obtiene


Z a ( ) X ( ) e j td e

j td T

donde X() est dada por la Ec. (3.95).


Si t 0 = 0 y T = , za(t) se degenera en la onda de la Fig. 3.22b donde
t 2
t 2
zb ( t ) A
A

El espectro se convierte entonces en

Zb ( ) A sinc 2 j 2sen 2
j A sinc2

El espectro es puramente imaginario porque zb(t) tiene simetra impar.

za(t)

zb(t)
T/2

T/2

td

td + T
t0

0
A

(a
)

(b)

Figura 3.22

204

Otra operacin en el eje del tiempo es un cambio de escala, el cual produce una imagen de x(t)
escalada horizontalmente al reemplazar t por t. La seal escalada x(t) ser expandida si 1 o
comprimida si 1 ; un valor negativo de produce inversin en el tiempo y tambin expansin o
compresin.
El cambio de escala en el dominio del tiempo se convierte en un cambio de escala recproco en el
dominio de la frecuencia. Es decir, si
x(t) X()
entonces
x ( t )

(3.96)

donde es una constante real. Se demostrar la Ec. (3.96) para el caso < 0 escribiendo y
haciendo ahora el cambio de variable t , y dt d , se tiene que

F x (

t)

x ( t )e

jt

dt

x ( ) e

x e

j /


X

1

3.6.2

Diferenciacin en el Dominio del Tiempo

Diferenciando la Ec. (3.68) (que da la definicin de la transformada inversa) con respecto al tiempo, se
obtiene
dx ( t )
dt

jX ( ) e
2

jt

Pero la expresin entre corchetes en el lado derecho, por definicin, es la transformada de la derivada
dx ( t ) dt y as obtenemos el par de transformadas
dx ( t )
dt

jX ( )

(3.97)

205

Es bastante sencillo demostrar para el caso general que

d n x (t )
dt n
3.6.3

(j)n X ( )

(3.98)

Integracin en el Dominio del Tiempo

Suponga que a partir de x(t) se genera otra funcin por integracin, es decir,
t

x ( ) d .

y (t )

El teorema de integracin dice que si

X (0)

x ( ) d 0

entonces
t

x ( ) d

X ( )

1
j 2 f

X(f)

(3.99)

La condicin de que el rea neta sea cero asegura que la seal integrada tiende a cero conforme t tiende
a infinito. Puesto que la integracin es el proceso inverso de la diferenciacin, la Ec. (3.99) muestra que
la operacin en el dominio de la frecuencia correspondiente a la integracin en el dominio del tiempo
es la multiplicacin por 1 j .
3.6.4

Dualidad

Comparando las relaciones para la transformada y la transformada inversa

x (t )

1
2

X ( ) e

jt

d X ( f ) e j 2 ft df

(3.100)

X ( )

x (t ) e

j t

dt X ( f )

(3.101)

se observa que hay una simetra bien definida, difiriendo solamente por la variable de integracin y el
signo en el exponente. De hecho, esta simetra conduce a una propiedad de la transformada de Fourier
conocida como dualidad. Especficamente, considere cambiar en la Ec. (3.101) por ; se obtiene
X

x (t ) e

jt

dt

(3.102)

Comparando esta ecuacin con la Ec. (3.100), vemos que si ahora intercambiamos y t, obtenemos

206

X ( t )

x e

jt

(3.103)

y se tiene que

x ( ) 21 F X ( t )

(3.104)

Es decir, si se nos da el par de transformadas de Fourier para la funcin temporal x(t):


x ( t ) X( )

(3.105)

y despus consideramos la funcin del tiempo X(t), su par de transformadas de Fourier es


X (t )

2 x ( )

x ( f )

(3.106)

Las implicaciones de estas dos ltimas ecuaciones son importantes. Por ejemplo, suponga que

1,

x (t )

0,

t M
t M

(3.107)

Entonces, de la Ec. (3.102),


X ( )

2sen M

M
2 M sinc

(3.108)

Ese resultado, junto con la Ec. (3.97) o, equivalentemente, la Ec. (3.105), produce el par de
transformadas en la Ec. (3.96) para M = /2, mientras que si usamos la Ec. (3.105) o la Ec. (3.107)
obtenemos el par en la Ec. (3.82) con M = W. Por lo tanto, la propiedad de dualidad permite obtener
ambas de estas transformadas duales a partir de una evaluacin de la Ec. (3.102). Esto a veces permite
una reduccin en la complejidad de los clculos involucrados en la determinacin de las transformadas
y las transformadas inversas. Como ilustracin de esta propiedad, consideraremos el siguiente ejemplo.

EJEMPLO 13. Supngase que se desea evaluar la transformada de Fourier de la seal

x (t )

2
t2 1

Si hacemos

x ( )

2
1
2

entonces, de la Ec. (3.105), tenemos el par de transformadas de Fourier

g ( t ) x ( )
Por el Ejemplo 6 sabemos que

2
1
2

207

g (t ) e

Adicionalmente, usando el par de transformadas dado por la Ec. (3.106), concluimos que puesto que
f(t) = x(t), entonces

X ( ) F x ( t ) 2 g ( ) 2 e

La propiedad de dualidad tambin puede usarse para determinar o sugerir otras propiedades de la
transformada de Fourier. Especficamente, si existen caractersticas de una funcin del tiempo que
tienen implicaciones sobre la transformada de Fourier, entonces las mismas caractersticas asociadas
con una funcin de la frecuencia tendrn implicaciones duales en el dominio del tiempo. Por ejemplo,
sabemos que una funcin del tiempo peridica tiene una transformada de Fourier que es un tren de
impulsos ponderados e igualmente espaciados. Debido a la dualidad, una funcin del tiempo que es un
tren de impulsos ponderados e igualmente espaciados tendr una transformada de Fourier que es
peridica en frecuencia. Esto es una consecuencia de las Ecs. (3.103) y (3.104). En forma similar,
algunas de las propiedades de la transformada de Fourier ya consideradas tambin implican
propiedades duales. Por ejemplo, vimos que la diferenciacin en el dominio del tiempo corresponde a
multiplicar por j en el dominio de la frecuencia. De la discusin anterior podramos entonces intuir
que la multiplicacin por jt en el dominio del tiempo corresponde a alguna forma de diferenciacin en
el dominio de la frecuencia. Para determinar la forma precisa de esta propiedad dual, procedemos en la
forma siguiente: Diferenciamos la ecuacin de sntesis con respecto a para obtener
dX ( )
d

jtx ( t ) e

j t

dt

Es decir,
jtx ( t )

dX ( )
d

(3.109)

que es la propiedad dual de la Ec. (3.97).


En forma similar, otras propiedades duales son:
e j0 t x ( t ) X ( 0 )

(3.110)

1
jt

x ( t ) x (0) ( t )

X ( ) d

(3.111)

que se obtiene a partir de la Ec. (3.115) ms adelante.


3.6.5

La Relacin de Parseval

Si x(t) y X() forman una par de transformadas de Fourier, entonces, la energa de una seal x ( t ) est
relacionada con su espectro por la relacin

208

x ( t ) dt
2

1
2

X ( ) d
2

X(f )

df

(3.112)

Esta expresin, conocida como la relacin de Parseval, se deduce de una aplicacin directa de la
transformada de Fourier. Especficamente,

x ( t ) dt x ( t ) x *( t ) dt x ( t ) 21 X *( ) e jt d dt

Invirtiendo el orden de integracin se obtiene

X *( ) x ( t ) e jt dt d

x ( t ) dt
2

1
2

Pero la cantidad entre corchetes es sencillamente la transformada de Fourier de x(t); en consecuencia,

x ( t ) dt
2

1
2

X ( ) d
2

X ( f ) df

La relacin de Parseval expresa que la energa total puede determinarse bien sea calculando la
energa por unidad de tiempo, | x(t)| 2 , e integrando para todo el tiempo o calculando la energa por
unidad de frecuencia, X , e integrando para todas las frecuencias. Por esta razn, a |X()|
2

tambin se le refiere como el espectro de la densidad de energa de la seal x(t). Con esto se quiere
decir que la energa en cualquier banda diferencial de frecuencias df es igual a X ( f ) df .

Ejemplo 14. Considere el par de transformadas

1,
x (t )

0,

t T1
t T1

X ( )

2sen T1

T
2T1 sinc 1

La energa total de la seal es

1
2

T1

4 T 2 sinc2 T1 d 12 dt 2 T
1
1

Como se observa, es mucho ms fcil integrar el lado derecho que el lado izquierdo.
3.7 La Propiedad de Convolucin
Esta propiedad juega un papel importante en el estudio de los sistemas LIT. La propiedad expresa que
si

209

x ( t ) X ( )
h ( t ) H ( )

entonces

y (t )

x ( ) h (t ) d x (t ) h (t )

Y ( ) X ( ) H ( )

(3.113)

La demostracin se obtiene directamente a partir de la definicin de la integral de convolucin, vale


decir, queremos determinar Y() usando la ecuacin

j t
x ( ) h ( t ) d e dt

Y ( ) F y ( t )

Intercambiando el orden de integracin y observando que x() no depende de t, tenemos

Y ( ) x ( ) h ( t ) e jt d dt

Por el teorema del retardo, el trmino entre corchetes es simplemente H()e j, y entonces

Y ( )

x ( ) e

H ( ) d

H ( )

x ( ) e

H ( ) X ( )

x(t)
X()

h(t)
LIT
H()

y(t) = x(t)h(t)
Y() = X()H()

Figura 3.23. Propiedad de convolucin de sistemas LIT.

As que una convolucin en el dominio del tiempo es equivalente a una multiplicacin en el dominio de
la frecuencia, la cual, en muchos casos, es conveniente y se puede hacer por inspeccin. El uso de la
propiedad de convolucin para sistemas LIT se muestra en la Fig. 3.23. El espectro de amplitud y el de
fase de la salida y(t) estn relacionados con los espectros de la entrada x(t) y la respuesta al impulso h(t)
en la forma

Y ( ) X ( ) H ( )
Y ( ) X ( ) H ( )

210

La funcin H() o H(f), la transformada de Fourier de la respuesta al impulso del sistema,


generalmente se conoce como la respuesta de frecuencia del sistema. Muchas de las propiedades de
sistemas LIT pueden interpretarse convenientemente en trminos de H(). Por ejemplo, la respuesta al
impulso de la conexin en cascada de dos sistemas LIT es la convolucin de las respuestas al impulso
de los sistemas individuales y la respuesta completa no depende del orden en el cual los sistemas estn
en la cascada (demustrelo!). Usando la Ec. (3.113) podemos definir esto en trminos de las respuestas
de frecuencia. Como se ilustra en la Fig. 3.24, la respuesta de frecuencia total de los dos sistemas en
cascada es simplemente el producto de las respuestas de frecuencia individuales, y de esto est claro
que la respuesta total no depende del orden de la cascada.

x(t)

H1()

y(t)

H2()
(a)

x(t)

y(t)

H1()H2()
(b)

x(t)

H2()

y(t)

H1()
(c)

Figura 3.24. Tres sistemas LIT equivalentes.

Ejemplo 15. La convolucin peridica f ( t ) x1 ( t ) x2 ( t ) se defini en el Ejemplo 11 del Cap. 2. Si


dn y en son los coeficientes de Fourier complejos de x1(t) y x2(t), respectivamente, demuestre que los
coeficientes de Fourier complejos ck de f (t) estn dados por
ck T0 dk ek

donde T0 es el perodo fundamental comn a x1(t), x2(t) y f (t).


Sabemos que
T0

f ( t ) x1 ( t ) x2 ( t )

x ( ) x
1

( ) d

Sean

x1 ( t )

d k e jk0 t

las series de Fourier para x1 y x2. Entonces

y x2 ( t )

e e
k

jk 0 t

211

T0

f (t )

x ( )
ek e jk 0 ( t ) d
k

T0

e e x () e
jk 0 t

jk 0

Como

dk

1
T0

T0

x ( ) e

jk 0

se obtiene que

f (t )

T d e e
0

jk 0 t

k k

la cual muestra que los coeficientes de Fourier complejos ck de f (t) son iguales a T0dkek.
3.7.1

Las Funciones Escaln y Signo

La falta de simetra en la funcin escaln crea un problema cuando tratamos de determinar su


transformada en el lmite. Para resolver este problema, comenzamos con la funcin signo, Fig. 3.25,
definida como
1
sgn t
1

t 0
t 0

la cual presenta simetra impar.

sgn t
1
0

Figura 3.25. La funcin signo.

La funcin signo puede considerarse como un caso lmite de la funcin

212

e bt

z ( t ) bt

t 0
t0

de manera que z ( t ) sgn t si b 0 . La transformada de z(t ) es

Z( f )

j 4 f
b (2 f )2
2

y, por tanto,

F sgn t lm Z ( f )
b0

1
jf

de donde se obtiene el par de transformadas


1

sgn t

j f

Las funciones escaln y signo estn relacionadas por la ecuacin


u (t )

1
2

sgn t 1

1
2

sgn t

1
2

y, por consiguiente,
u (t )

1
( f )
j 2 f 2

(3.114)

Ahora queremos derivar la propiedad de integracin cuando la seal integrada tiene un rea neta
diferente de cero. Esta propiedad se obtiene mediante la convolucin de u(t) con una seal de energa
arbitraria x(t),

x (t ) u (t )

x ( ) u (t ) d

x ( ) d

Pero, del teorema de la convolucin y la Ec. (3.114), se tiene que


1
1

( f )
j 2 f 2

F x (t ) u (t ) X ( f )
por lo que
t

x ( ) d

1
j 2 f

1
2

X (0) ( f )

la cual se reduce a nuestro teorema de integracin previo cuando X(0) = 0.

(3.115)

213

Ejemplo 16. Considere un sistema LIT con respuesta al impulso


h ( t ) e at u ( t )

y cuya excitacin es la funcin escaln unitario u(t). La transformada de Fourier de la salida es


1 1

Y ( ) F u ( t ) F e a t u ( t ) ( )

j a j

1
( )
a
j a j

1
1 1 1
( )

a
j a a j

Tomando la transformada de Fourier inversa de ambos lados resulta en


1
u ( t ) e at u ( t )
a
a
1
1 e at u ( t )
a

y (t )

3.8 Modulacin

La propiedad de convolucin expresa que la convolucin en el dominio del tiempo se corresponde con
una multiplicacin en el dominio de la frecuencia. A causa de la propiedad de dualidad entre los
dominios del tiempo y de la frecuencia, es de esperar que tambin se cumpla una propiedad dual.
Especficamente, si
x(t) X()
m(t) M()
entonces

x (t ) m(t )

1
2

X ( ) M ( ) X ( f ) M ( f )

(3.116)

La convolucin de seales de frecuencia se obtiene exactamente igual a la convolucin en el dominio


del tiempo; es decir,

X ( ) H ( )

X ( ) H ( ) d

H ( ) X ( ) d

La multiplicacin de la seal deseada x(t) por m(t) es equivalente a alterar o modular la amplitud de x(t)
de acuerdo con las variaciones en m(t), y por ello tambin se denomina modulacin de amplitud. La
importancia de esta propiedad se ilustrar con algunos ejemplos.

214

Ejemplo 17. Sea x(t) una seal cuyo espectro X() se muestra en la Fig. 3.26a. Tambin considere la
seal m(t) definida por
m ( t ) cos 0 t

Entonces
M ( ) ( 0 ) ( 0 )

como se muestra en la Fig. 3.28b, y el espectro (1/2) M()*X() de m(t)x(t) se obtiene aplicando la
Ec. (3.112):
1
2

M ( ) X ( ) 12 X ( 0 ) 12 X ( 0 ) R ( )

el cual se grafica en la Fig. 3.26c. Aqu hemos tomado 0 > 1 para que las partes diferentes de cero de
R() no se solapen. Vemos entonces que el espectro de la onda resultante de la multiplicacin en el
tiempo consiste de la suma de dos versiones desplazadas y escaladas de X().
S()
A

(a)
P()

R() 21 [S () P()]

A/2

01 0 0+1

01 0 0+1

(c)

(b)

Figura 3.26. La propiedad de modulacin.

De la Ec. (3.116) y de la Fig. 3.26 est claro que toda la informacin contenida en la seal x(t) se
preserva cuando la multiplicamos por una seal sinusoidal, aunque la informacin ha sido corrida hacia
frecuencias mayores. Este hecho forma la base para los sistemas de modulacin de amplitud sinusoidal
y en el prximo ejemplo le damos una mirada a cmo podemos recuperar la seal original x(t) a partir
de la seal modulada.

Ejemplo 18. Consideremos ahora la seal r(t) = m(t)x(t) en el Ejemplo 16 y sea


g(t) = r(t)m(t)

215

donde, de nuevo, m ( t ) cos 0 t . Entonces, R(), M() y G() son como se muestra en la Fig. 3.27.

R()
A/2

(a)

M()

G()

A/2
A/4

(c)

(b)

Figura 3.27. Espectros de las seales en el Ejemplo 18.

De la Fig. 3.27c y de la linealidad de la transformada de Fourier, vemos que g(t) es la suma de 12 x ( t )


y una seal con un espectro que es diferente de cero solamente para las frecuencias ms altas (centradas
alrededor de 20). Suponga que aplicamos la seal g(t) como la entrada a un sistema LIT con
respuesta de frecuencia H() que es constante para frecuencias bajas (digamos para 1 y cero
para las frecuencias altas (para 0 ). Entonces la salida de este sistema tendr como su espectro
H()G(), la cual, debido a la seleccin particular de H(), ser una rplica a escala de X(). Por lo
tanto, la salida misma ser una versin a escala de x(t).
3.9 Generacin de Otros Pares de Transformadas
El impulso y el escaln unitarios estn relacionados por la identidad
( t td )

d
dt

u ( t td )

(3.117)

la cual proporciona otra interpretacin del impulso en trminos de la derivada de una discontinuidad en
la forma de un escaln. Esta ecuacin, junto con la ecuacin de definicin del impulso, facilita ciertos
clculos de transformadas y ayuda a predecir el comportamiento de alta frecuencia de una seal. El
mtodo es el siguiente: Se diferencia repetidamente la seal bajo anlisis hasta que aparezcan por
primera vez una o ms discontinuidades escalonadas. La siguiente derivada, digamos la n-sima,
incluye entonces un impulso Ak ( t tk ) por cada discontinuidad de amplitud Ak, por lo que

216

dn
dt n

x (t ) y (t )

A (t t
k

(3.118)

donde y (t ) es una funcin que no contiene impulsos. Transformando la Ec. (3.118) da


( j 2 f )n X ( f ) Y ( f )

A e

j 2 f tk

(3.119)

la cual puede resolverse para obtener X ( f ) si se conoce Y ( f ) .

Ejemplo 19. La Fig. 3.28 muestra una forma de onda llamada el pulso coseno levantado porque est
definida como
x (t )

A
t
1 cos
2

t

2

Usaremos el mtodo de diferenciacin para determinar el espectro X ( f ) y el comportamiento en alta


frecuencia.
Al derivar se encuentra que

dx ( t )
dt

d 2 x (t )
dt 2

A
t t A
t
sen 1 cos ( t ) ( t )
2
2 2

A
t t
sen
2
2

A
t t A
t
cos
sen ( t ) ( t )
2
2 2

A
t t
cos
2
2
2

Observe que d 2 x(t)/dt2 es discontinua en y = y, por ello, a partir de esta ltima relacin se tiene que

d 3 x (t )
dt 3

dx ( t ) A


2
dt
2

( t ) ( t )

y tomando la transformada se obtiene

j 2 f

X ( f )

A
j 2 f X ( f ) e j 2 ft e j 2 ft
2
2

para obtener finalmente que

X(f )

jA sen 2 f
j 2 f ( )2 ( j 2 f )3

A sinc 2 f
1 (2 f ) 2

217

A
x(t )

A
2

(a)

dx
dt

0
2

A

2

d2x
dt 2
(b)

2
A
2

d 3x
dt3

A

2

(c)

A

2

A
X( f )

A
2 f 1 (2 f) 2

A
2

1 2

3 2

Figura 3.28. Pulso coseno levantado. (a) Forma de onda; (b) derivadas; (c)
espectro de amplitudes.

cuyo espectro de amplitudes se muestra en la Fig. 3.28 para f 0 . Observe que X ( f ) tiene un
comportamiento de tercer orden (n = 3), en tanto que un pulso rectangular con
X ( f ) sinc f (sen f ) ( f ) tendra solamente un comportamiento de primer orden.

3.10 Densidad Espectral de Potencia


Existe una clase importante de seales, a saber, las seales de potencia no-peridicas, para las cuales
todava no hemos desarrollado un modelo en el dominio de la frecuencia. La serie de Fourier no existe
para seales de potencia no peridicas que no estn limitadas en el tiempo. Para esas seales, la

218

transformada de Fourier puede existir o no, y no es aplicable el concepto de densidad espectral de


energa. Sin embargo, puesto que se supone que la potencia promedio es finita, podemos usar funciones
de la densidad espectral de potencia para describir seales de potencia no-peridicas en el dominio de
la frecuencia.
Suponga que se nos da una seal de potencia x(t). Formemos con ella una versin truncada xT ( t ) de
x(t), como se muestra en la Fig. 3.29. La potencia promedio normalizada de xT ( t ) est dada por

S
T
x

T 2

1
T

(3.120)

x ( t ) dt

T 2

x(t)

T/2

T/2

(a)
xT(t)

T/2

T/2

(b)

Figura 3.29. (a) Una seal de potencia. (b) Versin truncada de la seal.

Ahora se introduce la funcin de autocorrelacin RxxT ( ) de xT(t) definida como

R ( )
T
xx

1
T

T 2

xT ( t ) xT ( t ) d

(3.121)

T 2

T
La transformada de Fourier de Rxx
( ) es

R ( ) exp( j 2 f ) d
exp( j 2 f ) xT ( t ) xT ( t ) dt d
T

T
xx

1
T

xT ( t ) xT ( t ) exp( j 2 f ( t )) exp( j 2 ft ) dt d

xT ( t ) exp( j 2 ft ) xT ( t ) exp( j 2 f ( t )) d dt

1
(3.122)
XT ( f ) XT ( f )
T
1

219

donde X T ( f ) es la transformada de Fourier de xT(t).


Puesto que x(t) es real, tenemos que X T ( f ) X T ( f ) y, por tanto,

RxxT ( ) exp( j 2 f ) d

XT ( f )

Haciendo Rxx ( ) lm RxxT ( ) y tomando el lmite T en ambos lados de la ecuacin anterior, se


T

obtiene que

Rxx ( ) exp( j 2 f ) d lm

1
T

XT ( f )

(3.123)

El lector debe observar que el lmite en el lado derecho de la Ec. (3.123) puede no existir siempre. El
lado izquierdo de la Ec. (3.123) es la transformada de Fourier de la funcin de autocorrelacin de la
2
seal de potencia x(t). En el lado derecho tenemos a X T ( f ) , que es la densidad espectral de energa
2

de xT(t) y, por lo tanto, X T ( f ) T da la distribucin de potencia en el dominio de la frecuencia. Por


esta razn podemos usar la relacin en la Ec. (3.123) para definir la densidad espectral de potencia
(dep) Gx(f) de x(t) como
Gx ( f ) F{ Rxx ( )}
lm

XT ( f )

(3.124)
2

(3.125)

La relacin dada en la Ec. (3.123) se conoce como el teorema de Wiener Khintchine y tiene gran
importancia en la teora de seales aleatorias.
Usaremos la dep definida en la Ec. (3.123) como la descripcin en el dominio de la frecuencia de
seales de potencia no-peridicas. Ya se defini previamente la dep para seales de potencia peridicas
como
Gx ( f )

Cx ( nf 0 ) ( f nf 0 )
2

El lector puede verificar que la definicin dada en la Ec. (3.123) en efecto se reduce a la ecuacin
anterior para seales peridicas. En ambos casos, para una seal de potencia tenemos que

S x lm

1
T

T 2

x 2 ( t ) dt

T 2

Rxx (0)

G ( f ) df
x

(3.126)

220

Si x(t) es una seal de corriente o de voltaje que alimenta una resistencia de carga de un ohmio,
entonces S x tiene las unidades de vatios y por ello a Gx ( f ) se le da las unidades de vatios por hertz.
Si la resistencia de carga tiene un valor diferente de un ohmio, entonces Gx ( f ) usualmente se
especifica en trminos de volts2 por hertz.
Se debe sealar aqu que la funcin de la densidad espectral de potencia (y la funcin de
autocorrelacin) no describe en forma nica una seal. La dep retiene solamente la informacin de la
magnitud y se pierde la informacin de la fase. As que para una seal de potencia dada, hay una
densidad espectral de potencia, pero hay muchas seales que tienen la misma densidad espectral de
potencia. En contraste, las series de Fourier y las transformadas de Fourier de seales, cuando existen,
describen en forma nica una seal en todos los puntos de continuidad.
Ejemplo 20. La funcin de autocorrelacin de una seal de potencia no-peridica es
Rxx ( ) exp ( 2 2 2 )

Determine la dep y el contenido de potencia promedio normalizada de la seal.


Solucin: Por el teorema de Wiener Khintchine, la dep de la seal est dada por

Gx ( f )

exp(

2 2 ) exp( j 2 f ) d

2 2 exp (2 f )2 2
La potencia promedio normalizada est dada por

S X lm

1
T

T 2

x 2 ( t ) dt Rxx (0) = 1

T 2

Ejemplo 21. Determine la funcin de autocorrelacin y la densidad espectral de potencia de una forma
de onda rectangular con un perodo T0, una amplitud pico igual a A y un valor promedio de A/2.
Solucin
Puesto que x(t) es peridica, necesitamos obtener el promedio en el tiempo para la correlacin por
un perodo solamente, es decir,

Rxx ( )

1
T0

T0 2

x ( t ) x ( t ) dt

T0 2

En la Fig. 3.30 se muestran dibujos de x(t) y x(t + ). Para 0 T0 2 , el valor de Rxx ( ) es igual al
rea sombreada en la Fig. 3.30c:

Rxx ( )

A2 T0

2 1
A
T0 2

2 T0

T0
2

221

x()

,,,

(a)

,,,
T0

T0/2

T0/2

T0

x(t +)
A

,,,

(b)

T0/2

T0

T0/2

,,,

(c)

T0

x(t)x(t +)

A2

,,,
T0

T0/2

T0/2

T0

x(t)x(t +)
Rxx()

A /2
(d)

T0/2

,,,

,,,

,,,
T0

T0/2

T0/2

T0

Figura 3.30 (a) Seal x(t). (b) x(t + ), la cual es integrada desde
T0 2 hasta T0 2 para obtener R xx () . (d) R xx () .

Se puede verificar fcilmente que Rxx ( ) es una funcin par y que ser peridica con un perodo igual
a T0. En la Fig. 3.30d se muestra un dibujo de Rxx ( ) . La dep de x(t) est dada por

Gx ( f )

xx

( ) exp( j 2 f ) d

A2
4
1 2 2
4 n n
n impar

exp( j 2 nf 0 ) exp ( j 2 f ) d

donde el trmino entre corchetes dentro de la integral es la serie de Fourier para Rxx ( ) . Completando
la integracin obtenemos

A2
4
Gx ( f )
( f )
( f nf 0 )
2 2

4
n n

n impar

222

Problemas
3.1. La serie exponencial de Fourier de una cierta seal peridica x(t) est dada por
x(t ) j exp ( j 4t ) (3 j3) exp ( j3t ) (2 j 2) exp ( j 2t )
(2 j 2) exp( j 2t ) (3 j3) exp ( j3t ) j exp ( j 4t

(a) Grafique el espectro de magnitud y el de fase de los dos espectros bilaterales de frecuencia.
(b) Escriba x(t) en la forma trigonomtrica de la serie de Fourier.
3.2. La seal mostrada en la Fig. P3.2 es creada cuando una onda de voltaje o de corriente en forma de
coseno es rectificada por un solo diodo, un proceso conocido como rectificacin de media onda.
Deduzca la serie de Fourier exponencial para la seal rectificada de media onda.
x(t ) cos t

1
,,,

,,,

3
2

3
2

5
2

Figura P3.2

3.3. Determine la expansin en serie de Fourier trigonomtrica para la seal en el Problema 3.2.
3.4. Dada la onda peridica x ( t ) e t , 0 < t < T0, determine los coeficientes de la serie exponencial de
Fourier y dibuje los espectros de amplitud y de fase.
3.5. La seal en la Fig. P3.5 se crea cuando una onda de voltaje o de corriente en seno es rectificada
por un circuito con dos diodos, un proceso conocido como rectificacin de onda completa.
Determine la expansin en serie de Fourier exponencial para la seal rectificada de onda
completa.

x(t ) sen t
,,,

,,,
0

Figura P3.5

3.6. Halle la representacin en serie de Fourier trigonomtrica para la seal en el Problema 3.5.

223

3.7. Halle las representaciones en series de Fourier exponenciales de las seales mostradas en la Fig.
P3.7. En cada caso, grafique los espectros de magnitud y de fase.
3.8. Determine las representaciones en serie de Fourier trigonomtrica de las seales mostradas en la
Figura P3.7.
x(t)

x(t)

...

...
1

...

...

(a)

(b)

x(t)

x(t)

...

...

...
2 1

0 1

...

2
1 0

(c)

(d)
Figura P3.7

3.8. (a) Demuestre que si una seal peridica es absolutamente integrable, entonces cn < .
(b) Tiene representacin en serie de Fourier la seal peridica x ( t ) sen

2
t

? Explique su

respuesta.
(c) Tiene representacin en serie de Fourier la seal peridica x ( t ) tan 2 t ? Por qu?
3.9. (a) Demuestre que x ( t ) t 2 , t , x ( t 2 ) x ( t ) tiene la serie de Fourier
x (t )

1
1

4 cos t cos 2 t cos 3t


3
4
9

(b) Haga t = 0 para obtener la expresin

n 1

( 1)n 1
n2

2
12

3.10. Los coeficientes de Fourier de una seal peridica con perodo T vienen dados por

224

n0
0,

cn
n

1 exp j 3 2 exp jn , n 0

Representa esto una seal real? Por qu o por qu no? A partir de la forma de cn, deduzca la
seal de tiempo x(t). Ayuda: Use la relacin

exp( jn t ) ( t t ) dt exp( jn t )
1

3.11. (a) Grafique la seal

x(t )

1
4

n sen 4 cos 2 nt
2

n 1

Para M = 1, 3 y 5.
(d) Prediga la forma de x(t) conforme M .
3.12 Halle las series de Fourier exponencial y trigonomtrica para los trenes de impulsos mostrados en
la Fig. P3.12.
x(t)

x(t)

...

...

...

...
2

2 1 0

2
1

0 1

Figura P3.12

3.13 (a) Calcule la energa de la seal x(t) dada en la Figura P3.12.


(b) Si la energa de los primeros cuatro armnicos es igual a 0.0268 julios, calcule la energa
contenida en el resto de los armnicos.
3.14 Especifique los tipos de simetra para las seales mostradas en la Figura P3.14. Especifique
tambin los trminos que son iguales a cero.
3.15 Demuestre que el valor cuadrtico medio de una seal peridica real x(t) es igual a la suma de los
valores cuadrticos medios de sus armnicos.
3.16 Conociendo la expansin en serie de Fourier de x1(t) mostrada en la Figura P3.16a. determine los
coeficientes de x2(t) mostrada en la Figura P3.16b.
3.17 Considere la onda triangular mostrada en la Fig. P3.17. Usando la tcnica de diferenciacin,
determine (a) la serie de Fourier exponencial de x(t), y (b) la serie de Fourier trigonomtrica de
x(t).

225

x(t)

x(t)
2

...
2

...
0

...

...
4 2

(b)

(a)
x(t)

x(t)

...

...

...

...
1 2

3
T

2T

2 1 0

(c)

(d)
Figura P3.14
x2(t)
2

x1(t)

...

...
T

T T+

...

...
3 1 0

2 3

Figura P3.16

x(t)
A

T0

T0

2T0

Figura P3.17

3.18 La convolucin peridica o circular es un caso especial de la convolucin general. Para seales
peridicas con el mismo perodo T, la convolucin peridica se define mediante la integral
z (t )

1
T

x ( ) y (t ) d
T

226

(a) Demuestre que z(t) es peridica.


(b) Demuestre que la convolucin peridica es asociativa y conmutativa.
3.19 Determine la convolucin peridica de las dos seales mostradas en la Fig. P3.19.

x(t)

y(t)
1

...

...
2

1 0 1

...

...
2

Figura P3.19

3.20 Considere la seal peridica x(t) cuya serie exponencial de Fourier es

x(t )

exp( jn0 t ),

c0 0

(a) Integre trmino por trmino para obtener la expansin de Fourier de y(t) = x(t)dt y demuestre
que y(t) tambin es peridica.
(b) Cmo se comparan las amplitudes de los armnicos de y(t) con las amplitudes de los
armnicos de x(t)?
(c) La integracin les quita o les pone nfasis a las componentes de alta frecuencia?
(d) Usando la parte (c), es la onda integrada ms suave que la original? Explique.
3.21 La representacin en serie de Fourier de la seal triangular en la Figura P3.21(a) es
x (t )

8
1
1
1

sen
t

sen
3
t

sen
5
t

sen 7 t

2
9
25
49

Use este resultado para obtener la serie de Fourier para la seal en la Fig. P3.21b.

x(t)
x(t)

...

...

...

...

Figura P3.21

227

3.22 Un voltaje x(t) se aplica al circuito mostrado en la Figura P.3.22. Si los coeficientes de Fourier de
x(t) estn dados por

exp jn
n 1
3
1

cn

(a)
(b)
(c)
(d)
(e)

Demuestre que x(t) debe ser una seal real del tiempo.
Cul es el valor promedio de la seal?
Determine los tres primeros armnicos de y(t) diferentes de cero.
Qu le hace el circuito a los trminos de alta frecuencia de la entrada?
Repita las partes (c) y (d) para el caso donde y(t) es el voltaje en el resistor.
1

x (t )

1F

+
y (t )

Figura P3.22

3.23 Si el voltaje de entrada al circuito mostrado en la Fig. P3.22 es


x ( t ) 1 2(cos t cos 2t cos3t )

determine el voltaje de salida y(t).


3.24 La entrada

x (t )

exp( jn0 t )

se aplica a cuatro sistemas diferentes cuyas respuestas son

y1 ( t )

( 3) c

y2 ( t )

y3 ( t )

exp jn0 t t0

cn exp j n0 t n 2 0 t0

y4 ( t )

n c

exp j n0 t 3n0 t0

exp j n0 t

228

Cul de ellos, si hay alguno, es un sistema sin distorsin?


3.25 Para el circuito mostrado en la Fig. P3.25,
(a) Determine la funcin de transferencia H(j).
(b) Grafique H(j) y H(j).
(c) Considere la entrada x ( t ) 10exp( jt ). Cul es la frecuencia ms alta que se puede usar
de forma que

y (t ) x (t )
x (t )

0.01 ?

(d) Cul es la mayor frecuencia que se puede usar tal que H(j) se desve de la caracterstica
lineal ideal por menos de 0.02?
1 k

x (t )

1 nF

+
y (t )

Figura P3.25

3.26 Se pueden usar dispositivos no lineales para generar armnicos de la frecuencia de entrada.
Considere el sistema no lineal descrito por
y ( t ) A x ( t ) B x2 ( t )

Determine la respuesta del sistema a x ( t ) a1 cos 0 t a2 cos 2 0 t . Haga una lista de todos los
nuevos armnicos generados por el sistema y tambin sus amplitudes.
3.27 Se usa una fuente de frecuencia variable para medir la funcin del sistema H(j) de un sistema
LIT cuya respuesta al impulso es h(t). La salida de la fuente y ( t ) exp( j t ) se conecta a la
entrada del sistema LIT. La salida H ( ) exp( j t ) se mide para frecuencias diferentes. Los
resultados se muestran en la Fig. P3.27.

H ()

H ()

12
4103

4103

Figura P3.27

Determine la respuesta del sistema para la seal de entrada siguiente:

229

x(t)
1
...

...
700

500

300

200

200

300

500

700

3.28 Para el sistema mostrado en la Fig. P3.28, la entrada x(t) es peridica con perodo T. Demuestre
que yc(t) y ys(t) en cualquier tiempo t > T1 despus de aplicar la entrada se aproximan a Re{cn } e
Im{cn}, respectivamente. En efecto, si T1 es un mltiplo entero del perodo t de la seal de
entrada x(t), entonces las salidas son exactamente iguales a los valores deseados. Discuta las
salidas para los casos siguientes:
(a) T1 = T
(b) T1 = mT
(c) T1 >> T pero T1 T

x(t)

h(t)

yc(t)

h(t)

cos n0 t

sen n0 t

2
T

ys(t)

h(t)

1/T1

T1

Figura P3.28

3.29 La ecuacin diferencial siguiente es un modelo para un sistema lineal con entrada x(t) y salida
y(t):
d y (t )
dt

2 y (t ) x (t )

Si la entrada x(t) es una onda cuadrada de perodo 2 s, duracin de l pulso igual a 1 s y amplitud
unitaria, determine las amplitudes del primer y tercer armnicos en la salida.
3.30 Repita el Problema 3.29 para el sistema descrito por la ecuacin diferencial
y ( t ) 4 y ( t ) 3 y ( t ) 2 x ( t ) x ( t )

3.31 Considere el circuito mostrado en la Fig. P3.31. La entrada es la seal rectificada de media onda
del Problema 3.2. Halle la amplitud del segundo y cuarto armnico de la salida y(t).

230

500
+

100 F

x (t )

500

y (t )

Figura P3.31

3.32 Considere el circuito mostrado en la Fig. P3.32. La entrada es la seal rectificada de media onda
del Problema 3.2. Determine la amplitud del segundo y del cuarto armnico de la salida y(t).

0 .1 H
+

x (t )

100 F

1 k

y (t )

Figura P3.32

3.33 Considere el circuito mostrado en la Fig. P3.31. La entrada es la seal rectificada de onda
completa del Problema 3.5. Determine la componente cd y la amplitud del segundo armnico de
la salida y(t).
3.34 Considere el circuito mostrado en la Fig. P3.32. La entrada el la seal rectificada de onda
completa del Problema 3.5. Determine la componente cd y la amplitud del segundo y el cuarto
armnico de la salida y(t).
3.35 Demuestre que las relaciones siguientes son identidades:

1
sen n 0 t
N
2

(a) exp( jn0 t )


sen 0 t 2
n N
T 2

(b)

1
T

1
sen n 0 t
3

dt 1
sen 0 t 2

T 2

3.36 Para la seal x(t) mostrada en la Fig. P3.15a, mantenga T fijo y discuta el efecto de variarcon
la restriccin < Tsobre los coeficientes de Fourier.

231

3.37 Considere la seal mostrada en la Fig. P3.15a. Determine el efecto sobre la amplitud del segundo
armnico de x(t) cuando hay un error muy pequeo en la medicin de . Para hacer esto, haga =
0 , donde << 0, y determine la variacin del segundo armnico con respecto a . Determine
el cambio porcentual en c2 cuando T 10 y = 0.1.
3.38 En la Figura P3.38 se muestra una onda sinusoidal truncada.
(a) Determine los coeficientes de la serie de Fourier.
(b) Calcule la amplitud del tercer armnico para B = A/2.
(c) Obtenga t0 de manera que c3 Sea un mximo. Este mtodo se usa para generar contenido
armnico a partir de una forma de onda sinusoidal.
x(t)

Asen t
B

0 t0

t0

Figura P3.38

3.39 Para la seal x(t) mostrada en la Fig. P3.39 determine lo siguiente:


(a) Los coeficientes de la serie de Fourier.
(b) Resuelva por el valor de t0 para el cual c3 es mximo.
(c) Compare el resultado con la parte (c) del Problema 3.38.
x(t)

Asen t
B
t0
2 2t0

t0

0
t0

Figura P3.39

3.40 La seal mostrada en la Fig. P3.40 es la salida es la seal rectificada de media onda suavizada.
Las constantes t1, t2 y A satisfacen las relaciones siguientes:
t1 tan 1 ( RC )

232

t
A sen t1 exp 1
RC
t
A exp 2 sen t2
RC
RC 0.1 s
2 60 377 rad/s

(a) Verifique que t1 1.5973 rad, A = 1.0429, y t2 7.316 rad.


(b) Determine los coeficientes de la serie de Fourier exponencial.
(c) Determine la relacin entre las amplitudes del primer armnico y la componente cd.

sen t

A
1

A exp( 10t )

t0
0

t1

2 t2

Figura P3.40

3.41 Determine la transformada de Fourier de una seal peridica x(t) con perodo T0.
3.42 La transformada de Fourier de una seal x(t) est dada por [Fig. P3.42]
X ( ) 12 pa ( 0 ) 12 pa ( 0 )

Determine y dibuje x(t).


X()
1/2

Figura P3.42

3.43 Un sistema se excita mediante la seal

+a

233

t
x ( t ) 6 rect
4

y su respuesta es 10 1 e ( t 2 ) u ( t 2) 1 e ( t 2 ) u ( t 2 . Cul es su respuesta al impulso?


3.44 Usando el teorema de convolucin, determine la transformada de Fourier inversa de

X ( )

1
( a j )2

3.45 Dibuje una grfica de la seal producida por la convolucin de las dos funciones siguientes:
(a) 2rect( t ) rect( t )
t 1
t 1
(b) rect
rect

2
2

(c) 2 ( t ) 5sen t
3.46 Determine la transformada de Fourier de las seales siguientes:
(a) x(t) = u(t)
(b) x ( t ) eat u ( t ), a 0
3.47 Determine la transformada de Fourier del pulso gaussiano

x ( t ) e at

a0

3.48 Halle la transformada de Fourier inversa de


(a) X ( )
(b) X ( )

1
( a j ) N
1
2 j 3
2

3.49 Halle la transformada de Fourier de las seales siguientes, usando la propiedad de multiplicacin:
(a) x ( t ) cos 0 t u ( t )
(b) x ( t ) sen 0 t u ( t )
(c) x ( t ) e at cos 0 t u ( t ), a 0
(d) x ( t ) e at sen 0 t u ( t ), a 0
3.50 Sea x(t) una seal cuya transformada de Fourier est dada por

234

X ( )

1
1

Considere la seal

y (t )

d 2 x (t )
dt 2

Determine el valor de

y ( t ) dt

3.51 Dibuje la transformada de Fourier inversa de la funcin en la Fig. P3.51.


X(f )
2

X(f )

Figura P3.51

3.52 Sea x(t) una seal real cuya transformada de Fourier es X(). La seal analtica x+(t) asociada
con x(t) es una seal compleja definida por
x ( t ) x ( t ) j x ( t )

donde x (t ) es la transformada de Hilbert de x(t).


(a) Halle la transformada de Fourier X+() de x+(t).
(b) Halle la seal analtica x+(t) asociada con cos 0 t y su transformada de Fourier X+().
3.53 Considere una seal real x(t) y sea
X ( ) F { x ( t )} A ( ) j B ( )

y
x ( t ) x p ( t ) xi ( t )

235

donde xp(t) y xi(t) con las componentes par e impar de x(t), respectivamente. Demuestre que

xp (t )

A ( )

xi ( t )

jB ( )

3.54 Considere un sistema LIT de tiempo continuo con respuesta de frecuencia H(). Determine la
transformada de Fourier S() de la respuesta al impulso unitario s(t) del sistema.
3.55 Un sistema se excita mediante la seal
t
x ( t ) 4 rect
2

y su respuesta es

y ( t ) 10 1 e ( t 1) u ( t 1) 1 e ( t 1) u ( t 1)
Cul es la respuesta al impulso?

CAPTULO CUATRO

ANLISIS DE FOURIER
TIEMPO DISCRETO

4. 1 Introduccin

Las tcnicas del anlisis de Fourier en tiempo continuo desarrolladas en el captulo anterior tienen
mucho valor en el anlisis de las propiedades de seales y sistemas de tiempo continuo. En esta parte
nos dedicamos al estudio del anlisis de Fourier en tiempo discreto, dando primero un leve tratamiento
de las seales discretas. stas, como su nombre lo indica, son seales que estn definidas solamente en
instantes discretos del tiempo El enfoque sigue muy de cerca el tratamiento que se hizo del caso en
tiempo continuo y los resultados son muy semejantes a los obtenidos en el Captulo 3.

4.2 Seales Peridicas

Como ya se vio para los sistemas de tiempo continuo, estamos interesados en la respuesta de sistemas
lineales a excitaciones peridicas. Ya se estudi que una secuencia (seal de tiempo discreto) x[n] es
peridica con perodo N si existe un entero positivo N para el cual

x[ n + N ] = x[ n]

para toda n

(4.1)

En la Fig. 4.1 se muestra un ejemplo de una secuencia de este tipo.


De la Fig. 4.1 y la Ec. (4.1) se deduce que

x [ n + mN ] = x [ n ]

(4.2)

x[n]
...

...
N

Figura 4.1

2N

238

para toda n y cualquier entero m. El perodo fundamental N0 de x[n] es el menor entero positivo N para
el cual se cumple la Ec. (4.1). Una secuencia que no es peridica se denomina una secuencia no
peridica (o aperidica).
Para una seal de tiempo discreto x[n], el contenido de energa normalizada E de x[n] se define como

E=

x[ n]

(4.3)

n =

La potencia promedio normalizada P de x[n] se define como


P = lm

2 N +1

x[n]

(4.4)

n = N

Con base en estas definiciones, se definen las siguientes clases de seales:


1. Se dice que x[n] es una seal (secuencia) de energa si y slo si 0 < E < (y, en consecuencia, P =
0).
2. Se dice que x[n] es una seal (secuencia) de potencia si y slo si 0 < P < , implicando con ello que
E =.
3. A las seales que no satisfacen ninguna de estas propiedades no se les refiere ni como seales de
energa ni de potencia.
Observe que una seal peridica es una seal de potencia si su contenido de energa por perodo es
finito, y entonces la potencia promedio de esta seal slo tiene que evaluarse durante un perodo.

4.3

Serie de Fourier Discreta

4.3.1 Secuencias Peridicas

En la Sec. 1.3 se defini a una seal (secuencia) de tiempo discreto como peridica si exista un
entero positivo N para el cual
x[ n + N ] = x[ n]

para toda n

(4.5)

El perodo fundamental N0 de x[n] es el menor entero positivo N para el cual se satisface la Ec. (4.5).
Ya vimos en el Cap. 2, que la secuencia exponencial compleja
x [ n ] = e j ( 2 N 0 ) n = e j 0 n

(4.6)

donde 0 = 2 N 0 , es una secuencia peridica con perodo fundamental N0. Como ya se analiz
anteriormente, una diferencia muy importante entre la funcin exponencial compleja de tiempo discreto
y la de tiempo continuo es que las seales e j0 t son diferentes para valores diferentes de 0, pero las
secuencias e j0 n que difieren en frecuencia por un mltiplo de 2, son idnticas; es decir,
e j ( 0 + 2 ) n = e j0 n e j 2 kn = e j0 n

(4.7)

239

Sea
k [ n ] = e jk 0 n ,

0 =

2
N0

k = 0, 1, 2, K

(4.8)

Entonces por la Ec. (4.7) tenemos


0 [ n ] = N0 [ n ], 1 [ n ] = N0 +1 [ n ], K , k [ n ] = N0 + k [ n ], K

(4.9)

De modo que las secuencias k [ n ] son diferentes slo en un intervalo de N0 valores sucesivos de k.
Es decir, cuando k es cambiado por cualquier mltiplo entero de N0, se genera la secuencia idntica.

4.3.2

Representacin en Serie de Fourier Discreta

En analoga con la representacin de seales peridicas en tiempo continuo, se busca una


representacin en serie de Fourier discreta de una secuencia peridica x[n] con perodo fundamental N0,
en funcin de los armnicos correspondientes a la frecuencia fundamental 2 N 0 . Es decir, buscamos
una representacin para x[n] de la forma
x[ n] =

N 0 1

a e

j0 nk

k =0

0 =

(4.10)

N0

donde los valores ak son los coeficientes de Fourier y estn dados por
ak =

N 0 1

N0

n =0

x[ n]e

jk 0 n

(4.11)

La validez de la relacin dada por la Ec. (4.11) se demuestra en la forma siguiente: Usando la
condicin de ortogonalidad (la demostracin de sta se deja como ejercicio)

n= N

[ n ] k [ n ] =
=

jm ( 2 N ) n

j ( m k )( 2 N ) n

e jk ( 2 N ) n

n= N

n= N

N
=
0

m=k
mk

(4.12)
m, k < N

donde las secuencias { k [ n ]} son ortogonales en cualquier intervalo de longitud N. Por ejemplo, el
conjunto de exponenciales complejas
k [ n ] = e jk ( 2 N ) n

k = 0, 1, 2, K , N 1

(4.13)

es ortogonal en cualquier intervalo de longitud N.


Reemplazando la variable k de la sumatoria por m en la Ec. (4.10), tenemos
N 1

x [ n ] = am e jm ( 2 N0 ) n
m=0

(4.14)

240

Usando la Ec. (4.13) con N = N0, la Ec. (4.14) puede escribirse como
x[ n] =

N 0 1

m=0

m [ n]

(4.15)

Multiplicando ambos lados de la Ec. (4.15) por k [ n ] y sumando desde n = 0 hasta N0 1, se obtiene
N 0 1

n=0

N0 1

a

m m [n] k [n]
n=0
m=0

N 0 1

x [ n ] k [ n ] =

Intercambiando el orden de las sumatorias y usando la Ec. (4.12), obtenemos


N 0 1

n=0

N0 1

x [ n ] [ n ] = am m [ n ] k [ n ] = N 0 ak
m=0
n=0

N 0 1

y de aqu se obtiene la Ec. (4.11).


Usando la Ec. (4.9), las Ecs. (4.10) y (4.11) pueden escribirse como
x[ n] =

ak e jk 0 n ,

k = N0

ak =
donde

k = N0

N0

0 =

2
N0

x [ n ] e jk 0 n

(4.16)

(4.17)

n = N0

denota la sumatoria en k conforme k vara en un intervalo de N0 enteros sucesivos. As,

conforme n toma los n = 0, 1, , N0 1, las muestras x[n] de x(t) son aproximadas por la Ec. (4.16).
La Ec. (4.16) representa la ecuacin de sntesis y la Ec. (4.17) la ecuacin de anlisis. Haciendo k = 0
en la Ec. (4.17), se obtiene
a0 =

1
N0

x[ n]

(4.18)

n = N0

la cual indica que a0 es igual al valor promedio de x[n] en un perodo.


A los coeficientes de Fourier ak con frecuencia se les refiere como los coeficientes espectrales de
x[n]. Es fcil demostrar que a k = a k + N 0 (hgalo!). Es decir, si consideramos ms de N0 valores
secuenciales de k, los valores ak se repetirn peridicamente con perodo N0. Este hecho debe
interpretarse con cuidado. En particular, como solamente hay N0 exponenciales complejas distintas que
son peridicas con perodo N0, la representacin en serie de Fourier de tiempo discreto es una serie
finita con N0 trminos. Por consiguiente, si fijamos los N0 valores consecutivos de k para los cuales
definimos la serie de Fourier en la Ec.(4.16), obtendremos un conjunto de exactamente N0 coeficientes
a partir de la Ec. (4.17). Por otra parte, algunas veces ser conveniente usar diferentes conjuntos de N0
valores de k y, en consecuencia, es til considerar la Ec. (4.16) como una suma para cualquier conjunto
arbitrario de N0 valores sucesivos de k.

241

Ejemplo 1. En este ejemplo consideramos la onda cuadrada peridica en tiempo discreto mostrada en
la Fig. 4.2. Podemos evaluar la serie de Fourier de esta funcin usando la Ec. (4.17).

1
...

...
N0

N1

N0

Figura 4.2

Debido a la simetra de esta secuencia con respecto a n = 0, es conveniente seleccionar un intervalo


simtrico en el cual evaluar la sumatoria en la Ec. (4.17). Por ello, expresamos la Ec. (4.17) como

ak =

N1

1
N0

e jk ( 2 N0 ) n

n = N1

Haciendo m = n + N1, la ecuacin anterior se convierte en


ak =
=

1
N0

1
N0

2 N1

jk ( 2 N 0 )( m N1 )

m=0

e jk ( 2 N0 ) N1

2 N1

jk ( 2 N 0 ) m

m =0

cuya sumatoria consiste de los primeros (2N1 + 1) trminos en una serie geomtrica, la cual al ser
evaluada produce
ak =

1 e jk 2 ( 2 N1 +1 ) N0
e jk ( 2 N0 ) N1
1 e jk ( 2 2 N0 )
N0

jk ( 2 2 N 0 )
e jk 2 ( N1 + 2 ) N0 e jk 2 ( N1 + 2 ) N0 )
1 e

=
jk ( 2 2 N 0 )
jk ( 2 2 N 0 )
jk ( 2 2 N 0 )
N0
e

e
e
1

1 sen[2 k ( N1 + 1 2 ) N 0 ]
N0

sen (2 k 2 N 0 )

k 0, N 0 , 2 N 0 , K

y
ak =

2 N1
N0

k = 0, N 0 , 2 N 0 , K

Esta expresin puede escribirse en una forma ms compacta si los coeficientes se expresan como
muestras de una envolvente:

242

sen[(2 N1 + 1) 0 2]

N 0 ak =

sen ( 0 2)

0 = 2 k N 0

En la Fig. 4.3 se dibujan los coeficientes N0ak para 2N1 + 1 = 5 y N0 = 10.

N0a0

Envolvente

N0a1

2
10

Figura 4.3

Ejemplo 2. Determine la representacin en serie de Fourier discreta de la secuencia

x [ n ] = cos n + sen n .
3
4

Tomemos x [ n ] = cos n + sen n = x1[n] +x2[n], donde


3
4

x1 [ n ] = cos n = cos 1 n 1 =
3
3

x2 [ n ] = sen n = sen 2 n 2 =
4
4
Como 1 2 = 16 (= nmero racional), x1[n] es peridica con perodo fundamental N1 = 6, y como
2 2 = 18 (= nmero racional), x2[n] es peridica con perodo fundamental N2 = 8. Por tanto, x[n] es
peridica y su perodo fundamental est dado por el mnimo comn mltiplo de 6 y 8, es decir, N0 = 24
y 0 = 2 N 0 = 12 . Por la frmula de Euler tenemos que
x[ n] =

1
e j ( 3 ) n + e j ( 3 ) n +
e j ( 4 ) n e j ( 4 ) n
2
2j

1
1
1
e j 4 0 n + j e j 3 0 n j e j 3 0 n + e j 4 0 n
2
2
2
2

As que c3 = j ( 12 ) , c4 = 12 , c4 = c4+ 24 = c20 = 12 , c3 = c3+ 24 = c21 = j (


Por lo tanto, la serie de Fourier discreta de x[n] es

1
2

y todos los otros ck = 0 .

243

1
1
1
1
x [ n ] = j e j 3 0 n + e j 4 0 n + e j 20 0 n + j e j 21 0 n ,
2
2
2
2

4.3.3

0 =

12

Convergencia de la Serie de Fourier Discreta

Puesto que la serie de Fourier discreta de una secuencia x[n] es una serie finita, en contraste con el
caso de tiempo continuo, y definida completamente por los valores de la seal en un perodo, no hay
problemas de convergencia con la serie de Fourier discreta y no se presenta el fenmeno de Gibbs. En
otras palabras, el hecho de que cualquier secuencia peridica en tiempo discreto x[n] est
completamente especificada por un nmero finito de parmetros, a saber, los valores de la secuencia en
un perodo, es la razn por la cual no hay problemas de convergencia en general con la serie de Fourier
en tiempo discreto.

4.4

Propiedades de la Serie de Fourier Discreta

4.4.1 Periodicidad de los Coeficientes de Fourier

De las Ecs. (4.9) y (4.10) vemos que


ak + N0 = ak

(4.19)

la cual indica que los coeficientes de la serie de Fourier son peridicos con perodo fundamental N0. Es
decir, si consideramos ms de N0 valores secuenciales de k, los valores de ak se repetirn
peridicamente con perodo N0.
4.4.2 Dualidad

De la Ec. (4.19) vemos que los coeficientes de Fourier ak forman una secuencia peridica con perodo
fundamental N0. Entonces, escribiendo ak como a[k], la Ec. (4.17) puede escribirse de nuevo como
a[k ] =

n = N0

N0

x [ n ] e jk 0 n

(4.20)

Sea n = m en la Ec. (4.20). Entonces


a[k ] =

m = N0

N0

x [ m ] e jk 0 m

Haciendo ahora k = n y m = k en la expresin anterior, obtenemos


a[n] =

m = N0

1
N0

x [ k ] e jk 0 n

(4.21)

244

Comparando la Ec. (4.21) con la Ec. (4.16), vemos que los valores (1 N 0 ) x [ k ] son los coeficientes
de Fourier de a[n]. Si adoptamos la notacin

SFD

x[ n]

ak = a [ k ]

(4.22)

para denotar el par de series de Fourier discretas (SFD), entonces, por la Ec. (4.21), tenemos
1

SFD

a[n]

N0

x[ k ]

(4.23)

La Ec. (4.23) se conoce como la propiedad de dualidad de la serie de Fourier discreta. Dicho de otra
forma, puesto que los coeficientes de Fourier ak de una seal peridica x[n] son a su vez una secuencia
peridica, podemos expandir los coeficientes ak en una serie de Fourier. La propiedad de dualidad
descrita implica que los coeficientes de la serie de Fourier para la secuencia peridica ak son los valores
(1 N 0 ) x [ k ] .

4.4.3

Otras Propiedades

Cuando x[n] es real, entonces de la Ec. (4.11) [o la Ec. (4.17)] y la Ec. (4.19) se deduce que
a k = aN0 k = ak

(4.24)

donde, igual que antes, el asterisco (*) denota el conjugado complejo.

4.4.4

Secuencias Pares e Impares

Cuando x[n] es real, sea


x [ n ] = x p [ n ] + xi [ n ]

donde xp[n] y xi[n] son las partes par e impar de x[n], respectivamente y sea

SFD

x[ n]

ak

Entonces
xp [ n ]

Re{ ak }

xi [ n ]

Im{ ak }

SFD

SFD

(4.25)

Vemos entonces que si x[n] es real y par, entonces sus coeficientes de Fourier son reales, mientras
que si x[n] es real e impar, sus coeficientes son imaginarios.

245

Ejemplo 3. Sea x[n] una secuencia peridica real con perodo fundamental N0 y coeficientes de Fourier
ck = ak + jbk, donde ak y bk son reales.

(a) Demuestre que ak = ak y bk = bk.


(b) Demuestre que c N 0

es real si N0 es par.

(c) Demuestre que x[n] puede tambin expresarse como una serie de Fourier trigonomtrica de la
forma
x [ n ] = c0 + 2

( N 0 1) 2

( ak cos k 0 n bk sen k 0 n )

k =1

0 =

2
N0

si N0 es impar, o
x [ n ] = c0 + ( 1) cN0 2 + 2

( N 0 1 ) 2

k =1

( ak cos k 0 n bk sen k 0 n )

si N0 es par.

Solucin:
(a) Si x[n] es real, entonces de la Ec. (4.11) tenemos
c k =

N 0 1

N0

n =0

x[ n]e

jk 0 n

1
=
N0

N 0 1

x[ n]e

jk 0 n

n=0

= ck

As que
c k = a k + jb k = ( ak + jbk ) = ak jbk

y obtenemos
a k = ak

b k = bk

(b) Si N0 es par, entonces de la Ec. (4.11),


cN0

N 0 1

N0

n=0

x[ n]e

N 0 1

N0

n =0

( 1)

j ( N 0 2 )( 2 N 0 ) n

x[n]

N 0 1

N0

n=0

x[n]e

j n

real

(c) Escribamos de nuevo la Ec. (4.10) como


x[ n ] =

N 0 1

c e
k =0

jk 0 n

= c0 +

N 0 1

c e
k =1

jk 0 n

Si N0 es impar, entonces (N0 1) es par y podemos escribir a x[n] como

246

x [ n ] = c0 +

( N 0 1 ) 2

(c e

jk 0 n

k =1

+ cN 0 k e j ( N 0 k ) 0 n

Ahora, de la Ec. (4.24),


cN0 k = ck

y
e j ( N0 k ) 0 n = e jN0 0 n e jk 0 n = e j 2 n e jk 0 n = e jk 0 n

Por lo tanto,
( N 0 1) 2

(c e

x [ n ] = c0 +
= c0 +

k =1

( N 0 1) 2

2 Re ck e jk 0 n

k =1

= c0 + 2

+ ck e jk 0 n

= c0 + 2

jk 0 n

( N 0 1) 2

k =1

( N 0 1) 2

k =1

Re ( ak + jbk ) (cos k 0 n + j sen k 0 n )


( ak cos k 0 n bk sen k 0 n )

Si N0 es par, podemos escribir a x[n] como


x [ n ] = c0 +
= c0 +

N 0 1

c e
k =1

( N0 2 ) 2

k =1

jk 0 n

(c e

jk 0 n

+ cN0 k e

j ( N ) k ) 0 n

)+c

N0 2

e j ( N0

2 ) 0 n

Usando de nuevo la Ec. (4.24), se obtiene


y e j ( N0 k ) 0 n = e jk 0 n

cN0 k = ck

y
e j ( N0

2 ) 0 n

= e j ( N0

2 ) ( 2 N0 ) n

= e jn = ( 1) n

Entonces
x [ n ] = c0 + ( 1) cN0 2 +
n

( N0 2 ) 2

k =1

= c0 + ( 1) n cN0 2 + 2

2 Re ck e jk 0 n

( N0 2 ) 2

k =1

( ak cos k 0 n bk sen k 0 n )

247

4.5 Teorema de Parseval

Si x[n] est representada por la serie de Fourier discreta (4.16), entonces se puede demostrar que
1
N0

x[ n]

n = N0

ak

k = N0

(4.26)

La Ec. (4.26) se conoce como la identidad de Parseval (o el teorema de Parseval) para la serie de
Fourier discreta.

Demostracin: Sean x1[n] y x2[n] dos secuencias peridicas con igual perodo fundamental N0 y con
series de Fourier discretas dadas por

x1 [ n ] =

N 0 1

bk e jk0 n

N 0 1

c e

x2 [ n ] =

k =0

k =0

jk 0 n

0 =

2
N0

Entonces la secuencia x [ n ] = x1 [ n ] x2 [ n ] es peridica con el mismo perodo fundamental N0 (la


demostracin se deja para el lector) y se puede expresar como
x[ n] =

N 0 1

a e
k =0

jk 0 n

0 =

2
N0

donde a k est dada por


ak =

N 0 1

m=0

(4.27)

m k m

De esta relacin se obtiene que


ak =

N 0 1

N0

n=0

x1 [ n ] x2 [ n ] e jk 0 n =

N 0 1

m=0

m k m

Haciendo k = 0 en la expresin anterior, se obtiene


1

N 0 1

N0

n=0

x1 [ n ] x2 [ n ] =

N 0 1

N 0 1

m=0

k =0

bm c m =

la cual se conoce como la relacin de Parseval.


Ahora, sean
x[ n] =

N 0 1

a e
k =0

jk 0 n

b c
k

(4.28)

248

x *[ n ] =

N 0 1

b e

jk 0 n

1
=
N0

N 0 1

k =0

Entonces
bk =

1
N0

N 0 1

x *[ n ] e

jk 0 n

n =0

x[ n]e
n =0

jk 0 n

= a k

(4.29)

La Ec. (4.29) indica que si los coeficientes de Fourier de x[n] son los ak, entonces los coeficientes de
Fourier de x*[n] son los a k . Haciendo x1 [ n ] = x [ n ] y x2 [ n ] = x *[ n ] en la Ec. (4.28), se tiene que
bk = ak y ek = c k (o c k = ak ) y se obtiene
N 0 1

1
N0

x [ n ] x *[ n ] =

n=0

N 0 1

a a
k =0

(4.30)

o
1

N 0 1

N0

n=0

x[ n]

N 0 1

k =0

ak

que es la relacin buscada.

4.6 La Transformada de Fourier Discreta

A continuacin se considerar la representacin en el dominio de la frecuencia de seales de tiempo


discreto y de duracin finita que no son necesariamente peridicas.

4.6.1

Transformacin de la Serie de Fourier Discreta en la Transformada de Fourier

Sea x[n] una secuencia no peridica de duracin finita. Es decir, para algn entero positivo N1,
x[ n ] = 0

n > N1

Una secuencia as se muestra en la Fig. 4.9a. Sea xN [ n ] una secuencia peridica formada al repetir
0

x[n] con un perodo fundamental N0, como se muestra en la Fig. 4.9b. Si hacemos que N 0 ,
tenemos
lm xN0 [ n ] = x [ n ]

N 0

La serie de Fourier discreta para x N 0 [ n] est dada por

(4.31)

249

xN0 [ n ] =

ae

jk 0 n

0 =

xN0 [ n ] e jk 0 n

k = N0

(4.32)

N0

donde
ak =

1
N0

n = N0

(4.33)

x[n]

N1

N1

(a)
x N 0 [n]

...

...
N1

N0

N1

N0

(a)

Figura 4.4

Puesto que xN0 [ n ] = x [ n ] para n N1 y tambin como x[n] = 0 fuera de este intervalo, la Ec. (4.33)
puede escribirse de nuevo como

ak =

1
N0

N1

x [ n ] e jk 0 n =

n = N1

1
N0

x[ n]e

jk 0 n

(4.34)

n =

Definamos la envolvente X ( ) de N0ak como


X () =

x[ n ] e

jn

(4.35)

n =

Entonces, de la Ec. (4.34), los coeficientes de Fourier pueden expresarse como


ak =

1
N0

X ( k 0 )

(4.36)

250

donde 0 se usa para denotar el espacio muestral 2/N0. As pues, los coeficientes ak son
proporcionales a muestras igualmente espaciadas de esta funcin envolvente. Sustituyendo la Ec.
(4.36) en la Ec. (4.32), tenemos
xN 0 [ n ] =

k = N0

1
N0

X ( k 0 ) e jk 0 n

o
xN 0 [ n ] =

2 k = N0

X ( k 0 ) e jk 0 n

(4.37)

De la Ec. (4.35), X ( ) es peridica con perodo 2 y e jn tambin lo es. Por ello, el producto
X ( ) e jn tambin ser peridico con perodo 2. Como se muestra en la Fig. 4.5, cada trmino en la
sumatoria de la Ec. (4.37) representa el rea de un rectngulo de altura X ( k 0 ) e jk 0 n y anchura 0 .
Conforme N 0 , 0 = 2 N 0 se hace infinitesimal ( 0 0) y la Ec. (4.36) se convierte en una
integral. Tambin, puesto que la sumatoria en la Ec. (4.37) es sobre N0 intervalos consecutivos de
anchura 0 = 2 N 0 , el intervalo total de integracin siempre tendr una anchura de 2. As que
conforme N 0 y en vista de la Ec. (4.31), la Ec. (4.37) se convierte en
x[ n ] =

X () e
2

j n

(4.38)

Como X ( ) e jn es peridica con perodo 2, el intervalo de integracin en la Ec. (4.38) puede


tomarse como cualquier intervalo de longitud 2.

X () e jn

X (k 0 ) e jk0 n

0
2

k0

Figura 4.5

4.6.2

Par de Transformadas de Fourier

La funcin X ( ) definida por la Ec. (4.35) se denomina la transformada de Fourier de x[n] y la Ec.
(4.38) define la transformada de Fourier inversa de X ( ) . Especficamente, ellas se denotan por

251

x[ n]e

X ( ) = F { x [ n ]} =

jn

(4.39)

n =

x [ n ] = F 1 { X ( )} =

X () e
2

jn

(4.40)

y decimos que x[n] y X ( ) forman un par de transformadas de Fourier denotadas por

X ()

x[ n ]

Las Ecs. (4.39) y (4.40) son las contrapartes discretas de las ecuaciones para las transformadas en
tiempo continuo. La derivacin de estas ecuaciones indica cmo una secuencia aperidica puede verse
como una combinacin lineal de exponenciales complejas. En particular, la ecuacin de sntesis es en
efecto una representacin de x[n] como una combinacin lineal de exponenciales complejas
infinitesimalmente cercanas en frecuencia y con amplitudes X()(d/2) y proporciona la informacin
de sobre cmo x[n] est compuesta de exponenciales complejas en frecuencias diferentes.

Ejemplo 4. Determine la transformada de Fourier del pulso rectangular (Fig. 4.6)

x[n] = u[n] + u[n N ]


De la Ec. (4.38), la transformada de Fourier de x[n] est dada por
X () =

x[ n]e

jn

n =

1 e jn
1 e

n=0

= e j( N 1 ) 2

N 1

= (1) e jn

e jN

e 2

(e
(e

j N 2
j 2

e j N 2

e j 2

sen ( N 2)
sen ( 2)
x[n]

...
0 1 2

Figura 4.6

N1

252

4.6.3

Espectros de Fourier

La transformada de Fourier X ( ) de x[n] es, en general, compleja y puede expresarse como


X ( ) = X ( ) e j ( )

(4.41)

Igual que en tiempo continuo, la transformada de Fourier X ( ) de una secuencia no peridica x[n] es
la especificacin en el dominio de la frecuencia de x[n] y se conoce como el espectro (o espectro de
Fourier) de x[n]. La cantidad X ( ) es el espectro de magnitud de x[n] y ( ) es el espectro de
fase de x[n]. Adems, si x[n] es real, el espectro de amplitud X ( ) es una funcin par y el espectro
de fase ( ) es una funcin impar de .

Ejemplo 5. Considere la secuencia


x [ n ] = n u [ n ]

<1

Para este ejemplo,

X ( ) = n e j n =
n=0

1
1 e j

El espectro de magnitud est dado por


X () =

1
1 + 2 2 cos

y el de fase por
( ) = tan 1

4.6.4

sen

1 cos

Convergencia de X()

Igual que en el caso de tiempo continuo, la condicin suficiente para la convergencia de X ( ) es


que x[n] sea absolutamente sumable, es decir,

x[ n] <

(4.42)

n =

La demostracin de esto se deja para el lector. Por lo tanto, vemos que la transformada de Fourier en
tiempo discreto posee muchas semejanzas con el caso de tiempo continuo. Las diferencias principales
entre los dos casos son la periodicidad de la transformada de tiempo discreto X ( ) y el intervalo
finito de integracin en la ecuacin de sntesis. Ambas provienen de un hecho que ya se ha sealado:

253

Exponenciales complejas en tiempo discreto que difieren en frecuencia por un mltiplo de 2 son
idnticas.

4.7 Propiedades de la Transformada de Fourier

Hay muchas diferencias y semejanzas con el caso continuo. Estas propiedades son tiles en el anlisis
de seales y sistemas y en la simplificacin del trabajo con las transformadas directa e inversa

4.7.1

Periodicidad

Ya se vio que la transformada de Fourier discreta es siempre peridica en con perodo 2, de modo
que
X ( + 2) = X ()

(4.43)

Como una consecuencia de la Ec. (4.43), en el caso de tiempo discreto tenemos que considerar valores
de (radiantes) solamente en el intervalo 0 < 2 o < , mientras que en el caso continuo
tenemos que considerar valores de (radianes/segundo) en todo el intervalo < < .

4.7.2

Linealidad

Sean x1 [ n ] y x2 [ n ] dos secuencias con transformadas de Fourier X1() y X2(), respectivamente.


Entonces
a1 x1 [ n ] + a2 x2 [ n ]

a1 X 1 ( ) + a2 X 2 ( )

(4.44)

para cualesquiera constantes a1 y a2.

4.7.3

Desplazamiento o Corrimiento en el Tiempo

Por sustitucin directa en las ecuaciones de definicin de la transformada de Fourier, se obtiene que
x [ n n0 ]

e jn0 X ( )

La demostracin de la Ec. (4.45) es la siguiente: Por definicin, Ec. (4.39),

F { x [ n n0 ]} =

x[ n n

n =

Mediante el cambio de variable m = n n0, obtenemos

] e j n

(4.45)

254

F { x [ n n0 ]} =

x[ m]e

j ( m + n0 )

m =

= e j n0

x[m]e

j m

= e j n0 X ( )

m =

Por tanto,
x[ n]

e j n0 X ( )

Ejemplo 6. Determine (a) la transformada de Fourier X() de la secuencia en forma de pulso


rectangular mostrada en la Fig. 4.7a; (b) Grafique X() para N1 = 4 y N1 = 8.

(a) De la Fig. 4.7 vemos que


x [ n ] = x1 [ n + N1 ]

donde x1[n] se muestra en la Fig. 4.7b. Haciendo N = 2N1 + 1 en el resultado del Ejemplo 4,
tenemos
X1 ( ) = e

j N1

sen ( N1 + 12 )
sen ( 2)

x1[n

x[n]
1

...
N1

...

...
0 1 2

N1

0 1 2

(a)

(b)

Figura 4.7

Ahora, por la propiedad de desplazamiento en el tiempo, tenemos que


X () = e

j N1

sen ( N1 + 12 )
X1 ( ) =
sen ( 2)

(b) Haciendo N1 = 4 en la ecuacin anterior, se obtiene


X () =

sen (4.5 )
sen (0.5 )

la cual se grafica en la Fig. 4.8a. En forma similar, para N1 = 8 se obtiene

2N1

255

X () =

sen (8.5 )
sen (0.5 )

la cual se grafica en la Fig. 4.8b.


X()

X()

17

(a)

(b)

Figura 4.8

4.7.4

Desplazamiento en Frecuencia

Esta propiedad, originada al multiplicar la exponencial e j0 n por x[n], produce el par de transformadas
dado por

e j 0 n x [ n ]

X ( 0 )

La demostracin procede en la forma siguiente. Por la Ec. (4.39)

F { e j n x [ n ]} =
0

j 0 n

x [ n ] e j n

n =

x[ n]e

j ( 0 ) n

n =

= X ( 0 )

de donde
e j 0 n x [ n ]

X ( 0 )

Ejemplo 7. Determinar la transformada inversa de


X ( ) = 2 ( 0 )

, 0

De la ecuacin de definicin de la transformada de Fourier inversa, se tiene que


x[ n ] =

2 (
2

) e j 0 n d = e j 0 n

(4.46)

256

y de aqu se obtiene el par de transformadas


e j0 n 2 ( 0 )

Haciendo 0 = 0 en la relacin anterior, se obtiene otro par de transformadas:


x[ n ] = 1 2 ( ) ,

4.7.5

Conjugacin

Esta propiedad relaciona el conjugado de la funcin con su transformada y nos dice que
x *[ n ]

X *( )

(4.47)

De la Ec. (4.39)

F { x *[ n ]} =

x *[ n ] e

j n

n =

= x [ n ] e j n
n =

= x [ n ] e j ( n ) = X *( )
n =

y, por tanto,
x *[ n ]

X *( )

Debido a la naturaleza discreta del ndice del tiempo para seales de tiempo discreto, los escalamientos
en tiempo y en frecuencia resultan en tiempos discretos que toman una forma algo diferente de sus
contrapartes en tiempo continuo. Sea x[n] una seal con espectro X() y consideremos las dos
propiedades siguientes.
4.7.6

Inversin en el Tiempo

x[ n]

X ( )

(4.48)

Esta demostracin se deja para el lector. Aun cuando la Ec. (4.48) es anloga al caso de tiempo
continuo, surgen diferencias cuanto tratamos de escalar tiempo y frecuencia en vez de simplemente
invertir el eje del tiempo, como se ver a continuacin.
4.7.7

Escalamiento en el Tiempo

La propiedad de escalamiento de la transformada de Fourier en tiempo continuo se expres como


x ( at )


X
a
a
1

(4.49)

257

Sin embargo, en el caso de tiempo discreto, x[an] no es una secuencia si a no es un entero. Por otra
parte, si a es un entero, digamos 2, x[2n] consiste solamente de las muestras pares de x[n]. As que el
escalamiento en el tiempo en tiempo discreto toma una forma algo diferente de la Ec. (4.49).
Sea m un entero positivo y defina la secuencia
x [ n m ] = x [ k ] si n = km , k entero
x( m ) [ n ] =
si n km
0

(4.50)

Entonces tenemos
X ( m )

x( m ) [ n ]

(4.51)

La Ec. (4.51) es la contraparte en tiempo discreto de la Ec. (4.49). Expresa de nuevo la relacin
inversa entre el tiempo y la frecuencia. Es decir, conforme la seal se extiende en el tiempo ( m > 1) , su
transformada de Fourier se comprime. Observe que X(m) es peridica con perodo 2/m, ya que X()
es peridica con perodo 2.
De la Ec. (4.39)

F { x( m ) [ n ]} =

n =

(m)

[ n ] e j n

Haciendo el cambio de variable n = km en el lado derecho de esta ecuacin, obtenemos

F { x( m ) [ n ]} =

k =

j km
= x [ k ] e j ( m ) k = X ( m )
( m ) [ km ] e

De aqu que

x( m ) [ n ]
4.7.8

X ( m )

Dualidad

La propiedad de dualidad de una transformada de Fourier de tiempo continuo se expres como


X (t )

2 x ( )

(4.52)

En el caso discreto no hay una contraparte para esta propiedad. Sin embargo, hay una dualidad entre la
transformada de Fourier discreta y la serie de Fourier de tiempo continuo. Sea
x[ n ]

X ()

De las Ecs. (4.39) y (4.43),


X () =

x[ k ]e

j 2n

(4.53)

n =

X ( + 2) = X ()

(4.54)

258

Puesto que es una variable continua, haciendo = t y n = k en la Ec. (4.53), se obtiene

X (t ) =

x[ k ]e

jkt

(4.55)

k =

Como X(t) es peridica con perodo T0 = 2 y la frecuencia fundamental 0 = 2 T0 = 1 , la Ec. (4.55)


indica que los coeficientes de la serie de Fourier de X(t) sern iguales a x[k]. Esta relacin dual se
denota por
SF

ak = x [ k ]

X (t )

(4.56)

y donde SF denota la serie de Fourier y las ak son los coeficientes de Fourier.


4.7.9

Diferenciacin en Frecuencia

De nuevo, suponga que X() es la transformada de x[n]. Entonces


n x[ n]

dX ( )

(4.57)

De la definicin (4.39) sabemos que

x[ n]e

X () =

j n

n =

Diferenciando ambos lados de la expresin anterior con respecto a e intercambiando el orden de la


diferenciacin y la sumatoria, se obtiene
dX ( )
d

d
d
j n
x
[
n
]
e
=
x[ n]
( e j n )

d n =
d

n =

=j

n x[ n]e

j n

n =

Multiplicando ambos lados por j, vemos que

F { n x [ n ]} =

n x[ n]e

j n

= j

n =

y, por tanto,
n x[ n]

4.7.10 Diferencias

Para una sola diferencia, se tiene que

dX ( )
d

dX [ n ]
d

259

x [ n ] x [ n 1]

(1 e j ) X ( )

(4.58)

La secuencia x [ n ] x[ n 1] ya se defini como la primera diferencia. La Ec. (4.58) se obtiene


fcilmente a partir de la propiedad de linealidad, Ec. (4.44), y la propiedad de desplazamiento en el
tiempo, Ec. (4.45).

Ejemplo 8. Demuestre que

F {u [ n ]} = ( ) +

1 e j

Sea

F {u [ n ]} = X ( )
Observe ahora que

[ n ] = u [ n ] u [ n 1]
y tomando la transformada de Fourier de ambos lados de esta ecuacin, se obtiene
1 = ( 1 e j ) X ( )

Ahora bien, ( 1 e j ) = 0 para = 0 y, por tanto, X() debe ser de la forma


X () = A() +

1 e j

donde A es una constante. Para determinar A, procedemos en la forma siguiente: La componente par de
u[n] est dada por
u p [ n ] = 12 + 12 [ n ]
As que la componente impar de u[n] est dada por
ui [ n ] = u [ n ] u p [ n ] = u [ n ] 12 12 [ n ]
y

F {ui [ n ]} = A ( ) +

1
1 e

()

1
2

Pero la transformada de Fourier de una secuencia real impar debe ser puramente imaginaria; as que
debemos tener que A = y entonces

F {u [ n ]} = ( ) +

1
1 e j

X ()

260

4.7.11 Acumulacin

Esta propiedad dice que


n

x[ k ]

X (0) ( ) +

k =

1
1 e j

X ()

(4.59)

Observe que la acumulacin es la contraparte en tiempo discreto de la integracin. El trmino en


impulso en el lado derecho de la Ec. (4.59) refleja el valor promedio o CD que puede resultar de la
acumulacin.
La demostracin de esta propiedad se deja para el lector (use el resultado del Ejemplo 8).

Ejemplo 9. Determinar la transformada de Fourier de u[n] usando la propiedad de acumulacin, Ec.


(4.59).

Ya sabemos que
u[n] =

[ k ]

k =

y tambin que
[ n ]

Haciendo x[k] = [k] en la Ec. (4.59), se obtiene


x[ n ] = [ n ]

X () = 1

() +

y X (0) = 1

y, por tanto,
u[n] =

[ k ]

k =

1
1 e j

4.7.12 Convolucin

Para dos seales discretas x1[n] y x2[n], su convolucin y la transformada de sta cumplen con la
relacin
x1 [ n ] x2 [ n ]

X1 ( ) X 2 ( )

(4.60)

Esta propiedad juega un papel muy importante en el estudio de los sistemas LIT de tiempo discreto.
Por las definiciones (4.27) y (4.39), tenemos que

F { x1 [ n ] x2 [ n ]} =

x [ k ] x

n =

k =

[ n k ] e j n

261

Intercambiando el orden de las sumatorias, obtenemos

F { x1 [ n ] x2 [ n ]} =

x [ k ] x
1

k =

n =

[ n k ] e j n

Por la propiedad de desplazamiento en el tiempo, Ec. (4.45):

n =

[ n k ] e j n = e j n X 2 ( )

y as tenemos que

F { x1 [ n ] x2 [ n ]} =

x [k ]e

k =

j k

X 2 ()


= x1 [ k ] e j k
k =

X 2 ( ) = X1 ( ) X 2 ( )

y se verifica la relacin (4.60).

Ejemplo 10. Determine la transformada de Fourier inversa x[n] de


X () =

( 1 a e )
j

a <1

usando el teorema de convolucin.


La transformada inversa de
1

a <1

( 1 a e )
j

es la funcin a n u [ n ] (Ejemplo 5). Ahora bien,


X () =

(1 a e )
j

1
1

=
j
j
1 a e
1 a e

Entonces, usando el teorema de convolucin, Ec. (4.60), se obtiene


x[ n ] = a n u [ n ] a n u [ n ] =

a u[k ]a
k

k =
n

= a n 1 = ( n + 1) a n u [ n ]
k =0

Por consiguiente, tenemos el par de transformadas

nk

u[n k ]

262

( n + 1) a n u [ n ]

a <1

( 1 a e )
j

4.7.13 Propiedad de Multiplicacin o Modulacin

En el Cap. 3 se introdujo la propiedad de modulacin para seales de tiempo continuo y se indicaron


algunas de sus aplicaciones. Existe una propiedad anloga para seales de tiempo discreto y juega un
papel similar en aplicaciones. Esta propiedad es
1

x1 [ n ] x2 [ n ]

X1 ( ) X 2 ( )

(4.61)

donde el smbolo denota la convolucin peridica definida por


X1 ( ) X 2 ( ) =

() X 2 ( ) d

(4.62)

La propiedad de multiplicacin (4.61) es la propiedad dual de la Ec. (4.60) y su demostracin procede


en la forma siguiente:
Sea x [ n ] = x1 [ n ] x2 [ n ] . Entonces, por la definicin (4.39),
X () =

x [n] x

n =

[n]

Por la Ec. (4.39),


x1 [ n ] =

X
2

( ) e j n d

Entonces

X () =

n =

X 1 ( ) e j n d x2 [ n ] e j n

2 2

Intercambiando el orden de la sumatoria y la integracin obtenemos


X () =
=

X1 ( )
x2 [ n ] e j ( ) n d
2 2
n =

X
2
2

y as queda demostrada la propiedad.

() X 2 ( ) d =

1
2

X1 ( ) X 2 ( )

263

4.7.14 Propiedades Adicionales

Si x[n] es real, sea


x [ n ] = x p [ n ] + xi [ n ]
donde xp[n] y xi[n] son las componentes par e impar de x[n], respectivamente. Sea
x[ n]

X ( ) = A ( ) + jB ( ) = X ( ) e j ( )

(4.63)

Entonces
X ( ) = X *( )

(4.64)

xp [ n ]

Re{ X ( )} = A ( )

xi [ n ]

j Im{ X ( )} = jB ( )

(4.65)

La Ec. (4.64) es la condicin necesaria y suficiente para que x[n] sea real. De las Ecs. (4.64) y (4.63)
obtenemos
A ( ) = A ( ),

B ( ) = B ( )

X ( ) = X ( ) ,

( ) = ( )

(4.66)

De las Ecs. (4.65) y (4.66) se observa que si x[n] es real y par, entonces X() es real y par, mientras
que si x[n] es real e impar, X() es imaginaria e impar.

4.7.15 Relacin de Parseval

Si x[n] y X[] forman un par de transformadas de Fourier, entonces

x [n] x

n =

[n] =

n =

x[ n]

2 2
1

X1 ( ) X 2 ( ) d
X ()

(4.67)

(4.68)

La Ec. (4.68) se conoce como la identidad de Parseval (o el teorema de Parseval) para la transformada
de Fourier de tiempo discreto. En analoga con el caso de tiempo continuo, el lado izquierdo de la Ec.
2
(4.68) se conoce como la energa en x[n] y X ( ) como el espectro de la densidad de energa.

Como la energa en una secuencia peridica es infinita, la Ec. (4.68) no es de utilidad en ese caso. Para
seales peridicas se puede derivar una variante de la identidad de Parseval que relaciona la energa en
un perodo de la secuencia con la energa en un perodo de los coeficientes de la serie de Fourier, ella
es

264

1
N0

x[ n ]

n = N0

k = N0

ak

(4.69)

4.8 La Respuesta de Frecuencia de Sistemas LIT Discretos

Como ya se mostr en el Cap. 2, la salida y[n] de un sistema LIT discreto es igual a la convolucin de
la entrada x[n] con la respuesta al impulso h[n], suponiendo que las transformadas de Fourier de x[n],
y[n] y h[n] existen; es decir,

y [ n ] = x [ n ] h[ n ]

(4.70)

Entonces la propiedad de convolucin implica que

Y () = X () H ()

(4.71)

donde X(), Y() y H() son las transformadas de Fourier de x[n], y[n] y h[n], respectivamente. De
esta relacin obtenemos
H () =

Y ()

(4.72)

X ()

De la relacin (4.71), observe que


Y () = X () H ()

(4.73)

Y ( ) = X ( ) + H ( )

(4.74)

Igual que en el caso de tiempo continuo, la funcin H ( ) se conoce como la respuesta de magnitud
del sistema. Debido a la forma multiplicativa de la Ec. (4.73), a la magnitud de la respuesta de
frecuencia de un sistema LIT algunas veces se le refiere como la ganancia del sistema. Las relaciones
dadas por las Ecs. (4.71) y (4.72) se muestran en la Fig. 4.9.

[n]
x[n]

X[]

h[n]
H()

h[n]
y[n] = x[n]h[n]

Y[] = X()H()

Figura 4.9

265

Sea
H ( ) = H ( ) e jH ( )

(4.75)

La funcin H() se conoce como la respuesta de frecuencia del sistema, H ( ) como la respuesta
de magnitud del sistema y H ( ) como la respuesta de fase del sistema.

4.8.1

Sistemas LIT Caracterizados por Ecuaciones de Diferencias

Muchos sistemas LIT de tiempo discreto y de inters prctico son descritos por ecuaciones de
diferencias con coeficientes constantes de la forma
N

k =0

k =0

ak y [ n k ] = bk x [ n k ]

(4.76)

donde M N. En esta seccin usamos las propiedades de la transformada de Fourier en tiempo discreto
para obtener una expresin para la respuesta de frecuencia del sistema LIT descrito por la Ec. (4.76).
Tomando la transformada de Fourier de ambos lados de la Ec. (4.76) y usando las propiedades de
linealidad y de desplazamiento en el tiempo, obtenemos
N

k =0

k =0

ak e jk Y ( ) = bk e jk X ( )
o, en forma equivalente,
M

H () =

Y ()
X ()

b e
k =0
N

jk

a e
k =0

(4.77)
jk

Observe que H() es un cociente de polinomios en la variable e j . Los coeficientes del polinomio del
numerador son los mismos coeficientes que aparecen en el lado derecho de la Ec. (4.76) y los
coeficientes del denominador son los mismos coeficientes que aparecen en el lado izquierdo de la Ec.
(4.76). Esto implica que la respuesta de frecuencia de un sistema LIT especificado por la Ec. (4.77)
puede escribirse por inspeccin.

Ejemplo 11. Considere un sistema LIT inicialmente en reposo descrito por la ecuacin de diferencias
y [ n ] 34 y [ n 1] + 18 y [ n 2] = x [ n ]

De la Ec. (4.77), la respuesta de frecuencia es


H () =

Y ()
X ()

1
1 34 e j + 81 e j 2

266

Para determinar la respuesta al impulso, debemos obtener la transformada inversa de H(), por lo
que necesitamos expandir esta ltima expresin en fracciones parciales. Por lo tanto,
H () =

1
1 34 e

2
1 e
1
2

+ 81 e

j 2

( 1

1
2

e j ) ( 1 14 e j )

1
1 e j
1
4

La transformada inversa puede obtenerse por inspeccin y el resultado es


n
n
h [ n ] = 2 ( 12 ) ( 14 ) u [ n ]

4.8.2

Naturaleza Peridica de la Respuesta de Frecuencia

De la Ec. (4.43) sabemos que

H () = H ( + 2)
As que, a diferencia de la respuesta de frecuencia de los sistemas de tiempo continuo, la de todos los
sistemas LIT de tiempo discreto es peridica con perodo 2. Por consiguiente, solamente necesitamos
observar la respuesta de un sistema de tiempo discreto en la banda de frecuencias 0 < 2 o
< .

4.9

Respuesta del Sistema a Muestras de Sinusoides de Tiempo Continuo

4.9.1

Respuestas del Sistema

Denote por yc[n], ys[n] y y[n] las respuestas del sistema a las excitaciones cos n , sen n y e jn ,
respectivamente (Fig. 4.10). Puesto que e jn = cos n + j sen n , y la respuesta de un sistema LIT con
respuesta al impulso h[n] a una excitacin exponencial de la forma zn es
y[n] = H ( z ) zn ,

H (z) =

h[ n ] z

n =

se tiene que
y [ n ] = yc [ n ] + jys [ n ] = H ( ) e jn

yc [ n ] = Re{ y [ n ]} = Re { H ( ) e jn }
ys [ n ] = Im{ y [ n ]} = Im { H ( ) e jn }

(4.78)

267

ejn
cos n

H()

sen n

y[n] = H()ejn
y c [n] = Re [ H () e jn ]
y s [n] = Im [ H () e jn ]

Figura 4.10

Cuando una sinusoide cos n se obtiene por muestreo de una sinusoide de tiempo continuo cos t ,
con un intervalo de muestreo igual a Ts, es decir
cos n = cos t

t nTs

= cos Ts n

se pueden aplicar todos los resultados obtenidos en esta seccin si sustituimos Ts por :
= Ts
Para una sinusoide de tiempo continuo cos t existe una forma de onda nica para todo valor de en
el intervalo de 0 a . Un aumento en resulta en una sinusoide de frecuencia siempre creciente. En
contraste, la sinusoide de tiempo discreto cos n tiene una forma de onda nica solamente para
valores de en el intervalo de 0 a 2 porque

cos[ ( + 2 m ) n ] = cos ( n + 2 mn ) = cos n

m = entero

Este intervalo tambin est restringido por el hecho de que


cos ( ) n = cos n cos n m sen n sen n
= ( 1) n cos n

y, por lo tanto,
cos ( + ) n = cos ( ) n

(4.79)

La Ec. (4.79) muestra que una sinusoide de frecuencia( + ) tiene la misma forma de onda que una
con frecuencia( ). Por ello, una sinusoide con cualquier valor de fuera del intervalo 0 a es
idntica a una sinusoide con en el intervalo de 0 a . Concluimos entonces que toda sinusoide de
tiempo discreto con una frecuencia en la banda 0 < tiene una forma de onda nica y slo
necesitamos observar la respuesta de frecuencia de un sistema en la banda de frecuencias 0 < .

4.10

La Transformada de Fourier Discreta

Una de las razones para el crecimiento en el uso de los mtodos de tiempo discreto para el anlisis y
sntesis de seales y sistemas fue el desarrollo de herramientas muy eficientes para realizar el anlisis
de Fourier de secuencias de tiempo discreto. En el corazn de estos mtodos est una tcnica muy
cercana a las ideas que hemos presentado en las secciones anteriores y que est idealmente adaptada

268

para el uso en una computadora digital o para su implementacin en hardware digital. Esta tcnica es la
transformada de Fourier discreta (TFD) para seales de duracin finita y aun cuando puede
considerarse como una extensin lgica de la transformada de Fourier ya estudiada, no debe ser
confundida con la transformada de Fourier de tiempo discreto. Especficamente, en muchas
aplicaciones solamente se pueden verificar los valores de una funcin en un nmero finito de puntos
igualmente espaciados. Por ejemplo, un conjunto de esos valores podra ser una sucesin obtenida
mediante el muestreo instantneo de una seal continua en un conjunto de puntos con igual separacin
entre ellos. Entonces debemos hallar una serie de Fourier finita cuya suma en cada punto del dominio
de la funcin sea igual al valor correspondiente de la funcin en ese punto.

4.10.1 Definicin

Sea x[n] una secuencia finita de longitud N, es decir,


x [ n ] = 0 fuera de la banda 0 n N 1

(4.80)

La TFD de x[n], denotada como X[k], se define por


N 1

X [ k ] = x [ n ]WNkn
n=0

k = 0, 1, K , N 1

(4.81)

donde WN es la N-sima raz de la unidad dada por


WN = e j ( 2 N )

(4.82)

La TFD inversa (TFDI) est dada por


x[ n] =

N 1

X [ k ]W
N

kn
N

k =0

n = 0, 1, K , N 1

(4.83)

El par de TFD se denota por


x[ n]

X [k ]

(4.84)

La TFD tiene las siguientes caractersticas importantes:


1. Existe una correspondencia uno a uno entre x[n] y X[k].
2. Para su clculo est disponible un algoritmo extremadamente rpido llamado la transformada
de Fourier rpida (FFT por sus iniciales en ingls).
3. La TFD est ntimamente relacionada con la serie y la transformada de Fourier de tiempo
discreto y, por ello, exhibe algunas de sus propiedades importantes.
4. La TFD es la representacin de Fourier apropiada para realizacin en una computadora
digital ya que es discreta y de longitud finita tanto en el dominio del tiempo como en el de la
frecuencia.

Observe que la seleccin de N en la Ec. (4.81) no es fija, siempre que ella se escoja mayor que la
duracin de x[n]. Si x[n] tiene longitud N1 < N, deseamos suponer que x[n] tiene longitud N mediante la

269

simple adicin de (N N1) muestras con un valor de 0. Esta adicin de muestras de relleno se conoce
como relleno de ceros. Entonces la x[n] resultante se llama una secuencia o sucesin de N puntos, y a
la X[k] definida en la Ec. (4.81) se le refiere como una TFD de N puntos. Mediante una seleccin
juiciosa de N, tal como tomarla como una potencia de 2, se puede obtener una buena eficiencia
computacional.

Ejemplo 12. Determine la TFD de N puntos de las secuencias siguientes:

(a) x[n] = [n]


(b) x[n] = u[n] u[n N]
(a) De la definicin (4.81), tenemos
N 1

X [ k ] = [ n ]WNkn = 1
n=0

k = 0, 1, K , N 1

La Fig. 4.11 muestra x[n] y su TFD de N puntos.


(b) De nuevo, de la definicin (4.81) y usando la ecuacin
1 N

n = 1

n=0
N

N 1

=1

Obtenemos
N 1

1 WNkN

n =0

1 WNk

X [ k ] = WNkn =

=0

k0

puesto que WNkN = e j ( 2 N ) kN = e jk 2 = 1 y


N 1

N 1

n=0

n=0

X [0] = WN0 = 1 = N

X[k]

x[n]
1

N1

Figura 4.11

N n1

270

La Fig. 4.12 muestra x[n] y su TFD de N puntos X[k].

x[n]

X[k]

N1

N1

Figura 4.12

4.10.2 Relacin entre la TFD y la Serie de Fourier de Tiempo Discreto

Comparando las Ecs. (4.83) y (4.81) con las Ecs. (4.10) y (4.11), vemos que la X[k] de una secuencia
finita x[n] puede ser interpretada como los coeficientes ck en la representacin en serie de Fourier
discreta de su extensin peridica multiplicada por el perodo N0 y N0 = N. Es decir,
X [ k ] = N ck

(4.85)

En realidad, las dos pueden hacerse idnticas si incluimos el factor 1/N en la TFD y no en la
transformada de Fourier de tiempo discreto.

4.10.3 Relacin entre la TFD y la Transformada de Fourier

Por la definicin (4.38), la transformada de Fourier de la secuencia x[n] definida por la Ec. (4.80)
puede ser expresada como
N 1

X ( ) = x [ n ] e jn

(4.86)

n =0

Comparando la Ec. (4.86) con la Ec. (4.81), vemos que


X [k ] = X ()

= k 2 N

k 2
=X

(4.87)

As pues, la TFD X[k] corresponde a la transformada de Fourier X() muestreada uniformemente en las
frecuencias = k2/N para k entero.

271

4.10.4 Propiedades de la TFD

Debido a la relacin expresada por la Ec. (4.87) entre la TFD y la transformada de Fourier, es de
esperar que sus propiedades sean bastante semejantes, excepto que la TFD X[k] es una funcin de una
variable discreta mientras que la transformada de Fourier X() es una funcin de una variable continua.
Observe que las variables de la TFD n y k deben estar restringidas al intervalo 0 n, k < N, por lo que
los desplazamientos de la TFD x[n n0] o X[k k0] implican x [ n n0 ]md N o X[k k0]md N, donde la
notacin [m]md N significa que
0 [ m ]md N = m + i N

(4.88)

0 [ m ]md N < N

(4.89)

para algn entero i tal que

Por ejemplo, si x[n] = [n 3], entonces


x [ n 4]md 6 = [ n 7 ]md 6 = [ n 7 + 6]md 6 = [ n 1]

El desplazamiento en la TFD tambin se conoce como un desplazamiento circular.


Puesto que la TFD es evaluada en frecuencias en la banda [0, 2], las cuales estn separadas por
2/N, al considerar la TFD de dos seales simultneamente, las frecuencias correspondientes a la TFD
deben ser las mismas para que cualquier operacin tenga significado. Esto significa que la longitud de
las secuencias consideradas debe ser la misma. Si ste no es el caso, se acostumbra aumentar las
seales mediante un nmero apropiado de ceros, de modo que todas tengan la misma longitud. Algunas
propiedades bsicas de la TFD son las siguientes:
1.

Linealidad: Sean X1[k] y X2[k] las TFD de dos secuencias x1[n] y x1[n]. Entonces

a1 x1 [ n ] + a2 x2 [ n ]

a1 X 1 [ k ] + a2 X 2 [ k ]

(4.90)

para cualesquiera constantes a1 y a2.


2.

Desplazamiento en Tiempo: Para cualquier entero real n0,

x [ n n0 ]md N

WNkn0 X [ k ]

WN = e j ( 2 N )

(4.91)

donde el desplazamiento es un desplazamiento circular.


3.

Desplazamiento en Frecuencia:

WN kn0 x [ n ]
4.

X [ k k0 ]md N

(4.92)

Conjugacin:

x *[ n ]

X *[ k ]md N

donde el asterisco, igual que antes, denota el conjugado complejo.

(4.93)

272

5.

Inversin del Tiempo:

x[ n ]md N
6.

(4.94)

N x [ k ]md N

(4.95)

Dualidad:

X [n]
7.

X [ k ]md N

Convolucin Circular: En nuestras discusiones anteriores de diferentes transformadas vimos que


la transformada inversa del producto de dos transformadas corresponda a una convolucin de las
funciones del tiempo correspondiente. Con esto en mente, tenemos entonces que

x1 [ n ] x2 [ n ]

X1 [ k ] X 2 [ k ]

(4.96)

donde
N 1

x1 [ n ] x2 [ n ] = x1 [ i ] x2 [ n i ] md N

(4.97)

i =0

La suma de convolucin en la Ec. (4.97) se conoce como la convolucin circular de x1[n] y x2[n]. La
demostracin de esta propiedad se deja como un ejercicio.
Ejemplo 13. Considere dos secuencias x[n] y h[n] de longitud 4 dadas por


x [ n ] = cos n
2
1
h[ n ] =
2

n = 0, 1, 2, 3

n = 0, 1, 2, 3

(a) Calcule y [ n ] = x [ n ] h [ n ] usando la convolucin circular.


(b) Calcule y[n] usando la TFD.
(a) Las secuencias x[n] y h[n] pueden expresarse como
x [ n ] = {1, 0, - 1, 0} y h [ n ] = { 1, 12 , 14 ,

1
8

Por la Ec. (4.97),


3

y [ n ] = x [ n ] h [ n ] = x [ i ] h [ n i ] md
i =0

Las secuencias x[i] y h[n i]md 4 para n = 0, 1, 2, 3 se grafican en la Fig. 4.13a. Entonces, por la Ec.
(4.97) obtenemos
n=0

y [0] = 1(1) + ( 1) ( 14 ) =

3
4

273

y [1] = 1( 12 ) + ( 1) ( 18 ) =

n =1

3
8

y [ 2] = 1( 14 ) + ( 1) (1) = 34

n=2

y [3] = 1( 18 ) + ( 1) ( 12 ) = 83

n=3

y
y [ n ] = { 34 , 83 , 43 , 83 }

x[i]

h[i]

1
2

3 1

h[n i]md 4

h[n i]md 4

1
n=0

n=1

0 1 2 3

0 1 2 3

h[n i]md 4

h[n i]md 4

1
n=2

n =3
1
2

0 1 2 3

0 1 2 3

i
(a)
y[n]
1
1
2

12

2 3
0

(b)

Figura 4.13

274

(b) Por la Ec. (4.81)


3

X [ k ] = x [ n ]W4kn = 1 W42 k

k = 0, 1, 2, 3

n=0

H [ k ] = h [ n ]W4kn = 1 + 12 W4k + 14 W42 k + 81 W43 k

k = 0, 1, 2, 3

n =0

Entonces, por la Ec. (4.96), la TFD de y[n] es


Y [ k ] = X [ k ] H [ k ] = ( 1 W42 k )( 1 + 12 W4k + 14 W42 k + 18 W43 k )
= 1 + 12 W4k 43 W42 k 83 W43 k 14 W44 k 18 W45 k
Como W44 k = (W44 ) = 1k y W45 k = W4( 4+1 ) k = W4k , se tiene
k

3
Y [ k ] = 34 + 83 W4k 34 W42 k W43 k
8

k = 0, 1, 2, 3

y por la definicin de la TFD, Ec. (4.81), obtenemos


y [ n ] = { 34 , 83 , 43 , 83 }

Ejemplo 14, Demuestre que si x[n] es real, entonces su TFD X[k] satisface la relacin

X[N k] = X*[k]
De la Ec. (4.81)
N 1

N 1

n =0

n=0

X [ N k ] = x [ n ]WN( N k ) n = x [ n ] e j ( 2 N ) ( N k ) n

Ahora bien,
e j ( 2 N )( N k ) n = e j 2 n e j ( 2 N ) kn = e j ( 2 N ) kn

Por lo tanto, si x[n] es real, entonces x*[n] = x[n] y


N 1

X [ n k ] = x[ n]e

j ( 2 N ) kn

n =0

8.

N 1

= x [ n ] e j ( 2 N ) kn = X *[ k ]
n =0

Multiplicacin:
x1 [ n ] x2 [ n ]

donde

1
N

X1 [ k ] X 2 [ k ]

(4.98)

275
N 1

X 1 [ k ] X 2 [ k ] = X 1 [ i ] X 2 [ k i ]md N
i =1

9.

Propiedades Adicionales:

Cuando x[n] es real, sea


x [ n ] = x p [ n ] + xi [ n ]

donde xp[n] y xi[n] son las componentes par e impar de x[n], respectivamente. Sea
x[ n]

X [ k ] = A [ k ] + j B [ k ] = X [ k ] e j [ k ]

Entonces
X [ k ]md N = X *[ k ]

(4.99)

xp [ n ]

Re{ X [ k ]} = A[ k ]

xi [ n ]

j Im{ X [ k ]} = j B [ k ]

(4.100)

De la Ec. (4.99) tenemos


A[ k ] md N = A[ k ]
X [k ]

md N

B [ k ] md N = B [ k ]

= X [k ]

[ k ] md N = [ k ]

(4.101)

10. Relacin de Parseval:


N 1

n=0

x[n]

N 1

X [k ]

(4.102)

n=0

La Ec. (4.102) se conoce como la identidad de Parseval (o el teorema de Parseval) para la TFD.

276

Problemas
4.1 Un conjunto de secuencias es {k[n]} es ortogonal en un intervalo [N1, N2] si dos seales
cualesquiera m[n] y n[n] en el conjunto satisfacen la condicin
N2

n = N1

0
[ n ] k [ n ] =

mk
m=k

0. Demuestre que el conjunto de secuencias exponenciales complejas


k [ n ] = e jk ( 2 N ) n

k = 0, 1, K , N 1

es ortogonal en cualquier intervalo de longitud N.


4.2 Determine los coeficientes de Fourier para la secuencia peridica x[n] en la Fig. P4.2.

x[n]

...

...
4 3 2 1 0 1 2 3 4 5 6

Figura P4.2

4.3 Considere una secuencia

x[ n] =

[ n 3k ]

k =

(b) Dibuje x[n].


(c) Determine los coeficientes de Fourier ck de x[n].
4.4 Determine la representacin en serie de Fourier discreta para cada de las secuencias siguientes y
grafique la magnitud y fase de los coeficientes de Fourier:

(a) x [ n ] = cos n
4

(b) x [ n ] = 2sen

3
4

n sen

2
7

(c) x[n] es peridica con perodo 4 y

277

1
x[ n] =
3

(d) x [ n ] =

( 1)

( n k ) + sen

k =

0n3

3 n
4

4.5 Sea x[n] una secuencia peridica real con perodo fundamental N0 y coeficientes de Fourier
ck = ak + jbk , donde ak y bk son ambos reales.

(a) Demuestre que a k = ak y b k = bk .


(b) Demuestre que cN0

es real si N0 es par.

(c) Demuestre que x[n] tambin puede expresarse como una serie de Fourier trigonomtrica
discreta de la forma
x [ n ] = c0 + 2

( N 0 1) 2

k =1

( an cos k 0 n bk sen k 0 n )

0 =

2
N0

si N0 es impar o
x [ n ] = c0 + ( 1) n cN0 2 + 2

( N0 2 ) 2

k =1

( ak cos k 0 n bk sen k 0 n )

si N0 es par.
4.6 Determine la transformada de Fourier de cada una de las secuencias siguientes:

(a) x [ n ] = a n u [ n 1]
(b) x [ n ] = a ,
n

a real

a <1

(c) x [ n ] = sen ( 0 n ),

0 <

(d) x [ n ] = u [ n 1]
4.7 Sean x[n], h[n] y y[n] secuencias peridicas con el mismo perodo N0, y sean ak, bk y ck los
coeficientes de Fourier respectivos.

(a) Sea y [n] = x [n] h [n] . Demuestre que


ck = am bk m = ak m bm = ak bk
N0

N0

(b) Sea y [n] = x [n] h [n] . Demuestre que


ck = N 0 ak bk

4.8 Determine la transformada de Fourier de la secuencia en pulso rectangular

x { n} = u [ n ] u [ n N ]

278

4.9 Para la secuencia del pulso rectangular mostrado en la Fig. P4.9,

(a) Determine la transformada de Fourier X().


(b) Grafique X() para N1 = 4 y N1 = 10.

x[n]

...

...

N1

N1

Figura P4.9

4.10 Determine la transformada de Fourier inversa de

(a) X ( ) = 2 ( 0 )

(b) X ( ) = cos (2 )
(c) X ( ) = j
4.11 Determine la transformada de Fourier de la secuencia sinusoidal
x [ n ] = cos 0 n

4.12 Considere la secuencia x[n] definida por


1
x [ n] =
0

n 2
otros valores de n

(a) Dibuje x[n] y su transformada de Fourier X().


(b) Dibuje la secuencia escalada en el tiempo x( 2 ) [ n ] y su transformada de Fourier X ( 2 ) ( ) .
(c) Dibuje la secuencia escalada en el tiempo x( 3 ) [ n ] y su transformada de Fourier X ( 3 ) ( ) .
(d) Determine y [ n ] = x [ n ] x [ n ] .
(e) Exprese Y() en funcin de X().
4.13 Considere la secuencia y[n] dada por

x[ n]
y[n] =
0

Exprese Y[] en funcin de X[].

n par
n impar

279

4.14 Sea

1,
x[ n] =
0,

n 2
n >2

(a) Determine y [ n ] = x [ n ] x [ n ] .
(b) Halle la transformada de Fourier Y() de y[n].
4.15 Use el teorema de la convolucin para determinar la transformada de Fourier inversa de
X () =

( 1 a e )
j

a <1

4.16 Demuestre que

u[n] () +

1 e j

4.17 Verifique la propiedad de acumulacin, es decir, demuestre que

x[ k ]

X (0) ( ) +

k =

1
1 e j

X ()

4.18 Demuestre que

u[n]

[ n ] +

1
1 e j

4.19 Verifique el teorema de Parseval para la transformada de Fourier discreta.


4.20 Un sistema LIT causal de tiempo discreto es descrito por
y [ n ] 14 y [ n 1] + 81 y [ n 2] = x [ n ]

donde x[n] y y[n] son la entrada y salida del sistema, respectivamente.


(a) Determine la respuesta de frecuencia H() del sistema.
(b) Halle la respuesta al impulso h[n] del sistema.
(c) Halle y[n] si x [ n ] = (

1
2

u[n] .

4.21 Considere un sistema LIT causal de tiempo discreto con respuesta de frecuencia

H ( ) = Re{ H ( )} + j Im{ H ( )} = A ( ) + j B ( )
(a) Demuestre que la respuesta al impulso h[n] del sistema puede obtener en funcin de A() o
de B() solamente.
(b) Halle H() y h[n] si
Re{ H ( )} = A ( ) = 1 + cos

280

4.22 Determine la TFD de la secuencia x[n] = an, 0 n N 1.


4.23 Evale la convolucin circular y [ n ] = x [ n ] h [ n ] , donde

x [ n ] = u [ n ] u [ n 4]
h [ n ] = u [ n ] u [ n 3]
(a) Suponiendo N = 4.
(b) Suponiendo N = 8.

CAPTULO CINCO

LA TRANSFORMACIN DE LAPLACE
5.1

Introduccin

El concepto de transformar una funcin puede emplearse desde el punto de vista de hacer un
cambio de variable para simplificar la solucin de un problema; es decir, si se tiene un problema en la
variable x, se sustituye x por alguna otra expresin en trminos de una nueva variable, por ejemplo,
x = sen y , anticipando que el problema tendr una formulacin y una solucin ms sencillas en
trminos de la nueva variable y; luego de obtener la solucin en trminos de la nueva variable, se usa el
procedimiento opuesto al cambio previo y se obtiene entonces la solucin del problema original. El
logaritmo es un ejemplo sencillo de una transformacin a la que ya nos hemos enfrentado; su virtud es
que transforma un producto en una suma, que es una operacin mucho ms sencilla. Efectuando la
operacin inversa, el antilogaritmo, obtenemos el resultado del producto.

En el Captulo 3 se estudi la Transformada de Fourier. Sin embargo, esta transformacin est


restringida a funciones que tienden a cero lo suficientemente rpido conforme t de modo que la
integral de Fourier converja. Ahora se remover esa restriccin. Tambin queremos extender el
teorema de la integral de Fourier a aquellos casos donde deseamos la respuesta de un sistema lineal a
una excitacin que comienza en t = 0, es decir, se definen condiciones iniciales, y luego desarrollar
ciertas propiedades de la transformada modificada resultante, la cual identificaremos como la
transformada de Laplace.
Una transformacin que es de gran importancia en el clculo es la de integracin,
x

I { f ( t )} = f ( t ) dt = F ( x )
0

El resultado de esta operacin es una funcin F(x), la imagen de f(t) bajo la transformacin. Obsrvese
que la operacin inversa a la integracin es la derivacin; si se designa por D la operacin de derivar,
d/dt, entonces
D { F ( x )} = f ( x )

Con frecuencia es necesaria una transformacin ms complicada. Si se tiene una funcin f(t) de la
variable t, se define una transformada integral de f(t) como
b

Transformada integral de f ( t ) = T { f ( t )} = f ( t ) K ( s , t ) dt
a

(5.1)

282

La funcin K(s, t), la cual es una funcin de dos variables, se denomina el ncleo de la transformacin.
Obsrvese que la transformada integral ya no depende de t; es una funcin F(s) de la variable s, de la
cual depende el ncleo. El tipo de transformada que se obtiene y los tipos de problemas para los cuales
es de utilidad dependen de dos cosas: el ncleo y los lmites de integracin. Para ciertos ncleos K(s, t),
la transformacin (5.1) al aplicarse a formas lineales en f(t) dadas, cambia esas formas a expresiones
algebraicas en F(s) que involucran ciertos valores de frontera de la funcin f(t). Como consecuencia,
ciertos tipos de problemas en ecuaciones diferenciales ordinarias se transforman en problemas
algebraicos cuya incgnita es la imagen F(s) de f(t). Como ya se mencion, si se conoce una
transformacin inversa, entonces es posible determinar la solucin y(t) del problema original.
En general, recuerde que una transformacin T{f(t)} es lineal si para todo par de funciones f1(t) y f2(t)
y para todo par de constantes c1 y c2, ella satisface la relacin
T {c1 f1 ( t ) + c2 f 2 ( t )} = c1T { f1 ( t )} + c2 T { f 2 ( t )}

(5.2)

Es decir, la transformada de una combinacin lineal de dos funciones es la combinacin lineal de las
transformadas de esas funciones.
Para la seleccin particular del ncleo K ( s , t ) = e st y los lmites de integracin desde cero hasta
infinito en la Ec. (5.1), la transformacin definida as se denomina una transformacin de Laplace y la
imagen resultante una transformada de Laplace. La transformada de Laplace de f(t) es entonces una
funcin de la variable s y se denota por F(s) o L{f(t)}. La transformacin de Laplace es probablemente
la herramienta ms poderosa para estudiar los sistemas lineales descritos por ecuaciones diferenciales
con coeficientes constantes. Como un proceso, la transformacin convierte un problema en ecuaciones
diferenciales en uno que involucra una o ms ecuaciones algebraicas.
5.2 Definicin de la Transformada de Laplace

Dada una funcin f(t) definida para todos los valores positivos de la variable t, se forma la integral

f (t ) e

st

dt = F ( s )

(5.3)

la cual define una nueva funcin F(s) del parmetro s, para todo s para el cual converge la integral. La
funcin F(s) as formada se denomina la transformada de Laplace unilateral de f(t). Normalmente se
omitir el trmino unilateral y la transformada se denotar por F(s) o L{f(t)}. Es decir, la definicin
formal de la transformada de Laplace es
L { f (t )} = F ( s ) =

f (t ) e

st

dt

(5.4)

El lmite inferior de la Ec. (5.4) se escogi como 0 en vez de 0 o 0+ para incluir casos donde la funcin
f(t) pueda tener una discontinuidad de salto en t = 0 . Esto no debe considerarse una restriccin, ya que
en los estudios usuales de transitorios, el origen del tiempo siempre puede tomarse en el instante t = 0 o
en algn tiempo finito t > 0. La funcin en el lado derecho de la Ec. (5.4) no depende de t porque la

283

integral tiene lmites fijos. Como ya se mencion, la transformacin de Laplace es un proceso que
reduce un sistema de ecuaciones integro-diferenciales simultneas lineales a un sistema de ecuaciones
algebraicas simultneas lineales. La transformada de Laplace asocia una funcin en el dominio del
tiempo con otra funcin, la cual se define en el plano de frecuencia compleja.
La propiedad ms sencilla y ms obvia de la transformacin de Laplace es que es lineal. Esta
afirmacin es fcil de demostrar ya que ella est definida como una integral. Es decir, si f 1 (t) y f 2 (t)
poseen transformadas F1(s) y F2(s) y c1 y c2 son constantes,
L { c1 f1 ( t ) + c2 f 2 ( t )} = c1 F1 ( s ) + c2 F2 ( s )

(5.5)

La notacin
f (t ) F ( s )

significar que las funciones f(t) y F(t) forman un par de transformadas de Laplace, es decir, que F(s)
es la transformada de Laplace de f(t).
En general, la variable s es compleja (s = + j) pero, por los momentos, se tomar como real y ms
adelante se discutirn las limitaciones sobre el carcter de la funcin f(t) y sobre el recorrido de la
variable s. Puesto que el argumento st del exponente de e en la Ec. (5.3) o (5.4) debe ser adimensional,
entonces las dimensiones de s deben ser las de frecuencia y las unidades de segundos inversos ( s 1 ) .
Ahora se obtendrn las transformadas de algunas funciones elementales. La mayora de los ejemplos
estn basados en la integral

pt

dt =

1
p

p>0

(5.6)

cuya demostracin procede de la identidad


T

pt

dt =

1 e pT
p

En efecto, si p > 0, entonces epT 0 conforme T y se obtiene la Ec. (5.5).

Ejemplo 1

(a)

Se determinar la transformada de Laplace de la funcin f(t) = 1, t > 0. Insertando esta funcin en


la Ec. (5.3), se obtiene

L { 1 } = (1) e st dt = e st dt =

1
s

para s > 0. En la notacin indicada,


1
(b)

1
s

s>0

Considrese ahora la funcin f(t) = e c t , t > 0, donde c es una constante. En este caso,

(5.7)

284

L { ect } = ect e st dt = e ( s c ) t dt
La ltima integral es la misma que la de (5.6) con p = s c; por lo tanto, es igual a 1 ( s c ) , con
tal que s c > 0. Se concluye entonces que
1

ect

sc

s>c

(5.8)

Con la ayuda de mtodos elementales de integracin se pueden obtener las transformadas de otras
funciones. Por ejemplo,
t
sen at

1
s

t2

,
1

s2 + a2

cos at

2
s3
s

(5.9)

s2 + a2

para s > 0; ms adelante se estudiarn procedimientos ms sencillos para obtener estas transformadas.

Ejemplo 2

Usando la propiedad de linealidad de la transformada de Laplace se obtendr la transformada de la


funcin f ( t ) = senh at .
Usando la identidad
senh at =

1
e a t e at
2
2

entonces
1
2

1 1

2 sa

senh at

L {senh at } = L e at e at =

1 1
2 s+a

cuando s > a y s > a; es decir,


s2 + a2

( s>

Como la ecuacin de definicin de la transformada de Laplace contiene una integral en la cual uno de
sus lmites es infinito y por la propiedad de linealidad, una de las primeras preguntas a responder se
refiere a la existencia de la transformada. Un ejemplo sencillo de una funcin que no tiene una
transformada de Laplace es f ( t ) = exp exp ( t ) . Por ello, a continuacin se darn algunos teoremas

concernientes a la convergencia de la integral de Laplace.

285

5.3

Condiciones para la Existencia de la Transformada de Laplace

5.3.1 Funciones Seccionalmente Continuas

Se dice que una funcin f(t) es seccionalmente continua en un intervalo acotado a < t < b, si es
continua excepto en un nmero finito de puntos t1 < t2 < L < t N de (a , b) y si en cada punto de
discontinuidad posee lmites finitos conforme t tiende a cualquier extremo de los subintervalos desde el
interior (si x1 = a, el lmite por el lado derecho existe en t1, y si tN = b, el lmite por el lado izquierdo
debe existir en tN). Se usan los smbolos

f ( ti ),

f ( ti+ )

para denotar los lmites por el lado izquierdo y por el lado derecho, respectivamente, de f(t) en ti. La
funcin f(t) que se ilustra en la Fig. 5.1 es seccionalmente continua en (a, b). Tiene slo una
discontinuidad en t = t1 y
f ( t1 ) = A ,

f ( ti+ ) = B

f(t)

f(t)

B
A

t1

Figura 5.5

t1

Figura 5.2

La funcin que se ilustra en la Fig. 5.2 no es seccionalmente continua. Posee slo una discontinuidad
en t1, pero el lmite por el lado derecho de g(t) no existe en t1.
Teorema 1. Sean las funciones f(t) y g(t) seccionalmente continuas en todo intervalo de la forma
[c,T], donde c es fijo y T > c. Si |f(t)| g(t) para t c y si la integral

g ( t ) dt
c

converge, entonces la integral

286

f ( t ) dt
c

tambin converge.
Ms adelante se usar el Teorema 1 para establecer un conjunto de condiciones de suficiencia para la
existencia de la transformada de Laplace de una funcin. Sin embargo, primero se introducir la
notacin
f ( t ) = O [ g ( t )]

la cual debe leerse f(t) es del orden de g(t). Esta notacin significa que existen constantes M y N
tales que
f ( t ) Mg ( t )

cuando t N. En particular, si f(t) = O|e t | para alguna constante , se dice que f(t) es de orden
exponencial.
Teorema 2. Sea f(t) una funcin seccionalmente continua en todo intervalo de la forma [0,T], donde
T > 0 y sea f ( t ) = O et para alguna constante . Entonces la transformada de Laplace

L { f ( t )} = F ( s ) existe, al menos para s > .

Demostracin: De acuerdo con las hiptesis del teorema, existen constantes M y t0 tales que
s t
f ( t ) Met cuando t > t0. Entonces f ( t ) e st M e ( ) cuando s t0. Puesto que la integral

M e

( s ) t

dt

t0

converge cuando s > , la integral

st

dt

t0

tambin converge (Teorema 1). Puesto que

e
0

st

t0

f ( t ) dt = e
0

st

f ( t ) dt + e st f ( t ) dt ,

s>

t0

la transformada de Laplace L {f(t)} existe para s > .


Como una aplicacin importante del Teorema 2, se demostrar que si f(t) es de la forma
t n eat cos bt ,

t n e at sen bt

donde n es un entero no negativo, entonces L {f(t)} existe para s > a > 0. Primero obsrvese que

(5.10)

287

t n = O et
para todo nmero positivo . Como cos bt 1 y sen bt 1 para todo t, tenemos que

f ( t ) = O e(

a + ) t

Por el teorema 1, L { f ( t )} existe para s > a+ para todo nmero positivo . Por consiguiente,
L { f ( t )} existe para s > a.
El resultado anterior es importante en el estudio de ecuaciones diferenciales lineales con coeficientes
constantes. Considere la ecuacin homognea

P( D) x = 0
donde D = d/dt y P(D) es un operador polinomial. Toda solucin de esta ecuacin es una combinacin
lineal de funciones de la forma (5.10). Cualquier derivada de una solucin es tambin una combinacin
lineal de funciones de este tipo. Por lo tanto, se puede decir que toda solucin de la ecuacin, y toda
derivada de una solucin, es de orden exponencial y posee una transformada de Laplace.
Teorema 3. Sea f(t) una funcin seccionalmente continua en todo intervalo de la forma [0, T] y sea
f ( t ) = O et para alguna constante . Entonces la funcin h(t), donde
t

h ( t ) = f ( u ) du
0

es de orden exponencial. Si > 0, h ( t ) = O et y si < 0, h(t) = O[1].


Demostracin: Existen constantes positivas t0 y M1 tales que f ( t ) M 1 et para t t0. Tambin existe
una constante positiva M2 tal que |f(t)| M 2 para 0 t t0. Puesto que
t0

t0

h ( t ) = f ( u ) du + f ( u ) du
para t > t0, se tiene que
t0

t0

h ( t ) M 2 du + M 1 f ( u ) du
o
h ( t ) M 2 t0 +

Si > 0, entonces

M1

(e

et0

288

h ( t ) M 2 t0 + 1

t
e ,

t t0

y h ( t ) = O et .

Ejemplo 3

La funcin escaln unitario (previamente definida en el Cap. 1)


0 cuando 0 < t < t0
u ( t t0 ) =
1 cuando t > t0

es un ejemplo de una funcin seccionalmente continua en el intervalo 0 < t < T para todo nmero
positivo T (Fig. 5.3). Observe la discontinuidad en t = t0:
lm u ( t t0 ) = 0

lm u ( t t0 ) = 1

t t0

t t0+

La transformada de Laplace de esta funcin es

u (t t

)e

st

dt = e

st

t0

dt = e
s

st
t0

As que, siempre que s > 0,

L { u ( t t0 ) } =

e t0 s
s

u(t t0)
1

t
t0
Figura 5.3

Aqu se debe sealar un punto importante. La transformada de Laplace est definida solamente entre
0 y +. La conducta de la funcin f ( t ) para t < 0 nunca entra en la integral y por tanto no tiene
efecto sobre su transformada. Por ejemplo, las funciones f ( t ) = 1 y u(t) ( t0 = 1 en el Ejemplo 3) tienen
la misma transformada 1/s.

Las condiciones mencionadas en los teoremas para la existencia de la transformada de una funcin
son adecuadas para la mayora de nuestras necesidades; pero ellas son condiciones suficientes y no

289

necesarias. Por ejemplo, la funcin f(t) puede tener una discontinuidad infinita en, por ejemplo, t = 0,
es decir |f(t)| conforme t 0, con tal que existan nmeros positivos m, N y T, donde m < 1,
tales que |f(t)| < N/t m cuando 0 < t < T. Entonces, si en cualquier otra forma, f(t) cumple con las
condiciones mencionadas, su transformada todava existe porque la integral
T

sT

f ( t ) dt

existe.
5.3.2

Regin de Convergencia de la Transformada

El recorrido de valores de la variable compleja s para los cuales converge la transformada de Laplace
se denomina la regin de convergencia (RDC). Por ejemplo, sabemos que la seal x ( t ) = e at u ( t ) , a
real, tiene como transformada la funcin X ( s ) = 1 ( s + a ) , siempre que Re( s ) > a , puesto que
lm e ( s + a ) t = 0
t

slo si Re ( s + a ) > 0 o Re( s ) > a . As que la RDC para este ejemplo la especifica la condicin
Re( s ) > a y se muestra en el plano complejo como lo ilustra la Fig. 5.4 mediante el rea sombreada a
la derecha de la lnea Re( s ) = a , para el caso en que a > 0 .
j

a>0

Figura 5.4

5.4 Teoremas de la Derivada y de la Integral

Se desea expresar la transformada de Laplace

f (t ) e

st

dt

de la derivada f'(t) de una funcin f(t) en trminos de la transformada de Laplace F(s) de f(t).
Integrando por partes se obtiene

290

L { f ( t )} = f ( t ) e st dt = f ( t ) e st
0

+ s f ( t ) e st dt
0

Sea f(t) del orden de est conforme t tiende a infinito y continua. Entonces, siempre que s > a, el primer
trmino en el lado derecho se convierte en f(0) y por tanto

L { f ( t )} = sF ( s ) f (0)

(5.11)

As que la diferenciacin de la funcin objeto corresponde a la multiplicacin de la funcin resultado


por su variable s y la adicin de la constante f(0). La frmula (5.11) da entonces la propiedad
operacional fundamental de la transformacin de Laplace; sta es la propiedad que hace posible
reemplazar la operacin de diferenciacin en una ecuacin diferencial por una simple operacin
algebraica sobre la transformacin.

Ejemplo 4

Se desea resolver la ecuacin

y ( t ) + 3 y ( t ) = 0,

t >0

(5.12)

con la condicin inicial y(0) = 2.


Multiplicando ambos lados de la Ec. (5.12) por est e integrando de cero a infinito, se obtiene

[ y ( t ) + 3 y ( t )] e

st

dt = 0

(5.13)

Del teorema de la derivada, Ec. (5.11), se obtiene que

y (t ) e

st

dt = sY ( s ) y (0) = sY ( s ) 2

donde Y(s) = L{y(t)}. Sustituyendo en la Ec. (5.12) da


sY ( s ) 2 + Y ( s ) = 0

(5.14)

As que la transformada de Laplace Y(s) de la funcin incgnita y(t) satisface esta ecuacin.
Resolvindola, se obtiene
Y (s) =

(5.15)

s+3

Como se observa, la fraccin anterior es la transformada de la funcin 2 e3t . Por lo tanto, la solucin
de (5.12) es
y ( t ) = 2 e 3 t ,

t >0

291

5.4.1 La Transformada de Laplace Bilateral

La transformada de Laplace F(s) de una funcin f(t), como se defini en (5.3), involucra los valores de
la funcin f(t) para todo t en el intervalo (0, ). Es decir, el intervalo adecuado en la solucin de
ecuaciones diferenciales que son vlidas para t 0. En la teora de circuitos elctricos, sistemas de
control lineales y otras aplicaciones, algunas veces es deseable considerar los valores de f(t) en todo el
eje real y definir a F(s) en consecuencia. Esto conduce a la funcin

F (s) =

f ( t ) e st dt

(5.16)

conocida como la transformada de Laplace bilateral de f(t). Si la funcin f(t) es causal, es decir, si
f(t) = 0 para t < 0, entonces la integral en (5.16) es igual a la integral en (5.3). En este texto no se usar
la Ec. (5.16). La notacin F(s) se reservar slo para las transformadas unilaterales.
5.4.2 La Funcin Impulso

Un concepto importante ya presentado en el Cap. 1 es el de la funcin impulso. Esta funcin, tambin


conocida como la funcin delta de Dirac, se denota por ( t ) y se representa grficamente mediante
una flecha vertical, como en la Fig. 5.5. En un sentido matemtico estricto, la funcin impulso es un
concepto bastante sofisticado. Sin embargo, para las aplicaciones de inters es suficiente comprender
sus propiedades formales y aplicarlas correctamente. Las propiedades de esta funcin ya se estudiaron
en el Cap. 1 y no se repetirn aqu. Su transformada se derivar ms adelante.
(t )

Figura 5.5

5.4.3 El Teorema de la Derivada

Al comienzo de esta seccin se demostr que si F(s) = L{f(t)}, entonces

L { f ( t )} = sF ( s ) f (0)

(5.17)

Ahora se revisar el significado de f(0). Si f(t) es continua en el origen, entonces f(t) tiene un
significado claro: es el valor de f(t) para t = 0. Suponga, sin embargo, que f(t) es discontinua y que
f (0 + ) = lm f ( + ),
0

f (0 ) = lm f ( ),
0

>0

(5.18)

292

son sus valores en t = 0+ y t = 0, respectivamente (Fig. 5.7a). En este caso, el nmero f(0) en la Ec.
(5.17) depende de la interpretacin de f'(t). Si f'(t) incluye el impulso [f(0+) f(0)](t) debido a la
discontinuidad de f(t) en t = 0 (Fig. 5.7b), entonces f(0) = f(0). Si f'(t) es la derivada de f(t) para
+
t > 0 solamente y sin el impulso en el origen (Fig. 5.7c), entonces f(0) = f(0 ). La primera
interpretacin requiere aclarar el significado de la integral en la Ec. (5.3) cuando f(t) contiene un
impulso en el origen.
Como se sabe, la integral de (t) en el intervalo (0, ) no est definida porque (t) es un impulso en t
= 0. Para evitar esta dificultad, se interpretar a F(s) como un lmite de la integral f(t)e s t en el
intervalo (, ):

F ( s ) = lim

f (t ) e

st

dt =

f (t ) e

st

(5.19)

dt

donde > 0. Con esta interpretacin de F(s) se deduce que la transformada de (t) es igual 1:
(t ) 1

(5.20)

porque

(t ) e

st

dt = 1

f(t)

f'(t)

t0

f'(t)

t>0

f(0+) f(0)

f(0+)
0

f(0)
(a)

(b)

(c)

Figura 5.7

Adems, el trmino f(0) en la Ec. (5.29) es el lmite f(0) de f( ) conforme 0. Si F(s) se


interpreta como un lmite en el intervalo (, ), entonces f(0) = f(0+). En resumen,

f (t ) e

st

dt = sF ( s ) f (0 )

(5.21)

st

dt = sF ( s ) f (0+ )

(5.22)

f (t ) e

0+

293

La diferencia f(0+) f(0) entre estas dos integrales es igual a la transformada de Laplace del impulso
[f(0+) f(0)](t) en el origen y causada por la discontinuidad de f(t) en ese punto.
Si la funcin f(t) es continua en el origen, entonces debe quedar claro que f(0) = f(0+) = f(0) y las
frmulas (5.17), (5.21) y (5.22) son equivalentes. Si f(t) es continua para t 0 excepto por un salto
finito en t0, es fcil demostrar que la frmula (5.17) debe reemplazarse por la frmula
L { f ( t )} = sF ( s ) f (0) [ f ( t0 + 0) f ( t0 0) ] e st0

donde la cantidad entre corchetes es la magnitud del salto en t0.


Derivadas de Orden Superior. Sean f(t) y f'(t) continuas para t 0 y de orden exponencial y
tambin sea f'(t) seccionalmente continua en todo intervalo acotado. Entonces, como f"(t) es la
derivada de f'(t), la transformada de f'(t) menos el valor inicial f'(0) de f'(t), es decir

L { f ( t )} = sL { f ( t )} f (0)
= s [ sF ( s ) f (0) ] f '(0)

(5.23)

= s 2 F ( s ) sf (0) f '(0)

La aplicacin repetida del argumento anterior produce la relacin

L { f ( n ) ( t )} = s n F ( s ) s n1 f (0) s n2 f '(0) L f ( n1) (0)

(5.24)

donde se supone que f(t) y sus derivadas de orden hasta n 1 son continuas para t 0 y de orden
exponencial.
Aplicando (5.24) al impulso (t), se obtiene

L { ( n ) ( t )} = s n
porque la transformada de (t) es igual a 1 y los valores de sus derivadas en t = 0 son iguales a cero.

Ejemplo 5

Se desea obtener la transformada de f(t) = sen(at) a partir de la transformada de cos(at).


Si f(t) = cos(at), entonces f ( t) = asen(at) y aplicando (5.17), se obtiene

L { a sen at} = s L {cos at} 1


=

s2
s2 + a2

1 =

y por tanto

L {sen at} =

a
s + a2
2

a2
s2 + a2

294

Ejemplo 6

Determnese la transformada de f(t) = tu(t).


Solucin: La funcin f(t) = t y f'(t) son continuas y f(t) es de O ( et ) para cualquier positiva. Por lo
tanto,

L { f ( t )} = sL { f ( t )} f (0)

( s > 0)

o
L{1} = s L {t}

Como L {1} = 1/s, se tiene entonces que

L{ t } =

( s > 0)

s2

Ejemplo 7

Determnese la transformada de Laplace de f(t) = t n , donde n es cualquier entero positivo.


Solucin La funcin f(t) = t n cumple con todas las condiciones del Teorema 2 para cualquier
positiva. En este caso,
f (0) = f '(0) = L f (
f

(n)

f(

n 1)

(0) = 0

( t ) = n!

n +1)

(t ) = 0

Aplicando la frmula (5.24) se obtiene

L{ f

( n +1)

( t ) = 0 = s n +1L { t n } n !

y por tanto,

L{ tn } =

n!
s n +1

( s > 0)

5.4.4 El Teorema de la Integral

Usando el teorema de la derivada, se obtendr la transformada F(s) de la integral definida por


t

f (t ) =

y ( ) d

(5.25)

295

de una funcin y(t) en trminos de la transformada Y(s) de y(t). Se supone que f(t) es seccionalmente
continua y de orden exponencial.
La funcin f(t) en la Ec. (5.25) es continua y f(0) = 0. Tambin se tiene que y(t) = f'(t). Por lo tanto,
la transformada Y(s) de y(t) es igual la transformada sY(s) f(0) y, puesto que f(0) = 0, se concluye
que Y(s) = sF(s). Entonces,
t
1
F ( s ) = L y ( ) d = Y ( s )
0
s

(5.26)

Ahora bien, la formulacin de las leyes de Kirchhoff para una red, con frecuencia incluye una
integral con lmites de a t. Estas integrales pueden dividirse en dos partes,
0

y ( ) d =

y ( t ) dt + y ( ) d
0

en donde el primer trmino de la derecha es una constante. Cuanto y(t) es una corriente, esta integral es
el valor inicial de la carga, q (0 ) , y cuando y(t) es un voltaje, la integral es el enlace de flujo
(0 ) = L i (0 ) , donde L es la inductancia. En cualquier caso, este trmino debe incluirse en la
formulacin de la ecuacin; la transformada de una constante q (0 ) es

L { q (0 )} =

q (0 )
s

Y se puede escribir una ecuacin similar para (0 ).


5.4.5 Traslacin Compleja

Ahora se expresar la transformada

at
st
e f ( t ) e dt =

f (t ) e

( s + a ) t

dt

( a > 0)

del producto eatf(t) en trminos de la transformada F(s) de f(t). La ltima integral en la ecuacin
anterior es la misma integral de la ecuacin de definicin de la transformada, siempre que s se
reemplace por s a. Por lo tanto, es igual a F(s a) y se obtiene el par de transformadas
e at f ( t ) F ( s + a )

(5.27)

Esta propiedad nos dice que la transformada del producto de eat por una funcin de t es igual a la
transformada de la misma funcin con s reemplazada por s + a. Como herramienta para hallar
transformadas inversas, esta propiedad afirma que si s + a es reemplazada por s en la transformada de
una funcin f(t), entonces f(t) es igual al producto de eat por la inversa de la transformada modificada.

296

Ejemplo 8
(a)

Se desea evaluar L1
.
2
s ( s + 4)

Suprimiendo el factor 1/s, obtenemos Y ( s ) = 1 ( s 2 + 4) = 12 2 ( s 2 + 4) cuya transformada


inversa es igual a 12 sen 2t . Integrando esta ecuacin y usando la Ec. (5.26), tenemos que
t

1
cos 2 t

sen 2 t
L1 2
dt =
=
4
s ( s + 4) 0 2
(b)

1 cos 2 t

1
= sen 2 t
2

Ahora se usarn las Ecs. (5.25) y (5.26) para evaluar la integral


t

g ( t ) = e a d
0

ste es un caso especial de la Ec. (5.26) con Y(t) = ea t . Usando (5.26) con f(t) = 1, se tiene que
F(s) = 1/s y entonces

L { e at 1} =

1
s+a

Usando(5.26) con Y(s) = 1/(s+a), se obtiene


G(s) =

1
s(s + a)

1a
s

1a
s+a

y por tanto,
1
e a t
a a
1
= ( 1 e at )
a

g (t ) =

( t > 0)

Aplicando la Ec. (5.26) a las transformadas de sen(bt) y cos(bt) se demuestra fcilmente que
e at cos bt
e at sen bt

s+a

(s + a)

+ b2

(s + a)

+ b2

297

5.5 El Problema de Inversin

Si F(s) es la transformada de Laplace de una funcin f(t), entonces f(t) se denomina la transformada
de Laplace inversa de F(s). El problema de inversin es la determinacin de la transformada inversa
f(t) de una funcin F(s) dada. Este problema es bsico en las aplicaciones de la transformada de
Laplace. Considere, por ejemplo, la ecuacin diferencial
y ( t ) + 3 y ( t ) = 6,

y (0) = 0

(5.28)

Transformando esta ecuacin, se obtiene


6

sY ( s ) + 3Y ( s ) =

puesto que y(0) = 0 y la transformada de f(t) = 6 es igual a 6/s. Por lo tanto,


Y (s) =

(5.29)

s ( s + 3)

As que para determinar y(t) se debe hallar la transformada inversa de esta fraccin.
En general, hay dos mtodos de inversin fundamentales diferentes:
1. El Mtodo de la Frmula de Inversin. En este mtodo, la funcin f(t) se expresa directamente

como una integral que involucra la funcin F(s). Este resultado importante, conocido como el de la
frmula de inversin, se discute usualmente en el contexto de lo que se conoce como transformadas
de Fourier (tpico fuera del alcance de este texto).
2. Tablas. En este mtodo se intenta expresar la funcin F(s) como una suma de transformadas

F ( s ) = F1 ( s ) + F2 ( s ) + L + Fn ( s )

(5.30)

donde F1 ( s ), K ,Fn ( s ) son funciones con transformadas inversas f1 ( t ), K , f n ( t ) conocidas y


tabuladas. De la propiedad de linealidad de la transformada se determina que si F(s) puede ser
expandida como en la Ec. (5.30), entonces su transformada inversa f(t) est dada por
f ( t ) = f1 ( t ) + f 2 ( t ) + L + f n ( t )

(5.31)

Como una ilustracin se expande la fraccin (5.29) como una suma de dos fracciones con
transformadas conocidas:
Y (s) =

6
s ( s + 3)

2
s

2
s+3

(5.32)

sta muestra que la transformada inversa y(t) de Y(s) es la suma


y ( t ) = 2 e 3t ,

t>0

(Esta tcnica tambin se us en el Ejemplo 8).


La identidad en (5.32) proviene de la conocida tcnica de expansin de funciones racionales en
fracciones parciales, la cual se discutir ms adelante.

298

En el problema de inversin se deben considerar las siguientes preguntas:


1. Existencia. Posee toda funcin F(s) una transformada inversa? Hay funciones que no poseen

transformadas inversas. Sin embargo, esas funciones tienen un inters principalmente matemtico.
Todas las funciones consideradas en este texto poseen transformadas inversas.
2. Unicidad. Pueden dos funciones f1(t) y f2(t) tener la misma transformada F(s)? Si dos funciones

tienen la misma transformada, entonces ellas deben ser iguales para esencialmente todos los valores
de t. Sin embargo, pueden diferir en un conjunto discreto de puntos. Si las funciones son continuas,
entonces ellas deben ser idnticas.

5.5.1 Inversin de Transformadas Racionales (Fracciones Parciales)

Ahora se determinar la transformada inversa f(t) de la clase de funciones racionales, es decir, de


funciones de la forma
F (s) =

N (s)

(5.33)

D(s)

donde N(s) y D(s) son polinomios en s y no poseen factores comunes. Aqu se supone que F(s) es una
fraccin propia, es decir, que el grado de N(s) es menor que el de D(s). Las fracciones impropias
involucran funciones de singularidad y se considerarn posteriormente.
Primero, supngase que todas las races si, i = 1, 2, , n, del denominador D(s) son distintas. De
acuerdo con la teora de fracciones parciales, F(s) puede entonces expandirse como una suma, es decir,
F (s) =

N (s)
D(s)

c1
s s1

c2
s s2

+ L +

cn

(5.34)

s sn

Para determinar el valor de ci, se multiplican ambos miembros de la Ec. (5.34) por s si para obtener la
ecuacin

( s si ) F ( s ) = ( s si )

N (s)
D(s)

c1 ( s si )
s s1

+ L + ci + L +

cn ( s si )
s sn

es decir, se remueve del denominador el factor s si; evaluando ahora el resultado en s = si, se obtiene
ci = ( s si ) F ( s ) s = s = ( s si )
i

N (s)
D(s)

=
s = si

N (s)
D ( s )

(5.35)
s = si

donde D ( si ) = [ dD ds ]s = s = D ( s ) ( s si ) s = s . Puesto que la transformada inversa de la fraccin


i
i
1/(s s i ) es igual a e si t , de la Ec. (5.34) se concluye que la transformada inversa f(t) de la funcin
racional F(s) es una suma de exponenciales:
f ( t ) = c1 e s1 t + c2 e s2 t + L cn e n

s t

(5.36)

299

Ejemplo 9. Determine la transformada inversa de la funcin


F (s) =

s 2 + 29s + 30
s 3 + 7 s 2 + 10s

Solucin: El denominador de F(s) es de mayor grado que el numerador y posee factores reales y
distintos; stos son: s1 = 0, s2 = 2 y s3 = 5. Por lo tanto, se pueden determinar factores c1, c2, y c3
tales que
s 2 + 29 s + 30
s + 7 s + 10 s
3

s 2 + 29 s + 30
s ( s + 2) ( s + 3)

c1
s

c2
s+2

c3
s+5

y usando la Ec. (5.35) se obtiene


c1 = sF ( s ) s =0 = 3, c2 = ( s + 2 ) F ( s )

s =2

= 4,

c3 = ( s + 5 ) F ( s )

s =5

= 6

Por lo tanto,
f ( t ) = 3 + 4 e 2 t 6 e 5 t ,

t>0

Ahora se considerarn fracciones parciales para el caso en el cual el polinomio D(s) contiene factores
lineales repetidos de la forma (s s i )m. En este caso, la expansin de F(s) en fracciones parciales
consiste de trminos de la forma
ci 1
s si

ci 2

( s si )

+ L

cim

( s si )

(5.37)

donde los nmeros c i j , j = 1, 2, , m, son independientes de s y vienen dados por


ci , m r =

1 dr

( s si )m F ( s )
,
s = si
r ! ds

r = 0, 1, K , m 1

(5.38)

As que para evaluar el coeficiente ci,mr se remueve el factor (s s i )m del denominador de F(s) y se
evala la derivada r-sima del resultado en s = si. La componente de f(t) debida a la raz mltiple si es
la transformada inversa de la suma en (5.37) y viene dada por
ci 1 e si t + ci 2 t e si t + L +

cim

( m 1)!

t m 1 e si t

(5.39)

De lo anterior se concluye que la transformada inversa de una funcin racional F(s) es una suma de
exponenciales cuyos coeficientes son polinomios en t. Los exponentes si se denominan los polos de
F(s), es decir, los polos son las races del denominador D(s).

Ejemplo 10. La funcin

300

F (s) =

s2 + 2 s + 5

( s + 3)( s + 5)

c1

s+3

c21
s+5

c22

( s + 5)

(5.40)

tiene un polo sencillo en s1 = 3 y un polo mltiple en s2 = 5 con multiplicidad m = 2. En este caso,


c1 =

s2 + 2 s + 5

( s + 5)
c21 =

= 2,

c22 =

s2 + 2 s + 5
s+3

s =3

d s2 2 s + 5
s+3

ds

s2 + 6 s + 1

s =5

( s + 3)

= 10
s =5

= 1

2
s =5

Por tanto,
f ( t ) = 2 e 3t (1 + 10 t ) e 5 t ,

t>0

Observe que el coeficiente c21 puede determinarse sin diferenciacin. Puesto que (5.40) es vlida para
toda s, tambin es vlida para s = 0 (o cualquier otro nmero). Haciendo s = 0, por ejemplo, se obtiene
1
15

c1
3

c21
5

c22
25

Puesto que c1 = 2 y c22 = 10, la igualdad anterior produce c21 = 1.


Races Complejas

En los ejemplos anteriores, las races del denominador de la funcin F(s) eran reales. Se pueden
obtener resultados similares si D(s) tiene races complejas. Sin embargo, en este caso los coeficientes
correspondientes son complejos y f(t) contiene trminos exponenciales complejos. En el anlisis de
sistemas fsicos, la funcin F(s) tiene coeficientes reales. Por ello, las races complejas siempre ocurren
en pares conjugados y, como se demuestra a continuacin, las componentes correspondientes de f(t)
son ondas sinusoidales amortiguadas con coeficientes reales. Se comenzar con un ejemplo:
F (s) =

5 s + 13

s ( s + 4 s + 13)
2

En este caso, D(s) tiene dos polos complejos, s1 = 2 + j3, s2 = 2 j3, y un polo real, s3 = 0. La
expansin directa de (5.34) da
5 s + 13
s ( s + 4 s + 13)
2

c1
s ( 2 + j 3)

c2
s ( 2 j 3)

c3
s

donde c1 = (1+j)/2, c2 = (1j)/2 y c3 = 1 (determinados en la forma ya explicada). Por consiguiente,


f (t ) =

1+ j
2

e(

2 + j 3) t

1 j
2

e(

2 j 3 ) t

+ 1,

t>0

(5.41)

301

Esta expresin incluye cantidades complejas. Sin embargo, es una funcin real. Efectivamente,
2 j 3) t
insertando la identidad e(
= e 2 t ( cos 3 t j sen 3 t ) en (5.41), se obtiene
f ( t ) = 1 e 2 t ( cos 3 t sen 3 t ) ,

t>0

(5.42)

la cual es una expresin real.


Ahora se demostrar que la Ec. (5.42) puede determinarse directamente. El resultado est en el hecho
de que si F(s) es una funcin real con coeficientes reales y s1 y s2 son dos nmeros complejos
conjugados, entonces F(s2) = F ( s1* ) = F *( s1 ) (donde el asterisco indica el conjugado complejo).

Considere una funcin racional F(s) con coeficientes reales. Como se sabe, si s1 = + j es un polo
complejo de F(s), entonces su conjugado, s1* = j , tambin es un polo. Por lo tanto, la expansin
(5.34) de F(s) contiene trminos como
c1
s s1

c2
s s2

s1 = + j , s2 = j

(5.43)

Los coeficientes c1 y c2 se expresarn en trminos de la funcin


G(s) =

F (s)
j

( s s1 )( s s2 )

(5.44)

De la Ec. (5.35) se obtiene que


c1 = F ( s ) ( s s1 )

s = s1

j G ( s1 )
s s2

1
= G ( s1 )
2

puesto que s1 s2 = j2. En forma similar,


1
c2 = G ( s2 )
2

La funcin G(s1) es, en general, compleja con parte real Gr y parte imaginaria Gi, es decir,
G ( s1 ) = Gr + jGi

(5.45)

Como F(s2) = F*(s1), de (5.44) se obtiene que G(s2) = G*(s1) = Gr jGi, y por lo tanto,
c1 =

1
2

( Gr + jGi ) ,

c2 =

1
2

( Gr jGi )

La transformada inversa de la suma en la Ec. (5.43) es entonces igual a


c1 e s1 t + c2 e s2 t =

Insertando la identidad e(

j ) t

1
2

( Gr + jGi ) e(+ j) t + ( Gr jGi ) e( j) t


2

(5.46)

= et ( cos t j sen t ) en la Ec. (5.46), se obtiene finalmente la

transformada inversa f(t) de F(s) debida a los polos complejos conjugados s1 y s2, y la cual es igual a

302

e t ( Gr cos t Gi sen t )

(5.47)

En resumen: Para hallar el trmino en f(t) resultante de los polos complejos de F(s), se forma la
funcin G(s), como en (5.44), y se calcula su valor G(s1) para s = s1. El trmino correspondiente de f(t)
lo da la Ec. (5.47), donde Gr y Gi son las partes real e imaginaria de G(s).
El resultado anterior se aplicar a la funcin
F (s) =

5 s + 13
s ( s 2 + 4 s + 13)

ya considerada. En este caso,

( s s1 )( s s2 ) = s 2 + 4 s + 13,
G(s) =

F (s)
j

(s

+ 4 s + 13) =

s1 = 2 + j 3,

5 s + 13
j 3s

= 2, = 3

G ( s1 ) =

5 ( 2 + j 3) + 13
j 3( 2 + j 3)

Por tanto, Gr = 1, Gi = 1 y la Ec. (5.47) da


e 2 t ( cos 3 t + sen 3 t )

Este es el trmino de f(t) proveniente de los polos complejos de F(s) y concuerda con el resultado
(5.42).

Ejemplo 11

Obtener la transformada de Laplace inversa de la funcin


F (s) =

s
( s + 9) ( s + 2 )
2

c1
s j3

c2
s + j3

c3
s+2

El coeficiente c3 correspondiente al polo real s3 = 2 se determina directamente a partir de la Ec. (5.35):


c3 = ( s + 2 ) F ( s ) s =2 =

2
13

Los otros dos polos s1 = j3 y s2 = j3 de F(s) son imaginarios puros con = 0 y = 3. Puesto que

( s s1 )( s s2 ) = s 2 + 9
la funcin G(s) correspondiente en la Ec. (5.44) est dada por
G(s) =
Por lo tanto,

F (s)
j3

(s

+ 9) =

s
j 3( s + 2)

303

G ( s1 ) =

j3
j 3( j 3 + 2)

2
13

3
13

Agregando el trmino c3e2t debido al polo real s3 = 2, se obtiene


f (t ) =

2
13

cos 3t +

13

sen 3t

2
13

e 2 t

5.5.2 Inversin de Funciones Impropias

En la Seccin 5.5.1 se determin la transformada inversa de funciones racionales propias. Ahora se


considerarn funciones impropias, limitando la discusin a dos casos especiales.
Se comenzar con un ejemplo. Suponga que
F (s) =

3 s 2 + 15 s + 14
s2 + 3s + 2

Dividiendo se obtiene
3 s 2 + 15 s + 14
s2 + 3 s + 2

= 3+

6s + 8
s2 + 3 s + 2

= 3+

s +1 s + 2

y por tanto,
f ( t ) = 3 ( t ) + 2 e t + 4 e 2 t
Considrese otro ejemplo. Sea la funcin
F (s) =

s3 + 3 s 2 + s + 8
s2 + 4 s

Entonces, procediendo en la misma forma que en el ejemplo previo, se obtiene


s3 + 3 s 2 + s + 8
s2 + 4 s

= s 1+

2
s

3
s+4

y por lo tanto
f ( t ) = ( t ) ( t ) + 2 + 3 e 4 t
En general, para una funcin racional
F (s) =

N (s)
D(s)

donde el grado de N(s) es mayor o igual que el de D(s), se procede a la divisin para obtener

304

F ( s ) = cm n s m n + L + c1 s + c0 +

Q(s)
D(s)

= P(s) +

Q(s)
D(s)

donde P(s) es el cociente y Q(s) es el residuo; m es el grado del numerador y n el del denominador (m >
n). Ahora el grado de Q(s) es menor que el de D(s). La nueva funcin racional Q ( s ) D ( s ) es propia y
est preparada para su expansin. Se contina entonces con la expansin en fracciones parciales de
Q(s)/D(s) y luego se obtiene la transformada inversa de F(s). Obsrvese que el polinomio P(s)
producir funciones singulares. stas no aparecen con frecuencia, pero son de mucha utilidad en la
solucin de algunos problemas prcticos que estn fuera del alcance de este texto.
5.6 Los Valores Inicial y Final de f(t) a Partir de F(s)

A continuacin se demuestra que los valores de una funcin f(t) y sus derivadas en t = 0 pueden
expresarse en trminos de los valores de su transformada para valores grandes de s. Este resultado
permite determinar en una forma sencilla la conducta de f(t) cerca del origen. Tambin se determinar
el comportamiento de f(t) conforme t tiende a infinito usando su transformada y bajo ciertas
condiciones.
5.6.1 El Teorema del Valor Inicial

La funcin est tiende a cero conforme s tiende a infinito para t > 0 (la parte real de s mayor que cero).
A partir de esto se deduce que bajo ciertas condiciones generales

lm f ( t ) e s t dt = 0
s

(5.48)

para todo > 0. Si f(t) es continua para t 0, excepto posiblemente por un nmero finito de
discontinuidades finitas, y tambin de orden exponencial, entonces la integral en (5.48) tiende a F(s)
cuando 0 . Esto da como resultado que
lm F ( s ) = 0
s

(5.49)

Lo anterior podra no ser cierto si f(t) contiene impulsos u otras singularidades en el origen. Por
ejemplo, si f(t) = e a t , entonces F(s) = 1/(s a) tiende a cero cuando s . Sin embargo,
si f ( t ) = ( t ) , entonces su transformada F(s) = 1 no tiende a cero.
Aplicando (5.49) a la funcin f ' ( t) y usando la Ec. (5.17), se obtiene

lm
s

f (t ) e

st

dt = sF ( s ) f (0 ) = 0

Aqu se toma a f'(t) como seccionalmente continua y de orden exponencial.

305

Entonces se obtiene que


f (0 ) = lm sF ( s )

(5.50)

este resultado se conoce como el teorema del valor inicial. Se verificar con una ilustracin sencilla. Si
f(t) = 3e2t, entonces
F (s) =

3
s+2

lm sF ( s ) = lm
s

3s
s+2

=3

lo cual concuerda con la Ec. (5.50) porque, en este caso, f(0+) = f(0) = 3.

Ejemplo 12

Si
F (s) =

2s + 3
s + 7 s + 10
2

entonces,
lm sF ( s ) = lm
s

2 s2 + 3 s
s 2 + 7 s + 10

=2

y, po tanto, f(0) = 2.

El teorema del valor inicial tambin puede usarse para determinar los valores iniciales de las
derivadas de f(t). En efecto, como se obtiene a partir de la Ec. (5.24), la funcin
s 2 F ( s ) sf (0) f (0) es la transformada de Laplace de f"(t). Por lo tanto [ver la Ec. (5.48)], debe
tender a cero cuando [f"(t) debe cumplir con las condiciones necesarias]. Esto conduce a la
conclusin de que
f (0) = lm s 2 F ( s ) sf (0)
s

(5.51)

En una forma similar se pueden determinar los valores iniciales de derivadas de orden superior. En
todos estos casos se ha supuesto que f(t) es continua en el origen.

Ejemplo 13

Si
F (s) =

2s + 3
s 2 + 7 s + 10

entonces sF ( s ) 0, s 2 F ( s ) 0 y s 3 F ( s ) 1 cuando s . Por tanto,


f (0) = 0, f (0) = 0, f (0) = 1

306

5.6.2 El Teorema del Valor Final

Ahora se demostrar que si f(t) y su primera derivada son transformables en el sentido de Laplace,
entonces
lm f ( t ) = lm sF ( s )
t

(5.52)

s 0

Ya se ha demostrado que

f (t ) e

st

dt = sF ( s ) f (0 )

(5.53)

Cuando s tiende a cero, se obtiene entonces que

f ( t ) dt = lm f ( t ) dt
t

= lm f ( t ) f (0 )
t

Igualando este resultado con el de la Ec. (5.53), escrita para el lmite cuando s 0, se llega a la
conclusin de que
lm f ( t ) = lm sF ( s )
t

(5.54)

s 0

como se requera. La aplicacin de este resultado requiere que todas las races del denominador de
F(s) tengan partes reales negativas, ya que de otra manera no existe el lmite de f(t) cuando t tiende a
infinito.
Ejemplo 14

Para la funcin
f ( t ) = 5 3 e 2 t
es evidente que su valor final es 5. La transformada de f(t) es
F (s) =

5
s

3
s+2

2 s + 10
s ( s + 2)

y, segn la Ec. (5.54), el valor final de f(t) es


lm f ( t ) = lm sF ( s ) = lm
t

s 0

s 0

2 s + 10
s+2

=5

307

5.7 Teoremas Adicionales


5.7.1 El Teorema de Traslacin Real o de Desplazamiento

Una funcin f(t) trasladada en el tiempo se representa como f(t t 0 )u(t t 0 ), donde
f ( t t0 ),
f ( t t0 ) u ( t t0 ) =
0,

t > t0

(5.55)

t < t0

(vase la Fig. 5.8). Observe que la funcin f(t t 0 )u(t t 0 ) es idntica a f(t)u(t) excepto que est
retardada o trasladada en t0 seg. Para encontrar la transformada de esta funcin se aplica la Ec. (5.3) a
la Ec. (5.55):

f ( t t0 ) u ( t t0 ) e st dt = f ( t t0 ) e st dt = f ( t ) e

f(t t0)u(t t0)

f(t)u(t)

dt

t0

f(t)

s ( t + t0 )

t0

Figura 5.8

de donde se concluye que

L { f ( t t0 ) u ( t t0 )} = e st L { f ( t )}

(5.56)

Aplicando la propiedad (5.56) al par (t) 1, se obtiene


( t t0 ) e st0

Ejemplo 15

De los pares 1 1/s y t 1/s2, se obtienen los pares


u ( t t0 )

1
s

e st0 ,

( t t 0 ) u ( t t0 )

Aplicando lo anterior al pulso pT = u(t) u(t T). Se obtiene

1
s2

e st0

308

pT = u ( t ) u ( t T )

( 1 e )
s
sT

(5.57)

Este ltimo resultado puede verificarse aplicando la definicin dada por la Ec. (5.3) de la transformada.
Puesto que pT ( t ) = 1 para 0 < t < T y 0 para otros valores de t, su transformada es igual a

pT ( t ) e st dt = e st dt =

( 1 e )
sT

acorde con (5.57).

Ejemplo 16

Si se da la funcin
f ( t ) = 6 e 2 t u ( t ) + 4 e

3 ( t t0 )

u ( t t0 )

entonces, aplicando la Ec. (5.56), se obtiene


F (s) =

6
s+2

4
s+3

e st0

Supngase que F1(s), F2(s), , Fm(s) son funciones con transformadas inversas conocidas
f1 ( t ), f 2 ( t ), K , f m ( t ) . De la Ec. y la propiedad de linealidad de la transformada se obtiene que la
transformada inversa de la suma
F ( s ) = F1 ( s ) e st1 + F2 ( s ) e st2 + L + Fm ( s ) e stm

(5.58)

f ( t ) = f1 ( t t1 ) u ( t t1 ) + f 2 ( t t2 ) u ( t t2 ) + L + f m ( t tm ) u ( t tm )

(5.59)

es la suma

Esto se ilustrar mediante un ejemplo.

Ejemplo 17

Se desea determinar la transformada inversa de la funcin


F (s) =

3 + 3 se sT + 6 e 2 sT
s 2 + 7 s + 10

Solucin: Esta funcin es una suma igual que en la Ec. (5.58), donde

309

F1 ( s ) =

3
s + 7 s + 10
2

F2 ( s ) =

3s
s + 3 s + 10
2

F3 ( s ) =

6
s + 7 s + 10
2

y t1 = 0, t2 = T y t3 = 2T. Usando expansin en fracciones parciales, se obtiene


f1 ( t ) = e 2 t e 5 t ,

f 2 ( t ) = 5 e5 t 2 e2 t ,

f 3 ( t ) = 2 e2 t 2 e5 t

y aplicando la Ec. (5.59), se obtiene


f ( t ) = f1 ( t ) u ( t ) + f 2 ( t T ) u ( t T ) + f3 ( t 2T ) u ( t 2T )

5.7.2 El Teorema de Escala

Este teorema relaciona los cambios de escala en el dominio de s con los cambios correspondientes en el
dominio de t. El trmino cambio de escala significa que s o t se multiplican por una constante positiva.
Dada una funcin f(t), se cambia de escala al formar una nueva funcin f ( t/t 0 ). Su transformada se
encuentra como sigue: a partir de la ecuacin de definicin se tiene que

L { f ( t t0 )} = f ( t t0 ) e st dt
0

= t0 f ( t t0 ) e

( t0 s ) t t0

d ( t t0 )

si ahora se hace t/t 0 = x, entonces la ltima ecuacin se convierte en

L { f ( t t0 )} = t0 f ( x ) e t sx dx
0

Obsrvese que la integral define a F(t 0 s), de tal modo que se puede escribir

L { f ( t t0 ) } = t0 F ( t 0 s )

(5.60)

La transformada inversa correspondiente es


f ( t t0 ) = t0 L1 { F ( t0 s ) }

Ejemplo 18

Para la transformada
F (s) =

1
s ( s + 1)

(5.61)

310

el valor correspondiente de f ( t) es
f ( t ) = 1 e t

(5.62)

El teorema de escala indica que la nueva funcin


f1 ( t ) = L1 {2 F ( 2 s )} = 1 et 2

(5.63)

est relacionada con f(t) en la Ec. (5.62) por un simple cambio en la escala del tiempo.

5.7.3 Derivadas de Transformadas

Cuando la integral de Laplace

F (s) =

f (t ) e

st

(5.64)

dt

es diferenciada formalmente con respecto al parmetro s, se obtiene la frmula


dF ( s )
dt

[ t f ( t )] e

st

dt

lo que implica que


t f (t )

dF ( s )
ds

(5.65)

es decir, la multiplicacin de una funcin f(t) por t en el dominio del tiempo equivale a diferenciar la
transformada F(s) de f(t) con respecto a s y luego cambiar de signo en el dominio de la frecuencia
compleja..
Se debe sealar que f(t)e-st y su derivada parcial de cada orden con respecto a s cumplen con las
condiciones necesarias para que la diferenciacin con respecto a s se pueda ejecutar dentro del signo de
integracin; se obtiene as el siguiente teorema:
Teorema 4. La diferenciacin de la transformada de una funcin corresponde a la multiplicacin por
t :

F(

n)

( s ) = L { ( t )n f ( t )} ,

( n = 1, 2, K )

(5.66)

Adicionalmente F(n)(s) 0 conforme s . Estas propiedades se cumplen siempre que f(t) sea
seccionalmente continua y del orden de et t, si s > en la frmula (5.66).

311

Ejemplo 19

Ya se sabe que

L {sen at} =

( s > 0)

s + a2
2

y, por la Ec. (5.66),

L { t sen at} =

d a

ds s 2 + a 2

2 as

= 2
( s + a 2 )2

de donde se obtiene la frmula

L { t sen a t } =

2a s

( s2 + a2 )

(5.67)

Ejemplo 20

Determinar la transformada de Laplace de f ( t ) = t e a t cos 5 t .


Si se hace f1 ( t ) = cos 5 t y f 2 ( t ) = t cos 5 t , se obtiene
F1 ( s ) =

s
s + 25
2

Usando el teorema de la multiplicacin por t, se obtiene


F2 ( s ) =

d s
s 2 25
=

ds s 2 + 25 ( s 2 + 25) 2

y finalmente, usando la propiedad de la traslacin compleja,


F (s) =

( s + 2)

25

( s + 2 )2 + 25

s 2 + 4 s 21

(s

+ 4 s + 29 )

5.7.4 La Transformada de una Funcin Peridica

Considere la funcin peridica f(t) con un perodo T que satisface f(t + nT) = f(t), donde n es un entero
positivo o negativo. La transformada de esta funcin es

312

F (s) =

f (t ) e

st

dt

f (t ) e

st

(5.68)

2T

dt +

f (t ) e

st

dt + L

Trasladando sucesivamente cada trmino de la transformada por e-sT, en donde n es el nmero de


traslados necesarios para hacer que los lmites de las expresiones integrales sean todos de 0 a T, se
tiene que

F ( s ) = (1 + e

sT

+e

+ L) f ( t ) e st dt

2 sT

y utilizando el teorema del binomio para la identificacin de la serie, se obtiene


F (s) =

1
1 e

T
Ts

f (t ) e

st

dt

(5.69)

La integral en esta ecuacin representa la transformada de la funcin f(t) como si ella estuviese
definida slo de 0 a T. Denotando esta transformada por F1(s), se obtiene
F (s) =

F1 ( s )

1 e Ts

(5.70)

Esta ecuacin relaciona la transformada de una funcin peridica con la transformada de esa funcin
sobre el primer ciclo (o cualquier otro ciclo).

Ejemplo 21

Se desea determinar la transformada de un tren de pulsos con un perodo T, donde cada pulso tiene una
amplitud unitaria y una duracin a < T.
Solucin: Aplicando la Ec. (5.70), se tiene
F1 ( s ) =

f (t )e

st

dt

= e st dt =
0

(1 e )
s

y por tanto,
F (s) =

1 1 e a s
s 1 e T s

as

313

5.8 Aplicacin de la Transformada de Laplace a Ecuaciones Diferenciales Ordinarias

En esta seccin se usan transformadas de Laplace para resolver ecuaciones diferenciales lineales con
coeficientes constantes. Se supone siempre que todas las ecuaciones son vlidas para t 0 y las
soluciones se determinan para diferentes formas de excitacin.
Una ecuacin diferencial lineal de orden n con coeficientes constantes es una ecuacin de la forma
an y ( ) ( t ) + an 1 y (
n

n 1)

( t ) + L + a1 y ( t ) + a0 y ( t ) = x ( t )

(5.71)

donde x(t), la excitacin, es una funcin conocida y a0, a1, , an son constantes dadas.
Una solucin de (5.71) es cualquier funcin y(t) que satisfaga la ecuacin. Como se ver, la Ec.
(5.71) tiene muchas soluciones. Sin embargo, su solucin es nica si se especifican los valores iniciales
de y(t) y sus primeras n 1 derivadas:
y (0) = y0 , y (0) = y1 , K , y (

n 1)

(0) = yn 1

(5.72)

Estos valores se denominan condiciones iniciales.


Una solucin particular es una solucin y(t) que satisface unas condiciones iniciales especficas. Si no
se especifican los valores iniciales, entonces y(t) es una solucin general. As que una solucin general
es una familia de soluciones que depende de los n parmetros y0, y1, , yn1.
A una ecuacin diferencial se le puede dar una interpretacin de sistema. En esta interpretacin, la
Ec. (5.71) especifica un sistema con entrada (excitacin) x(t) y salida (respuesta) y(t). La salida as
especificada, y(t), es la solucin nica de la Ec. (5.71) bajo las condiciones iniciales especificadas.
El estado inicial del sistema es el conjunto (5.72) de condiciones iniciales. La respuesta de estado
cero del sistema es la solucin, y(t) = y(t), de (5.71) con cero condiciones iniciales:
( n1)

y ( 0 ) = y ( 0 ) = L y

(0) = 0

(5.73)

La respuesta de entrada cero, y(t) = y(t). Es la solucin de (5.71) cuando x(t) = 0. Es decir, la
respuesta de entrada cero y(t) es la solucin de la ecuacin homognea
an y ( ) ( t ) + an 1 y ( n 1 ) ( t ) + L + a1 y ( t ) + a0 y ( t ) = 0
n

(5.74)

La aplicacin de la transformada de Laplace para resolver la Ec. (5.71) comprende los siguientes pasos:
st

1. Se multiplican ambos lados de la ecuacin por e

y se integra de cero a infinito. Puesto que la

ecuacin es vlida para t 0, resulta la ecuacin

(n)
st
an y ( t ) + L +a0 y ( t ) e dt =

x (t ) e

st

dt

(5.75)

Se supone que todas las funciones son transformables en el sentido de Laplace. Ello implica que el
lado derecho es igual a la transformada X(s) de la funcin conocida x(t), y el lado izquierdo puede
expresarse en trminos de la transformada Y(s) de y(t) y de las condiciones iniciales (5.73).
2. Se resuelve la ecuacin en la transformada Y(s) resultante.

314

3. Se determina la transformada inversa y(t) de Y(s) usando fracciones parciales u otros mtodos de

inversin.
A continuacin se ilustra el mtodo con varios ejemplos.

Ejemplo 22

Resolver la ecuacin diferencial


a1 y ( t ) + a0 y ( t ) = x ( t )
sujeta a la condicin inicial y(0) = y0.
Tomando transformadas en ambos lados se obtiene
a1 [ sY ( s ) y0 ] + a0 Y ( s ) = X ( s )

Por tanto,
Y (s) =

ay
X ( s)
+ 1 0
a1s + a0 a1s + a0

As que Y(s) = Y+Y, donde


Y ( s ) =

X (s)

a1 s + a0

es la respuesta de estado cero y


Y =

1
s + a0 a1

y0

es la respuesta de entrada cero. Su inversa es la exponencial


y = y0 e s1t

donde s1 = a 0 /a 1 .
Si, por ejemplo, a0 = 1, a1 = 2, x(t) =8t y y(0) = 5, entonces la ecuacin es
y (t ) + 2 y (t ) = 8 t ,

y (0) = 5,

y su ecuacin transformada es
Y (s) =

8 s2
s+2

5
s+2

2
7

+
s2 s s + 2

La solucin completa es
y ( t ) = 4 t 2 + 7 e 2 t ,

(t 0)

315

Ejemplo 23

Resolver la ecuacin diferencial


d2 y
dt 2

+4

dy
dt

+ 5 y = 5u ( t )

sujeta a las condiciones


y ( 0 ) = 1,

dy
dt

=2
t =0

La transformacin de Laplace de esta ecuacin diferencial produce


5
s 2 Y ( s ) s y (0) y (0) + 4 [ sY ( s ) y (0) ] + 5Y ( s ) =
s

y al incluir las condiciones iniciales, se obtiene


Y ( s ) ( s 2 + 4 s + 5) =

5
s

+s+6

o
Y (s) =

s2 + 6 s + 5
s ( s 2 + 4 s + 5)

Desarrollando ahora en fracciones parciales,


1
j
j
Y (s) = +
+
s s + 2 j1 s + 2 + j1
y tomando la transformada inversa da la solucin
y ( t ) = 1 + 2 e 2 t sen t ,

t 0

Ejemplo 24

Determine la solucin de la ecuacin diferencial


y ( t ) y ( t ) 6 y ( t ) = 2
sujeta a las condiciones
y ( 0 ) = 1,

y (0) = 0

Aplicando la transformacin a ambos lados de la ecuacin diferencial, se obtiene la ecuacin


algebraica

316

sY ( s ) s sY ( s ) + 1 6Y ( s ) =

2
s

Por tanto,

(s

s 6)Y ( s ) =

s2 s + 2

s2 s + 2

o
Y (s) =

s ( s 3)( s + 2)

s 3

C
s+2

Evaluando los coeficientes, se encuentra que


Y (s) =

11

3 s 15 s 3

4 1
5 s+2

y la solucin y(t) es
1 8
4
y ( t ) = + e3t + e 2 t ,
3 15
5

t0

Ejemplo 25

Determine la solucin del sistema de ecuaciones diferenciales


dy1
dt
dy2
dt

+ 20 y1 10 y2 = 100 u ( t )
+ 20 y2 10 y1 = 0

sujeto a las condiciones iniciales y1(0) = 0 y y2(0) = 0.


Las ecuaciones transformadas son
( s + 20) Y1 ( s ) 10Y2 ( s ) =

100Y1 ( s ) + ( s + 20) Y2 ( s ) = 0
Resolviendo este sistema, se obtiene

100
s

317

100 ( s + 20 )

Y1 ( s ) =

s ( s + 40 s + 300)
1000
2

Y2 ( s ) =

s ( s + 40 s + 300)
2

=
=

20 1
3 s
10 1
3 s

5
s + 10
5
s + 10

3 s + 30
5

3 s + 30

y la solucin es
5
5 e 10 t e30 t ,
3
3
10
5
y2 ( t ) = 5 e 10 t + e 30 t ,
3
3
y1 ( t ) =

20

t0
t0

5.9 La Convolucin

La operacin de convolucin encuentra aplicaciones en muchos campos, incluyendo la teora de redes


elctricas y controles automticos. Una aplicacin sobresaliente es la que permite evaluar la respuesta
de un sistema lineal a una excitacin arbitraria cuando se conoce la respuesta al impulso [respuesta
cuando la excitacin es un impulso unitario ( t ) ].
Sean las dos funciones f1(t) y f2(t) transformables en el sentido de Laplace y sean F1(s) y F2(s) sus
transformadas respectivas. El producto de F1(s) y F2(s) es la transformada de Laplace de la convolucin
de f1(t) y f2(t); es decir,

L { f ( t )} = F ( s ) = F1 ( s ) F2 ( s )
t

(5.76)

f ( t ) = f1 ( t ) f 2 ( t ) = f1 ( ) f 2 ( t ) d = f1 ( t ) f 2 ( ) d
0

(5.77)

Las integrales en la Ec. (5.77) se conocen como integrales de convolucin y el asterisco (*) indica la
operacin de convolucin. De acuerdo con la relacin f ( t ) = f1 ( t ) f 2 ( t ) , se observa que
F ( s ) = L { f1 ( t ) f 2 ( t )} = L { f 2 ( t ) f1 ( t )}
= F1 ( s ) F2 ( s )

(5.78)

As que la transformada inversa del producto de las transformadas F1(s) y F2(s) se determina mediante
la convolucin de las funciones f1(t) y f2(t) usando cualquiera de las frmulas en la Ec. (5.77)
(obsrvese que la convolucin es conmutativa).
Para deducir estas relaciones, observe que la transformada F(s) = F1(s)F2(s) se puede expresar como
un producto de las integrales que definen sus transformadas de Laplace en la forma
F (s) =

la cual puede expresarse como

f ( t ) e st dt =

t
st
0 0 f1 ( t ) f 2 ( ) d e dt

318

st
0 0 f1 ( t ) u ( t ) f 2 ( ) d e dt

F (s) =

puesto que u(t ) = 0 para > t. Intercambiando el orden de integracin, se obtiene

F (s) =

f 2 ( ) f1 ( t ) u ( t ) e st dt d
0

Definiendo ahora
x=t
se tiene que

s x +
F ( s ) = f 2 ( ) f1 ( x ) u ( x ) e ( ) dx d
0

Pero u(x) hace cero el valor de la integral entre corchetes para x < 0, y por tanto

F (s) =

s x +
f 2 ( ) f1 ( x ) e ( ) dx d
0

la cual puede ser expresada como el producto de dos integrales:

s
=

F ( s ) f 2 ( ) e d f1 ( x ) e sx dx = F2 ( s ) F1 ( s )
0
0

o tambin
t

F2 ( s ) F1 (s)

(t ) f2 ( ) d

(5.79)

la que demuestra la validez de una de las Ecs. (5.77). Si se intercambian f1(t) y f2(t), se puede utilizar un
proceso similar para derivar la otra ecuacin en (5.77).
A continuacin se mostrar mediante un ejemplo, que la convolucin se puede interpretar de acuerdo
con cuatro pasos: (1) reflexin, (2) traslacin, (3) multiplicacin y (4) integracin.
Ejemplo 26

En este ejemplo, sean F1(s) = 1/s y F2 ( s ) = 1 ( s + 1) , de manera que f1(t) = u(t) y f 2 ( t ) = e t u ( t ) . Se


desea determinar la convolucin de f1(t) y f2(t); es decir, se desea hallar
t

f ( t ) = f1 ( t ) f 2 ( t ) = u ( t ) e d
0

Los pasos para aplicar la convolucin a estas dos funciones se ilustran en la Fig. 5.9, en la cual f1(t) y
f2(t) se muestran en la (a) y f1() y f2() en (b). En (c) se han reflejado las funciones respecto de la lnea

319

t = 0 y en (d) se ha trasladado algn valor tpico de t. En (e) se ha efectuado la multiplicacin indicada


dentro de la integral de las Ecs. (5.77). La integracin del rea sombreada da un punto de la curva f(t)
para el valor seleccionado de t. Al efectuar todos los pasos anteriores para diferentes valores de t, se
obtiene la respuesta f(t), tal como se seala en (f) de la misma figura.

f1(t)

f2(t)

(a)

f1()

f2()

0
(b)

f1()

f1()

f2()

(c)
f1(t )

f2(t )

(d)
f1(t )f2()

f1()f2(t )

0
(e)

f(t) = f1(t)* f2(t)

0
(f)

Figura 5.9

320

Para este ejemplo, la integracin de la Ec. (5.79) es sencilla y da


t

f ( t ) = e d = 1 e t
0

que es, por supuesto, la transformada inversa del producto F1(s)F2(s),


1
1
1 1
f ( t ) = L1
=L

s s + 1
s ( s + 1)
= 1 e t
Ejemplo 27

Como otro ejemplo, considere ahora la transformada


F (s) =

1
( s 2 + a 2 )2

En este caso se puede tomar


F1 ( s ) = F2 ( s ) =

a s + a2
2

de manera que
f1 ( t ) = f 2 ( t ) =

1
a

sen at

y, por tanto,

1
= 2 sen at sen at
2
2
2
( s + a ) a
1 t
= 2 sen a sen a ( t ) d
a 0

L1

1
2a 2

( sen at at cos at )

321

5.10 Propiedades de la Integral de Convolucin

Ahora se derivarn algunas propiedades de la integral de convolucin.


Propiedad 1 La operacin de convolucin es conmutativa, distributiva y asociativa:
f1 ( t ) f 2 ( t ) = f 2 ( t ) f1 ( t )

(a )

f ( t ) [ f1 ( t ) + f 2 ( t ) +L + f k ( t ) ] = f ( t ) f1 ( t ) + f ( t ) f 2 ( t ) + L + f ( t ) f k ( t )

f1 ( t ) [ f 2 ( t ) f 3 ( t ) ] = [ f1 ( t ) f 2 ( t ) ] f 3 ( t )

(b )

(5.80)

(c )

Solamente se verificar la relacin (5.80) (c), dejando las otras dos como ejercicios. Sean G1 ( s ) y
G2 ( s ) las transformadas de Laplace de las funciones g1 ( t ) = f 2 ( t ) f 3 ( t ) y g 2 ( t ) = f1 ( t ) f 2 ( t ) ,
respectivamente. Por el teorema de convolucin sabemos que
G1 ( s ) = F2 ( s ) F3 ( s ),

G2 ( s ) = F1 ( s ) F2 ( s )

(5.81)

donde Fi ( s ) (i = 1, 2, 3) denota la transformada de Laplace de f i (t ) . Esto da

L { f1 ( t ) [ f 2 ( t ) f3 ( t ) ] } = L { f1 ( t ) g1 ( t ) } = F1 ( s ) G1 ( s )
= F1 ( s ) F2 ( s ) F3 ( s ) = G2 ( s ) F3 ( s ) = L { g 2 ( t ) f3 ( t ) }

(5.82)

= L { [ f1 ( t ) f 2 ( t ) ] f3 ( t ) }

Tomando la transformada de Laplace inversa de ambos lados produce la identidad deseada (5.80) (c).
Propiedad 2 Si las funciones f1 (t ) y f 2 (t ) son diferenciables para t > 0 y continuas para t = 0,
entonces su convolucin es diferenciable para t > 0:

df ( t )
dt

f
1

( )

df 2 ( t )

dt

(5.83)

df1 ( t )

dt

=
0

d + f1 ( t ) f 2 (0)

f 2 ( ) d + f1 (0) f 2 ( t )

t>0

Para demostrar esto, aplicamos la regla de Leibnitz para diferenciar dentro de una integral, la cual
dice que si
b(t )

h(t ) =

g (t , ) d

(5.84)

a(t )

donde a(t) y b(t) son funciones diferenciables de t y g ( t , ) y g ( t , ) t son continuas en t y ,


entonces

322

dh ( t )
dt

b(x)

g ( t , )
t

d + g (t , b)

db ( t )
dt

g (t , a )

da ( t )
dt

(5.85)

a(x)

Aplicando (5.85) a la ecuacin de definicin de la integral de convolucin con h ( t ) = f ( t ) ,


g ( t , ) = f1 ( ) f 2 ( t ) o f1 ( t ) f 2 ( ) , a = 0 y b = t+, se obtiene la relacin (5.83).
Observe que la Ec. (5.83) no necesita realmente la hiptesis de que ambas f1 ( t ) y f 2 ( t ) sean
diferenciables. De hecho, si cualquiera de las funciones es diferenciable y la otra continua, entonces su
convolucin es diferenciable. Desde el punto de vista de la operacin de convolucin, la Ec. (5.83)
puede escribirse tambin como
df ( t )
dt

= f1 ( t )

df 2 ( t )
dt

+ f1 ( t ) f 2 (0) =

df1 ( t )
dt

f 2 ( t ) + f1 (0) f 2 ( t )

(5.86)

Propiedad 3 Sea
f ( t ) = f1 ( t ) f 2 ( t )

df ( t )
dt

= f1 ( t )

df 2 ( t )
dt

+ f1 ( t ) f 2 (0) =

df1 ( t )
dt

f 2 ( t ) + f1 (0) f 2 ( t )

y escriba
g1 ( t ) = f1 ( t T1 ) u ( t T1 ),

T1 0

(5.87)

g 2 ( t ) = f 2 ( t T2 ) u ( t T2 ),

T2 0

(5.88)

g ( t ) = f ( t T1 T2 ) u ( t T1 T2 )

(5.89)

donde u(t) denota la funcin escaln unitario. Entonces


g ( t ) = g1 ( t ) g 2 ( t )

(5.90)

Esta propiedad expresa que si las funciones f1 ( t ) y f 2 ( t ) son retrasadas por T1 y T2 segundos,
respectivamente, entonces la convolucin de las dos funciones retrasadas es igual a la convolucin de
las funciones originales, retrasada por T1 + T2 segundos. La demostracin de esta propiedad se deja para
el lector.
5.11 Ecuaciones Diferenciales e Integrales

Con la ayuda de la propiedad de convolucin se pueden resolver algunos tipos de ecuaciones integrodiferenciales no homogneas, lineales y con coeficientes constantes. Se darn algunos ejemplos.

Ejemplo 28. Determine la solucin general de la ecuacin diferencial

y ( t ) + k 2 y ( t ) = f ( t )

(5.91)

323

en trminos de la constante k y la funcin f(t).


Suponiendo que todas las funciones en (5.91) son transformables, la ecuacin transformada es
s 2 Y ( s ) s y (0) y '(0) + k 2 Y ( s ) = F ( s )

donde y(0) y y(0) son, por supuesto, las condiciones iniciales. De aqu se obtiene
Y (s) =

k s +k
2

F ( s ) + y (0)

s
s +k
2

y (0)

s + k2
2

y por tanto,
y (t ) =

( sen kt ) f ( t ) + y (0) cos kt +

y '(0)
k

sen kt

Esta solucin general de la Ec. (5.91) puede entonces escribirse en la forma


y (t ) =

1
k

f ( ) sen k ( t ) d + C

cos kt + C2 sen kt

donde C1 y C2 son constantes arbitrarias.


Ejemplo 29. Resuelva la ecuacin integral
t

y ( t ) = at + y ( ) sen ( t ) d
0

Esta ecuacin se puede escribir en la forma


y ( t ) = at + y ( t ) sen t
y, transformando ambos miembros, se obtiene la ecuacin algebraica
Y (s) =

a
s2

+Y (s)

1
s2 + 1

cuya solucin es
1 1
Y (s) = a 2 + 4
s
s
y por tanto,
1
y (t ) = a t + t3
6

La ecuacin integral general del tipo de convolucin tiene la forma

324

y (t ) = f (t ) + g (t ) y ( ) d

(5.92)

donde las funciones f(t) y g(t) son dadas y y(t) debe determinarse. Puesto que la ecuacin transformada
es

Y ( s ) = F ( s ) + G ( s )Y ( s )
la transformada de la funcin buscada es
Y (s) =

F (s)

(5.93)

1 G ( s)

Si la Ec. (5.92) es modificada reemplazando y(t) por combinaciones lineales de y(t) y sus derivadas,
la transformada de la ecuacin modificada sigue siendo una ecuacin algebraica en Y(s).

Ahora se proceder a resolver la ecuacin de estado para sistemas LIT estudiada en el Captulo 2,
utilizando la transformada de Laplace.
Aplicando la transformacin de Laplace a la ecuacin

dx ( t )
dt

= A x ( t ) + Bu ( t ) , con condicin inicial

x(0), se obtiene

sX ( s ) x (0) = AX ( s ) BU ( s )
o

X ( s )[ sI A ] = x (0) + BU ( s )
de donde

X ( s ) = [ sI A ]1 [ x (0) + BU ( s )]
= ( s )[ x (0) + BU ( s )]
por lo que

x ( t ) = L1 { [ sI A ]1 [ x (0) + BU ( s )] }

(5.94)

donde (s) = [sI A]1 es la matriz resolvente. Se debe observar que (t) = L 1 {(s)} = eA t . En la
seccin anterior ya vimos que la matriz (t) se conoce como la matriz de transicin y ms adelante se
darn algunas de sus propiedades.

Ejemplo 30. Resolver el sistema


0 6
0
x& =
x + u ( t ),

1 5
1

1
x (0) =
2

Tomando u(t) = 1 y ejecutando las operaciones indicadas en la Ec. (5.94), obtenemos

325

1
[ sI A ] = s
0

0 0 6 s

=
1 1 5 0

0 0 6 s

=
s 1 5 1

6
s + 5

de donde
s +5
s + 5 6 ( s + 2 )( s +3 )
( s ) = [ sI A ] = 2
=

s ( s + 2)(1s +3 )
s + 5 s + 6 1

(s+2)(s+3)

6
(s+2)(s+3)

y
1 0
+ 1
s
2
( s + 2 )( s + 3 )
s

( s + 2s )(+5s +3 )
X ( s ) = 1
( s + 2 )( s +3 )

6
( s +1)( s + 2 )

o
s 2 + 17 s + 6
6
1 s ( s + 2)( s + 3)
( s +1 )( s + 2 )

1 =
s

2
s
2
+
( s + 2 )( s + 3 )

s
( s + 2)( s + 3)

( s + 2s )(+5s +3 )
X ( s ) = 1
( s + 2 )( s +3 )

y por tanto,
X1 ( s ) =

s 2 + 17 s + 6
s ( s + 2)( s + 3)

K1
s

K2
s+2

1 12
12
= +

s+3 s s+2 s+3


K3

x1 ( t ) = 1 + 12 e 2 t 12 e3 t

X2 (s) =

2s
( s + 2)( s + 3)

K1
s+2

K2
s+3

4
s+2

6
s+3

x2 ( t ) = 4 e 2 t + 6 e 3 t

Ejemplo 31. Resolver el sistema


1 0
2
(
t
)
+
x& ( t ) =
x

3 ,
0 2

5
x (0) =
1

Procediendo igual que en el Ejemplo 30, se obtiene


s + 1 0
[ sI A ] =
,
0 s+2

s +1 0

[ sI A ]1 =
1

0 s+2

326

y
1
2 5s + 2
s + 1 0 5 + s s ( s + 1)

=
X( s ) =

1 3
s+3

0
1
+

s+2 s s ( s + 2)
y por tanto,

X1 ( s ) =

5s + 2
s ( s + 1)
s+3

s s +1
1.5 0.5
=

X2 (s) =
s ( s + 2)
s s+2

x1 ( t ) = 2 + 3 e t

x2 ( t ) = 1.5 0.5 e 2 t

Ejemplo 32. Resolver el sistema


2
4
0
x& ( t ) =
x(t ) + ,

1 2
2

3
x (0) =
1

Aqu
s + 4
[ sI A ] =
1

2
s + 2

s + 2 2

s + 6 s + 10 1 s + 4
1

y
3s2 + 8 s + 4
3

1
1
s + 2 2
s

=
X( s ) = 2
2

s + 6 s + 10 1 s + 4 1 + s 2 + 6 s + 10 s 2 + 3 s + 8
s

X1 ( s ) =

3s2 + 8 s + 4

s ( s + 6 s + 10 )

K1 = 0.4,

K2 =

K1
s

K2

K3

s +3 j s +3+ j

3( 3 + j ) 2 + 8( 3 + j ) + 4
( 3 + j ) j 2

= 1.703 40.236 = K 3

x1 ( t ) = 0.4 + 3.406 e 3 t sen( t + 130.24 )

327

X2 (s) =

s2 + 3s + 8
s ( s 2 + 6 s + 10)

K1

K2

s +3 j

K3
s +3+ j

K 2 = 1.204 85.24 = K 3

K1 = 0.8,

x2 ( t ) = 0.8 + 2.41e 3 t sen( t + 175.24 )

Ejemplo 33
Resolver el sistema
1
1
(
t
)
+
x
0 t ,
3

0
x& ( t ) =
2

0
x (0) =
2

Procedemos igual que en los ejemplos previos y obtenemos:


1
s
sI A =
,
2 s + 3

( s +1s)(+3s + 2 )
X( s) = 2
( s +1 )( s + 2 )
X1 ( s ) =

( s +1s)+(3s + 2 )
[sI A ] =
2
( s +1 ) ( s + 2 )

s
( s +1 )( s + 2 )

1
(s+1)(s+2)

2 s 2 + s + 3
0 + 1 s 2 ( s +1)( s + 2

s = 2 s + 2
s

( s +1 )( s + 2 )
2 + 0 s 2 ( s +1)( s + 2 )

-1
(s+1)(s+2)

2 s 2 + s + 3
s 2 ( s + 1)( s + 2)

1.5 1.75 1.75

+
s
s+2
s2

x1 ( t ) = 1.5 t 1.75 + 1.75 e 2 t

X2 (s) =

2 s3 + 2
s ( s + 1)( s + 2)
2

1
s

1.5
s

3.5
s+2

x2 ( t ) = t 1.5 + 3.5 e 2 t

5.12 Polos y Ceros de la Transformada


Usualmente, la transformada F(t) de una funcin f(t) es una funcin racional en s, es decir,
F (s) =

a0 s m + a1 s m 1 + L + am 1 s + am
b0 s n + b1 s n 1 + L + bn 1 s + bn

a0 ( s z1 )( s z2 )L ( s zm )
b0 ( s p1 )( s p2 )L ( s pn )

(5.95)

Los coeficientes ak y bk son constantes reales y m y n son enteros positivos. La funcin F(s) se
denomina una funcin racional propia si n > m, y una funcin racional impropia si n m. Las races

328

del polinomio del numerador, zk se denominan los ceros de F(s) porque F(s) = 0 para esos valores de s.
De igual forma, las races del polinomio del denominador, pk, se denominan los polos de F(s) ya que
ella se hace infinita para esos valores de s. En consecuencia, los polos de F(s) estn fuera de la regin
de convergencia (RDC) ya que F(s), por definicin, no converge en los polos. Por otra parte, los ceros
pueden estar dentro y fuera de la RDC, Excepto por un factor de escala, a0/b0, F(s) puede especificarse
completamente por sus polos y ceros, lo que nos da una forma compacta de representar a F(s) en el
plano complejo.
Tradicionalmente se usa una para indicar la ubicacin de un polo y un para indicar cada cero.
Esto se ilustra en la Fig.5.10 para la funcin dada por
F (s) =

2s + 4
s + 4s + 3
2

=2

s+2

Re( s ) > 1

( s + 1)( s + 3)
j

3 2

Figura 5.10

329

Problemas
1. Determinar la transformada de Laplace de las siguientes funciones:

(a) f ( t ) = 2sen 12 t

(b) f ( t ) = 3 e 2 t sen 3t

(c) f ( t ) = 4 e t sen 5 t + t 2 cos 5 t

(d) f ( t ) = t 3 e2 t sen t

2. Determine la transformada de Laplace de las funciones en las grficas.


5

3
6

0
5

1
(a)

(b)
2

1
0
1

1 2

3
t

1 2

2
(c)

(d)

3. Encontrar la transformada de Laplace inversa de las siguientes funciones usando desarrollo en

fracciones parciales.

(a )

F (s) =

(c) F ( s ) =
(e) F ( s ) =
(g) F ( s ) =

2 s2 + 3s + 4
s3 + 5 s 2 + 4 s
4s 2 + 6 s + 10

s3 + 5 s 2 + 8 s + 4
2s 2 + 5 s + 4
s3 + 7 s 2 + 16 s + 12
14s+42
s 4 + 8 s 3 + 14 s 2 + 12 s

(b) F ( s ) =
(d) F ( s ) =

6s 2 + 10 s + 4
3s 2 + 24 s + 48
2 s2 + 6 s + 8

(f) F ( s ) =
(g) F ( s ) =

(s

+ 2 s + 5)
8 ( s+10 )

s ( s 2 + 10 s + 20 )
12s+48
s 4 + 6 s 3 + 16 s 2 + 56 s + 80

4. Resolver las siguientes ecuaciones diferenciales mediante la aplicacin directa de la transformacin

de Laplace.

330

d2x

dx

+ 5 x = 2 t + 6.
x ( 0 ) = 2, x ( 0 ) = 1.
dt
dt
d2x
dx
x ( 0 ) = 2, x ( 0 ) = 8.
(b) 2 2 + 12 + 10 x = 6 cos 4 t ,
dt
dt
(a)

+4

d3 x

d2x

dx

+ 3 x = 4,
x ( 0 ) = 1, x ( 0 ) = 2, x ( 0 ) = 5.
dt
dt
d2x
dx
+ 7 2 + 12 = 2.
x ( 0 ) = 3, x ( 0 ) = 1, x ( 0 ) = 2.
(d)
3
dt
dt
dt
(c)

dt
d3x

+3

5. Halle las transformadas inversas de Laplace de las siguientes funciones:

(a)

1 + e s

(b)

s ( s + 3)

e 2 s s e s
s2 + 6 s + 5

6. Halle las transformadas de Laplace de las funciones ilustradas en la figura.


f(t)

f(t)

3
2

2
1
0

(a)

(b)

7. Determine la transformada inversa de las siguientes funciones usando la integral de convolucin.

(a) F ( s ) =
(c) F ( s ) =

(b) F ( s ) =

s ( s + 4)
2

10 s

(d) F ( s ) =

s + 2s + 4s + 8
3

1
s ( s + 4)

2
s + 6 s 2 + 13 s
3

8. Demuestre que la solucin del sistema de ecuaciones diferenciales

x ' ( t ) 2 y ( t ) = f ( t ) , x ( t ) -y ( t ) +y ( t )=0

bajo las condiciones x ( 0 ) = x ( 0 ) = y ( 0 ) = y ( 0 ) = 0, tal que f(0) = 0, es


t

x ( t ) = f ( ) d 2 f ( ) cos ( t ) d ,
0

y ( t ) = f ( ) cos ( t ) d
0

9. Resuelva el siguiente sistema de ecuaciones y verifique su resultado:

331

x ( t ) + y ( t ) = f ( t ) , y ( t ) + x ( t ) = 1, x ( 0 ) = 1, y ( 0 ) = 0.
10. Resuelva por y(t) y verifique su solucin:
t

y ( ) d y (t ) = t ,

y (0) = 2

11. Halle la solucin de la ecuacin integral


t

y ( t ) = a sen bt + c y ( ) sen b ( t ) d
0

(a) cuando b2 > bc; (b) cuando b = c.


12. Sea F(s) la transformada de Laplace de f(t). Demuestre que

f (t )
= F ( s ) ds
t s

L
13. Demuestre que para real y positiva

t d n t n 2 t
Le
e
dt n n !

( s )n
=
n +1
( s + )

14. Usando la propiedad demostrada en el Problema 12, determine las transformadas de Laplace de las

siguientes funciones:
a) t 1 cos 0 t

(b) t 1 (1 e t )

(c) t 1 (senh t + cosh t )

CAPTULO SEIS

LA TRANSFORMADA Z
6.1 Introduccin
En el Captulo 5 se introdujo la transformada de Laplace. En este captulo presentamos la transformada
Z, que es la contraparte en tiempo discreto de la transformada de Laplace. La transformada Z puede
considerarse una extensin o generalizacin de la transformada de Fourier discreta, as como la
transformada de Laplace puede considerarse como una extensin de la transformada de Fourier. La
transformada Z se introduce para representar seales en tiempo discreto (o secuencias) en el dominio de
la variable compleja z, y luego se describir el concepto de la funcin del sistema para un sistema LIT
en tiempo discreto. Como ya se estudi, la transformada de Laplace convierte ecuaciones ntegrodiferenciales en ecuaciones algebraicas. Ahora veremos que, en una forma similar, la transformada Z
convierte ecuaciones en diferencias recursivas en ecuaciones algebraicas, simplificando as el anlisis
de los sistemas en tiempo discreto.
Las propiedades de la transformada Z son muy parecidas a las de la transformada de Laplace, de
manera que los resultados de este captulo son semejantes a los del Captulo 5 y, en algunos casos, se
puede pasar directamente de la una transformada a la otra. Sin embargo, veremos algunas diferencias
importantes entre las dos transformadas.
6.2 La Transformada Z
En la Seccin 4.9 vimos que para un sistema LIT de tiempo discreto con respuesta al impulso dada
por h[n], la salida y[n] del sistema a una entrada exponencial de la forma zn viene dada por

y[ n ] = T {z n } = H ( z ) z n

(6.1)

donde

H (z) =

h[ n ] z

(6.2)

n =

Para z = e j con real (es decir, con z = 1 ), la sumatoria en la Ec. (6.2) corresponde a la
transformada de Fourier discreta de h[n]. Lo anterior nos conduce a la definicin siguiente para la
transformada Z de una secuencia x[n].

334

6.2.1. Definicin

La funcin H(z) en la Ec. (6.2) se conoce como la transformada Z de h[n]. Para una seal en tiempo
discreto general x[n], la transformada Z, X[z], se define como

X (z) =

x[ n ] z

(6.3)

n =

La variable z es generalmente compleja y en forma polar se expresa como

z = r e j

(6.4)

donde r es la magnitud de z y es el ngulo de z. La transformada Z definida en la Ec. (6.3) con


frecuencia se denomina la transformada Z bilateral para distinguirla de la transformada Z unilateral,
estudiada ms adelante en la Sec. 6.7, y la cual se define como

X (z) =

x[ n ] z

(6.5)

n =0

Claramente, ambas transformadas son equivalentes slo si x[n] = 0 para t < 0 (causal). En lo que sigue,
omitiremos la palabra bilateral excepto cuando sea necesario para evitar ambigedades.
Igual que en el caso de la transformada de Laplace, algunas veces la Ec. (6.3) se considera como un
operador que transforma una secuencia x[n] en una funcin X(z), simblicamente representada por
X ( z ) = Z { x [ n ]}

(6.6)

Las funciones x[n] y X(z) forman un par de transformadas Z; esto se denotar por

x[ n ]

X (z)

(6.7)

que significa que las funciones x[n] y X(z) forman un par de transformadas Z, es decir F(z) es la
transformada Z de x[n].

Existen varias relaciones importantes entra la transformada Z y la transformada de Fourier. Para


estudiar estas relaciones, consideremos la expresin dada por la Ec. (6.5) con la variable z en forma
polar. En trminos de r y , la Ec. (6.3) se convierte en
X ( r e j ) =

x[ n ](r e )
j

(6.8)

n =

o, en forma equivalente,
X ( r e j ) =

{ x[ n ] r } e
n

j n

(6.9)

n =

A partir de esta ltima ecuacin vemos que X ( re j ) es la transformada de Fourier de la secuencia x[n]
multiplicada por una exponencial real rn, es decir,
X ( re j ) = F { x [ n ] r n }

(6.10)

335

La funcin de ponderacin exponencial rn puede estar decreciendo o creciendo con n creciente,


dependiendo de si r es mayor o menor que la unidad. En particular, se observa que para r = 1, la
transformada Z se reduce a la transformada de Fourier, vale decir,
X (z)

z = e j

= F { x[ n ] }

(6.11)

6.2.2. La Regin de Convergencia de la Transformada Z

Como en el caso de la transformada de Laplace, la banda de valores de la variable compleja z para la


cual converge la transformada Z se denomina la regin de convergencia (RDC). En el caso de tiempo
continuo, la transformada de Laplace se reduce a la transformada de Fourier cuando la parte real de la
variable de transformacin es cero; es decir, la transformada de Laplace se reduce a la de Fourier en el
eje imaginario. Como contraste, la reduccin de la transformada Z a la de Fourier se produce cuando la
magnitud de la variable de transformacin z es igual a la unidad. De manera que la reduccin se
produce en el contorno del plano z complejo correspondiente a un crculo de radio unitario, el cual
jugar un papel importante en la discusin de la regin de convergencia de la transformada Z.
La suma en la Ec. (6.3) tiene potencias de z positivas y negativas. La suma de las potencias negativas
converge para z mayor que alguna constante r1, y la suma de las potencias positivas converge para
z menor que alguna otra constante r2. Esto muestra que la regin de existencia de la transformada z

bilateral es un anillo cuyos radios r1 y r2 dependen de x[n].


Para ilustrar la transformada Z y la RDC asociada, consideremos algunos ejemplos.
Ejemplo 1. Considere la secuencia

x[ n ] = a n u [ n ]

(6.12)

a real

Entonces, por la Ec. (6.3), la transformada Z de x[n] es

X (z) =

a u[ n ] z
n

n =

( az )

1 n

n =0

Para que X(z) converja se requiere que

a z 1

<

n =0

En consecuencia, la RDC es la banda de valores para los cuales az 1 < 1 o, en forma equivalente,
z > a , para cualquier valor finito de a. Entonces

X (z) =

( az )
n =0

1 n

1
1 az 1

z > a

(6.13)

Alternativamente, multiplicando el numerador y el denominador de la Ec. (6.12) por z, podemos


escribir X(z) como

336

X (z) =

z > a

za

(6.14)

Ambas formas de X(z) en las Ecs. (6.12) y (6.13) son de utilidad dependiente de la aplicacin. De la Ec.
(6.13) vemos que X(z) es una funcin racional de z. En consecuencia, igual que con las transformadas
de Laplace racionales, puede caracterizarse por sus ceros (las races del polinomio del numerador) y sus
polos (las races del polinomio del denominador). De la Ec. (6.13) vemos que hay un cero en z = 0 y un
polo en z = a. La RDC y el diagrama de polos y ceros para este ejemplo se muestran en la Fig. 6-1. En
las aplicaciones de la transformada Z, al plano complejo se le refiere comnmente como el plano z.

Crculo

Im(z)

Im(z)

1 a

Re(z)

Re(z)

a>1

0<a<1

Im(z)

Im(z)

Re(z)

1 < a < 0

Re(z)

a < 1

Figura 6-1. RDC de la forma z > a

Ejemplo 2. Considere la secuencia

x [ n ] = a n u [ n 1]

(6.15)

De la Ec. (6.3), tenemos

X (z) =

a n u [ n 1] z n =

n =

a z
n

n =

( a z)
n =1

= 1

( a z)
n =0

337

Ahora bien,

(a z)
1

n =0

1 a z

a 1 z < 1 o

z < a

por lo que
X ( z ) = 1

1 a 1 z

a 1 z
1 a 1 z

z < a

za

(6.16)

La transformada Z, X(z) viene dada entonces por


X (z) =

1
1 az 1

z < a

(6.17)

Como lo indica la Ec. (6.16), X(z) tambin puede escribirse como


X (z) =

z
za

z < a

(6.18)

As pues, la RDC y la grfica de polos y ceros para este ejemplo se muestran en la Fig. 6-2.
Comparando las Ecs. (6.13) y (6.17) [o las Ecs. (6.14) y (6.18)], vemos que las expresiones algebraicas
de X(z) para dos secuencias diferentes son idnticas, excepto por las RDC. As que, igual que en la
transformada de Laplace, la especificacin de la transformada Z requiere tanto la expresin algebraica
como la regin de convergencia.
Im(z)

Im(z)

Re(z)

0<a<1

Re(z)

Im(z)

1 < a < 0

a>1

Im(z)

Re(z)

Re(z)

a < 1

Figura 6-2. RDC de la forma |z | < |a |.

Ejemplo 3. Una sucesin finita x[n] se define como x [ n ] 0, N1 n N 2 , donde N1 y N2 son finitos, y
x[n] = 0 para cualquier otro valor de n. Para determinar la RDC procedemos en la forma siguiente:

338

De la Ec. (6.3) se tiene


N2

X (z) =

x[ n ] z

(6.19)

n = N1

Para z diferente de cero o infinito, cada trmino en la Ec. (6.19) ser finito y por tanto X(z) converger.
Si N1 < 0 y N2 > 0, entonces la Ec. (6.19) incluye trminos con potencias de z tanto positivas como
negativas. Conforme z 0 , los trminos con potencias de z negativas se convierten en no acotados,
y conforme z , los trminos con potencias de z positivas se vuelven no acotados. Por tanto, la
RDC es todo el plano z excepto para z = 0 y z = . Si N1 0, la Ec. (6.19) contiene slo potencias
negativas de z, y por ende la RDC incluye z = . Si N2 0, la Ec. (6.19) contiene slo potencias
positivas de z y, por tanto, la RDC incluye el punto z = 0.

6.2.3. Propiedades de la Regin de Convergencia

Como vimos en los Ejemplos 1 y 2, la RDC de X(z) depende de la naturaleza de x[n]. Las propiedades
de la RDC se resumen a continuacin. Se entiende que X(z) es una funcin racional de z.
1. La RDC no contiene ningn polo.
2. Si x[n] es una secuencia finita, es decir, x[n] = 0 excepto en un intervalo finito N1 n N 2 , donde
N1 y N2 son finitos, y X(z) converge para algn valor de z, entonces la RDC es todo el plano z
excepto posiblemente z = 0 o z = .
3. Si x[n] es una secuencia lateral derecha, es decir, x[n] = 0 para n < N1 < , y X(z) converge para
algn valor de z, entonces la RDC es de la forma
z > rmx

> z > rmx

donde rmx es igual a la mayor magnitud de cualquiera de los polos de X(z). As pues, la RDC es el
exterior del crculo z = rmx en el plano z con la posible excepcin de z = .
4. Si x[n] es una secuencia lateral izquierda, es decir, x[n] = 0 para n > N2 > , y X(z) converge para
algn valor de z, entonces la RDC es de la forma
z < rmn

0 < z < rmn

donde rmn es igual a la menor magnitud de cualquiera de los polos de X(z). As pues, la RDC es el
interior del crculo z = rmn en el plano z con la posible excepcin de z = 0
5. Si x[n] es una secuencia bilateral, es decir, x[n] es una secuencia de duracin infinita que no es ni
lateral izquierda ni lateral derecha, y X(z) converge para algn valor de z, entonces la RDC es de la
forma
r1 < z < r2

donde r1 y r2 son las magnitudes de dos de los polos de X(z). As que la RDC es una regin anular
en el plano z entre los crculos z < r1 y z < r2 que no contienen polos.

339

Observe que la Propiedad 1 se deduce inmediatamente de la definicin de polos; es decir, X(z) es


infinita en un polo.
Ejemplo 4. Considere la secuencia
an
x[ n ] =
0

0 n N 1, a > 0
otros valores de n

Determinar X(z) y graficar sus polos y ceros.


De la Ec. (6.3), obtenemos
N 1

X (z) =

a z
n

n =0

N 1

(a z )

1 n

1 ( a z 1 )

n =0

1 a z 1

1 zN aN
z N 1

za

(6.20)

De la Ec. (6.20) vemos que hay un polo de orden (N 1) en z = 0 y un polo en z = a. Como x[n] es una
secuencia de longitud finita y es cero para n < 0, la RDC es z > 0 (la RDC no incluye el origen
porque x[n] es diferente de cero para algunos valores positivos de n}. Las N races del polinomio del
numerador estn en
zk = ae j ( 2 k N )

k = 0, 1, K , N 1

(6.21)

La raz en k = 0 cancela el polo en z = a. Los ceros restantes de X(z) estn en


zk = ae j ( 2 k N )

k = 1, K , N 1

(6.22)

El diagrama de polos y ceros con N = 8 se muestra en la Fig. 6-3.


Im(z)
Plano z
Polo de orden
(N 1)

Polo-cero se
cancelan

Re(z)

Figura 6-3 Diagrama de polos y ceros con N = 8.

En general, si x[n] es la suma de varias secuencias, X(z) existe solamente si existe un conjunto de
valores de z para los cuales convergen las transformadas de cada una de las secuencias que forman la
suma. La regin de convergencia es entonces la interseccin de las regiones de convergencia
individuales. Si no hay una regin de convergencia comn, entonces la transformada X(z) no existe.

340

6.3
Transformadas Z de Secuencias Importantes
6.3.1. La Secuencia Impulso unitario [n]

De la definicin dada en la Ec. (1.79) y la Ec. (6.3), tenemos

X (z) =

[ n ] z

= z 0 = 1

(6.23)

n =

y, por consiguiente,
[ n ] 1

todo z

(6.24)

z k

(6.25)

Es fcil demostrar que


[n k ]

6.3.2. La Secuencia Escaln Unitario u[n]

Haciendo a = 1 en las Ecs. (6.12)) a (6.14), obtenemos


u[ n ]

1 z

z >1

z 1

(6.26)

6.3.3. Funciones Sinusoidales

Sea x [ n ] = cos 0 n . Escribiendo x[n] como


x[ n ] =

(e
2

j0 n

+ e j 0 n

y usando el resultado dado en la Ec. (6.14), se obtiene que


X (z) =

z
j0

2 z e
2 z e j0
z ( z cos 0 )

(6.27)

z 2 2 z cos 0 + 1

En forma similar, la transformada Z de la secuencia x [ n ] = sen 0 n est dada por


X (z) =

z sen 0
z 2 2 z cos 0 + 1

(6.28)

341

6.3.4. Tabla de Transformadas Z

En la tabla al final del captulo se tabulan las transformadas Z de algunas secuencias encontradas con
frecuencia.
6.4 Propiedades de la Transformada Z

A continuacin se presentan algunas propiedades bsicas de la transformada Z y la verificacin de


algunas de esas propiedades. Estas propiedades hacen de la transformada Z una valiosa herramienta en
el estudio de seales y sistemas de tiempo discreto.
6.4.1

Linealidad

Si x1[n] y x2[n] son dos secuencias con transformadas X1(z) y X2(z) y regiones de convergencia R1 y
R2, respectivamente, es decir,
x1 [ n ]

X1 ( z )

RDC = R1

x2 [ n ]

X2 ( z)

RDC = R2

entonces
a1 x1 [ n ] + a2 x [ n ]

a1 X 1 ( z ) + a2 X 2 [ n ]

R R1 R2

(6.29)

donde a1 y a2 son constantes arbitrarias, es decir, la transformada Z de una combinacin lineal de


secuencias es igual a la combinacin lineal de las transformadas Z de las secuencias individuales.
La demostracin de esta propiedad se obtiene directamente de la definicin de la transformada Z, Ec.
(6.3). Como se indica, la RDC de la combinacin es al menos la interseccin de R1 y R2.
Ejemplo 5. Halle la transformada Z y dibuje el diagrama de polos y ceros (partes b y c) con la RDC
para cada una de las secuencias siguientes:

(a) x [n ] = 2 [n ] + 3 [n 2] [n 5] .
n

1
1
(b) x [ n ] = u [ n ] + u [ n ]
2
3
1
1
(c) x [ n ] = u [ n ] + u [ n 1]
2
3
(a) A partir del par [n k ] z k y de la Ec. (6.29), se sigue que la transformada Z de la sucesin
dada es
X ( z ) = 2 + 3 z 2 z 5

342

(b) De la tabla de transformadas al final del captulo, se obtiene


n

1
u[ n ]
2

1
u[ n ]
3

z >

1
2

z >

z 13

(6.30)

2
1

(6.31)

Vemos que la RDC en las Ecs. (6.30) y (6.31) se solapan y, de esta manera, usando la propiedad
de linealidad, se obtiene
X (z) =

z
z

1
2

z
z

1
3

2 z ( z 125 )

z >

( z )( z )
1
2

1
3

1
2

(6.32)

De la Ec. (6.32) vemos que X(z) tiene dos ceros en z = 0 y z = 5/12 y dos polos en z = y z = 1/3,
y que la RDC es z > 12 como se dibuja en la Fig. 6-4.

Im(z)


1
3

Re(z)

Figura 6-4

(c) De la parte (b)


n

1
u[ n ]
2

z >

1
2

1
2

y de la tabla de transformadas,
n

1
u [ n 1]
3

z
z

1
3

z <

1
3

(6.33)

Vemos que las RDC de estas dos ltimas relaciones no se solapan y no hay una RDC comn; as
pues, x[n] no tiene transformada Z.
Ejemplo 6. Sea

x[ n ] = a

a>0

(6.34)

343

Hallar X(z) y dibujar el diagrama de polos y ceros y la RDC para a < 1 y a > 1.
La sucesin x[n] se dibuja en la Fig. 6-5.
x[n] = a

x[n] = a

0<a<

a>1
1

0
(a)

0
(b)

Figura 6-5

Puesto que x[n] es una secuencia bilateral, podemos expresarla como


x [ n ] = a n u [ n ] + a n u [ n 1]

(6.35)

De la tabla de transformadas

an u [ n ]

a n u [ n 1]

z >a

za

z <

z 1 a

(6.36)
1

(6.37)

Si a < 1, vemos que la RDC en las Ecs. (6.36) y (6.37) se solapan y entonces

X (z) =

z
za

z
z 1 a

a2 1

( z a ) ( z 1 a )

a< z <

1
a

(6.38)

De la Ec. (6.38) vemos que X(z) tiene un cero en el origen y dos polos en z = a y z = 1/a y que la RDC
es a < z < 1 a , como se ilustra en la Fig. 6-6. Si a > 1, vemos que las RDC en las Ecs. (6.36) y (6.37)
no se solapan y no hay una RDC comn y, por tanto, x[n] no tendr una X(z).
Im(z)
Crculo
unitario

Figura 6-6

1/a Re(z)

344

6.4.2

Desplazamiento (Corrimiento) en el Tiempo o Traslacin Real

Si

x[ n ]

X (z)

RDC = R

entonces
x [ n n0 ]

R = R {0 < z < }

z n0 X ( z )

(6.39)

Demostracin:

Por la definicin en la Ec. (6.3),

Z {x [ n n0 ] } =

x[n n ] z

n =

Mediante el cambio de variables m = n n0, obtenemos

Z {x [ n n0 ] } =

x[m] z

( m + n0 )

m =

= z n0

x[m] z

= z n0 X ( z )

m =

Debido a la multiplicacin por z n0 , para n0 > 0, se introducen polos adicionales en z = 0 y se


eliminarn en z = . En la misma forma, si n0 < 0, se introducen ceros adicionales en z = 0 y se
eliminarn en z = . Por consiguiente, los puntos z = 0 y z = pueden aadirse o eliminarse de la
RDC mediante corrimiento en el tiempo. De este modo tenemos entonces que
x [ n n0 ]

z n0 X ( z )

R = R {0 < z < }

donde R y R' son las RDC antes y despus de la operacin de desplazamiento. En resumen, la RDC de
x [ n n0 ] es la misma que la RDC de x[n] excepto por la posible adicin o eliminacin del origen o
infinito.
Casos especiales de la propiedad definida en la Ec. (6.39) son los siguientes:
x [ n 1]
x [ n + 1]

z 1 X ( z )
zX ( z )

R = R {0 < z < }
R = R {0 < z < }

(6.40)
(6.41)

Debido a estas ltimas relaciones, z1 a menudo se le denomina el operador de retardo unitario y z se


conoce como el operador de avance(o adelanto) unitario. Observe que en la transformada de Laplace
los operadores s1 = 1/s y s corresponden a integracin y diferenciacin en el dominio del tiempo,
respectivamente.

345

6.4.3

Inversin en el Tiempo

Si la transformada Z de x[n] es X(z), es decir,

x[ n ]

X (z)

RDC = R

x[ n ]

1
X
z

entonces
R =

(6.42)

En consecuencia, un polo (o cero) en X(z) en z = zk se mueve a 1/zk luego de inversin en el tiempo. La


relacin R' = 1/R indica la inversin de R, reflejando el hecho de que una secuencia lateral derecha se
convierte en lateral izquierda si se invierte el tiempo, y viceversa. La demostracin de esta propiedad se
deja como ejercicio.
6.4.4

Multiplicacin por z0n o Corrimiento en Frecuencia

Si
x [ n] X ( z )

RDC = R

entonces
z0n x [ n ]

z
X
z0

R = z0 R

(6.43)

Demostracin

Por la definicin dada en la Ec. (6.3), tenemos que

Z {z x [ n ] } =
n
0

z
x[ n ]
z0
n =

( z x[ n ] ) z
n
0

n =

z
=X
z0

Un polo (o cero) en z = zk en X(z) se mueve a z = z0zk luego de la multiplicacin por z0n y la RDC se
expande o contrae por el factor z0 , y la propiedad especificada por la Ec. (6.42) queda demostrada.
Un caso especial de esta propiedad es la relacin

e j 0 n x [ n ]

X e j 0 z

R = R

(6.44)

En este caso especial, todos los polos y ceros son simplemente rotados en un ngulo 0 y la RDC no
cambia.
Ejemplo 7. Determine la transformada Z y la RDC asociada para cada de las secuencias siguientes:

(a) x[n] = [n n0]

346

(b) x[ n ] = u [ n n0 ]
(c) x[ n ] = a n +1 u [ n + 1]
(d) x[ n ] = u [ n ]

Solucin
(a) De la Ec. (6.24)

[ n ]

toda z

Aplicando la propiedad de corrimiento en el tiempo (6.38), se obtiene


0 < z , n0 > 0

z n0

[ n n0 ]

z < , n0 < 0

(6.45)

(b) De la Ec. (6.26),


z

u[n]

z >1

z 1

Aplicando de nuevo la propiedad de desplazamiento en el tiempo, obtenemos


u [ n n0 ]

z n0

z ( n0 1)

z 1

1< z <

z 1

(6.46)

(c) De las Ecs. (6.12) y (6.14) se tiene que


an u [ n ]

z > a

za

y por la Ec. (6.41)


n +1

a u[n]

z
za

z2
za

a < z <

(6.47)

(d) De la Ec. (6.26)


u[n]

z >1

z 1

y por la propiedad de inversin en el tiempo (6.42), obtenemos


u[n]

6.4.5

1z

1 z 1 1 z

z <1

Multiplicacin por n (o Diferenciacin en el Dominio de z)

Si x[n] tiene transformada z con RDC = R, es decir,

(6.48)

347

X (z)

x[ n ]

RDC = R

entonces

nx [ n ]

dX ( z )

R = R

dz

(6.49)

Demostracin

Partiendo de la definicin (6.3)

X (z) =

x[ n ] z

n =

y diferenciando ambos lados con respecto a z, se obtiene


dX ( z )
dz

n x[ n ] z

n 1

n =

por lo que

dX ( z )
{ nx[n] } z n = Z { nx[n] }
z
=
dz
n =

de donde sigue la Ec. (6.49).


Por diferenciacin sucesiva con respecto a z, la propiedad especificada por la Ec. (6.49) puede ser
generalizada a

Z {n x [ n ] } = ( z )
k

dk
dz k

(6.50)

X (z)

Ejemplo 8. Determine la transformada Z de la secuencia x[ n] = na n u[n] .

De las Ecs. (6.12) y (6.14) sabemos que


an u[ n ]

z
za

z > a

(6.51)

Usando la propiedad de la multiplicacin por n dada por la Ec. (6.49), se obtiene


na n u [ n ]

6.4.6

d z
az

=
dz z a ( z a ) 2

z > a

Acumulacin

Si la secuencia x[n] tiene transformada Z igual a X(z) con regin de convergencia R, es decir,

x[ n ]

X (z)

RDC = R

(6.52)

348

entonces
1

x(k )

1 z

k =

Observe que la expresin

n
k =

X (z) =

z
z 1

R R { z > 1}

X (z)

(6.53)

x [ k ] es la contraparte en tiempo discreto de la operacin de

integracin en el dominio del tiempo y se denomina acumulacin. El operador comparable de la


transformada de Laplace para la integracin es 1/s. La demostracin de esta propiedad se deja como
ejercicio.
6.4.7

Convolucin

Si x1[n] y x2[n] son tales que


x1 [ n ]

X1 ( z )

RDC = R1

x2 [ n ]

X2 ( z)

RDC = R2

entonces la transformada de la convolucin de estas secuencias es dada por


x1 [ n ] x2 [ n ]

R R { z > 1}

X1 ( z ) X 2 ( z )

(6.54)

Esta relacin juega un papel importante en el anlisis y diseo de sistemas LIT de tiempo discreto, en
analoga con el caso de tiempo continuo.
Demostracin

De la Ec. (2.9) sabemos que

y [ n ] = x1 [ n ] x2 [ n ] =

x [k ] x [n k ]
1

k =

entonces, por la definicin (6.3)

=
x
[
k
]
x
[
n
k
]
z
x
[
k
]
x2 [ n k ] z n

1
2
1
n = k =
k =

n =

Y (z) =

Observando que el trmino entre parntesis en la ltima expresin es la transformada Z de la seal


desplazada, entonces por la propiedad de corrimiento en el tiempo (6.39) tenemos

x1 [ k ] z k X 2 ( z ) =
x1 [ k ] z k
k =
k =

Y (z) =

X 2 ( z ) = X1 ( z ) X 2 ( z )

con una regin de convergencia que contiene la interseccin de la RDC de X1(z) y X2(z). Si un cero de
una de las transformadas cancela un polo de la otra, la RDC de Y(z) puede ser mayor. As que
concluimos que
x1 [ n ] x2 [ n ]

X1 ( z ) X 2 ( z )

R R { z > 1}

349

6.5

La Transformada Z Inversa

La inversin de la transformada Z para hallar la secuencia x[n] a partir de su transformada Z X(z) se


denomina la transformada Z inversa y simblicamente se denota como
x [ n ] = Z 1 { X ( z )}

(6.55)

6.5.1. Frmula de Inversin

Igual que en el caso de la transformada de Laplace, se tiene una expresin formal para la transformada
Z inversa en trminos de una integracin el plano z; es decir,
x[ n ] =

1
2 j

X (z)z

n 1

dz

(6.56)

donde C es un contorno de integracin con sentido antihorario que encierra el origen. La evaluacin
formal de la Ec. (6.55) requiere de la teora de una variable compleja.
6.5.2. Uso de Tablas de Pares de Trasformadas Z

En el segundo mtodo para la inversin de X(z), intentamos expresar X(z) como una suma
X ( z ) = X1 ( z ) + X 2 ( z ) + L + X n ( z )

(6.57)

donde X1(z), X2(z), , Xn(z) son funciones con transformadas inversas conocidas x1[n], x2[n], ,
xn[z], es decir, estn tabuladas (tabla al final del captulo). Entonces, de la propiedad de linealidad de la
transformada Z se deduce que la transformada Z inversa viene dada por
x[ n ] = x1 [ n ] + x2 [ n ] + L + xn [ z ]

(6.58)

6.5.3. Expansin en Series de Potencias

La expresin que define la transformada Z [Ec. (6.3)] es una serie de potencias donde los valores de la
secuencia x[n] son los coeficientes de zn. As pues, si se da X(z) como una serie de potencias en la
forma

X (z) =

x[n ] z

n =

(6.59)

= L + x [ 2] z 2 + x [ 1] z + x [0] + x [1] z 1 + x [2] z 2 + L

podemos determinar cualquier valor particular de la secuencia determinando el coeficiente de la


potencia apropiada de z1. Puede pasar que este enfoque puede no proporcione una solucin en forma

350

cerrada pero es muy til para una secuencia de longitud finita donde X(z) puede no tener una forma ms
sencilla que un polinomio en z1. Para transformadas Z racionales, se puede obtener una expansin en
serie de potencias mediante divisin de polinomios, como se ilustrar con algunos ejemplos.
Ejemplo 9. Hallar la transformada Z inversa de

X ( z ) = z 2 ( 1 12 z 1 )( 1 z 1 )( 1 + 2 z 1 ) ,

0< z <

Multiplicando los factores en esta ecuacin, podemos expresar X(z) como

X ( z ) = z 2 + 12 z 52 + z 1
Entonces, por la definicin (6.3),
X ( z ) = x[ 2] z 2 + x[ 1] z + x[0] + x[1] z 1
y obtenemos
x [ n ] = {L ,1, 12 , 52 , 1, 0,L}

Ejemplo 10. Usando la tcnica de la expansin en serie de potencias, determine la transformada Z


inversa de las transformadas siguientes:

(a) x ( z ) =

1
1 a z 1

z <a

(b) X ( z ) = log
,
1
1 a z

(c) X ( z ) =

z >a

z <

2 z 3 z +1
2

1
2

(a) Como la RDC es z < a , es decir, el interior de un crculo de radio a, x[n] es una secuencia lateral
derecha. Por tanto, debemos dividir de manera que obtengamos una serie en potencias de z en la
forma siguiente. Multiplicando el numerador y el denominado de X(z) por z, tenemos
X (z) =

z
za

y procediendo a la divisin, obtenemos


X (z) =

1
1 a z

z
za

y por la definicin (6.3), obtenemos

= a 1 z a 2 z 2 a 3 z 3 L a k z k L

351

x[ n ] = 0

n0

x[ 1] = a 1 , x [ 2] = a 2 , x [ 3] = a 3 , x[ k ] = a k , L
de modo que
x[ n ] = a n u [ n 1]
(b) La expansin en serie de potencias para log (1 r ) es dada por

log (1 r ) =

n r

r <1

n =1

Ahora
1

= log (1 a z 1 )
X ( z ) = log
1
1 a z

z > a

Puesto que la RDC es z > a , es decir, az 1 < 1 , entonces X(z) tiene la expansin en serie de
potencias

X (z) =

( a z 1 ) =
n

n =1

n a z
n

n =1

de la cual podemos identificar x[n] como


(1 n ) a n
x[ n ] =
0

n 1
n0

o
x[ n ] =

1
n

a n u [ n 1]

(c) Puesto que la RDC es z < 12 , x[n] es una secuencia lateral izquierda. As pues, debemos dividir
para obtener una serie de potencias en z. Procedemos entonces a la divisin para obtener
z
1 3 z + 2 z2

= z + 3 z 2 + 7 z 3 + 15 z 4 + L

Entonces
X ( z ) =L + 15 z 4 + 7 z 3 + 3 z 2 + z
y, por la definicin (6.3), se obtiene

x[ n ] = K ,15, 7,3,1, 0

352

6.5.4. Expansin en Fracciones Parciales

Igual que en el caso de transformada de Laplace inversa, el mtodo de expansin en fracciones


parciales generalmente proporciona el mtodo ms til para hallar la transformada Z inversa,
especialmente cuando X(z) es una funcin racional de z. Sea
X (z) =

N (z)

=K

D( z)

( z z1 )( z z2 )L ( z zm )

(6.60)

( z p1 )( z p2 )L ( z pn )

Suponiendo que n m, es decir, el grado de N(z) no puede exceder el grado de D(z), y que todos los
polos son sencillos, entonces la fraccin X(z)/z * es una funcin propia y puede ser expandida en
fracciones parciales
X (z)
z

c0
z

c1
z p1

c2
z p2

+L +

cn
z pn

c0

ck

k =1

z pk

(6.61)

donde
c0 = X ( z )

ck = ( z pk )

z =0

X (z)
z

(6.62)
z = pk

Por lo tanto, obtenemos


X ( z ) = c0 + c1

z
z p1

+ c2

z
z p2

+ L + cn

z
z pn

= c0 +

k =1

z
z pk

(6.63)

Determinando la RDC para cada trmino en la Ec. (6.63) a partir de la RDC total de X(z) y usando una
tabla de transformadas, podemos entonces invertir cada trmino, produciendo as la transformada Z
inversa completa.
Si m > n en la Ec. (6.60), entonces se debe aadir un polinomio en z al lado derecho de la Ec. (6.63),
cuyo orden es (m n). Entonces, para m > n, la expansin en fracciones parciales tendran la forma
mn

q =0

k =1

z pk

X ( z ) = bq z q + ck

(6.64)

Si X(z) tiene polos de orden mltiple, digamos que pi es el orden del polo mltiple con multiplicidad
r, entonces la expansin de X(z)/z consistir de trminos de la forma
1
z pi

( z pi )

+L+

( z pi )

(6.65)

donde
z k =

1 dk
r X (z)
( z pi )
k
k ! dz
z

(6.66)
z = pi

La expansin es de X(z)/z debido a que las fracciones individuales tienen como denominador el factor de la forma
1 az 1 y no ( z a ) como aparece en la expansin.

353

Ejemplo 11

(a) Usando expansin en fracciones parciales, resuelva de nuevo el problema en el Ejemplo 10(c)
X (z) =

z <

2 z2 3 z +1

1
2

Usando expansin en fracciones parciales, obtenemos


X (z)
z

1
2 z 3 z +1
2

2 ( z 1) ( z 12 )

c1
z 1

c2
z 12

donde
c1 =

1
2( z

1
2

=1

c2 =

2 ( z 1)

z =1

= 1
z =1 2

Por tanto,
X (z) =

z
z 1

z <

1
2

1
2

(b) Si
F (z) =

30 z 2 12 z
6 z2 5 z +1

entonces
F (z)
z

30 z 12
6 z2 5 z +1

3
z 1 2

2
z 1 3

Por tanto,
F (z) =

3z
z 1 2

2z
z 1 3

y
n

1
1
f [ n ] = 3 + 2
2
3

Ejemplo 12. Hallar la transformada Z inversa de


X (z) =

z
( z 1) ( z 2) 2

Usando expansin en fracciones parciales, tenemos que

z >2

354

X (z)
z

1
( z 1) ( z 2)

c1
z 1

z2

2
( z 2) 2

(6.67)

donde
c1 =

1
( z 2)

=1

2 =

z =1

1
z 1

=1
z =2

Sustituyendo estos valores en la Ec. (6.67), se obtiene


1
( z 1) ( z 2) 2

1
z 1

1
z2

1
( z 2) 2

Haciendo z = 0 en la expresin anterior (la expresin es vlida para cualquier valor de z), se tiene que

1
4

= 1

1
2

1
4

de donde 1 = 1 y entonces
z
z
z

+
z 1 z 2 ( z 2) 2

X (z) =

z> 2

Como la RDC es z > 2 , x[n] es una secuencia lateral derecha y de la tabla de transformadas
obtenemos
x [ n ] = ( 1 2n + n 2n 1 ) u [ n ]
Ejemplo 13. Calcule la transformada Z inversa de

X (z) =

z3 5 z2 + z 2

z <1

( z 1) ( z 2)

Si expandimos el denominador obtenemos


X (z) =

z3 5 z 2 + z 2
( z 1) ( z 2)

z3 5 z2 + z 2
z2 3 z + 2

que es una funcin racional impropia; realizamos la divisin y tenemos


X (z) = z 2+

7 z + 2
z 3z + 2
2

= z 2+

7 z + 2
( z 1) ( z 2)

Ahora, sea
X1 ( z ) =

Entonces

7 z + 2
( z 1) ( z 2)

355

X1 ( z )
z

7 z + 2
z ( z 1) ( z 2)

5
z 1

6
z2

y
5z

X1 ( z ) = 1 +

z 1

6z

z2

Por consiguiente
X ( z ) = z 1 +

5z
z 1

6z

z<1

z2

Puesto que la RDC de X(z) es z < 1 , x[n] es una secuencia lateral izquierda y de la tabla de
transformadas, obtenemos
x [ n ] = [ n + 1] [ n ] + 5 u [ n 1] 6 2n u [ n 1]
= [ n + 1] [ n ] + ( 5 6 2n 1 ) u [ n 1]

Ejemplo 14. Hallar la transformada Z inversa de


X (z) =

z >3

z 3

X(z) puede escribirse como


X (z) =

z
= 4 z 1

z 3
z 3
4

z >3

Como la RDC es z > 3 , x[n] es una secuencia lateral derecha y de la tabla de transformadas
obtenemos

3n u [ n ]

z
z 3

Usando la propiedad de corrimiento en el tiempo, se tiene


3n 1 u [ n 1]

1
z
z 1
=
z 3 z 3

y concluimos que
x [ n ] = 4 (3) n 1 u [ n 1]
Ejemplo 15. Hallar la transformada Z inversa de
X (z) =

(1 az 1 )

z2
( z a )2

z > a

356

De la Ec. (6.68) se sabe que


na n 1 u [ n ]

z
( z a )2

z > a

(6.69)

Ahora, X(z) puede escribirse como


z

X (z) = z
2
( z a)

z > a

y aplicando la propiedad de corrimiento en el tiempo a la Ec. (6.69), obtenemos


x [ n ] = ( n + 1) a n u [ n + 1] = ( n + 1) u [ n ]
ya que x[1] = 0 en n = 1.
6.6

La Funcin del Sistema: Sistemas LIT en Tiempo Discreto

6.6.1. La Funcin del Sistema

En la Sec. 2.3 se demostr que la salida y[n] de un sistema LIT de tiempo discreto es igual a la
convolucin de la entrada x[n] con la respuesta al impulso h[n]; es decir,

y [ n ] = x [ n ] h[ n ]

(6.70)

Aplicando la propiedad de convolucin de la transformada Z, Ec. (6.54), obtenemos

Y ( z) = X (z)H (z)

(6.71)

donde Y(z), X(z) y H(z) son las transformadas Z de y[n], x[n] y h[n], respectivamente. La Ec. (6.71)
puede expresarse como
H (z) =

Y (z)
X (z)

(6.72)

La transformada Z H(z) de h[n] se conoce como la funcin del sistema (o la funcin de transferencia
del sistema). Por la Ec. (6.72), la funcin del sistema H(z) tambin puede ser definida como la relacin
entre las transformadas Z de la salida y[n] y de la entrada x[n]. La funcin del sistema caracteriza
completamente al sistema. La Fig. 6-9 ilustra la relacin de las Ecs. (6.70) y (6.71).
h[n]

y[n] = x[n] h[n]

x[n]

Y ( z) = X ( z) H ( z)

X(z)
H[z]

Figura 6-9 Respuesta al impulso y funcin del sistema.

357

Ejemplo 16. La entrada x[n] y la respuesta al impulso h[n] de un sistema LIT de tiempo discreto
vienen dados por

x[ n ] = u [ n ]

h[ n ] = n u [ n ]

0 < <1

Determine la salida y[n] usando la transformada Z.


De la tabla de transformadas obtenemos
x[ n ] = u [ n ]

h[ n ] = n u [ n ]

X (z) =

z > 1

z 1
z

H (z) =

z >

Entonces, por la Ec. (6.71),


Y (z) = X (z) H (z) =

z2

z > 1

( z 1) ( z )

Usando ahora expansin en fracciones parciales, se obtiene


Y (z)
z

z
( z 1) ( z )

c1

z 1

c2
z

donde
c1 =

z =1

c2 =

z
z 1

=
z =

de manera que
Y (z) =

z >1

1 z 1 1 z

cuya transformada Z inversa es


y[ n ] =

1
1

u[n]

1 n 1
n u [ n ] =
1
1

u [ n ]

Ejemplo 17. La respuesta al escaln s[n] de un sistema LIT de tiempo discreto viene dada por

x [ n ] = n u [ n ],

0 < <1

Determine la respuesta al impulso h[n] del sistema.


Sean x[n] y y[n] la entrada y salida del sistema. Entonces
x[ n ] = u [ n ]

y[ n ] = n u [ n ]

X (z) =

z >1

z 1

Y ( z) =

z
z

z >

358

Entonces, por la Ec. (6.71),


H (z) =

Y (z)
X (z)

z 1

z >

Usando expansin en fracciones parciales, se obtiene


H (z)
z

z 1

1 1 1 1

z
z

z ( z )

o
H (z) =

z >

Tomando la transformada Z inversa, obtenemos


h[ n ] =

[ n ]

n u [ n ]

Cuando n = 0,
h[0] =

y por tanto
n=0

1
h[ n ] =
n 1
(1 )

n 1

por lo que h[n] puede escribirse como


h [ n ] = [ n ] (1 ) n 1 u [ n 1]
Ejemplo 18. Se tiene que la salida y[n] de un sistema LIT de tiempo discreto es 2 (

entrada x[n] es el escaln unitario u[n].


(a) Calcule la respuesta al impulso h[n] del sistema.
(b) Determine la salida y[n] cuando la entrada x[n] es

( 12 )

u[n] .

Solucin:
(a)
x[ n ] = u [ n ]

X (z) =

1
y[ n ] = 2 u[ n ]
3

z >1

z 1

Y (z) =

Usando expansin en fracciones parciales, se obtiene

2( z 1)
z

1
2

z >

1
3

1 n
3

u [ n ] cuando la

359

H (z)
z

2( z 1)

z ( z 13 )

6
z

4
z 13

y
H (z) = 6 4

z >

z 1

Tomando la transformada Z inversa, obtenemos


n

1
h[ n ] = 6 [ n ] 4 u [ n ]
3
(b)
n

1
x[ n ] = u [ n ]
2

X (z) =

z >

z 12

1
2

Entonces
Y (z) = X (z)H (z) =

2 z ( z 1)

( z )( z )
1
2

1
2

11
2

Usando expansin en fracciones parciales una vez ms, tenemos que


Y (z)
z

2( z 1)

( z 12 ) ( z 13 )

6
z 12

8
z 13

As que
Y ( z ) = 6

z
z

1
2

+8

z
z

1
3

z >

1
2

y la transformada Z inversa de Y(z) es


n
1 n
1
y [ n ] = 6 + 8 u [ n ]
2
3

6.6.2. Caracterizacin de Sistemas LIT en Tiempo Discreto

Muchas de las propiedades de los sistemas LIT de tiempo discreto puede asociarse ntimamente con las
caractersticas de la funcin de transferencia H(z) en el plano z y en particular con las ubicaciones de
los polos y la regin de convergencia (RDC).
1.

Causalidad

Para un sistema LIT de tiempo discreto, tenemos que

360

h[n] = 0

n<0

Como h[n] es una seal unilateral derecha, el requisito correspondiente sobre H(z) es que su RDC debe
ser de la forma
z > rmx

Es decir, la RDC es el exterior de un crculo que contiene todos los polos de H(z) en el plano z. En
forma similar, si el sistema es anticausal, es decir,
h[n] = 0

n0

entonces h[n] es una seal lateral izquierda y la RDC de H(z) debe ser de la forma
z < rmn

Es decir, la RDC es el interior de un crculo que no contiene polos de H(z) en el plano z.


2.

Estabilidad

En la Sec. 2.5 se estableci que un sistema LIT de tiempo discreto es estable (estabilidad de entrada
acotada-salida acotada, que se abreviar EASA) si y slo si [Ec. (2.53)]

h[ n ] <

n =

El requisito correspondiente sobre H(z) es que su RDC contenga el crculo unitario, es decir, z = 1 .

Ejemplo 19. Si un sistema LIT de tiempo discreto es estable (entrada acotada-salida acotada, EASA),
demuestre que su funcin del sistema H(z) debe contener el crculo unitario, es decir, z = 1 .

Un sistema LIT de tiempo discreto tiene estabilidad EASA si y slo si su respuesta al impulso h[n] es
absolutamente sumable, es decir,

h[ n ] <

n =

Ahora,

H (z) =

h[n] z

n =

Sea z = e j de manera que z = e j = 1 . Entonces


H ( e j ) =

h[n ]e

j n

n =

h[ n ]e

n =

j n

h[ n ] <

n =

361

En consecuencia, vemos que si el sistema es estable, entonces H(z) converge para z = e j . Es decir,
para LIT de tiempo discreto estable, la RDC de H(z) debe contener el crculo unitario z = 1 .

3.

Sistemas Causales y Estables

Si el sistema es causal y estable, entonces todos los polos de H(z) deben estar ubicado en el interior del
crculo unitario del plano z ya que la RDC es de la forma z > rmx , y como el crculo unitario es
incluido en la RDC, debemos tener rmx < 1 .

6.6.3. Funcin del Sistema para Sistemas LIT Descritos por Ecuaciones de Diferencias Lineales
con Coeficientes Constantes.

En la Sec. 2.9 se consider un sistema LIT de tiempo discreto para el cual la entrada x[n] y la salida
y[n] satisfacen la ecuacin de diferencias lineal con coeficientes constantes de la forma
N

a y[ n k ] = b x[ n k ]
k

k =0

(6.73)

k =0

Aplicando la transformada Z y usando las propiedades de corrimiento en el tiempo, Ec. (6.39), y de


linealidad, Ec. (6.29), de la transformada Z, obtenemos
N

ak z k Y ( z ) =

k =0

b z

X (z)

k =0

o
N

Y (z)

ak z k = X ( z )

k =0

b z

(6.74)

k =0

As pues,
M

Y (z)
H (z) =
=
X ( z)

b z

k =0
N

a z

(6.75)
k

k =0

Por tanto, H(z) siempre es racional. Observe que la RDC de H(z) no es especificada por la Ec. (6.75)
sino que debe inferirse con requerimientos adicionales sobre el sistema; requerimientos como la
causalidad o la estabilidad.
Ejemplo 20. Un sistema LIT de tiempo discreto causal es descrito por la ecuacin en diferencias
y[ n ]

3
4

y [ n 1] +

1
8

y [ n 2] = x[ n ]

(6.76)

362

donde x[n] y y[n] son la entrada y salida del sistema, respectivamente.


(a) Determine la funcin del sistema H(z).
(b) Halle la respuesta al impulso h[n] del sistema.
(c) Halle la respuesta al escaln s[n] del sistema.
(a) Tomando la transformada Z de la Ec. (6.75), se obtiene
Y (z)

1
z 1Y ( z ) + z 2 Y ( z ) = X ( z )
4
8

o
3 1 1 2

1 z + z Y ( z ) = X ( z )
4
8

As que
Y (z)

H (z) =

X ( z)

1
1 34 z 1 + 81 z 2

z2

z >

( z )( z )
1
2

1
8

z2
z 2 43 z + 81
1
2

(b) Usando expansin en fracciones parciales, se obtiene


H (z)
z

( z )( z )
1
2

1
4

2
z

1
2

1
z 14

y
H (z) = 2

z
z 12

z >

z 14

cuya transformada Z inversa es


1 n 1 n
h[ n ] = 2 u [ n ]
2 4

(c)
x[ n ] = u [ n ]

X ( z)

z
z 1

z >1

Entonces
Y ( z) = X (z)H (z) =

z3
( z 1) ( z 12 )( z 14 )

Usando de nuevo expansin en fracciones parciales, se obtiene

z >1

363

Y (z)
z

z2
( z 1) ( z

1
2

)( z )
1
4

83
z 1

2
z

1
2

13
z 14

o
Y (z) =

8 z
3 z 1

z
z 12

1 z
3 z 14

z >1

y la transformada Z inversa de Y(z) es


n
n
8
1 1 1
y[ n ] = s[ n ] = 2 + u [ n ]
3
2 3 4

Ejemplo 21. Considere un sistema LIT en el cual la entrada x[n] y la salida y[n] satisfacen la ecuacin
en diferencias lineal con coeficientes constantes
y[ n ]

1
y [ n 1] = x [ n ] + x[ n 1]
4
3

Aplicando la transformada Z en ambos lados de esta ecuacin y usando las propiedades de linealidad
y corrimiento en el tiempo, obtenemos
Y (z)

1
z 1Y ( z ) = X ( z ) + z 1 X [ n ]
4
3

o
1
1 + z 1
3
Y (z) =
X (z)
1 1
1 z
4
o
1
1 + z 1
3
=
H (z) =
1
X (z)
1 z 1
4
Y (z)

Ejemplo 22. Consideremos ahora un ejemplo de la aplicacin de la transformada Z a la solucin de una


ecuacin sencilla en comparacin con el uso de frmulas recursivas. Deseamos resolver la ecuacin

y [ n ] 3 y y [ n 1] = 6
con la condicin inicial y[1] = 4.
Podemos hallar y[n] en forma recursiva: haciendo n = 0, 1, 2, , obtenemos

y [0] = 18,

y [1] = 60,

y [2] = 186, etc.

364

Para determinar y[n] para cualquier n, usamos la transformada Z. Esto produce


Y ( z ) 3{ z 1Y ( z ) + y [ 1] } =

6z
z 1

Por tanto,
Y (z) =

18 z 2 12 z
( z 3) ( z 1)

21 z
z 3

3
z 1

y el resultado es
y [ n ] = 21( 3) 3
n

6.6.4. Interconexin de Sistemas

Para dos sistemas LIT (con respuestas al impulso h1[n] y h2[n], respectivamente) en cascada, la
respuesta al impulso total h[n] viene dada por
h [ n ] = h1 [ n ] h2 [ n ]

(6.77)

As que las funciones de los sistemas estn relacionadas por el producto


H ( z ) = H1 ( z ) H 2 ( z )

R R1 R2

(6.78)

En forma similar, la respuesta al impulso de una combinacin en paralelo de dos sistemas LIT est
dada por
h [ n ] = h1 [ n ] + h2 [ n ]

(6.79)

y
H ( z ) = H1 ( z ) + H 2 ( z )

R R1 R2

(6.80)

Ejemplo 23. Considere el sistema de tiempo discreto de la Fig. 6-10. Escriba una ecuacin de
diferencias que relacione la salida y[n] con la entrada x[n].

Suponga que la entrada al elemento de retardo unitario es q[n]. Entonces, de la Fig. 6-10 vemos que
q [ n ] = 2 q [ n 1] + x [ n ]
y [ n ] = q [ n ] + 3 q [ n 1]
Tomando la transformada Z de estas ecuaciones, se obtiene
Q ( z ) = 2 z 1 Q ( z ) + X ( z )
Y ( z ) = Q ( z ) + 3 z 1 Q ( z )
Reacomodando, obtenemos

365

x[n]

y[n]

q[n]
+

+
Retardo
unitario

+
3

q[n 1]

Figura 6-10

( 1 2 z )Q( z ) = X ( z )
( 1+ 3z )Q( z ) = Y ( z )
1

de donde
H (z) =

Y (z)
X (z)

1 + 3 z 1
1 2 z 1

Por lo tanto,

( 1 2 z )Y ( z ) = ( 1+ 3 z ) X ( z )
1

o
Y ( z ) 2 z 1Y ( z ) = X ( z ) + 3 z 1 X ( z )
Tomando ahora la transformada inversa y usando la propiedad de corrimiento en el tiempo, se obtiene
la ecuacin en diferencias para y[n]
y [ n ] 2 y [ n 1] = x [ n ] + 3 x [ n 1]

Ejemplo 24. Considere el sistema de tiempo discreto mostrado en la Fig. 6-11. Para qu valores de k
es el sistema estable EASA?
x[n]

+
k/2

a[n 1]

z1

q[n]

k/3

+ y[n]
Figura 6-11

366

En la figura se observa que


q[ n ] = x[ n ] +

y[ n ] = q[ n ] +

2
k

q [ n 1]

q [ n 1]

Tomando la transformada Z de las ecuaciones anteriores, se obtiene


Q( z) = X ( z) +

Y ( z) = Q( z) +

2
3

z 1 Q ( z )
z 1 Q ( z )

Reacomodando, tenemos que


k 1

1 z Q ( z ) = X ( z )
2

k 1

1 z Q ( z ) = Y ( z )
3

y de sta obtenemos
k 1
z
Y (z)
z+k 3
3
=
=
H (z) =
k
X (z)
z+k 2
1 + z 1
2
1+

z >

k
2

la cual muestra que el sistema tiene un cero en z = k/3 y un polo en z = k/2 y que la RDC es
z > k 2 . Entonces, como se mostr anteriormente, el sistema es estable EASA slo si k < 2 .

6.7

La Transformada Z Unilateral

6.7.1. Definicin

La transformada Z unilateral XI(z) de una secuencia x[n] se define como

XU [ n ] =

x[ n ]z

(6.81)

k =0

y difiere de la transformada bilateral en que la sumatoria se calcula para solamente n 0. As, la


transformada Z unilateral de x[n] puede considerarse como la transformada bilateral de x[n]u[n]. Como
x [ n ] u [ n ] es una secuencia lateral derecha, la RDC de XU(z) est siempre fuera de un crculo en el
plano z.

367

6.7.2. Propiedades Bsicas

La mayora de las propiedades de la transformada Z unilateral son las mismas que la de la transformada
Z bilateral. La transformada unilateral es til en el clculo de la respuesta de un sistema causal a una
entrada causal cuando el sistema es descrito por una ecuacin en diferencias lineal de coeficientes
constantes con condiciones iniciales diferentes de cero. La propiedad bsica de la transformada Z
unilateral que es de utilidad en esta aplicacin es la propiedad de corrimiento en el tiempo siguiente, la
cual es diferente de la misma propiedad para la transformada bilateral.
Propiedad de Corrimiento en el Tiempo

Si x[n] XU(z), entonces para m 0,


x [ n m ] z m X U ( z ) + z m +1 x [ 1] + z m + 2 x [ 2] + L + x [ m ]

(6.82)

x [ n + m ] z m X U ( z ) z m x [0] z m 1 x [1] L zx [ m 1]

(6.83)

6.7.3. La Funcin del Sistema

De manera similar al caso del sistema LIT de tiempo continuo, con la transformada Z unilateral, la
funcin del sistema H(z) = Y(z)/X(z) se define bajo la condicin de que el sistema est en reposo, es
decir, todas las condiciones iniciales son iguales a cero.
6.7.4. Valores Inicial y Final
Teorema del Valor Inicial

Sea x[n] una secuencia causal con transformada Z dada por X(z). Entonces
x [0] = lm X ( z )
z

(6.84)

que es el teorema del valor inicial para la transformada Z.


Como x[n] = 0 para n < 0, tenemos que

X (z) =

x[ n ] z

= x [0] + x [1] z 1 + x [2] z 2 + L

n =0

Conforma z , z 0 para n > 0, y da como resultado la Ec. (6.83).


Teorema del Valor Final

Sea x[n] una secuencia causal con transformada Z igual a X(z). Entonces, si X(z) es una funcin
racional con todos sus polos estrictamente en el interior del crculo unitario excepto posiblemente por
un polo de primer orden en z = 1, se tiene que

368

lm x [ N ] = lm ( 1 z 1 ) X ( z )

(6.85)

z 1

que es el teorema del valor final para la transformada Z.


De la propiedad de corrimiento en el tiempo, Ec. (6.86), tenemos

Z { x[ n ] x[ n 1] } = ( 1 z 1 ) X ( z )

(6.87)

El lado izquierdo de esta ltima ecuacin puede escribirse como

{ x [ n ] x [ n 1] } z n

= lm

n =0

{ x [ n ] x [ n 1] } z

n =0

Si ahora hacemos que z 1, entonces esta podemos escribir esta ecuacin como
lm ( 1 z 1 ) X ( z ) = lm
z 1

{ x [ n ] x [ n 1] } = lm x [ N ] = x [ ]
N

n =0

Ejemplo 25. Considere un sistema de tiempo discreto cuya entrada x[n] y salida y[n] estn relacionadas
por

y [ n ] ay [ n 1] = x [ n ],

a constante

Determine y[n] con la condicin auxiliar y[1] = y1 y x [ n ] = K bn u [ n ] .


Sea

y[ n ]

YI ( z )

Entonces, de la Ec. (6.82),


y [ n 1]

z 1YU ( z ) + y [ 1] = z 1YU ( z ) + y1

De la tabla de transformadas tenemos la relacin


x[ n ]

XU ( z ) = K

z > b

z b

Tomando la transformada Z unilateral de la Ec. (6.87), se obtiene


YU ( z ) a [YU ( z ) + y1 ] = K

z
z b

o
z
za

YU ( z ) = ay1 + K
z b
z
Entonces
YU ( z ) = a y1

z
za

+K

z2
( z a) ( z b)

369

y usando expansin en fracciones parciales, obtenemos


YU ( z ) = a y1

z
za

K
z
z
a
b

b a ( z b)
za

Tomando ahora la transformada Z inversa, se obtiene el resultado


b

y [ n ] = ay1 a n u [ n ] + K

bn u [ n ] K

ba
ba
n +1
b a
y1 a n +1 + K
u[n]
b a

an u [ n ]

n +1

Ejemplo 26. Para la ecuacin en diferencias


3 y [ n ] 4 y [ n 1] + y [ n 2] = x [ n ], con x [ n ] = ( 12 ) , y [ 1] = 1, y [ 2] = 2
n

Tomando la transformada Z unilateral de la ecuacin dada, obtenemos


3YU ( z ) 4 { z 1YU + y [ 1] } + { z 2 YU + z 1 y [ 1] + y [ 2] } = X U ( z )
Sustituyendo las condiciones auxiliares y[1] = 1, y[2] = 2, y XI(z) en la expresin anterior, se obtiene

(3 4 z

+ z 2 ) YI ( z ) = 2 z 1 +

z
z 12

o
3( z 1) ( z 13 )
z2

YU ( z ) =

3 z 2 2 z + 12
z ( z 12 )

Entonces
YU ( z ) =
=

z ( 3 z 2 2 z + 12 )

3( z 1) ( z 12 ) ( z 13 )
3 z
2 z 1

z
z

1
2

2 z 13

y, por tanto,
n

3 1 11
y[ n ] = +
2 2 23

n 2

Ejemplo 27. El sistema de la Fig. 6-12 consiste un elemento de retardo y un multiplicador. Tiene como
variable la entrada y[n] al elemento de retardo y su estado inicial es y[1] = 8. Determinaremos y[n]
para cualquier n 0 usando la transformada Z.

370

8
y[n]
4

1
2

y[n 1]

z 1

0.5

1 0 1 2 3 4

y[n]

Figura 6-12

Como vemos en el diagrama,


y[ n ] =

1
2

y [ n 1]

Aplicando la transformada Z en ambos lados de esta ecuacin, se obtiene


1

Y (z) =

{z

Y ( z ) + y [ 1] }

Por tanto,
Y (z) =

4z
z 0.5

y la transformada Z inversa produce la solucin


y [ n ] = 4 ( 0.5 )

la cual se indica en la Fig. 6-12.


6.8

La Transformada de Laplace y la Transformada Z

Si representamos la secuencia x[n] = x(nT) como un tren de impulsos separados por el intervalo de
tiempo T, el perodo de muestreo, el impulso del n-simo instante, (t nT), tiene el valor de
ponderacin x(nT). Por consiguiente, la relacin entre la secuencia x(nT) y la seal x*(t) se puede
expresar como

x *(t ) =

x ( nT ) ( t nT )

(6.88)

n =0

Tomando la transformada de Laplace de ambos lados de la Ec. (6.88), se obtiene

X *( s ) = L [ x *( t )] =

x{nT ) e

nTs

(6.89)

n+0

Comparando la Ec. (6.89) con la Ec. (6.81) para la transformada Z unilateral, vemos que esta ltima y
la transformada de Laplace se relacionan a travs de la equivalencia

z = ets

(6.90)

371

En verdad, la transformada Z unilateral definida por la Ec. (6.87) puede considerarse como un caso
especial cuando T = 1. En consecuencia, la definicin de la transformada Z unilateral se puede resumir
como
X ( z ) = L [ x ( kT )] = L [ x *( t )] = Z [ X *( z )]
= X *( s )

z = ets

(6.91)

372

Pares Ordinarios de Transformadas Z

X(z)

RDC

[n]

Toda z

u[n]

1
z
,
1
z 1
1 z

z >1

u[n 1]

1
z
,
1
z 1
1 z

z <1

[n m]

zm

Toda z excepto 0 si m > 0 o si m


<0

a n u[n]

1
z
,
1
za
1 az

z >a

a n u[ n 1]

1
z
,
1
za
1 az

z <a

na n u[ n]

az 1

z >a

x[n]

(1 az )

1 2

az 1

na n u[ n 1]

(1 az )

1 2

(n + 1)a nu[n]

(1 az )

1 2

(cos 0 n )u[n]

az

(z a )2

az

z <a

(z a )

z
,

z a

z >a

z 2 (cos 0 ) z

z >1

(sen 0 ) z
(2 cos 0 ) z + 1

z >1

z (2 cos 0 ) z + 1
2

(sen 0 n )u[n]
z

(r

cos 0 n u[ n]

z
0 n N 1
otros valores de n

z 2 (r cos 0 ) z

z >r

(r sen 0 ) z
(2r cos 0 ) z + r 2

z >r

z 2 (2r cos 0 ) z + r 2

r n (sen 0 n )u[n]

an

1 a N z N
1 az 1

z >0

373

Problemas

6.1 Halle las transformadas Z de las secuencias siguientes:

(a) x [ n ] = { 14 ,1. 15 }
(b) x[ n ] = 6 [ n + 5] 4 [ n 2]
n

1
(c) x [ n ] = 2 u [ n ] 3(2) n u [ n 1]
3
6.2 Dado que
X (z) =

z ( z 4)
( z 1) ( z 2) ( z 3)

Especifique todas las regiones de convergencia posibles. Para cul RDC es X(z) la transformada Z
de una secuencia causal?
6.3 Demuestre la propiedad de inversin dada por la Ec. (6.41).
6.4 Determine la transformada Z de la seal x1 [ n ] = ( a n cos 0 n ) u [ n ] a partir de la transformada de

la seal x2 [ n ] = ( cos 0 n ) u [ n ] , usando la propiedad de escalamiento.


6.5 Demuestre que si x[n] es una secuencia lateral derecha y X(z) converge para algn valor de z,
entonces la RDC de X(z) es de la forma
z > rmx

> z > rmx

donde rmx es la magnitud mxima de cualquiera de los polos de X(z).


6.6 Hallar la transformada Z inversa de
X (z) =

2 + z 2 + 5 z 3
z2 + 3 z + 2

z >0

6.7 Determine las transformadas Z de las x[n] siguientes:

(a)

x[ n ] = ( n 2) u [ n 2]

(b) x[ n ] = u [ n 2] u [ n 4]
(c)

x [ n ] = n {u [ n ] u [ n 4] }

6.8 Usando la relacin


an u [ n ]

z
za

z >a

halle la transformada Z de (a) x [ n ] = na n 1 u [ n ] ; (b) x [ n ] = n ( n 1) a n 2 u [ n ] ; y (c)

374

x [ n ] = n ( n 1)L ( n k + 1) a n k u [ n ]
6.9 Determine la transformada inversa de

X ( z ) = log ( 1 13 z 1 )
(a) usando la expansin en serie de potencias log (1 a ) =

(a k ),

k =1

a < 1 , y (b)

diferenciando X(z) y usando las propiedades de la transformada Z.


6.10 Determine la transformada Z y la regin de convergencia de la secuencia

2n cos 3 n

x[ n ] = 1 n
cos 3 n
4

n<0
n0

6.11 Determine la transformada Z inversa de cada una de las transformadas Z dadas mediante dos de
los mtodos explicados en el captulo y compare los valores para n = 0, 1, 2, y 3. En cada caso,
seale la RDC para su la validez de su resultado.

(a) X ( z ) =
(b) X ( z ) =
(c) X ( z ) =
(d) X ( z ) =
(e) X ( z ) =
(f) X ( z ) =

z
( z 1) ( z 0.8)
z ( z + 1)
( z 1) ( z 0.8)
1
( z 1) ( z 0.8)
1
z ( z 1) ( z 0.8)

z2
( z 1) 2 ( z 2) 2
2z +3
( z 2)3

6.12 Determine la convolucin de las secuencias causales siguientes:


n

1
h[ n ] = ,
2

1,
x[ n ] =
0,

0 n 10
otros valores de n

6.13 Halle la funcin de transferencia del sistema mostrado en la Fig. P.6.13 si

h1 [ n ] = ( n 1) u [ n ]
h2 [ n ] = [ n ] + n u [ n 1] + [ n 2)
n

1
h3 [ n ] = u [ n ]
3

375

h1[n]

h3[n]

x[n]

y[n]

+
h2[n]

Figura P.6.13

6.14 Determine la respuesta al escaln del sistema con funcin de transferencia

H (z) =

z 12
z 2 + 56 z + 16

6.15 Considere el sistema mostrado en la Fig. P.6.15

(a) Determine la funcin del sistema H(z).


(b) Halle la ecuacin de diferencias que relaciona la salida y[n] con la entrada x[n].

x[n]

y[n]
+

+
1
4

z 1

Figura P.6.15

6.16 Considere un sistema LIT de tiempo discreto cuya funcin de sistema H(z) es dada por

H (z) =

z
z

1
3

z >

1
3

(a) Calcule la respuesta al escaln s[n].


(b) Determine la salida y[n] cuando la entrada es x[n] = u[n].
6.17 Demuestre que un criterio simplificado para que el polinomio X ( z ) = z 2 + a1 z + a2 tenga todos
sus polos en el interior del crculo unitario en el plano z lo da

376

X (0) < 1,

X ( 1) > 0,

X (1) > 0

Use este criterio para hallar los valores de K para los cuales el sistema dado por
H (z) =

0.8 K z
( z 0.8) ( z 0.5)

sea estable.
6.18 (a) Cuando se aplica la excitacin x [ n ] = ( 12 ) se aplica a un sistema LIT, la salida y[n] viene
n

dada por y [ n ] = 2 ( 13 ) . Determine la funcin de transferencia del sistema. (b) Cul es la


n

respuesta al impulso correspondiente?


6.19 Resuelva la ecuacin en diferencias

y [ n ] 5 y [ n 1] + 6 y [ n 2] = 2,

y [ 1] = 6,

y [ n 2] = 4

6.20 Considere un sistema causal de tiempo discreto cuya salida y[n] y entrada x[n] estn relacionadas
por

y [ n ] 127 y [ n 1] + 121 y [ n 2] = x[ n ]
(a) Halle la funcin del sistema H(z).
(b) Calcule la respuesta al impulso h[n]
6.21 Demuestre que la solucin general de la ecuacin en diferencias

y [ n ] 2 y [ n 1] + y [ n 2] = 0
puede escribirse en la forma y [ n ] = C cosh n + D senh n , donde cosh = y C y D son dos
constantes arbitrarias. Determine y[n] si y[0] = E, y[10] = 0 y = 1.25.
6.22 (a) Demuestre que la salida y[n] del sistema de la Fig. P.6.22 satisface la ecuacin

2 y [ n ] y [ n 1] = 4 x [ n ] + 2 x [ n 1]
(b) El estado inicial del sistema es q[1] = 2. Halle la respuesta de entrada cero. (c) Halle la
funcin del sistema H(z). (d) Determine la respuesta al impulso h[n] y la respuesta al escaln.
x[n]
q[1] = 2
1/2
q[n 1]

z 1

q[n]
2

y[n]
Figura P.6.22

377

6.23 Use la transformada Z unilateral para resolver las ecuaciones de diferencias siguientes con las
condiciones iniciales dadas.

(a) y [ n ] 3 y [ n 1] = x[ n ], con x[ n ] = 4 u [ n ], y [ 1] = 1
(b) y [ n ] 5 y [ n 1] + 6 y [ n 2] = x[ n ], con x[ n ] = u [ n ], y [ 1] = 3, y [ 2] = 2
n

1
(c) y [ n ] y [ n 1] + y [ n 2] = x[ n ], y [ 1] = 0, y [ 2] = 1, x [ n ] = u [ n ]
2
4
3
1

CAPTULO SIETE

MODULACIN DE AMPLITUD
7.1 Introduccin
Ahora nos ocuparemos de la transmisin de mensajes formados por seales continuas (analgicas).
Cada seal de mensaje se selecciona de un nmero infinito de formas de onda posibles. Por ejemplo, en
la transmisin de radio y televisin se tiene un nmero infinito de mensajes posibles y no todas las
formas de ondas son conocidas. Esa coleccin de mensajes y formas de ondas puede ser modelada
convenientemente mediante procesos aleatorios continuos, en donde cada funcin miembro del proceso
aleatorio corresponde a una forma de onda del mensaje. Para el anlisis se define la transmisin de
seales analgicas como la transmisin por un canal dado de una seal x(t) de pasabajas, arbitraria y de
energa finita. En algunos casos tomaremos a x(t) como una seal de un solo tono (sinusoidal o de
potencia).
Si el canal es de pasabajas por naturaleza, la seal de pasabajas portadora de la informacin (o seal
del mensaje) puede transmitirse por el canal sin modificaciones. Esta clase de transmisin se conoce
como comunicacin en la banda base. La transmisin de esa seal por un canal de comunicaciones de
pasabandas, como una lnea telefnica o un canal satelital, requiere una adaptacin obtenida mediante
un corrimiento de la banda de frecuencias contenidas en la seal a otra banda de frecuencias adecuada
para la transmisin. Este corrimiento o traslacin se alcanza mediante el proceso conocido como
modulacin.
La modulacin es una operacin realizada en el transmisor para obtener una transmisin eficiente y
confiable de la informacin y consiste en la variacin sistemtica de algn atributo de una onda
portadora o modulada, como por ejemplo la amplitud, la fase o la frecuencia, de acuerdo con una
funcin de la seal del mensaje o seal moduladora. Aunque hay muchas tcnicas de modulacin, es
posible identificar dos tipos bsicos de ellas: la modulacin de onda portadora continua (OC) y la
modulacin de pulsos. En la modulacin OC, la onda portadora es continua (usualmente una onda
sinusoidal), y se cambia alguno de sus parmetros proporcionalmente a la seal del mensaje. En la
modulacin de pulsos, la onda portadora es una seal de pulsos (con frecuencia una onda de pulsos) y
se cambia un parmetro de ella en proporcin a la seal del mensaje. En ambos casos, el atributo de la
portadora puede ser cambiado en una forma continua o discreta. La modulacin de pulsos discretos
(digital) es un proceso discreto y es especialmente apropiado para mensajes que son discretos por
naturaleza, como, por ejemplo, la salida de un teletipo. Sin embargo, con la ayuda del muestreo y la
cuantizacin, se pueden transmitir seales del mensaje que varan continuamente (analgicas) usando
tcnicas de modulacin digital.
La modulacin, adems de usarse en los sistemas de comunicacin para adaptar las caractersticas de
la seal a las caractersticas del canal, tambin se utiliza para reducir el ruido y la interferencia, para

380

transmitir simultneamente varias seales por un mismo canal y para superar limitaciones fsicas en el
equipo.
El anlisis de Fourier se adapta extremadamente bien para el anlisis de seales moduladas; este
estudio es el objetivo principal de este captulo.
7.1.1

Necesidad de la Modulacin

Antes de comenzar una discusin cuantitativa de sistemas de modulacin, se examinarn las ventajas
de usar seales moduladas para la transmisin de informacin. Ya hemos mencionado que se requiere
modulacin para adaptar la seal al canal. Sin embargo, esta adaptacin involucra varios aspectos
importantes que merecen una explicacin adicional.
Modulacin para Facilidad de Radiacin. Si el canal de comunicacin consiste del espacio libre,
entonces se necesitan antenas para radiar y recibir la seal. La radiacin electromagntica eficiente
requiere de antenas cuyas dimensiones sean del mismo orden de magnitud que la longitud de onda de la
seal que est siendo radiada. Muchas seales, incluyendo las de audio, tienen componentes de
frecuencia que llegan a 100 Hz o menos. Para estas seales, seran necesarias antenas de alrededor de
300 Km de longitud si la seal se fuese a radiar directamente. Si se usa modulacin para imprimir la
seal del mensaje sobre una portadora de alta frecuencia, digamos a 100 MHz, entonces las antenas no
necesitan tener una longitud de ms de un metro (longitud transversal).
Modulacin para Concentracin o Multicanalizacin. Si ms de una seal usa un solo canal, la
modulacin puede usarse para trasladar diferentes seales a posiciones espectrales diferentes
permitiendo as al receptor seleccionar la seal deseada. Las aplicaciones de la concentracin
(multiplexing en ingls) incluyen la telemetra de datos, radiodifusin FM estereofnica y telefona
de larga distancia.
Modulacin para Superar Limitaciones en el Equipo. El rendimiento de los dispositivos de
procesamiento de seales tales como filtros y amplificadores, y la facilidad con la cual estos
dispositivos pueden construirse, depende de la situacin de la seal en el dominio de la frecuencia y de
la relacin entre las frecuencias ms alta y ms baja de la seal. La modulacin puede ser usada para
trasladar la seal a una posicin en el dominio de la frecuencia donde se cumplan fcilmente los
requerimientos de diseo. La modulacin tambin puede usarse para convertir una seal de banda
ancha (una seal para la cual la relacin entre la frecuencia mayor y la menor es grande) en una seal
de banda angosta.
Ocasionalmente, en aplicaciones de procesamiento de seales, la banda de frecuencias de la seal a
procesar y la banda de frecuencias del aparato procesador pueden no adaptarse. Si el procesador es
elaborado y complejo, puede ser mejor dejar que opere en alguna banda de frecuencias fija y, ms bien,
trasladar la banda de frecuencias de la seal para que se corresponda con esta banda fija del equipo. La
modulacin puede usarse para obtener esta traslacin de frecuencias.

381

Modulacin para Asignacin de Frecuencias. La modulacin permite que varias estaciones de radio
o televisin se transmitan simultneamente con frecuencias portadoras diferentes y permite sintonizar
diferentes receptores para seleccionar estaciones diferentes.
Modulacin para Reducir el Ruido y la Interferencia. El efecto del ruido y la interferencia no
pueden ser eliminados completamente en un sistema de comunicacin. Sin embargo, es posible
minimizar sus efectos usando ciertos tipos de esquemas de modulacin. Estos esquemas generalmente
requieren un ancho de banda de transmisin mucho mayor que el ancho de banda de la seal del
mensaje. Por esta razn se intercambia ancho de banda por reduccin de ruido un aspecto importante
del diseo de sistemas de comunicacin.
7.2 Tipos de Modulacin Analgica
Los tipos bsicos de modulacin analgica son la modulacin de onda continua (OC) y la de pulsos.
En la modulacin de onda continua, se usa una seal sinusoidal xc ( t ) = Ac cos ( c t + ) como una
seal portadora. Entonces una seal portadora modulada general puede ser representada
matemticamente como
xc ( t ) = A ( t ) cos [ c t + ( t ) ] , c = 2 f c

(7.1)

En la Ec. (7.1), fc se conoce como la frecuencia portadora, A(t) es la amplitud instantnea de la


portadora y (t) es el ngulo o desviacin de fase instantnea de la portadora. Cuando A(t) est
relacionada linealmente con la seal del mensaje x(t), el resultado es modulacin de amplitud. Si (t) o
su derivada est linealmente relacionada con x(t), entonces tenemos modulacin de fase o de
frecuencia. Se usa el nombre comn de modulacin angular para denotar tanto la modulacin de fase
como la de frecuencia.
Mientras la modulacin es el proceso de transferir informacin a una portadora, la operacin inversa
de extraer la seal portadora de la informacin de la portadora modulada se conoce como
demodulacin. Para diferentes tipos de esquemas de modulacin consideraremos diferentes mtodos de
demodulacin y supondremos que la demodulacin se hace en la ausencia de ruido. El efecto del ruido
sobre la calidad de la seal de salida de diferentes mtodos de transmisin modulada ser el objetivo de
la discusin en un captulo posterior.
En el anlisis de los esquemas de modulacin OC se prestar mucha atencin a tres parmetros
importantes: la potencia transmitida, el ancho de banda de transmisin y la complejidad del equipo para
modular y demodular. Estos parmetros, junto con la calidad de la seal de salida en la presencia de
ruido, proporcionarn la base para la comparacin de diferentes esquemas de modulacin.
En la modulacin de pulsos, un tren peridico de pulsos cortos acta como la seal portadora.
7.3 Transmisin de Seales de Banda Base Analgicas
Los sistemas de comunicacin en los cuales ocurre la transmisin de seales sin modulacin se
denominan sistemas de banda base. En la Fig. 7.1 se muestran los elementos funcionales de un
sistemas de comunicacin de banda base. El transmisor y el receptor amplifican la potencia de la seal

382

y realizan las operaciones de filtrado apropiadas. En el sistema no se ejecutan operaciones de


modulacin ni demodulacin. El ruido y la distorsin de la seal debidos a las caractersticas no ideales
del canal hacen que la seal de salida y(t) sea diferente de la seal de entrada x(t). Ahora se
identificarn diferentes tipos de distorsin, sus causas y las curas posibles. En un captulo posterior se
discutirn los efectos del ruido sobre la calidad de la seal y el diseo ptimo del transmisor y receptor
que minimiza esos efectos.

Ruido
x(t)
a
Seal de
entrada

Transmisor

Canal
Hc(f)

Receptor

b
Seal de
salida + ruido

Figura 7.1 Un sistema de comunicacin de banda base.

7.3.1 Distorsin de la Seal en la Transmisin en la Banda Base


Se dice que la seal de salida y(t) no est distorsionada si se parece a la seal de entrada x(t). Ms
especficamente, si y(t) difiere de x(t) por una constante de proporcionalidad y un retardo temporal
finito, entonces se dice que la transmisin no est distorsionada. Es decir,
y ( t ) = Kx ( t td )

(7.2)

para transmisin sin distorsin. La constante K es la atenuacin y td es el retardo temporal. La prdida


de potencia en la transmisin es 20 log10 K y en la Tabla 1 se dan valores tpicos de prdidas de
transmisin para varios medios.
El requisito para transmisin sin distorsin expresado por la Ec. (7.2) puede cumplirse si la funcin
de transferencia total del sistema entre los puntos a y b en la Fig. 7.1 es
H ( f ) = K exp ( j 2 ftd )

para

f < fx

(7.3)

donde fx es el ancho de banda de la seal en la banda base. Si suponemos que el transmisor y el


receptor no producen distorsin de la seal, entonces la respuesta del canal tiene que satisfacer
H c ( f ) = K exp ( j 2 ftd )

para

f < fx

(7.4)

para una transmisin sin distorsin.


La condicin dada por la Ec. (7.4) es bastante fuerte y, en el mejor de los casos, los canales reales
slo pueden satisfacer esta condicin aproximadamente. Por ello, siempre ocurrir algo de distorsin en
la transmisin de seales aunque se puede minimizar mediante un diseo apropiado. Un enfoque
conveniente para minimizar la distorsin de una seal es identificar diferentes tipos de distorsin e
intentar minimizar sus efectos dainos por separado.

383

Tabla 1. Valores tpicos de prdidas de transmisin


Medio de Transmisin

Frecuencia

Prdida,
dB/km

Par de alambres (0.3 cm de dimetro)

1 kHz

0.05

Par de alambres trenzados (calibre 16)

10kHz

100 kHz

300kHz

100 kHz

1 MHz

3 MHz

Cable coaxial (1 cm de dimetro)

Cable coaxial (15 cm de dimetro)


Gua de onda rectangular (52.5 cm)
Gua de onda helicoidal (5 cm de
dimetro)
Cable de fibra ptica

100 MHz

1.5

10 GHz

100 GHz

1.5

3.61014 Hz

2.5

2.41014 Hz

0.5

14

1.810 Hz

0.2

Los tres tipos comunes de distorsin encontrados en un canal son:


1. Distorsin de amplitud debida a |Hc(f)| K.
2. Distorsin de fase (o retardo) debida a que
ngulo{H c ( f )} 2 ftd m (m es un entero > 0)
3. Distorsin no lineal debida a elementos no lineales presentes en el canal.
Las primeras dos categoras se conocen como distorsin lineal y la tercera como distorsin no lineal.
Ahora las examinaremos por separado.
7.3.2 Distorsin Lineal
Si la respuesta de amplitud del canal no es plana en la banda de frecuencias para las cuales el espectro
de la entrada es diferente de cero, entonces diferentes componentes espectrales de la seal de entrada
son modificados en forma diferente. El resultado es distorsin de amplitud. Las formas ms comunes
de la distorsin de amplitud son la atenuacin excesiva o el realce de las bajas frecuencias en el
espectro de la seal. Resultados experimentales indican que si |Hc(f)| es constante hasta dentro de 1 dB
en la banda del mensaje, entonces la distorsin de amplitud ser despreciable. Ms all de estas

384

observaciones cualitativas, no se puede decir mucho sobre la distorsin de amplitud sin un anlisis ms
detallado.
Si el desplazamiento de fase es arbitrario, diferentes componentes de la seal de entrada sufren
retardos temporales diferentes lo cual resulta en distorsin de fase o de retardo. Una componente
espectral de la entrada con frecuencia f sufre un retardo td(f),

td ( f ) =

ngulo de { H ( f )}

(7.5)

2 f

El lector puede verificar que un ngulo de { H ( f )} = 2 td f m resultar en una respuesta


y ( t ) = x ( t td ) , es decir, no ocurre distorsin. Cualquier otra respuesta de fase, incluyendo un
desplazamiento constante de fase , m , producir distorsin.
La distorsin por retardo es un problema crtico en la transmisin de pulsos (datos). No obstante, el
odo humano es sorprendentemente insensible a esta distorsin y por tanto la distorsin por retardo no
es preocupante en la transmisin de audio.
7.3.3 Compensacin

El remedio terico para la distorsin lineal es la compensacin mostrada en la Fig. 7.2. Si la funcin de
transferencia del compensador satisface la relacin
H eq =

K exp ( j 2 ftd )
Hc ( f )

para

f < fx

(7.6)

tenemos entonces que H c ( f ) H eq ( f ) = K exp ( j 2 ftd ) y no se tendr distorsin. Sin embargo, es

muy raro que se pueda disear un compensador que satisfaga exactamente la Ec. (7.6). Pero son
posibles excelentes aproximaciones, especialmente con un filtro transversal como el mostrado en la
Fig. 7.3.

x(t)

Canal
Hc(f)

Compensador
Heq(f)

salida

Figura 7.2 Compensador del canal.

La salida del compensador mostrado en la Fig. 7.3 puede escribirse como


y ( t ) = c1 z ( t ) + c0 z ( t ) + c1 z ( t 2 )
a partir de la cual obtenemos la funcin de transferencia del filtro como
H eq ( f ) = c1 + c0 exp ( j ) + c1 exp ( j 2 ), = 2 f
Generalizando esta relacin a un compensador con 2M + 1 derivaciones, tenemos entonces que

385

H eq ( f ) = exp ( jM ) cm exp ( jm )
m = M

(7.7)

que est en la forma de una serie de Fourier exponencial con periodicidad 1/. Por lo tanto, si se va a
compensar el canal en la banda fm del mensaje, podemos aproximar el lado derecho de la Ec. (7.6)
mediante una serie de Fourier (en el dominio de la frecuencia) con periodicidad 1/ 2fm. Si la
aproximacin en serie de Fourier tiene 2M + 1 trminos, entonces se necesita un compensador con 2M
+ 1 derivaciones.

entrada
x(t)

Retardo

Retardo

c1

c0

c1

+
+

y(t)

salida

Figura 7.3 Un filtro transversal compensador de tres derivaciones.

7.3.4 Distorsin No Lineal y Compansin

Los canales y dispositivos electrnicos prcticos, tales como amplificadores, con frecuencia exhiben
caractersticas de transferencia no lineales que resultan en una distorsin no lineal de la seal. En la
Fig. 7.4 se muestra un ejemplo de la caracterstica de transferencia de un elemento no lineal sin
memoria. En general, estos dispositivos actan linealmente cuando la entrada x(t) es pequea, pero
distorsionan la seal cuando la amplitud de la entrada es grande.

Entrada
x(t)

Aproximacin
lineal

Caracterstica de
transferencia real

Salida
y(t)

Figura 7.4 Caracterstica de transferencia de un dispositivo no lineal.

386

Para investigar la naturaleza de la distorsin no lineal de la seal, supongamos que la caracterstica de


transferencia del dispositivo no lineal puede ser modelada por la relacin
y ( t ) = a1 x ( t ) + a2 x 2 ( t ) + a3 x3 ( t ) + L

(7.8)

Ahora, si la entrada es la suma de dos ondas coseno, digamos cos 2 f1 t + cos 2 f 2 t , entonces la salida
contendr trminos de distorsin armnica en las frecuencias 2f1, 2f2 y trminos de distorsin de
intermodulacin en las frecuencias f1 f 2 , 2 f 2 f1 , 2 f1 f 2 , y as sucesivamente. En un caso
general, si x(t) = x1(t) + x2(t), entonces y(t) contendr los trminos x12 ( t ), x22 ( t ), x1 ( t ) x2 ( t ) , y as
sucesivamente. En el dominio de la frecuencia es fcil ver que aunque X1(f) y X2(f) puedan estar
separadas en frecuencia, el espectro de x1 ( t ) x2 ( t ) [obtenido a partir de X 1 ( f ) X 2 ( f ) ] puede
solaparse con X1(f) o X2(f) o con ambas. Esta forma de distorsin por intermodulacin (o diafona) es
de importancia en sistemas donde varias seales son concentradas (multicanalizadas) y transmitidas por
el mismo canal.
La caracterstica de transferencia mostrada en la Fig. 7.4 sugiere que una solucin para minimizar la
distorsin no lineal es mantener la amplitud de la seal dentro de la banda lineal de operacin de la
caracterstica. Esto se obtiene usualmente usando dos dispositivos no lineales, un compresor y un
expansor, como se muestra en la Fig. 7.5.
Un compresor esencialmente reduce la banda de amplitudes de una seal de entrada de manera que
caiga dentro de la banda lineal del canal. Para una seal x(t) de valores positivos, por ejemplo, podemos
usar un compresor con una caracterstica de transferencia g comp [ x ( t )] = log e [ x ( t )] . Puesto que un
compresor reduce la banda de la seal de entrada, tambin reduce la banda de la seal de salida. La
seal de salida es expandida al nivel apropiado mediante el expansor que opera a la salida del canal.
Idealmente, un expansor tiene una caracterstica de transferencia gexp que produce
g exp { g comp [ x ( t )]} = x ( t ) . Por ejemplo, si g comp [ x ( t )] = log e [ x ( t )] , entonces g exp [ y ( t )] = exp[ y ( t )]
producir g exp { g comp [ x ( t )]} = x ( t ) . La operacin combinada de comprimir y expandir se denomina
compansin. La compansin se usa extensivamente en sistemas telefnicos para compensar por la
diferencia en el nivel de la seal entre oradores altos y bajos.

x(t)

Canal
(supuesto
no lineal)

Compresor

Expansor

y(t)

Figura 7.5 Compansin.

7.4 Esquemas de Modulacin Lineales OC

La modulacin lineal se refiere al corrimiento directo de frecuencias del espectro del mensaje usando
una portadora sinusoidal. La portadora modulada es representada por
xc ( t ) = A ( t ) cos c t

(7.9)

387

en la cual la amplitud de la portadora A(t) est relacionada linealmente con la seal del mensaje x(t).
Dependiendo de la naturaleza de la relacin espectral entre x(t) y A(t), tenemos los siguientes tipos de
esquemas de modulacin lineal: modulacin de banda lateral doble (DSB, por sus siglas en ingls),
modulacin de amplitud (AM), modulacin de banda lateral nica (SSB por sus siglas en ingls) y
modulacin de banda lateral residual (VSB por sus siglas en ingls). Cada uno de estos esquemas tiene
sus propias ventajas distintivas, desventajas y aplicaciones prcticas. Ahora estudiaremos estos
diferentes tipos de esquemas de modulacin lineal recalcando tpicos tales como los espectros de las
seales, potencia y ancho de banda, mtodos de demodulacin y la complejidad de transmisores y
receptores.
En nuestra discusin sobre esquemas de modulacin lineales, usaremos uno de tres modelos
diferentes para la seal del mensaje x(t): un solo tono de frecuencia, fx, una combinacin de tonos
restringidos en frecuencia a menores o iguales que fx, o una seal arbitraria de pasabajas de energa
finita con una transformada de Fourier X(f), la cual es idnticamente igual a cero para f > f x .

7.4.1

Modulacin de Banda Lateral Doble (DSB)

La modulacin de banda lateral doble (DSB, por sus iniciales en ingls) resulta cuando la amplitud
A(t) es proporcional a la seal del mensaje x(t), es decir, el mensaje de pasabajas x(t) es multiplicado
por una forma de onda portadora Ac cos c t , como se muestra en la Fig. 7.6a. La seal modulada xc(t)
es
xc ( t ) = Ac x ( t ) cos c t = A ( t ) cos c t ,

c = 2 f c

(7.10)

y se llama la seal modulada en banda lateral doble. La Ec. (7.10) revela que la amplitud instantnea
de la portadora A(t) es proporcional a la seal del mensaje x(t). Un ejemplo en el dominio del tiempo de
la seal modulada x(t) se muestra en la Fig. 7.6d para una seal del mensaje sinusoidal.
Del teorema de modulacin se deduce que el espectro de la seal DSB dada en la Ec. (7.10) es
X c ( f ) = 12 Ac [ X ( f + f c ) + X ( f f c )]

(7.11)

donde fc = c/2. Las representaciones en el dominio de la frecuencia de X(f) y Xc(f) se muestran en


las Figs. 7.6e y 7.6f para una seal de mensaje de pasabajas.
La banda espectral ocupada por la seal del mensaje se llama la banda de frecuencias de la banda
base y la seal del mensaje usualmente se conoce como la seal de la banda base. La operacin de
multiplicar seales se llama mezclado o heterodinaje. En la seal trasladada, la parte del espectro de la
seal de la banda base que est sobre fc aparece en el intervalo fc a fc + fx y se denomina la seal de la
banda lateral superior. La parte de la seal modulada que est entre fc fx y fc se llama la seal de la
banda lateral inferior. La seal portadora de frecuencia fc tambin se conoce como la seal del
oscilador local, la seal mezcladora o la seal heterodina. Como se observa en la Fig. 7.6f, el espectro
de Xc(f) no tiene una portadora identificable. Por ello, este tipo de modulacin tambin se conoce como
modulacin de banda lateral doble con portadora suprimida (DSB-SC). La frecuencia portadora fc es
normalmente mucho ms alta que el ancho de banda de la seal de la banda base fx. Es decir,
f c >> f x

(7.12)

388

x(t

xc(t

z(t)

xr(t)

Ac cos c t

Filtro de
pasabajas

y(t)

2 cos c t

(a) Modulador

(b) Demodulador sincrnico

(c) Moduladora sinusoidal


Inversin de fase

(d) Seal modulada


X(f)

fx

fx

(e) Espectro del mensaje


Banda lateral
inferior 2fx

Xc(f)

Banda lateral
superior
fc fx fc fc + fx

fc fx fc

fc + fx

(e) Espectro DSB


Z(f)

2fc

fx

Respuesta del filtro


de banda base

fx

2fc fx 2fc 2fc + fx

(f)

Figura 7.6 Modulacin de banda lateral doble. (a) Modulador. (b) Demodulador sincrnico (o coherente).
(c) Seal moduladora sinusoidal. (d) Seal modulada. (e) Espectro del mensaje para una x(t) arbitraria. (f)
Xc(f). (g) Z(f).

389

Potencia y Ancho de Banda de la Seal Transmitida. De la Fig. 7.6f vemos que el ancho de banda
BT requerido para transmitir una seal del mensaje con ancho de banda fx usando modulacin de banda
lateral doble es 2fx Hz:

BT = 2 f x

(7.13)

Para calcular la potencia transmitida promedio ST de la seal modulada, supongamos que x(t) es una
seal de potencia. Entonces,

ST = lm

1
T

T 2

Ac2 x 2 ( t ) cos 2 ( c t ) dt

T 2

T 2
T 2

2
A
1 Ac2 2
c
2

x ( t ) dt +
x ( t ) cos 2 c t dt
= lm
T T

2
2

T 2
T 2

El valor de la segunda integral es cero, y si definimos la potencia promedio de la seal Sx como

S x = lm

1
T

T 2

x 2 ( t ) dt

T 2

entonces
ST = Sc S x

(7.14)

donde Sc = Ac2 2 es la potencia promedio de la portadora.


Demodulacin de la Seal de la Banda Base. Si suponemos que el canal es ideal, entonces la seal
recibida xr(t) tendr la misma forma que xc(t). Es decir,
xr ( t ) = ac x ( t ) cos c t

donde ac/Ac es la atenuacin del canal. La seal del mensaje en la banda base x(t) puede ser recuperada
de la seal recibida xr(t) multiplicando xr(t) por una portadora local y filtrando a pasabajas la seal
producto. La salida del multiplicador es
z ( t ) = [ ac x ( t ) cos c t ]2 cos c t
= ac x ( t ) + ac x ( t ) cos 2 c t
y el espectro de Z(f) est dado por
Z ( f ) = ac X ( f ) + 12 ac [ X ( f 2 f c ) + X ( f + f c )]
El espectro de Z(f) se muestra en la Fig. 7.6g, de la cual es obvio que si
f x < 2 fc f x

o fc > f x

entonces no hay solapamiento de X(f) con X(f 2fc) o con X(f + 2fc). Por tanto, filtrando Z(f) mediante
un filtro de pasabajas con una frecuencia de corte B, fx < B < 2fc fx producir una seal de salida y(t),
y(t) = acx(t)

390

que es una rplica de la seal del mensaje transmitida x(t).


Aunque el ancho de banda del filtro de pasabajas puede estar entre fx y 2fc fx, l debe ser tan
pequeo como sea posible para reducir los efectos de cualquier ruido que pueda acompaar la seal
recibida. Si hay ruido presente, entonces se debe insertar un filtro de pasabandas con una frecuencia
central fc y un ancho de banda de 2fx antes del multiplicador en la Fig. 7.6b para limitar la potencia de
ruido que entra al demodulador.
El esquema de recuperacin de la seal mostrado en la Fig. 7.6b se denomina un esquema de
demodulacin sincrnico o coherente. Este esquema requiere que en el receptor est disponible una
seal de un oscilador local que est perfectamente sincronizado con la seal portadora usada para
generar la seal modulada. ste es un requisito bastante rgido y no puede obtenerse fcilmente en
sistemas prcticos. La falta de sincronismo resultar en distorsin de la seal. Suponga que la seal del
oscilador local tiene una desviacin de frecuencia igual a y y una desviacin de fase igual a .
Entonces la seal producto z(t) tendr la forma
z ( t ) = ac x ( t ) cos( t + ) + trminos de frecuencia doble

y la seal de salida y(t) ser


y ( t ) = ac x ( t ) cos ( t + )

(7.15)

Ms adelante se verificar que cuando = 0 y = /2, la seal se pierde completamente. Cuando


= 0 , entonces y ( t ) = ac x ( t ) cos t variar provocando una seria distorsin de la seal. Este
problema es bastante grave ya que usualmente fc >> fx de modo que aun un error porcentual pequeo
en fc ocasionar una desviacin f que puede ser comparable o mayor que fx! La evidencia
experimental indica que para seales de audio, una f > 30 Hz se convierte en inaceptable. Para
seales de audio puede ser posible ajustar manualmente la frecuencia y la fase de la portadora local
hasta que la salida suene bien. Desafortunadamente, las desviaciones de fase y de frecuencia de la
portadora con frecuencia son cantidades que varan con el tiempo requiriendo entonces ajustes casi
continuos.
Existen varias tcnicas usada para generar una portadora coherente para la demodulacin. En el
mtodo mostrado en la Fig. 7.7, se extrae una componente de la portadora de la seal DSB usando un
circuito cuadrtico y un filtro de pasabandas. Si x(t) tiene un valor CD igual a cero, entonces xc(t) no
tiene ninguna componente espectral en fc. No obstante, x2(t) tendr una componente CD diferente de
cero y por tanto se puede extraer una componente de frecuencia discreta en 2fc del espectro de xr2 ( t )
usando un filtro con una pasabanda angosta. La frecuencia de esta componente puede ser reducida a la
mitad para proporcionar la portadora deseada para la modulacin.

Seal DSB
xr(t)

Circuito
cuadrtico

BPF centrado
en 2fc

Divisor de
frecuencia

Figura 7.7 Un sincronizador cuadrtico.

Seal de
sincronizacin

391

En el segundo mtodo mostrado en la Fig. 7.8, una pequea seal portadora (piloto) se transmite
junto con la seal DSB; en el receptor, la portadora piloto puede extraerse, amplificarse y usarse como
una portadora local sincronizada para la demodulacin (Fig. 7.8b).
Si la amplitud de la portadora insertada es lo suficientemente grande, entonces la seal recibida
puede ser demodulada sin tener que generar la portadora en el receptor. Una seal DSB con una
componente de portadora discreta grande se llama una seal modulada en amplitud (AM).

x(t)

xc(t)

Seal DSB
+
portadora

LPF
Filtro de
portadora

Ac cos c t

y(t)

Amplificador
(b)

(a)

Figura 7.8 Sistema DSB con portadora piloto. (a) Transmisor. (b) Receptor

Ejemplo 1. Evale el efecto de un error de fase en el oscilador local en la demodulacin de banda


lateral doble sincrnica.
Solucin. Suponga que el error de fase del oscilador local es . Entonces la portadora local es
expresada como cos ( c t + ) . Ahora,

xDSB ( t ) = m ( t ) cos c t
donde m(t) es la seal del mensaje y si designamos la salida del multiplicador en la Fig. 7.6b por d(t),
entonces
d ( t ) = [ m ( t ) cos c t ]cos ( c t + )
=
=

1
2
1

m ( t )[cos + cos (2 c t + )

1
m ( t ) cos + m ( t ) cos (2 c t + )
2
2

El segundo trmino en el lado derecho es eliminado por el filtro de pasabajas y obtenemos


y (t ) =

1
2

m ( t ) cos

(7.16)

Esta salida es proporcional a m(t) cuando es una constante. La salida se pierde completamente
cuando = 2 . As pues, el error de fase en la portadora local produce atenuacin en la seal de
salida sin ninguna distorsin siempre que sea constante pero diferente de /2. Si el error de fase
vara aleatoriamente con el tiempo, entonces la salida tambin variar aleatoriamente y es decir
indeseable.

392

Ejemplo 2. Evale el efecto de un pequeo error de frecuencia en el oscilador local en la


demodulacin DSB sincrnica.
Solucin. Suponga que el error de frecuencia del oscilador local es . La portadora local es
expresada entonces como cos (c + ) t . As que
d ( t ) = m ( t ) cos c t cos ( c + ) t
=

1
m ( t ) cos ( ) t + m ( t ) cos 2 c t
2
2

y
y (t ) =

1
2

m ( t ) cos( ) t

(7.17)

Es decir, la salida es la seal m(t) multiplicada por una sinusoide de baja frecuencia. Es decir un efecto
de batido y, como ya se mencion, es una distorsin muy indeseable.
7.4.2 Modulacin de Amplitud Ordinaria (AM)

Una seal modulada en amplitud es generada aadiendo una componente grande de portadora a la seal
DSB. La seal AM tiene la forma
xc ( t ) = Ac [1 + x ( t )]cos c t

(7.18)

= A ( t ) cos c t

(7.19)

donde A(t) es la envolvente de la portadora modulada. Para una recuperacin fcil de la seal usando
esquemas de demodulacin sencillos, la amplitud de la seal tiene que ser pequea y la componente
CD de la seal tiene que ser igual a cero, es decir,
x ( t ) < 1 y lm

1
T

T 2

x ( t ) dt = 0

T 2

Ms adelante se explicar la necesidad de estas restricciones.


En el dominio de la frecuencia, el espectro de la seal AM est dado por
X c ( f ) = 12 Ac [ X ( f f c ) + X ( f + f c )]
+ 12 Ac [ ( f f c ) + ( f + f c )]

(7.20)

En la Fig. 7.9 se muestran ejemplos de seales AM en el dominio del tiempo y en el de la frecuencia.

393

x(t)

xc(t)

envolvente

(a)

(b)
X(f)

fx

fx

(c)
portadora
banda lateral
banda lateral
inferior
superior
0
fc f x f c f c + f x
(d)

fc

Figura 7.9 Modulacin de amplitud. (a) seal del mensaje sinusoidal. Seal AM. (c) Espectro del
mensaje para una seal arbitraria x(t). (d) Espectro de la seal modulada.

7.4.3 ndice de Modulacin

Dos caractersticas nicas de la seal AM son que est presente una componente con frecuencia de la
portadora y que la envolvente A(t) de la portadora modulada tiene la misma forma que x(t) siempre que
f c >> f x y que A ( t ) = Ac [1 + x ( t )] no se haga negativa. Nuestra suposicin de que |x(t)| < 1 garantiza
que A(t) no se har negativa. Si x(t) es menor que 1, entonces A(t) se hace negativa y resulta una
distorsin de envolvente, como se muestra en la Fig. 7.10.
Un parmetro importante de una seal AM es su ndice de modulacin m, el cual se define como
m=

[ A ( t )]mx [ A ( t )]mn
[ A ( t )]mx + [ A ( t )]mn

(7.21)

Cuando m es mayor que 1 se dice que la portadora est sobremodulada, resultando en distorsin de
envolvente.
7.4.4 Potencia y Ancho de Banda de la Seal Transmitida

De la Fig. 7.9d vemos que el ancho de banda de la seal AM es


BT = 2fx
B

Suponiendo que x(t) es una seal de potencia, podemos calcular la potencia promedio de la seal
transmitida como

394

T 2

ST = lm

Ac2 [1 + x ( t )]2 cos 2 c t dt

T 2

T 2

lm
T

T 2

Ac2
2

[1 + x 2 ( t ) + 2 x ( t )][1 + cos 2 c t ] dt

= Sc + Sc S x

(7.22)

donde Sc = Ac2 2 y Sx es la potencia promedio normalizada de la seal.


La onda portadora por s sola, sin modulacin, no transporta ninguna informacin hasta el receptor.
Por ello, podemos concluir que una porcin de la potencia transmitida ST es desperdiciada en la
portadora. Ms adelante veremos que la simplicidad de los demoduladores AM depende de esta
potencia y, por tanto, la portadora no es del todo una prdida.

x c(t)

A(t)
distorsin de
envolvente

Ac

Ac
t

Ac

(b)

(a)

Figura 7.10 Distorsin de envolvente de una seal AM. (a) Seal modulada. (b) Envolvente A(t).

Para seales AM, el porcentaje de la potencia total que lleva informacin se usa como una medida de
la eficiencia de potencia. sta se denota por y la definimos como
=

Sc S x
Sc + Sc S x

(7.23)

Se deja como un ejercicio demostrar que la mxima eficiencia para una seal arbitraria x(t) es 50% y,
como se demostrar ms adelante, la mxima eficiencia para una seal de mensaje en onda seno es
33.3% (recuerde que |x(t)| < 1 y por tanto Sx 1).
Ejemplo 3. Una estacin AM comercial est transmitiendo con una potencia promedio de 10 kW. El
ndice de modulacin es 0.707 para una seal del mensaje sinusoidal. Determine la eficiencia de
potencia de transmisin y la potencia promedio en la componente de portadora de la seal transmitida.

395

Solucin. Para una seal del mensaje sinusoidal con un ndice de modulacin de 0.707, la seal
modulada est dada por

xc ( t ) = Ac (1 + 0.707 cos x t ) cos c t


Por lo tanto,
S x = 12 (0.707) 2 = 0.25
=

0.25S c
= 20%
Sc + 0.25S c

Ahora, Sc + 0.25Sc = 10 kW, y de aqu que Sc = 8 kW. Observe la proporcin entre la potencia usada
para transmitir informacin y la usada para transmitir la portadora.
Ejemplo 4. Otra forma de escribir la eficiencia de la AM ordinaria es como el porcentaje de la
potencia total llevada por las bandas laterales, es decir,
=

Ps

(7.24)

Pt

donde Ps es la potencia transportada por las bandas laterales y Pt es la potencia total de la seal AM. (a)
Determine para m = 0.5 (50 % de modulacin). (b) Demuestre que para AM de un solo tono, mx es
33.3 % para m = 1.
Solucin. Para modulacin de un solo tono

x ( t ) = am cos m t
el ndice de modulacin es
m=

am
Ac

Por lo tanto,
x ( t ) = am cos m t = mAc cos m t
y la seal AM es entonces
xc ( t ) = [ Ac + x ( t )]cos c t = Ac [1 + m cos m t ]cos c t
o
xc ( t ) = Ac cos c t + mAc cos m t cos c t
= Ac cos c t + 12 mAc cos ( c m ) t + 12 mAc cos ( c + m ) t
entonces
Pc = potencia en la portadora =

1
2

Ac2

396

Ps = potencia en las bandas laterales =

1
2

( 1 mA )2 + ( 1 mA )2 = 1 m 2 A2
c
c
2
2 c
4

La potencia total Pt es
Pt = Pc + Ps = 12 Ac2 + 14 m 2 Ac2 =

1
2

( 1+

1
2

m 2 ) Ac2

As pues,
=

Ps
Pt

100% =

1
4

1
2

m 2 Ac2

+ 14 m 2 ) Ac2

100% =

m2
2 + m2

100%

con la condicin que m 1.


(a) Para m = 0.5,

(0.5) 2
2 + (0.5) 2

100% = 11.1%

(b) Como m 1, se puede ver que mz ocurre para m = 1 y est dada por
= 12 100% = 33.3%
7.4.5 Demodulacin de Seales AM

La ventaja de la modulacin AM sobre la DSB es que un esquema muy sencillo, conocido como
deteccin de envolvente, puede ser usado para la demodulacin si se transmite suficiente potencia de
portadora. La seal del mensaje en la banda base x(t) puede ser recuperada de la seal AM xr(t) usando
el circuito sencillo mostrado en la Fig. 7.11a. La Ec. (7.18) muestra que siempre que |x(t)| < 1, la
envolvente de la seal recibida nunca pasar por cero y la porcin positiva de la envolvente se
aproxima a la seal del mensaje x(t) sin depender de la fase o frecuencia exactas de la portadora. La
parte positiva de la envolvente es recuperada mediante la rectificacin de xr(t) y suavizando la onda
rectificada usando una red RC. Durante el semiciclo positivo de la seal de entrada, el diodo es
polarizado directamente y el capacitor C se carga rpidamente hasta el valor pico de la seal. Conforme
la seal de entrada cae por debajo de su mximo, el diodo se abre y es decir seguido por una descarga
lenta del capacitor a travs del resistor R hasta el prximo semiciclo positivo, cuando la seal de
entrada excede el voltaje del capacitor y el diodo conduce de nuevo. El capacitor se carga hasta el
nuevo valor pico, y el proceso se repite.
Para una mejor operacin, la frecuencia de la portadora debe ser mucho ms alta que fx, y la
constante de tiempo de la descarga del circuito RC debe ser ajustada de modo que la pendiente mxima
negativa de la envolvente nunca exceder la tasa de descarga exponencial. Si la constante de tiempo es
demasiado grande, entonces el detector de envolvente no puede seguir a la envolvente (Fig. 7.11c). Si
la constante es demasiado pequea se generar una onda demasiado distorsionada (Fig. 7.11d) y la
demodulacin se hace ineficiente.
Bajo condiciones de operacin ideales, la salida del demodulador es
z ( t ) = k1 + k2 x ( t )

397

donde k1 es una desviacin de CD debida a la portadora y k2 es la ganancia del circuito demodulador.


Se puede usar un capacitor de acoplamiento o un transformador para remover la desviacin CD; sin
embargo, cualquier trmino CD en la seal del mensaje x(t) tambin ser eliminado [una de las razones
para nuestra suposicin de que el valor de x(t) es igual a cero]. Adems de remover componentes CD,
el filtro de remocin CD atenuar las componentes de baja frecuencia de la seal del mensaje. Por ello,
la AM no es adecuada para transmitir seales de mensajes que contienen contenidos significativos de
baja frecuencia.
Ejemplo 5. La entrada a un detector de envolvente (Fig. 7.11) es una seal AM de un solo tono
xc ( t ) = A (1 + cos m t ) cos c t , donde es una constante, 0 < < 1 y c >> m.

(a) Demuestre que si la salida del detector va a seguir la envolvente de xc(t), se requiere que en todo
instante t0
sen m t0
m

RC
1 + cos m t0
1

(7.25)

(b) Demuestre que si la salida del detector va a seguir la envolvente todo el tiempo, se requiere que
RC

1 2

(7.26)

Solucin

(a) La Fig. 7.12 muestra la envolvente de xc(t) y la salida del detector (el voltaje en el capacitor en la
Fig. 7.11). Suponga que el capacitor se descarga de su valor pico E0 = A (1 + cos 0 t ) en t0 = 0.
Entonces el voltaje vc(t) en el capacitor de la Fig. 7.11 est dado por
vc ( t ) = E0 e

t ( RC )

El intervalo entre dos picos sucesivos de la portadora es 1/fc = 2/c y RC >> 1/c. Esto significa
que la constante de tiempo RC es mucho mayor que el intervalo entro dos picos sucesivos de la
portadora. En consecuencia, vc(t) puede ser aproximada por
t

vc ( t ) E0 1

RC

As que si vc(t) va a seguir la envolvente de xc(t), se requiere que en cualquier instante t0


1

(1 + cos m t0 ) 1
RCf c

1 + cos m t0 +
fc

398

xr(t)

z(t)

(a)
Envolvente
z(t)
Portadora

ac
t

(b)

Envolvente
z(t)
Portadora

ac
t

(c)
Envolvente
z(t)
Portadora

ac
t

(d)

Figura 7.11 Demodulacin de envolvente de seales AM. (a) Detector


de envolvente. (b) Demodulacin correcta. (c) RC demasiado grande.
(d) RC demasiado pequea.

Ahora, si m << c, entonces



1

1 + cos m t0 + = 1 + cos m t0 + m
fc
fc

= 1 + cos m t0 cos
1 + cos m t0

y, por tanto,

m
fc
m
fc

sen m t0 sen
sen m t0

m
fc

399

1 m
(1 + cos m t0 )
sen m t0

fc
RCf c

o
sen m t0
m

RC
1 + cos m t0
1

vc(t)

Envolvente

E0

t0

t0 +1/fc

Figura 7.12

(b) Escribiendo de nuevo la Ec. (7.25), tenemos


1

RC

RC

cos m t0 m sen m t0

o
1

1
cos m t0
m sen m t0
RC

RC
o
2

1 1

1
1

sen m t0 tan
m TC RC
RC

2
m

Como esta desigualdad debe cumplirse para todo t0, se debe tener que
2

1
1
+

RC
RC
2
m

o
2 1 2 1 2
m +

RC RC
2

A partir de la cual se obtiene la relacin


RC

1 2

400

La seal AM tambin puede ser demodulada pasando xr(t) a travs de un dispositivo de ley
cuadrtica (o cualquier otro tipo de alinealidad que no tenga simetra de funcin impar) y filtrando la
salida al cuadrado. Los detalles de un demodulador de ley cuadrtica se dejan como un ejercicio.
Los demoduladores de envolvente y de ley cuadrtica para seales AM no requieren de una seal
sincronizada (coherente) de un oscilador local. Esto detectores son simples, eficientes y su
construccin es de bajo costo. Solamente los ahorros en el costo de construir los receptores justifican
su uso en muchas aplicaciones de la AM, como, por ejemplo, en la radio comercial. Los factores que
conspiran contra las ventajas de la simplicidad del equipo son la potencia desperdiciada en la portadora
y la pobre respuesta de baja frecuencia de los sistemas AM que usan demoduladores de envolvente o
de ley cuadrada.
7.4.6 Modulacin de Banda Lateral nica (SSB)

El requisito de potencia del transmisor y el ancho de banda de transmisin son parmetros importantes
de un sistema de comunicacin. Los ahorros en el requerimiento de potencia y en el ancho de banda
son altamente deseables. El esquema AM despilfarra tanto potencia transmitida como ancho de banda
de transmisin. El esquema de modulacin DSB tiene menos requerimientos de potencia que la AM
pero usa el mismo ancho de banda que ella. Ambas, la modulacin DSB y la AM, retienen las bandas
laterales superior e inferior de la seal del mensaje resultando en un ancho de banda de transmisin que
es el doble del ancho de banda de la seal del mensaje.
El espectro de cualquier seal x(t) de valores reales debe exhibir la condicin de simetra dada por
X ( f ) = X *( f )

y por ello las bandas laterales de la AM y DSB estn relacionadas en forma nica entre s por la
simetra. Dadas entonces la amplitud y la fase de una, siempre podemos reconstruir la otra. De aqu que
el ancho de banda puede ser reducido a la mitad si se elimina completamente una banda lateral. Esto
conduce a la modulacin de banda lateral nica (SSB, por sus siglas en ingls). En la modulacin
SSB, el ahorro en ancho de banda es acompaado por un aumento considerable en la complejidad del
equipo.
Adems de la complejidad del equipo, los sistemas SSB prcticos tienen una pobre respuesta de baja
frecuencia. Es posible una reduccin en la complejidad del equipo y mejoras en la respuesta de baja
frecuencia si solamente se suprime parcialmente una banda lateral en lugar de eliminarla
completamente. Los esquemas de modulacin en los cuales se transmite una banda lateral ms un
residuo de la segunda banda lateral se conocen como esquemas de modulacin de banda lateral
residual (VSB por sus siglas en ingls). La modulacin VSB se usa ampliamente para transmitir
seales de mensajes que tienen anchos de banda muy grandes y contenidos significativos de baja
frecuencia (tales como en la transmisin de datos de alta velocidad y en televisin).
En la modulacin SSB slo se transmite una de las dos bandas laterales que resultan de la
multiplicacin de la seal del mensaje x(t) con una portadora. En la Fig. 7.13a se muestra la generacin
de una seal SSB de banda lateral superior mediante el filtrado de una seal DSB, esto se conoce como
el mtodo de discriminacin de frecuencia. La recuperacin de la seal de la banda base mediante
demodulacin sincrnica se muestra en la Fig. 7.13b En las Figs. 7.13c, d y e muestran la

401

representacin en el dominio de la frecuencia de las operaciones importantes en un esquema de


modulacin SSB. La descripcin en el dominio del tiempo de seales SSB es algo ms difcil, excepto
por el caso de modulacin de tono

Seal DSB
x(t)

Filtro de
banda lateral

xc(t)

xc(t)

Ac cos c t

z(t)

Filtro de
banda base

y(t)

2 cos c t

(a) Modulador

(b) Demodulador

X(f)

fx

fx

(c) Espectro de la seal


HSSB(f)

fc fx

fc

fc

fc + f x

(d) Filtro de banda lateral ideal


Xc(f)

fc fx

fc

fc

fc + f x

(e) Espectro de la seal transmitida


Xc(f)

fx

fx

(f) Seal reconstruida

Figura 7.13 Modulacin de banda lateral nica.

De la Fig. 7.13 se puede verificar que el ancho de banda de la seal SSB es


BT = f x

(7.27)

402

Y el promedio de la potencia transmitida es


ST = 12 Sc S x

(7.28)

Las operaciones de modulacin y demodulacin para la seal SSB como se muestran en la Fig. 7.13
parecen muy sencillas. Sin embargo, la implementacin prctica es bastante difcil por dos razones.
Primero, el modulador requiere de un filtro de banda lateral ideal; segundo, el demodulador requiere de
una portadora sincrnica.
Las caractersticas agudas del corte requerido del filtro de banda lateral HSSB(f) no pueden ser
sintetizadas exactamente. Por ello, se debe atenuar una parte de la banda lateral deseada o pasar una
porcin de la banda lateral no deseada. Afortunadamente, muchas (no todas) seales de mensaje tienen
poco o ningn contenido de bajas frecuencias. Estas seales (por ejemplo, de voz o msica) tienen
agujeros a frecuencia cero y estos agujeros aparecen como un espacio vacante centrado en la
frecuencia de la portadora. La regin de transicin de un filtro de banda lateral prctico puede ser
acomodada en esta regin como se muestra en la Fig. 7.14. Como una regla emprica, la relacin 2/fc
no puede ser menor que 0.01 si se desea una frecuencia de corte razonable. La anchura de la regin de
transicin 2 est limitada a la anchura del agujero en el espectro y para una fc dada, puede no ser
posible obtener un valor razonable para la relacin 2/fc. Para esos casos, el proceso de modulacin
puede hacerse en dos o ms etapas usando una o ms frecuencias portadoras.

Xc(f)

X(f)

fx

HSSB(f)

fc

2
(a) Espectro del mensaje

(b) Espectro DSB

Figura 7.14 Caractersticas del filtro de banda lateral. (a) Espectro del mensaje. (b) Espectro DSB.

La seal SSB puede ser generada por otro mtodo denominado el mtodo de desplazamiento o de
corrimiento de fase, el cual no requiere de un filtro de banda lateral. Para ilustrar cmo trabaja este
mtodo, supongamos que la seal del mensaje tiene la forma
n

x ( t ) = X i cos (2 f i t + i ),
i =1

fn f x

Entonces la seal SSB (banda lateral superior) correspondiente a x(t) est dada por
xc ( t ) =
Podemos re-escribir xx(t) como

Ac
2

X
i=1

cos[2 ( f c + fi ) t + i ]

(7.29)

403

xc ( t ) =
=

Ac
2

Ac n

X i cos (2 fi t + i ) cos 2 f c t X i sen (2 f i t + i ) sen 2 f c t


2 i =1

i =1

[ x ( t ) cos 2 f c t ]

Ac
2

x ( t ) sen 2 f c t

(7.30)

donde x ( t ) se define como


n

x ( t ) = X i sen (2 f i t + i )

(7.31)

i =1

Las Ecs. (7.29), (7.30) y (7.31) sugieren que una seal SSB puede ser generada a partir de dos seales
de doble banda lateral (DSB) que tienen portadoras en cuadratura 12 Ac cos c t y 12 Ac sen c t
moduladas por x(t) y x ( t ) . La componente de la seal en cuadratura x ( t ) [conocida como la
transformada de Hilbert de x(t)], se obtiene a partir de x(t) desplazando la fase de cada componente
espectral de x(t) por 90. En la Fig. 7.15 se muestra un modulador SSB de desplazamiento de fase
consistente de dos moduladores DSB (de producto) y redes apropiadas de desplazamiento de fase. El
diseo de circuitos para el desplazamiento de fase no es trivial y un diseo imperfecto generalmente
resulta en distorsin de las componentes de baja frecuencia.

1
2

Ac x ( t ) cos c t

Ac cos c t

x(t)
Desplazamiento
de 90

Desplazamiento
de 90

1
2

Seal SSB

Ac x ( t ) sen c t

Figura 7.15 Modulador SSB por desplazamiento de fase.

En lugar de usar un demodulador sincrnico podemos aadir una componente de portadora a la seal
SSB (preferiblemente en el transmisor) e intentar demodular la seal SSB usando un demodulador de
envolvente. Sin embargo, este procedimiento conducir a alguna distorsin de la seal y a desperdiciar
potencia transmitida como se discute en la seccin siguiente.

Ejemplo 6. Demuestre que si la salida del modulador de corrimiento de fase (Fig. 7.16) es una seal
SSB, (a) la diferencia de las seales en la unin de suma produce la SSB de banda lateral superior

404

(USB, por sus siglas en ingls) y (b) la suma produce la seal SSB de banda lateral inferior (LSB por
sus siglas en ingls). Es decir,
xc ( t ) = xUSB ( t ) = m ( t ) cos c t m ( t ) sen c t

(7.32)

es una seal SSB de banda lateral superior y


xc ( t ) = xLSB ( t ) = m ( t ) cos c t + m ( t ) sen c t

(7.33)

es una seal SSB de banda lateral inferior.


Solucin
(a) Suponga que
m (t )

M ( )

y m ( t )

M ( )

Entonces aplicando el teorema de modulacin o la propiedad de corrimiento de frecuencia de la


transformada de Fourier, tenemos
m ( t ) cos c t

m ( t ) sen c t

1
M ( c ) + M ( + c )
2
2

1
1
M ( c )
M ( + c )
2j
2j

Tomando la transformada de Fourier de la Ec. (7.32), se obtiene


X c ( ) =

1
1
1

M ( c ) + M ( + c ) M ( c )
M ( + c )
2j
2
2
2 j

Tambin sabemos que


m ( t ) cos c t

cos c t

2
sen c t

m ( t )sen c t

Figura 7.16

405

M ( c ) = j sgn ( c ) M ( c )
M ( + c ) = j sgn ( + c ) M ( + c )

y as

X c ( ) =

1
2

1
M ( c ) + M ( + c )
2
2
1
1

sgn ( c ) M ( c ) + sgn ( + c ) M ( + c )
2
2

M ( c )[1 + sgn ( c )] +

1
2

M ( + c )[1 sgn ( + c )]

Puesto que
2
1 + sgn ( c ) =
0

> c
< c

y
2
1 sgn ( + c ) =
0

< c
> c

tenemos
0

X c ( ) = M ( + c )

M ( c )

< c
< c
> c

la cual se dibuja en la Fig. 7.17b. Vemos que xc(t) es una seal SSB de banda lateral superior.
(b) En una forma similar, tomando la transformada de Fourier de la Ec. (7.33), se obtiene que
X c ( ) =

1
M ( c )[1 sgn ( c )] + M ( + c )[1 + sgn ( + c )]
2
2

Como
2
1 sgn ( c ) =
0

< c
> c

y
2
1 + sgn ( + c ) =
0

Tenemos

> c
< c

406

M()

M 0 M

(a)
M( + c)

Xc(

M( c)

(b)
M( + c)

Xc(

M( c)

(c)

Figura 7.17

X c ( ) = M ( c )
M ( + )
c

> c
< c
> c

la cual se dibuja en la Fig. 7.17c. Vemos que xc(t) es una seal SSB de banda lateral inferior.
Ejemplo 7. Demuestre que una seal SSB puede ser demodulada por el detector sincrnico de la Fig.
7.18, (a) Dibujando el espectro de la seal en cada punto y (b) obteniendo la expresin en el dominio
del tiempo de las seales en cada punto.

xSSB

d(t)

LPF

y(t)

cos c t

Figura 7.18 Detector sincrnico

407

Solucin
(a) Sea M(), el espectro del mensaje m(t), como se muestra en la Fig. 7.19a. Suponga tambin que
xSSB(t) es una seal SSB de banda lateral inferior y que su espectro es XSSB(), como se muestra en
la Fig. 7.19b. La multiplicacin por cos c t desplaza el espectro de XSSB() hasta c y
obtenemos D(), el espectro de d(t), Fig. 7.19c. Despus de un filtrado de pasabajas, obtenemos
Y ( ) = 12 M ( ) , el espectro de y(t). As pues, obtenemos y ( t ) = 12 m ( t ) , que es proporcional a
m(t).

M()

0 M

(a)

XSSB()

(b)

D()

(c)

Y() = M()

0
(d)

Figura 7.19

(b) De la Ec. (7.30), la seal xSSB(t) puede ser expresada como


xSSB ( t ) = m ( t ) cos c t m m ( t ) sen c t
As que

408

d ( t ) = xSSB ( t ) cos c t
= m ( t ) cos 2 c t m m ( t ) sen c t cos c t
= 12 m ( t ) (1 + cos 2 c t ) m 12 m ( t ) sen 2 c t
= 12 m ( t ) + 12 m ( t ) cos 2 c t m 12 m ( t ) sen 2 c t

Por lo tanto, despus del filtrado de pasabajas se obtiene


y ( t ) = 12 m ( t )

7.4.7 Modulacin de Banda Lateral Residual (VSB)

Muchas seales de mensajes como la de video en TV, facsmile y seales de datos de alta velocidad
tienen un ancho de banda muy grande y un contenido significativo de baja frecuencia. La modulacin
SSB tiene una pobre respuesta de baja frecuencia. Aun cuando la DSB trabaja bien para mensajes con
alto contenido de bajas frecuencias, el ancho de banda de transmisin de la DSB es el doble del de la
SSB. Un esquema de modulacin que ofrece el mejor compromiso entre la conservacin del ancho de
banda, respuesta de baja frecuencia mejorada y mejor eficiencia de potencia es la modulacin de banda
lateral residual (VSB, por sus siglas en ingls).
La modulacin VSB se deriva filtrando seales DSB o AM en una forma tal que se pasa casi
completamente una banda lateral pero solo un residuo de la otra banda. En la Fig. 7.20 se muestra una
funcin de transferencia de un filtro VSB tpico. Un requisito importante y esencial del filtro de VSB,
HVSB(f), es que debe tener simetra impar con respecto a fc y una respuesta relativa de en fc.

x(t)

Modulador de
amplitud

VSB + portadora

HVSB(f)

Ac cos c t
(a) Modulador
HVSB(f)
HSSB(f)
H(f) = HSSB(f) HVSB(f)
H(f)
fc

fc +

(b) Caractersticas del filtro

Figura 7.20 Modulacin VSB. (a) Modulador. (b) Caractersticas del filtro.

409

El filtro de banda lateral VSB tiene un intervalo de transicin de anchura 2 Hz y el ancho de banda
de transmisin de la seal VSB es
BT = f x + ,

< fx

(7.34)

Para derivar una expresin en el dominio del tiempo para la seal VSB, expresemos H VSB ( f ) como
H VSB ( f ) = H SSB ( f ) [ H ( f )]

(7.35)

donde H ( f ) representa la diferencia entre la respuesta de los filtros SSB y VSB. Se requiere que
H ( f ) tenga simetra impar con respecto a fc (la razn de este requerimiento se aclarar cuando se
trabaje el Prob. 3.26). La entrada al filtro VSB es Ac [1 + x ( t )]cos c t y la seal de salida puede
expresarse en la forma
xc ( t ) = 12 Ac cos c t + 12 Ac [ x ( t ) cos c t x ( t ) sen c t ] 12 Ac x ( t ) sen c t
{
14243 1444442444443
VSB +
portadora
seal SSB
portadora
14444444442444444444
3

(7.36)

seal VSB

En la Ec. (7.36), 12 Ac x ( t ) sen c t es la respuesta de H ( f ) a la entrada Ac x ( t ) cos c t . La Ec.


(7.36) tambin se puede escribir como
xc ( t ) = 12 Ac [1 + x ( t )]cos c t 12 Ac ( t ) sen c t

(7.37)

donde ( t ) = x ( t ) + x ( t ) . Si (t ) = 0 , entonces la Ec. (7.37) se reduce a una seal AM y cuando


( t ) = x ( t ) , tenemos una seal SSB + portadora.
Aunque no es fcil derivar una expresin exacta para la potencia promedio transmitida en la
modulacin VSB, podemos obtener cotas para ST como
Sc + 12 Sc S x ST Sc S x + Sc

(7.38)

donde Sc es la potencia de la portadora y Sx es la potencia de la seal.


El lector puede verificar que la seal VSB puede ser demodulada mediante un demodulador
sincrnico. Sin embargo, resulta que podemos demodular una seal VSB con una pequea distorsin
usando demodulacin de envolvente si se ha aadido una componente grande de portadora a la seal
VSB en el transmisor.
Demodulacin de Envolvente de Seales de Banda Lateral Suprimida. A menudo es deseable
combinar la demodulacin de envolvente de la AM con la conservacin del ancho de banda de las
seales de banda lateral suprimida. La demodulacin de envolvente perfecta y libre de distorsin
requiere ambas bandas laterales y una seal portadora grande. Aadiendo una portadora a la seal
VSB, tenemos que

xc ( t ) = Ac { [1 + x ( t ) ] cos c t ( t ) sen c t}

(7.39)

Para AM, ( t ) = 0 ; y ( t ) = x ( t ) para SSB + portadora. Para VSB + portadora, ( t ) toma un valor
intermedio.

410

La envolvente de xc ( t ) se encuentra escribiendo


xc ( t ) = R ( t ) cos[ c t + ( t )]
donde R ( t ) es la envolvente dada por
R ( t ) = Ac

{ [1 + x ( t )]

+ [ ( t )]2

12

12

( t ) 2
= Ac [1 + x ( t )] 1 +

1 + x ( t )

(7.40)

La Ec. (7.40) muestra que la envolvente est distorsionada (la envolvente sin distorsin, igual que en el
caso AM, es Ac [1 + x ( t )] ). Sin embargo, si ( t ) << 1, la distorsin es despreciable y
R ( t ) Ac [1 + x ( t )], igual que en el caso AM. As que la clave para el xito de la deteccin de
envolvente de seales de banda lateral suprimida es mantener pequeo el componente de cuadratura
(t ) .
Para la seal SSB + portadora, ( t ) = x ( t ) y, por tanto, (t) no puede ignorarse. Adicionalmente, en
la portadora se desperdicia una cantidad substancial de potencia, mucho ms que en la AM. Para una
seal VSB con una banda lateral no demasiado pequea, la mayor parte del tiempo ( t ) es pequea
comparada con

x ( t ) . As que la demodulacin de envolvente puede usarse sin una distorsin

excesiva. Tambin, para una seal VSB + portadora, se puede demostrar que la potencia transmitida
promedio es
ST Sc + Sc S x
que es esencialmente la misma que en AM.
La anchura permisible de la banda lateral residual depender de las caractersticas espectrales de x(t)
y de la cantidad de distorsin que se pueda tolerar. Las transmisiones de TV comercial utilizan VSB +
portadora con un 30% de banda lateral residual. Mientras que la distorsin puede ser bastante
apreciable, la evidencia experimental indica que la calidad de la imagen no se degrada mucho.
Posteriormente discutiremos varios aspectos interesantes de las seales de la TV comercial.
7.5

Conversin de Frecuencias (Mezclado)

La traslacin de frecuencia, tambin conocida como conversin de frecuencia o mezclado, es la


operacin ms importante en los sistemas de modulacin lineal. La modulacin traslada el espectro del
mensaje hacia frecuencias superiores y la demodulacin es bsicamente una operacin de traslacin de
frecuencias hacia abajo. La traslacin de frecuencias tambin se usa a menudo para trasladar una seal
de pasa-bandas con una frecuencia portadora hasta una nueva frecuencia central.. Esto se puede obtener
multiplicando la seal de pasabandas por una seal peridica como se indica en la Fig. 7.21.

411

x ( t ) cos 1t

x ( t ) cos 2 t

BPF
centrado en
2

2 cos ( 1 + 2 ) t

Figura 7.21 Conversin de frecuencia o mezclado.

Modulador Balanceado. Un modulador balanceado que genera una seal DSB (producto) se muestra
en la Fig. 7.22. El modulador balanceado consiste de dispositivos de suma (amplificadores
operacionales) y dos elementos no lineales acoplados (como, por ejemplo, diodos polarizados
apropiadamente). Si suponemos que la no linealidad puede representarse mediante una caracterstica en
serie de potencias con dos trminos, entonces
y ( t ) = a1 [ Ac cos c t + x ( t )] + a2 [ Ac cos c t + x ( t )] 2 a1 [ Ac cos c t x ( t )] a2 [ Ac cos c t x ( t )] 2
= 2 a1 x ( t ) + 4 a2 x ( t ) Ac cos c t

x1 (t )
Alinealidad

x (t )

Ac cos c t

+ x (t )
2
Alinealidad

a1 x1 + a2 x12
BPF
centro fc
BW = fx

xc (t )

a1 x2 + a2 x22

(a)

xc (t )

x (t )

Ac cos ct
(b )

Figura 7.22 Modulador balanceado. (a) Diagrama de bloques de un modulador balanceado.


(b) Diagrama del circuito de un modulador balanceado.

412

Si x(t) est limitada en banda a fx y si fc > 2fx, entonces la salida del filtro de pasabandas ser
x ( t ) = (4 a2 Ac ) x ( t ) cos c t
que es la seal producto deseada. Con frecuencia se usan diodos semiconductores como los
dispositivos no lineales en los moduladores balanceados. El rendimiento de este tipo de modulador
depende de lo bien que se puedan acoplar las caractersticas de los diodos.
Modulador de Conmutacin. Otro circuito que se usa para mezclar dos seales se muestra en la Fig.
7.23. Cuando el voltaje de la portadora es positivo, hay un voltaje de salida v ( t ) ; y cuando la portadora
es negativa, el voltaje de salida es igual a cero.. As que los diodos operan como conmutadores con una
frecuencia fc y podemos escribir la salida v(t) como

v (t ) = x (t ) s (t )
donde s(t) es una funcin de conmutacin con un perodo 1/fc. Suponiendo un valor CD igual a cero
para la seal del mensaje x(t) y usando la expansin en serie de Fourier para s(t), podemos escribir v(t)
como
v ( t ) = k0 x ( t ) + k1 x ( t ) cos( c t ) + k3 x ( t ) cos( 3 c t ) + L

(7.41)

= trmino de la banda base + trmino DSB + armnicos


Al derivar la Ec. (7.41) hemos supuesto que s(t ) es una onda cuadrada simtrica, lo que implica que
los coeficientes de la serie de Fourier k2 , k4 , K son todos iguales a cero. Mediante filtrado de pasabandas de v (t ) obtenemos la salida como
xc ( t ) = k1 x ( t ) cos c t
que es la seal DSB deseada.

Mensaje

x(t )

Ac cos c t
v(t )

Seal DSB
BPF
cen tro f c
BW = 2 fx

Figura 7.23 Modulador de conmutacin.

xc (t )

413

En los sistemas prcticos, los osciladores, dispositivos de suma y filtros se construyen usando
componentes RLC y redes activas tales como transistores y amplificadores operacionales en circuitos
integrados. Para frecuencias de microondas, estos dispositivos se convierten en sistemas de parmetros
distribuidos.
7.6

Multicanalizacin por Divisin de Frecuencias

La transmisin simultnea de varias seales de mensajes por un solo canal se denomina


multicanalizacin (multiplexing en ingls). Hay dos tipos bsicos de tcnicas de multicanalizacin:
multicanalizacin por divisin de frecuencias (FDM, por sus siglas en ingls) y multicanalizacin por
divisin de tiempo (TDM, por sus siglas en ingls). En la FDM, el ancho de banda disponible en el
canal es dividido en varias ranuras que no se solapan y a cada seal de mensaje se le asigna una
ranura de frecuencias dentro de la pasa-banda del canal. Las seales individuales pueden ser extradas
de la seal FDM mediante un filtrado apropiado. La FDM se utiliza en la telefona de larga distancia,
telemetra de sondas espaciales y en otras aplicaciones.
El principio de la FDM se ilustra en la Fig. 7.24 para tres seales de mensajes que se supone estn
limitadas en banda. En general, si las seales de mensajes no estn estrictamente limitadas en banda,
entonces ser necesario un filtrado de paso bajo. Las seales limitadas en banda modulan
individualmente las subportadoras con frecuencias f c1 , f c2 y f c3 . La modulacin de subportadoras
mostrada en el ejemplo es SSB, pero se puede emplear cualquier tcnica de modulacin. Las seales
moduladas son sumadas para producir una seal multicanalizada completa x (t ) cuyo espectro se
muestra en la Fig. 7.24c.
Si se escogen adecuadamente las frecuencias subportadoras, entonces cada mensaje de seal ocupa
una ranura de frecuencias sin ningn solapamiento. Aunque los mensajes individuales estn claramente
identificados en el dominio de la frecuencia, la seal multicanalizada no tendr ningn parecido con las
seales de mensajes en el dominio del tiempo. La seal multicanalizada x ( t ) puede ser transmitida
directamente o usada para modular otra portadora de frecuencia f c antes de la transmisin.
La recuperacin de las seales de mensajes individuales se muestra en la Fig. 7.25. El primer paso en
la recuperacin es la demodulacin para extraer x ( t ) a partir de xc(t). Un filtrado de pasa-bandas de
xc ( t ) separa a xc1 ( t ), xc2 (t) y xc3 ( t ) . Finalmente, los mensajes son recuperados demodulando
individualmente a xc1 ( t ), xc2 (t) y xc3 ( t ) . Al equipo de multicanalizacin y de des-multicanalizacin a
menudo se le refiere por las siglas MUC.
Uno de los problemas principales con la FDM es la diafona, es decir, el acoplamiento cruzado
indeseado entre un mensaje y otro. La diafona (intermodulacin) surge principalmente a causa de nolinealidades en el sistema y se deben tomar precauciones considerables para reducir las no-linealidades
en dispositivos que procesan seales FDM. Una segunda fuente de diafona es una separacin espectral
imperfecta de las seales debido a filtrado imperfecto y a derivas en las frecuencias de las
subportadoras. Para reducir la posibilidad de solapamiento espectral, los espectros modulados son
separados en frecuencia mediante bandas de guarda, en las cuales se puedan acomodar las regiones de
transicin del filtro.

414

fc1

X1(f)
x1(t)
fx1

fx1
X2(f)

xc1(t)

LPF

Modulador SSB
fc2

x2(t)
fx2

LPF

fx2
X3(f)

xc2(t)

Modulador SSB
fc3

x3(t)
fx3

fx3

x(t)

Modulador
de portadora

xc(t)

xc3(t)

LPF

(a) Espectros del mensaje

Modulador SSB

(b) Transmisor FDM

X(f)

Banda de guarda

fc1 fc1 + fx1 fc2 fc2 + fx2

Filtro receptor para xc3

fc3 fc3 + fx3

(c) Espectro de la seal multicanalizada

Figura 7.24 Multicanalizacin por divisin de frecuencia (FDM) (a) Espectrodel mensaje. (b)
Transmisor FDM. (c) Espectro de la seal multicanalizada.

BPF
fc1

LPF
fx1

x1(t)

LPF
fx2

x2(t)

LPF
fx3

x3(t)

fc1
xc(t)

Demodulaci
n de
d

BPF
fc2
fc2
BPF
fc3
fc3

Figura 7.25 Receptor de FDM.

El ancho de banda mnimo de una seal FDM es igual a la suma de los anchos de banda de todas las
seales de mensajes. Si se usa un esquema de modulacin diferente de la SSB para multicanalizar, el
ancho de banda de la seal FDM ser mayor. La presencia de las bandas de guarda aumenta an ms el
ancho de banda.

415

Problemas

7.1 Dos seales x1 (t ) y x 2 (t ) cuyas transformadas de Fourier X 1 ( f ) y X 2 ( f ) se muestran en la Fig.


7.26, se combinan para formar la seal

y ( t ) = x1 ( t ) + 2 x2 ( t ) cos 2 f c t ,

f c = 20000 Hz

(a) Determine el ancho de banda de la seal y(t).


(b) Dada y(t), cmo se separaran x1(t) y 2 x2 ( t ) cos 2 f c t ?

104

X1(f)

X2(f)

104

f (kHz)

10

10 f (kHz)

Figura 7.26

7.2 Una seal m(t ) tiene una transformada de Fourier


1,
M(f )=
0,

f1 f f 2 , f1 = 1 kHz; f 2 = 10 kHz
otros valores de f

Suponga que se forma una seal y ( t ) = m ( t ) cos 2 (10)6 t . Halle la banda de frecuencias para la
cual y(t) tiene componentes espectrales diferentes de cero. Tambin determine la relacin entre las
frecuencias ms alta y ms baja [para las cuales Y ( f ) 0 ] de y(t). Compare esta relacin con
f2/f1. Es y(t) una seal de banda angosta? (Se dice que una seal de pasa-bandas es una seal de
banda angosta si f alta f baja 1. )
7.3 Considere un sistema con la amplitud y respuesta de fase mostradas en la Fig. 7.27 y las tres
entradas siguientes:
x1 (t ) = cos 500t + cos 2000t
x 2 (t ) = cos 500t + cos 2500t
x 3 (t ) = cos 2500t + cos 3500t

(a) Determine las salidas y1 ( t ), y2 ( t ) y y3 ( t ) .


(b) Identifique el tipo de distorsin, si la hay, sufrida por cada una de las seales de entrada.
7.4 Demuestre que un filtro RC de pasabajas da una transmisin casi libre de distorsin si la entrada al
filtro est limitada en banda a f x << f 0 = 1 2 RC .

416

H(f)

arg H ( f )

90

1.5
2 1

(kHz)

1.5

(kHz)

90
(a)

(b)

Figura 7.27 (a) Respuesta de amplitud. (b) Respuesta de fase

7.5 Suponga que una funcin de transferencia con rizos en la respuesta de amplitud puede ser
aproximada por

(1 + cos t0 ) exp ( jtd ) ,


H( f )=
otros valores de f
0

< 1,

f < fx

donde fx es el ancho de banda de la seal de entrada x(t). Demuestre que la salida y(t) es
y ( t ) = x ( t td ) +

[ x ( t t d + t0 ) + x ( t t d t0 ]

es decir, y(t) tiene un par de ecos.


7.6 La funcin de transferencia de un canal se muestra en la Fig. 7.28. La entrada al canal es una seal
de pasabajas x(t) con un ancho de banda igual a fx. Disee un compensador de cinco derivaciones
para este canal. (Ayuda: Expanda 1 H c ( f ) como una serie de Fourier en el intervalo [ f x , f x ] y
use los coeficientes de la serie para ajustar las ganancias de las tomas del compensador.)

f 2
H c ( f ) = exp

2 fx

Figura 7.28 Hc(f) para el Problema 7.6.

7.7 Un elemento no-lineal en un sistema de comunicacin tiene la caracterstica de transferencia

y ( t ) = x ( t ) + 0.2 x 2 ( t ) + 0.02 x3 ( t )
La salida deseada es el primer trmino.
x ( t ) = cos 700 t + cos150 t , determine:

Si

la

entrada

(a) los trminos de distorsin en las frecuencias de la seal de entrada;


(b) los trminos de distorsin de segundo armnico;

la

no-linealidad

es

417

(c) los trminos de distorsin de tercer armnico;


(d) los trminos de distorsin por intermodulacin.
7.8 Considere una seal x ( t ) = x2 ( t ) + x1 ( t ) cos 2 ft , donde x1(t) y x2(t) tienen los espectros
mostrados en la Fig. 7.26 y fc = 2000 Hz. Suponga que x(t) se aplica a una no-linealidad con una
caracterstica de transferencia y ( t ) = x ( t ) + 0.002 x 2 ( t ) . Dibuje las componentes que forman el
espectro de y(t) e identifique los trminos de intermodulacin (productos cruzados).
7.9 Una seal de pasabajas x(t) con un ancho de banda de 10 kHz es multiplicada por cos c t para
producir xc(t). Determine el valor de fc para que el ancho de banda de xc(t) sea un 1% de fc.
7.10 Una seal de pasabajas x ( t ) = 2 cos 2000 t + sen 4000 t es aplicada a un modulador DSB que
opera con una frecuencia de portadora igual a 100 kHz. Dibuje la densidad espectral de potencia
de la salida del modulador.
7.11 Se pueden generar seales DSB multiplicando la seal del mensaje por una portadora nosinusoidal como se muestra en la Fig. 7.29.

(a) Demuestre que el esquema mostrado en la figura trabajar si g(t) no tiene componente CD y
la frecuencia de corte del filtro es fc + fx, donde fc es la frecuencia fundamental de g(t) y fx es
el ancho de banda de x(t).
(b) Suponga que x ( t ) = 2 cos1000 t y que g(t) es como se muestra en la Fig. 7.27. Determine el
ancho de banda del filtro. Escriba una expresin para la salida xc(t).
(c) Cmo modificara el sistema si g(t) tiene una componente CD?

g(t
x(t)
LPF

g(t)
(Seal peridica)

xc(t)
1

1
1 seg

Figura 7.29 Modulador DSB para el Prob. 7.11

7.12 Demuestre que es posible demodular una seal DSB xc ( t ) = Ac x ( t ) cos 2 f c t multiplicndola
por una onda rectangular con un perodo T = 1 f c y luego pasando la salida por un filtro de
pasabajas (suponga que la onda rectangular es una funcin par de t).
7.13 Demuestre que la potencia promedio de una seal DSB xc ( t ) = Ac x ( t ) cos c t es Sc S x [Ec.
(3.14)] probando que

418

lm

1T2

t T 2

x 2 ( t ) cos 2 c tdt = 0

Suponga que x ( t ) est limitada en banda a fx y que f c >> f x .


7.14 Una forma de onda modulada en amplitud tiene la forma

xc ( t ) = 10(1 + 0.5cos 2000 t + 0.5cos 4000 t ) cos 20000 t


(a) Dibuje el espectro de amplitudes de xc(t).
(b) Determine la potencia promedio contenida en cada componente espectral incluyendo la
portadora.
(c) Halle la potencia total, la potencia en las bandas laterales y la eficiencia en potencia.
(d) Cul es el ndice de modulacin?
7.15 En la Fig. 7.30 se muestra una forma de onda AM. Suponga que la seal del mensaje es
sinusoidal.

(a) Determine el ndice de modulacin.


(b) Calcule Sc, Sxy la eficiencia en potencia.

xc(t)
10 V
5
0
5
10 V

Figura 7.30 Forma de onda para el Prob. 7.15.

7.16 Un transmisor AM desarrolla una salida de potencia no modulada de 400 vatios a travs de una
carga resistiva de 50 ohmios. La portadora es modulada por un solo tono con un ndice de
modulacin de 0.8.

(a) Escriba la expresin para la seal AM xc(t) suponiendo que fx = 5 kHz y fc = 1 MHz.
(b) Halle la potencia promedio total de la salida del modulador.
(c) Halle la eficiencia de potencia del modulador.
7.17 Los moduladores prcticos con frecuencia tienen una limitacin de potencia pico adems de una
limitacin de potencia promedio. Suponga que un modulador DSB y un modulador AM estn
operando con una seal

x ( t ) = 0.8cos 200 t

419

y con una forma de onda portadora igual a 10 cos 2 ft ( f c >> 100 Hz).
(a) Determine la potencia pico (instantnea) de las seales DSB y AM.
(b) Obtenga la relacin entre la potencia pico y la potencia promedio en las bandas laterales para
las seales DSB y AM y compare las relaciones.
7.18 Considere el modulador de conmutacin mostrado en la Fig. 7.31.

(a) Suponiendo que mx x ( t ) << Ac y que el diodo acta como un conmutador ideal, demuestre
que
v0 ( t ) Ac [ cos(2 f c t ) + mx ( t ) ] g p ( t )

donde gp(t) es un tren de pulsos rectangulares con perodo 1/fc y un ciclo de trabajo de .
(b) Sustituyendo la serie de Fourier para gp(t) en la ecuacin anterior, demuestre que v0(t) tiene
un componente de la forma A [1 + mx ( t ) ] cos(2 f c t ) .
(c) Suponiendo que x(t) es una seal de pasabajas limitada en banda a fx Hz ( f x << f c ) ,
demuestre que es posible generar una seal AM mediante un filtrado de pasa-bandas de v0(t).
Ac cos 2f c t

x (t )

vi (t )

v0 (t )

(a )

v0 (t )

v0 = vi

(b)

vi (t )

Figura 7.31 (a) Modulador de conmutacin. (b) Caractersticas del diodo ideal.

7.19 Una seal AM de la forma Rx ( t ) cos(2 f c t ) pasa por un canal de pasabandas con una funcin de

transferencia H ( f ) = K exp [ j ( f ) ] . La respuesta de fase del canal es de tal forma que puede

ser aproximada por una serie de Taylor con dos trminos como
( f + fc ) ( fc ) + f

d ( f )
df

f = fc

Demuestre que la seal en la salida del canal puede ser representada por
y ( t ) = KRx ( t t R ) cos[2 f c ( t tc )]
donde el retraso de la portadora tc y el retraso de la envolvente tR estn dados por

420

tR =
tc =

1 d ( f )
2

df

f = fc

( fc )
2 f c

7.20 Considere el demodulador de ley cuadrtica para seales AM mostrado en la Fig. 7.32.

% ).
(a) Dibuje el espectro de la salida x(t
(b) Demuestre que si x ( t ) << 1 , entonces x% ( t ) a + kx ( t ) , donde a y k son constantes.

xc ( t ) = Ac 1 + x ( t ) cos c t

z
y
No linealidad
z = ay2

Filtro de
pasabajas
Frecuencia
de corte fx

Figura 7.32 Demodulador de ley cuadrtica para seal AM.

7.21 La seal xc ( t ) = 2 (1 + 0.4 cos 6000 t ) cos106 t es aplicada a un dispositivo de ley cuadrtica

con una caracterstica de transferencia y = ( x + 4) 2 . La salida del dispositivo de ley cuadrtica es


filtrada por un LPF ideal con una frecuencia de corte de 8000 Hz. Dibuje el espectro de
amplitudes de la salida del filtro.
x ( t ) = 2 cos1000 t + cos 2000 t es multiplicada por una portadora igual a
10 cos10 t . Escriba la expresin para los trminos de la banda lateral superior de la seal
producto.

7.22 Una seal


5

7.23 Con frecuencia se usa un esquema de modulacin de multietapas para generar una seal SSB
usando filtros con 2 f c < 0.01 (Fig. 7.14). Suponga que queremos usar el esquema mostrado en
la Fig. 7.33 para generar una seal SSB con una frecuencia portadora fc = 1 MHz. El espectro de
la seal moduladora se muestra en la Fig. 7.34. Suponga que se tienen filtros de pasa-bandas que
proporcionarn 60 dB de atenuacin en un intervalo de frecuencias que es aproximadamente 1%
de la frecuencia central del filtro. Especifique las frecuencias portadoras y las caractersticas del
filtro para esta aplicacin.

421

x(t)

Filtro

xc(t)

Filtro

Portadora
fc 1

Portadora
fc 2

Figura 7.33 Un modulador SSB de dos etapas.

X(f)

3000

300

300

3000

Figura 7.34 Espectro de la seal para el Prob. 7.23.

7.24 La Fig. 7.35 muestra el modulador SSB de Weaver. Analice su operacin tomando
x ( t ) = cos 2 f x t ( f x < 2 B ) . Demuestre que xc(t) es una seal SSB.

LPF
BW = B
f1

x(t )

f1 = B
f2 = fc B
+ f x < 2B

f2

90o

90o

xc (t )

LPF
BW = B

Figura 7.35 Modulador SSB de Weaver (compare este modulador con el de la Fig. 7.15).

7.25 Dibuje el diagrama esquemtico de un demodulador sincrnico para una seal VSB. Demuestre
que el filtro VSB que se usa para general la seal VSB debe tener la simetra mostrada en la Fig.
7.20.
7.26 Verifique la afirmacin que sigue a la Ec. (7.35).
7.27 Obtenga una expresin para una seal VSB generada con x ( t ) = cos 2 f x t y
H VSB ( f c + f x ) = 0.5 + a , H VSB ( f c f x ) = 0.5 a , (0 < a < 0.5). Escriba la respuesta en la forma

422

de envolvente y de fase y en la forma de cuadratura. Tome a = 0.25 y evale el trmino de


distorsin en la Ec. (7.40).
7.28 Dibuje el diagrama de bloques para un modulador AM que usa un dispositivo no-lineal cuya
caracterstica de transferencia es vsal = a1 ven + a3 ven3 .
7.29 Suponga que los elementos no-lineales usados en un modulador balanceado (Fig. 7.22a) no estn
sintonizados. Es decir, uno de ellos tiene la caracterstica de transferencia
vsal = a11 ven + a12 ven2 + a13 ven3 , mientras que el segundo tiene la caracterstica de transferencia

vsal = a21 ven + a22 ven2 + a23 ven3 . Halle la seal de salida.
7.30 Dada una seal real m(t), defina una seal

m+ = m ( t ) + jm ( t )
donde m (t ) es la transformada de Hilbert de m(t) y m+(t) se llama una seal analtica.
(a) Demuestre que
2 M ( ) > 0
<0
0

F [ m+ ( t )] = M + ( t ) =
(b) Demuestre que

Re[ m ( t ) e jc t ]
es una SSB de banda lateral superior y
Re[ m+ ( t ) e jc t ]
es una seal SSB de banda lateral inferior.
7.31 Dos seales de mensaje x1(t) y x2(t) pueden ser moduladas sobre la misma portadora usando el
esquema de multicanalizacin de cuadratura mostrado en la Fig. 7.36.

(a) Verifique la operacin de este esquema de multicanalizacin de cuadratura.


(b) Si el oscilador local en el receptor tiene una desviacin de fase igual a con respecto a la
portadora del transmisor, determine las salidas y1(t) y y2(t) (suponga que << 1 ).
7.32 Sesenta seales de voz de grado telefnico son multicanalizadas usando FDM. El ancho de banda
de la seal de voz es 3 kHz y se requiere una banda de guarda de 1 kHz entre canales de voz
adyacentes. La modulacin de la subportadora es SSB (USB) y f c1 = 0 .

(a) Dibuje el espectro tpico de la seal multicanalizada.


(b) Si todos los canales se multicanalizan directamente, calcule el nmero de osciladores y
moduladores SSB requeridos.
(c) Suponga que la multicanalizacin se hace usando cinco grupos de 12 canales cada uno para
formar un supergrupo de 60 canales. Dibuje un diagrama de bloques del multicanalizador
indicando todas las frecuencias de las subportadoras. Cuntos osciladores y moduladores se
necesitan para implementar este esquema de multicanalizacin?

423

Filtro

x1 (t )
+

y1 (t )

Canal

90o

90o
+

x2 (t )

Filtro

y2 (t )

Sinc

fc

fc

Figura 7.36 Esquema de multicanalizacin en cuadratura.

7.33 Los BPF en el receptor FDM para el problema anterior tienen

f f c 2 n
H ( f f c) = 1 +

B

1 2

donde f c' es la frecuencia de la subportadora +1.5 kHz y B es el ancho de banda de 3 dB del


filtro. El ancho de banda del filtro debe ser 3 kHz y se requiere que el filtro tenga una
atenuacin de al menos 20 dB en la regin de rechazo, es decir,

H ( f f c' ) < 20 dB para

f f c' > 1.5 kHz

Determine un valor adecuado de n.


7.34 Dos seales x1(t) y x2(t) son multicanalizadas para formar

x ( t ) = x1 ( t ) + x2 ( t ) cos 2 f c t
x1(t) y x2(t) son seales de paso bajo limitadas en banda a 5 kHz con X 1 ( f ) = X 2 ( f ) = 0.0001
para f < 5 kHz y fc = 15 kHz. El canal por el cual se va a transmitir x(t) tiene una caracterstica
de transferencia no-lineal y la salida del canal es
y ( t ) = x ( t ) + 0.2 x 2 ( t )
(a) Dibuje el espectro de x(t) y de y(t). Explique las dificultades asociadas con la demodulacin
de x1(t) y x2(t) a partir de y(t).
(b) Cul de las seales demoduladas sufre la peor distorsin?
7.35 Con referencia al Prob. 7.34, suponga que la seal multicanalizada es

x ( t ) = x1 ( t ) cos 2 f c1 t + x2 ( t ) cos 2 f c2 t

424

f c1 >> 5 kHz y f 2 = f c1 + 20 kHz


(a) Dibuje el espectro de x(t) y de y(t).
(b) Pueden recuperarse x1(t) y x2(t) a partir de y(t).
7.36 En la Fig. 7.37a se muestra el espectro de una seal de mensaje m(t). Para asegurar privacidad en
la comunicacin, esta seal es aplicada a un sistema (conocido como un perturbador. (Scrambler
en ingls) mostrado en la Fig. 7.37b. Analice el sistema y dibuje el espectro de la salida x(t).

M ()

m (t )

HPF
c

(a )
B

M
C

LPF
c

x (t )

2 cos 2t

2 cos ct

2 = c + M
(b)

Figura 7.37 (HPF = filtro de pasaaltas, LPF = filtro de pasabajas).

425

REFERENCIAS
1.

Bracewell, Ronald N.: The Fourier Transform and its Applications. McGraw Hill, (2000).

2.

Carlson, A. Bruce, Crilly, Paul B., Rutledge, Janet C.: Communication Systems. McGraw Hill,
(2002).

3.

Chen, Wai-Kai: Linear Networks and Systems. World Scientific, (1990).

4.

Churchill, Ruel V.: Operational Mathematics. McGraw Hill, New York (1972).

5.

Davis, Harry F.: Fourier Series and Orthogonal Functions. Dover, New York (1989).

6.

Hsu, Hwei P.: Signals and Systems. McGraw Hill, (1995).

7.

Kuo, Benjamn C.: Sistemas de Control Automtico. Prentice-Hall, (1995).

8.

LePage, Wilbur R.: Complex Variables and the Laplace Transform for Engineers. Dover, (1980).

9.

Oppenheim, Alan V., Willsky, Alan S., Young, Ian T.: Signals and Systems. Prentice-Hall, (1983).

10. Papoulis, A.: The Fourier Integral and its Applications. McGraw-Hill, (1962)
11. Picinbono, Bernard: Principles of Signals and Systems: Deterministic Signals. Artech House,
(1988).
12. Sansone, G.: Orthogonal Functions. Dover, New York (1991).
13. Shanmugan, K. Sam: Digital and Analog Communication Systems. Wiley, (1979).
14. Sneddon, Ian N.: Fourier Transforms. Dover, (1995).
15. Tolstov, G. P.: Fourier Series. Dover, (1962).
16. Wylie, C. R., Barret, L. C.: Advanced Engineering Mathematics. McGraw-Hill, (1995).
17. Zadeh, L., Desoer, C.: Linear System Theory. Dover, (2008)

You might also like